numero 09 - outubro de 2007

498
www.famat.ufu.br Revista Científica Eletrônica da Faculdade de Matemática - FAMAT Universidade Federal de Uberlândia - UFU - MG Número 09 - Outubro de 2007 f À W e-mail: [email protected] Comitê Editorial: Márcio José Horta Dantas Valdair Bonfim - Famat/Ufu Marcos Antônio da Câmara - Famat/Ufu Weyder Orlando Brandão Junior - Petmat - Famat/Ufu - Famat/Ufu Gabriela Aparecida dos Reis - Petmat - Famat/Ufu Ernani Magno de Freitas Júnior - Petmat - Famat/Ufu FAMAT em Revista

Upload: truongcong

Post on 07-Jan-2017

306 views

Category:

Documents


18 download

TRANSCRIPT

Page 1: Numero 09 - Outubro de 2007

www.famat.ufu.br

Revista Científica Eletrônica daFaculdade de Matemática - FAMAT

Universidade Federal de Uberlândia - UFU - MG

Número 09 - Outubro de 2007

f��

e-mail: [email protected]

Comitê Editorial: Márcio José Horta Dantas Valdair Bonfim - Famat/Ufu

Marcos Antônio da Câmara - Famat/Ufu

Weyder Orlando Brandão Junior - Petmat - Famat/Ufu

- Famat/Ufu

Gabriela Aparecida dos Reis - Petmat - Famat/Ufu

Ernani Magno de Freitas Júnior - Petmat - Famat/Ufu

FAMAT em Revista

Page 2: Numero 09 - Outubro de 2007

����� �� ���� ���� �������

����� � ��������

� ������� �� � ��������

������� ��������� ���������� �� ������ ����������� �� ��� !���� �" "#

$����������� ������� �� $%���&���� $�$ '

Page 3: Numero 09 - Outubro de 2007

Editorial.

O Comitê Editorial da FAMAT em Revista, com muita satisfação, vem disponibilizar à comunidade acadêmica o seu nono número. A FAMAT em Revista é a revista eletrônica da comunidade acadêmica da Faculdade de Matemática da Universidade Federal de Uberlândia – MG. A sua finalidade é promover a circulação de idéias, estimular o estudo da Matemática e despertar a curiosidade intelectual dos estudantes e de todos aqueles que se interessam pelo estudo de Matemática.

Gostaríamos de externar nosso contentamento com a aceitação de nossa revista; a quantidade de artigos completos de iniciação científica vem se mantendo expressiva desde a terceira edição, o que tomamos como índice de nossos esforços, em prol do estudo de matemática e de mantermos uma revista voltada para os trabalhos de graduação, estão logrando certo êxito.

Em relação ao conteúdo do nono número da revista, foram contempladas as atividades desenvolvidas no segundo semestre de 2006 e no primeiro semestre de 2007. Abaixo, apresentamos de modo sucinto, as diversas contribuições e matérias que compõe cada seção.

Em Artigos Completos de Iniciação Científica, contamos com treze trabalhos muito interessantes, todos desenvolvidos em projetos de Iniciação Científica orientados por professores da FAMAT ou resultantes das atividades de cursos de Extensão. Sem dúvida, a leitura dos mesmos irá enriquecer a formação de estudantes de matemática.

Na Seção Problemas e Soluções, apresentamos as resoluções de quatro problemas propostos no número anterior. Além disso, quatro novos desafiadores problemas são propostos neste número.

Na Seção Eventos, disponibilizamos aos nossos leitores uma lista dos eventos ligados à matemática a serem realizados no segundo semestre de 2007 e no primeiro semestre de 2008. Damos particular ênfase à realização da VII Semana da Matemática que será realizada de 27 à 30 de novembro.

Na Seção Reflexões sobre o Curso de Matemática, temos um artigo do Coordenador do Curso de Matemática, Prof. Luiz Antônio Benedetti, sobre a Beleza da Matemática. Cremos que será muito instrutivo para os nossos leitores.

Na Seção Em Sala de Aula temos onze artigos. Vários deles são os trabalhos finais do curso de Modelagem Matemática ministrado pela Profa Rosana.

Na Seção Iniciação Científica em Números trazemos uma descrição dos atuais projetos de Iniciação Científica e de Ensino da FAMAT – UFU desenvolvido por alunos do Curso de Licenciatura e Bacharelado em Matemática.

Page 4: Numero 09 - Outubro de 2007

Na Seção E o meu Futuro Profissional, apresentamos uma entrevista com o Prof. João Carlos, Coordenador do Curso de Matemática da UFU, Campus do Pontal.

Na Seção Merece Registro, destacamos as atividades e os fatos que mereceram destaque na FAMAT no período de abril a setembro de 2007. Além disso, temos um artigo, extremamente interessante, do Prof. Antônio Carlos Nogueira, sobre as Olimpíadas de Matemática. O Prof. Antônio Carlos é Coordenador Regional da Olimpíada Brasileira de Matemática das Escolas Públicas (OBMEP). Também temos uma entrevista com o Prof. Luis Alberto Salomão, que é Professor Orientador da OBMEP.

Finalmente, esperamos que os nossos leitores apreciem os trabalhos aqui publicados e lembramos que críticas e sugestões produtivas são sempre bem-vindas.

Comitê Editorial

Page 5: Numero 09 - Outubro de 2007

Indice de Secoes

Secao 1: Trabalhos Completos de Iniciacao Cientıfica 7

Secao 2: Problemas e Solucoes 269

Secao 3: Eventos 277

Secao 4: Reflexoes sobre o Curso de Matematica 285

Secao 5: Em Sala de Aula 291

Secao 6: Iniciacao Cientıfica em Numeros 459

Secao 7: E o meu Futuro Profissional? 469

Secao 8: Merece Registro 475

Page 6: Numero 09 - Outubro de 2007
Page 7: Numero 09 - Outubro de 2007

FAMAT em Revista

Revista Científica Eletrônica daFaculdade de Matemática - FAMAT

Universidade Federal de Uberlândia - UFU - MG

Número 09 - Outubro de 2007www.famat.ufu.br

Trabalhos Completos deIniciação Científica

PBIIC-FAPEMIG-UFU - Programa de Bolsas Institucionais de Iniciação Científica da Fundação de Amparo à Pesquisa do Estado de Minas Gerais

PETMAT-UFU - Programa de Educação Tutorial da Faculdade de Matemática

PIBIC-CNPq-UFU - Programa Institucional de Bolsas de Iniciação Científica do Conselho Nacional de Desenvolvimento Científico e Tecnológico

PROMAT-UFU - Programa Institucional de Iniciação Científica e Monitoria da Faculdade de Matemática

IM-AGIMB - Instituto do Milênio - Avanço Global e Integrado da Matemática Brasileira

���

Page 8: Numero 09 - Outubro de 2007

Comitê Editorial da Seção Trabalhos Completos de Iniciação Científica

do Número 09 da FAMAT EM REVISTA:

Márcio José Horta Dantas (coordenador da seção) Valdair Bonfim

Marcos Antônio da Câmara

Page 9: Numero 09 - Outubro de 2007

Instrucoes para submissao de Trabalhos

A Secao de Trabalhos de Iniciacao Cientıfica visa divulgar trabalhos que estejam as-sociados a projetos cadastrados na(o) PBIIC-FAPEMIG / PETMAT / PIBIC-CNPq /PROMAT ou IM-AGIMB e orientados por docentes da FAMAT.

Trabalhos completos em nıvel de iniciacao cientıfica dos programas acima listadossubmetidos para publicacao na Revista Eletronica “Famat em Revista” estarao sujeitosa apreciacao pelo Comite Editorial responsavel por essa secao de artigos e, se for o caso,por consultores ad hoc ligados a area ou subarea do trabalho. Caso se faca necessario,sugestoes para o aperfeicoamento do trabalho serao dirigidas aos interessados pelo ComiteEditorial.

Alem da redacao clara e concisa que todo trabalho submetido a boa qualidade devepossuir, pede-se evitar o estilo arido e extremamente tecnico caracterıstico de algumaspublicacoes matematicas, nao perdendo de vista que o publico-alvo ao qual se destina arevista e constituıdo por alunos de graduacao.

Os trabalhos submetidos ate o final de um semestre letivo serao publicados na edicaoda revista lancada no inıcio do semestre letivo subsequente.

Quanto as normas tecnicas para submissao dos trabalhos:

1) Formato do arquivo: PDF

2) Tamalho da Folha: A4

3) Margens: 2,5 cm (portanto, area impressa: 16 cm x 24,7 cm)

4) Tamanho de fonte (letra): 12 pontos (exceto tıtulos, subtıtulos, notasde rodape, etc, que ficam submetidos ao bom senso)

5) Espacamento entre linhas: Simples

6) Orientador(es), tipo de programa e orgao de fomento (se houver)devem constar no trabalho.

Envio:Por e-mail: [email protected]

Page 10: Numero 09 - Outubro de 2007
Page 11: Numero 09 - Outubro de 2007

Índice de Trabalhos

Uma Introdução à Teoria de Pontos Críticos 13 Carlos Henrique Tognon e Antônio Carlos Nogueira

Ordenação de monômios, divisão em anéis de polinômios de várias variáveis e as Bases de Groebner 25 Danilo Adrian Marques e Cícero Carvalho

Evolução do Número de Alunos Reprovados nas Disciplinas do Curso deMatemática da UFU 39 Flávia Borges Arantes, Ródney Silva Abreu, Heyder Diniz Silva eRogério de Melo Costa Pinto

O Problema da Condução do Calor em Dimensão Maior que 1 e o Teorema Espectral 69

Karla Barbosa de Freitas e Valdair Bonfim

Perfil sócio-econômico dos candidatos do paies/ufu: Subprograma 2002-2005 83 Kátia Alessandra De Souza Caetano, Ednaldo Carvalho Guimarães,Rogério De Melo Costa Pinto, Marcelo Tavares

Um Texto Sobre Superfícies Parametrizadas Regulares 95 Laís Bássame Rodrigues e Edson Agustini

O teorema de Barlotti 147 Luciana Yoshie Tsuchiya, Gabriela Aparecida dos Reis e Edson Agustini

Sobre a necessidade das hipóteses no Teorema do Ponto Fixo de Banach 175 Marcelo Lopes Vieira e Valdair Bonfim

Heurísticas e Equações Diofantinas 181 Michelle Crescêncio de Miranda e Luiz Alberto Duran Salomão

Obtenção dos projetos ótimos de gráficos de X Utilizando o Matlab 189 Robson Silva Rossi e Aurélia Aparecida de Araújo Rodrigues

Formas Quadráticas e Cônicas 201Stela Zumerle Soares e Antônio Carlos Nogueira

Soma de Quadrados 217 Stela Zumerle Soares e Antônio Carlos Nogueira

Complexidade Algébrica em Demonstrações de Geometria Euclidiana Plana: o Teorema de Napoleão e Propriedades 231 Luciana Yoshie Tsuchiya, Gabriela Aparecida dos Reis e Edson Agustini

Introdução à Teoria das Curvas Algébricas Afins 259 Patrícia Borges dos Santos e Cícero Fernandes

Page 12: Numero 09 - Outubro de 2007
Page 13: Numero 09 - Outubro de 2007

UMA INTRODUÇÃO À TEORIA DE PONTOS CRÍTICOS

Carlos Henrique Tognon1 e Antônio Carlos Nogueira2

1. INTRODUÇÃO

Hoje em dia, um dos mais produtivos e atraentes ramos da Matemática é a Teoria de Singularidades. A Teoria de Singularidades não é uma teoria no sentido axiomático usual. Na verdade, é precisamente sua dimensão abrangente, suas vagas fronteiras e suas interações com outros ramos, não só da Matemática, mas da ciência em geral que a tornam tão atraente.

É comum se pensar que a Teoria de Singularidades é um descendente direto do Cálculo Diferencial e uma vez que este é ferramenta, por excelência, para se estudar física, equações diferenciais e a geometria de curvas e superfícies é de se esperar que a Teoria de Singularidades tenha aplicações nestas áreas.

De certa forma, a Teoria de Singularidades é uma extensão de vasto alcance do estudo de funções em pontos de máximo e mínimo; neste caso as funções são substituídas por famílias de funções.

Hoje a Teoria de Singularidades é uma das áreas mais desenvolvidas da Matemática e ainda muito tem a ser feito; além disso, esta teoria possui diversas aplicações em variados campos, como por exemplo, física e robótica.

A melhor maneira de se introduzir a Teoria de Singularidades é mostrando em ação uma situação concreta, onde não há dúvida sobre o que está ocorrendo e também que requeira um mínimo necessário de conhecimento prévio, como por exemplo, em algumas situações geométricas que de uma maneira oportuna serão introduzidas.

2. PONTOS CRÍTICOS

Um dos problemas centrais da Teoria de Singularidades é a classificação de tipos de pontos críticos; inicia-se nesta seção esta classificação provando-se o Lema de Morse, que classifica pontos críticos para qualquer número de variáveis. Estes pontos críticos serão chamados pontos críticos de Morse.

2.1 CLASSIFICAÇÃO DE PONTOS CRÍTICOS

Seja uma função de classe , isto é, uma função que possui derivada de todas as ordens e cada uma dessas derivadas é uma função contínua.

nf : C

Um ponto é dito um ponto crítico de se as derivadas parciais de se anulam em , isto é, se

nu f fu

021 nx

ufxuf

xuf , onde nxxxu ,,, 21 .

____________________________________1Aluno do PROMAT – FAMAT 2Orientador

Page 14: Numero 09 - Outubro de 2007

O valor é então chamado um valor crítico de .uf fGeometricamente, pontos críticos ocorrem quando o gráfico de possui uma

tangente horizontal. Se , ou seja, tem-se uma função , os pontos críticos de são classificados como máximo local, mínimo local e pontos de inflexão.

f1n :f

f

Para , quando , existem mais possibilidades. Os mais comuns são os máximos, mínimos e as selas. Exemplos destes casos são dados respectivamente por, ; ; .

2n 2:f

22, yxyxf 22, yxyxf 22, yxyxfObserve que nestes três casos o ponto crítico é a origem. Existe, entretanto

uma grande variedade de tipos mais complicados de funções, onde a análise de pontos críticos não é tão trivial. Por exemplo, as funções ;

; .

23 3, xyxyxf2, xyxf 22, yxyxf

Destes três últimos a função é a menos complicada, no sentido de que o ponto crítico é isolado, ou seja, suficientemente próximo a ele não existe outro ponto crítico.

23 3, xyxyxf

Nos outros dois casos, a origem não é um ponto crítico isolado: no caso tem-se uma reta de pontos críticos e no caso têm-se duas

retas de pontos críticos.

2, xyxf 22, yxyxf

Apesar de aparecerem em situações tão simples e corriqueiras, os pontos críticos não isolados não ocorrem com freqüência nas aplicações.

A distinção mais importante, entretanto, se faz entre pontos críticos não degenerados e pontos críticos degenerados.

Definição 1: Seja e . Considere que u seja um ponto crítico de . Se a matriz Hessiana de em possui determinante não-nulo, então

é um ponto crítico não degenerado de . Caso contrário, é um ponto crítico degenerado de .

nu nf :f f u

u f uf

A matriz Hessiana de em um ponto é a matriz quadrada de ordem dada por,

nf : nun

jiu xx

ffHess2

.

Por exemplo, se , então 22, yxyxf

2

22

2

2

2

y

fxy

f

yxf

x

f

fHess .

Assim, a matriz Hessiana da função calculada na origem será f

Page 15: Numero 09 - Outubro de 2007

2002

0,0fHess .

Esta matriz tem determinante igual a quatro, então a origem é um ponto crítico não degenerado da função .22, yxyxf

Por outro lado, se , então segue que ,

cujo determinante é nulo.

23 3, xyxyxf0000

0,0fHess

Desta forma a origem é um ponto crítico degenerado para a função .23 3, xyxyxf

Pode-se provar que pontos críticos não degenerados são sempre isolados, porém a recíproca não é verdadeira.

2.2 O LEMA DE MORSE

Os pontos críticos não degenerados são completamente classificados por um teorema, conhecido na literatura matemática como Lema de Morse.

Teorema 1 (Lema de Morse): Seja um ponto crítico não degenerado da função de classe . Então existe uma mudança de coordenadas

nunf : C

em , isto é, uma função , onde U é uma vizinhança do ponto , tal que a função

n nU: uUf : é dada por,

,221

222

21 nll yyyyyufuf

para todo ,Uyyyy n,,, 21 nl1 .

Na prova do Lema de Morse faz-se uso do seguinte lema, aceito sem demonstração.

Lema 1: Seja uma função de classe em uma vizinhança da

origem, com . Então existem funções , de classe ,

definidas em uma vizinhança da origem tais que com

nf : C

00f nif : niC ,,1,

n

iiii xfxf

1,

000i

i xff .

Agora, coloca-se o

Lema 2: Seja ( como no Lema 1, para 2:f 2n ).

Page 16: Numero 09 - Outubro de 2007

1. Se 0yf

xff em 0,0 , para , então existem funções

, definidas em uma vizinhança de

2, yx

321 ,, ggg 0,0 , tas que .32

212 gyxyggxf

2. Se 02

22

2

2

y

fyx

f

x

fyf

xff em 0,0 , para , então

existem funções , definidas em uma vizinhança de tais que

.

2, yx

4321 ,,, gggg 0,0

43

32

22

13 gygxyygxgxf

3. Se é como no item 2, então existem funções , definidas em uma

vizinhança de tais que .

f 21, gg

0,0 22

12 gygxf

Demonstração:

1. Seja tal que 2:f 0yf

xff em 0,0 , para . Pelo

Lema 1 existem funções definidas em uma vizinhança da

origem tais que , com

2, yx

22

21 :: fef

21 yfxff 00,00,000,00,0 21 yffe

xff .

Agora, pode-se aplicar o Lema 1 à função , obtendo-se funções

definidas em uma vizinhança de tais que . Analogamente, aplicando o Lema 1 à função , tem-se .

1f2

122

11 :: fef 0,0

12111 yfxff 2f

22212 yfxffAssim, 22

2211211

222211211 fyxyfxyffxyfxfyyfxfxf , portanto

.222

2112112 fyffxyfxfDesta maneira, existem funções , definidas em uma vizinhança da

origem em tais que , que é o resultado desejado. 321 ,, ggg

23

221

2 gyxyggxf

2. Tem-se agora 02

2

2

2

y

fyx

f

x

fyf

xff em , para

.

0,0

2, yxDo item anterior tem-se que 22211211 yfxfyyfxfxf . Desta forma

00000

2

21

11x

fx

fxx

ff , por hipótese. Aplicando o Lema 1 à função

, consegue-se obter funções 11f 22

21 :: fef definidas em uma

vizinhança de tais que 0,0 2111 fyfxf .

Temosyx

fxy

fx

fyy

ff

00000

221

12 , pois é de classe .f 2C

Page 17: Numero 09 - Outubro de 2007

Assim 00

02

12 yxff , por hipótese. Aplica-se, então, o Lema 1 à função

; logo se escreve 12f 121112 fyfxf , onde 212

211 :: fef são definidas

em uma vizinhança de .0,0

Também 0000

02

221 yx

fy

fxx

ff , por hipótese. Pelo Lema 1

aplicado à função , tem-se 21f 222121 fyfxf , onde 222

221 :: fef

são definidas em uma vizinhança de 0,0 .

Finalmente, 00000

2

22

22y

fy

fyy

ff , por hipótese. Então pelo

Lema 1 escreve-se , onde são definidas em uma vizinhança de

2122ˆˆ fyfxf 2

22

1 :ˆ:ˆ fef0,0 .

Daí, )ˆˆ()()()( 212221121121 fyfxyfyfxxyfyfxyfyfxxxf .Trabalhando-se esta expressão, se observa que existem funções

, definidas em uma vizinhança da origem, tais que

, como se queria demonstrar. 4321 ,,, gggg

43

32

22

13 gygxyygxgxf

3. Tem-se como no item anterior. Desta forma pode-se escrever da

seguinte maneira, .

f f

432

212 ygxgyygxgxf

Então existem funções definidas em uma vizinhança de tais que

, o que demonstra o item 3. 21 , gg 0,0

22

12 gygxf

A demonstração do Lema de Morse será feita para o caso ; o caso geral segue exatamente a mesma linha de raciocínio.

2n

Prova do Lema de Morse, caso 2n :

Pode-se supor, sem perda de generalidade, que o ponto crítico é a origem, ou seja,

2u0,0u .

Como é um ponto crítico da função , deve-se ter u 2:f 0yf

xf

em . Desenvolvendo a função em série de Taylor, até a ordem 2, em torno do

ponto obtém-se (note que é de classe ):

0,0 f

0,0 f C

Page 18: Numero 09 - Outubro de 2007

yxgyy

f

xyyx

fxx

fyy

fxx

ffyxf

,00,0

21

00,0

00,0

210

0,00

0,00,0,

22

2

22

2

2

onde são os termos de ordem superior no desenvolvimento. Como yxg ,

00,000,0y

fex

f , tem-se yxgcybxyaxfyxf ,2210,0, 22 , onde

2

22

2

2 0,00,0,0,0

y

fceyx

fbx

fa . Observe que como é de classe , se

tem que

f 2C

xyf

yxf 0,00,0 22

.

Note que 0,00,00,0 gff , logo 00,0g ;x

gx

fx

f 0,00,00,0 ,

logo 00,0x

g e y

gy

fy

f 0,00,00,0 , logo 00,0y

g ; também

00,0

00,0

2

2

2

2

y

gex

g .

Assim, e todas as suas derivadas de primeira e segunda ordem se anulam em .

g0,0A hipótese de que é um ponto crítico não degenerado de é

equivalente à condição , uma vez que

0,0u f

02 acb

cbba

fHess 0,0 ,

então o determinante de é igual a que é diferente de zero, pois

é ponto crítico não degenerado. Observe que equivale a .

0,0fHess 2bac

0,0 02bac 02 acbAssim, se 0a pode-se completar quadrados na parte quadrática de

obtendo-se,f

222

22 2 ya

bcyabxacybxyax .

Também se 0c , tem-se

222

22 2 xc

baxcbyccybxyax .

Se , então e assim pode-se escrever 0ca 0b

Page 19: Numero 09 - Outubro de 2007

22222

22 yxyxbbxycybxyax .

No caso em que , faz-se a seguinte mudança de coordenadas 0a

21

2,

abacyYay

abxX

e então a parte quadrática de toma a forma .f 22 YX

Tem-se que a aplicação dada por 22: YXyx ,, é invertível. Na verdade é um difeomorfismo (ver definição abaixo).

Fórmulas semelhantes se verificam para os outros dois casos; assim deduz-se que existe uma mudança de coordenadas em (ou seja, um difeomorfismo

local) tal que a composta tem a forma ,onde .

2

2:f yxhyxdyx ,, 22

0,0fdObserve que os sinais são independentes e, portanto existem quatro

casos. Também se tem que e todas as suas derivadas de ordem 1 e 2 se anulam em .

h0,0Aplicando o item 3 do Lema 2 à função , obtêm-se funções definidas

em uma vizinhança de tais que e então a expressão acima de

h 21 heh

0,0 22

12 hyhxh

f se torna

22

12 11, hyhxdyx ,

onde se anulam em .21 heh 0,0Para valores pequenos de yex , yxheyxh ,1,1 21 são não nulos.

Colocando yxhyYeyxhxX ,1,1 21 , tem-se que a aplicação

dada por é um difeomorfismo em uma vizinhança de .

2:f 22, YXdyx0,0Usando esta mudança de coordenadas obtêm-se as seguintes formas

normais:

22

22

22

,,,

YXdyxYXdyxYXdyx

onde . Isto conclui a demonstração. 0,0fd

Definição 2: Um difeomorfismo é uma aplicação de que é diferenciável, invertível e sua inversa é diferenciável.

nn em

Page 20: Numero 09 - Outubro de 2007

Observações referentes ao Lema de Morse: 1. Não é necessário incluir a forma , pois 22 YXd xyyx ,, é uma

mudança de coordenadas, o que significa que se trocando x por e por y y x se tem o mesmo resultado.

2. Os três casos do Lema de Morse correspondem, respectivamente, a um mínimo, uma sela e um máximo para a função em f 0,0 (figura 1).

3. O Lema de Morse diz que a função não apenas se comporta como uma das três formas normais acima, além disso, é igual a uma delas a menos de uma mudança de coordenadas no plano.

f

Máximo Mínimo Sela

Figura 1

2.3 PONTOS CRÍTICOS DEGENERADOS

Os pontos críticos não degenerados são completamente classificados pelo Lema de Morse. Quanto aos pontos críticos degenerados, a situação é diferente.

Inicia-se nesta seção o estudo dos pontos críticos degenerados no caso mais simples, ou seja, de uma função :f . Assume-se que tem um ponto crítico na origem e que . Desta maneira, deve-se ter

f00f 00'f .

Pelo que foi visto, a origem é ponto crítico não degenerado da função se, e somente se, :f 00''f , uma vez que 0''0 ffHess .

Pelo Lema de Morse existe uma mudança de coordenadas : ,definida em uma vizinhança U da origem tal que :f é dada por

, e como 200 xff 00f se tem que .20 xf

Neste caso, , se 20 xf 00''f e , se .20 xf 00''fEntretanto, se , obtém-se uma classificação mais refinada tomando-

se mais termos da série de Taylor de . Esta classificação, porém, não diz nada

sobre funções tais como

00''ff

21exp para as quais a série de Taylor é zero.

x

Page 21: Numero 09 - Outubro de 2007

Nota 1: Considere a função :f dada por

0,00,1exp2

xsexfexsex

xf .

Expandindo esta função em série de Taylor em torno do ponto obtém-se que todos os termos da expansão são iguais à zero, visto que qualquer derivada da função no ponto é igual à zero (a função definida desta forma é conhecida como função chata). Por esta razão se diz que a série de Taylor desta função é zero.

0x

f 0x f

f

Lema 3: Seja uma função de classe tal que, :q C

000''0'0 kqqqq .Então em alguma vizinhança da origem, existe uma função :l de

classe tal que e, além disso, se C xlxxq k 1 001kq , então .00l

Demonstração: A prova é feita por indução sobre .kQuando , o Lema 1 se aplica e segue o resultado. 0kPara , usa-se o mesmo Lema para mostrar que , onde

é uma função de classe .

0k xlxxq 1

:1l C

Como é de classe , diferenciando esta relação vezes, obtém-se que q C m

xlmxlxxq mmm 111 .

Observe que esta relação é obtida uma vez que,

xlmxlxxqxlxxlxlxlxqxlxxlxlxqxlxxlxq

mmm 111

'''1

''1

''1

''1

''1

'1

'1

'11

;;''';'';'

Fazendo 0x deduz-se que 00000 11

''1

'11

kllll .Como a função satisfaz a hipótese de indução, tem-se que em alguma

vizinhança da origem, existe uma função 1l

:l de classe tal que

. Assim, .

C

xlxxl k1 xlxxlxxxq kk 1

Note que, xlxxlkxxq k ')1(' ,

xlxkxlxxlxxlkxkxq kk '')2('')1('' 1 ,

xlxkxlxxlxkxlkxq kkkkkkk 11 )12(!1!1

logo, e daí, se 0!101 lkq k 001kq segue que 00l . Isto conclui a demonstração.

Page 22: Numero 09 - Outubro de 2007

Com este Lema em mãos, pode-se enunciar o

Teorema 2: Seja uma função de classe , tal que :f C

000''0'0 1kffff ,

mas . Então existe uma mudança de coordenadas sob a qual toma a forma , se é ímpar, e ou , se é par.

00kf fkx k kx kx k

Demonstração: Pelo Lema 3 tem-se que , com .xlxxf k 00lConsidere os casos: 1. é ímpar: Defina k kxlxxh 1 . Observe que

xlxlkxxlxh kkk '1' 11 , logo 000' 1 klh , então é um difeomorfismo em alguma vizinhança da origem.

h

Além disso, se tem que . Desta maneira segue que a

composta

xfxlxxh kk

kxxhfxhf 11 , visto que colocando , tem-se

, logo

xhy 1

kyhyf kkkk xxIdxhhyhxhf 11 .

2. é par: Têm-se duas possibilidades: ka) : neste caso, define-se xxl ,0 kxlxxh 1 , e o resultado segue

como no item 1, isto é, se transforma em .f kx

b) : neste caso define-se xxl ,0 kxlxxh 1 . Desta maneira

. Assim, tem-se que a composta xfxlxxlxxh kkk

kxxhfxhf 11 , visto que colocando , tem-se ,

logo

xhz 1 kzhzfkkkk xxIdxhhzhxhfxhf 111 .

Estes dois casos concluem a demonstração.

O Lema de Morse (teorema 1) e o teorema 2 motivam uma importante noção.

Definição 3: Sejam , onde são vizinhanças dos pontos , respectivamente. Diz-se que

nnnn UUuu 2121 ,,, 21 ,UU

21 , uu 11 :Uf e são funções 22 :Uf R - equivalentes se existirem vizinhanças , com , um difeomorfismo e uma constante

ii UV2,1, iVu ii 21: VVh c tal que 21 uuh

e .121 , Vucuhfuf

Nota 2: Seja . Então denota uma função definida em alguma vizinhança de . Duas tais funções são equivalentes se elas coincidem em

alguma vizinhança de .

nu 0 0,: uf n

0u

0u

Page 23: Numero 09 - Outubro de 2007

A sentença, , é uma relação de equivalência (é reflexiva, simétrica e transitiva).

0udevizinhançaumaemcoincidemgeffunçõesasgf

As funções que se relacionam com segundo f , formam uma classe de equivalência que é chamada um germe de em .f 0u

Definição 4: Suponha que 0,: tf seja A - equivalente a .Então, para , dizemos que tem tipo em , ou uma singularidade em

.

1kx0k f kA 0t kA

0tPor exemplo, tipo significa simplesmente que 0A 0' 0tf .

Um dos problemas centrais na Teoria de Singularidades é classificar funções segundo R -equivalência. Um exemplo disto já foi feito no Lema de Morse, onde uma função definida em uma vizinhança de um ponto crítico não degenerado é f R -

equivalente à função dada por .g 221

221 nll yyyyg

No teorema 2, se tem que uma função :f com série de Taylor não

nula é equivalente a para algum .kx k

Como existe o Lema de Morse para pontos críticos não degenerados, também se tem um resultado para os pontos críticos degenerados, que permite encontrar formas normais para uma função em uma vizinhança de tal ponto em dimensões maiores que 1. Este resultado é conhecido como Splitting lemma e é enunciado a seguir.

f

Teorema 3 (Splitting lemma): Seja uma função de classe ,com derivadas parciais de primeira ordem iguais a zero na origem e cuja matriz Hessiana na origem tem posto

nf : C

r . Então é f R -equivalente, na origem, a uma função da forma

nrr xxgxx ,,122

1 ,

onde é uma função de classe .rng : C

Este teorema mostra que o comportamento de uma função próximo a um ponto crítico degenerado pode ser determinado estudando-se uma função envolvendo um número de variáveis menor (igual a rn : este número é chamado o coposto (ou corank) de ). Esta redução do número de variáveis é que torna o Splitting lemma tão útil e surpreendente.

f

Exemplo: Seja uma função , com a origem sendo um ponto crítico degenerado; sob certas condições pode-se mostrar com o auxílio do Splittinglemma, que esta função é R-equivalente a uma das seguintes formas normais:

2:f

Page 24: Numero 09 - Outubro de 2007

)(,)(,

)(,

42

33

23

parabólicoumbílicoyyxyxohiperbólicumbílicoyxyx

elípticoumbílicoxyxyx

Umbílico parabólico

Figura 2

Umbílico hiperbólico Umbílico elíptico

Referências Bibliográficas

[1] Bruce, J.W. e Giblin, P.J.; Curves and Singularities; Segunda edição; Cambridge University Press; 1992.

[2] Tenenblat, Keti; Introdução à Geometria Diferencial; Editora Universidade de Brasília; 1988.

[3] Saunders, P.T.; An Introduction to Catastrophe Theory; Cambridge University Press; 1980.

Page 25: Numero 09 - Outubro de 2007

Ordenacao de monomios, divisao em aneis de polinomios

de varias variaveis e as Bases de Groebner

Danilo Adrian Marques∗ Prof. Cıcero Carvalho†

Faculdade de Matematica - FAMAT

Universidade Federal de Uberlandia - UFU

38408-100, Uberlandia - MG

Junho 2007

1 Ordens Sobre Monomios

Examinando em detalhes o algoritmo da divisao em K [x] e o escalonamento para sistemas deequacoes lineares (ou matrizes), veremos que uma nocao de ordem de termos e um ingredientechave de ambos (embora isto nao seja frequentemente enfatizado). Por exemplo, dividindof (x) = x5 − 3x2 + 1 por g (x) = x2 − 4x + 7 pelo metodo padrao, nos farıamos:

i) escreverıamos os termos do polinomio em ordem decrescente de grau de x

ii) no primeiro passo, o termo lıder (o termo de maior grau) em f e: x5 = x3 · x2 =x3 · (termo lıder em g). Entao, nos subtraımos x3 · g (x) de f para cancelar o termo lıder,ficando 4x4 − 7x3 − 3x2 + 1

iii) entao, nos repetıriamos o mesmo processo sobre f (x) − x3 · g (x), etc. ate obtermos umpolinomio de grau menor que 2

Para o algoritmo da divisao sobre polinomios de uma variavel, entao, nos lidamos com aordem de grau sobre monomios de uma variavel:

. . . > xm+1 > xm > . . . > x2 > x > 1 (1)

Similarmente, no algortimo de escalonamento sobre matrizes, em alguma linha dada, nostrabalhamos sistematicamente com a primeira entrada da esquerda - as entradas lıderes saoaquelas entradas nao nulas a extrema esquerda da linha. No nıvel de equacoes lineares, este eexpressado pela ordem das variaveis x1, . . . , xn como a seguir:

x1 > x2 > . . . > xn (2)

Nos escrevemos os termos nas nossas equacoes em ordem decrescente. Alem disso, numsistema na forma escalonada (onde a primeira entrada nao nula de cada linha e 1, e todas asoutras entradas na coluna contendo um lıder 1 sao zero) as equacoes sao listadas com seus

∗Orientando de Iniciacao Cientıfica: FAPEMIG. E-mail: [email protected]†Professor Orientador - E-mail: [email protected]

Page 26: Numero 09 - Outubro de 2007

termos lıderes em ordem decrescente (de fato, a definicao precisa de um sistema na formaescalonada poderia ser dada em termos desta ordem).

Da evidencia acima, podemos imaginar que uma componente muito importante de algumaextensao da divisao e escalonamento para polinomios arbitrarios em varias variaveis e uma or-dem de termos em polinomios em K [x1, . . . , xn]. Aqui, discutiremos as propriedades desejaveisque as ordens poderiam ter, e construiremos varios exemplos diferentes que satisfarao nossasnecessidades. Cada uma destas ordens sera usada em diferentes contextos.

Primeiro observamos que podemos reconstruir o monomio xα = xα11 . · · · .xαn

n a partir dan-upla de expontes α = (α1, . . . , αn) ∈ Z

n≥0. Esta observacao estabelece uma correspondencia

bijetiva entre monomios em K [x1, . . . , xn] e o conjunto Zn≥0. Alem disso, qualquer ordem >

sobre o espaco Zn≥0 nos dara uma ordem sobre monomios: se α > β, de acordo com esta ordem,

nos tambem diremos que xα > xβ.Existem varias maneiras diferentes de se definir uma ordem sobre Z

n≥0, mas exigimos sempre

que tais ordens sejam compatıveis com a estrutura algebrica de aneis polinomiais.Para comecar, como um polinomio e uma soma de monomios, nos gostarıamos ser capazes

de organizar os termos em um polinomio sem ambiguidade na ordem decrescente (ou crescente).Para fazer isto, nos temos que ser capazes de comparar todo par de monomios para estabelecersua posicao relativa. Entao exigimos que a ordem seja total, i.e. para todo par de monomiosxα e xβ, exatamente uma das tres condicoes seja verdadeira:

xα > xβ ou xα = xβ ou xα < xβ

A seguir, nos temos que levar em conta o efeito da soma e do produto sobre polinomios.Quando adicionamos polinomios podemos simplesmente reorganizar os termos na ordem apro-priada para a presente soma sem dificuldades. Produtos sao mais sutis, entretanto. Como amultiplicacao no anel polinomial distribui sobre adicao, e suficiente considerar o que acontecequando nos multiplicamos um monomio por um polinomio. Entao, nos exigimos que todas asordens de monomios tenham a seguinte propriedade adicional: se xα > xβ e xγ e um monomioqualquer, entao xα ·xγ > xβ ·xγ. Em termos dos vetores de expoentes, esta propriedade significaque se α > β na nossa ordem sobre Z

n≥0, entao, para todo γ ∈ Z

n≥0, α + γ > β + γ.

Com essas consideracoes em mente, nos fazemos a seguinte definicao.

Definicao 1 Uma ordem de monomios sobre K [x1, . . . , xn] e uma relacao > sobre Zn≥0, ou

equivalentemente, uma relacao no conjunto dos monomios xα, α ∈ Zn≥0, satisfazendo:

i) > e uma ordem total sobre Zn≥0;

ii) Se α > β e γ ∈ Zn≥0, entao α + γ > β + γ;

iii) > e uma boa ordenacao sobre Zn≥0. Isto significa que todo conjunto nao vazio de Z

n≥0 tem

um elemento mınimo em relacao a >.

O Lema a seguir nos ajudara a entender o que a condicao da boa ordenacao da parte (iii)da definicao significa.

Lema 1.1 Uma relacao de ordem > sobre Zn≥0 e uma boa ordenacao se e somente se toda

sequencia estritamente decrescente em Zn≥0

α (1) > α (2) > α (3) > . . .

eventualmente termina.

Page 27: Numero 09 - Outubro de 2007

Demonstracao:Provaremos a contrapositiva: > nao e uma boa ordenacao se e somente se existe uma

sequencia estritamente decrescente infinita em Zn≥0.

Se > nao e uma boa ordenacao, entao algum subconjunto nao vazio S ⊂ Zn≥0 nao tem um

menor elemento. Agora pegue α (1) ∈ S. Ja que α (1) nao e o menor elemento, nos podemosencontrar α (2) ∈ S tal que α (1) > α (2) em S. Entao α (2) tambem nao e o menor elemento,entao existe α (3) tal que α (2) > α (3) em S. Continuando este processo, nos temos umasequencia estritamente decrescente infinita: α (1) > α (2) > α (3) > . . ..

Por outro lado, dada uma sequencia infinita, entao {α (1) , α (2) , α (3) , . . .} e um subcon-junto nao vazio de Z

n≥0 sem o menor elemento, e entao > nao e uma boa ordenacao. �

Esse lema sera usado para mostrar que varios algoritmos podem ser terminados, por quealguns termos sao estritamente decrescentes (com respeito a uma determinada ordem fixada)em cada passo do algoritmo.

Como um exemplo simples de uma ordem de monomios, vemos que a ordem numerica usual

. . . > m + 1 > m > . . . > 3 > 2 > 1 > 0

nos elementos de Z≥0 satisfaz as tres condicoes da Definicao 1. Entao, a ordenacao grau (1)sobre monomios em K [x] e uma ordem de monomios.

Nosso primeiro exemplo de uma ordem sobre n-uplas sera a ordem lexicografica (ou ordemlex, abreviadamente).

Definicao 2 (Ordem Lexicografica) Sejam α = (α1, . . . , αn), β = (β1, . . . , βn) ∈ Zn≥0. Nos

dizemos que α >lex β se no vetor diferenca α − β ∈ Zn a primeira entrada nao nula a partir

da esquerda e positiva. Escrevemos xα >lex xβ se α >lex β.

Exemplo 1.1 i) (1, 2, 0) >lex (0, 3, 4) ja que α − β = (1,−1,−4);

ii) (3, 2, 4) >lex (3, 2, 1) ja que α − β = (0, 0, 3);

iii) As variaveis x1, . . . , xn foram ordenadas do jeito usual [veja 2] pela ordem lex:

(1, 0, . . . , 0) >lex (0, 1, 0, . . . , 0) >lex . . . >lex (0, 0, . . . , 1),

entao x1 >lex x2 >lex . . . >lex xn

Na pratica, quando trabalhamos com polinomios em duas ou tres variaveis, chamamos asvariaveis de x, y, z em vez de x1, x2, x3. Tambem assumiremos que a ordem alfabetica x >y > z sobre variaveis e usada para definir a ordem lexicografica a menos que dissermos outraexplicitamente.

A ordem Lex e analoga a ordem de palavras usadas em dicionarios (por isso o nome).

Proposicao 1.1 A ordem lex sobre Zn≥0 e uma ordem de monomios.

Demonstracao: Ver [1] �

Existem varias ordens lex, dependendo de como as variaveis sao ordenadas. Ate agora, nostemos usado a ordem lex com x1 > x2 > . . . > xn, mas dada qualquer ordem das variaveisx1, x2, . . . , xn, existe uma ordem lex correspondente. Por exemplo, se as variaveis sao x e y,entao temos uma primeira ordem lex com x > y e uma segunda com y > x. No caso geral de nvariaveis, existem n! ordens lex. No que segue, a frase “ordem lex”se referira a primeira, comx1 > x2 > . . . > xn, a menos que explicitada de outra forma.

Page 28: Numero 09 - Outubro de 2007

Observe que na ordem lex, independentemente do grau total, uma variavel e maior quequalquer monomio envolvendo variaveis menores, por exemplo, utilizando a ordem lex x > y >z, temos x >lex y5z3.

Para alguns propositos, queremos considerar o grau total dos monomios e ordenar monomiosde maior grau primeiro. Nossa primeira forma de se fazer isto e a ordem lexicografica graduada(ou ordem grlex).

Definicao 3 (Ordem Grau-lex): Seja α, β ∈ Zn≥0. Dizemos que α >grlex β se:

|α| =n∑

i=1

αi > |β| =n∑

i=1

βi ou |α| = |β| e α >lex β

Assim, podemos concluir que as ordens grlex sao dadas pelo grau total em primeiro lugar eentao “desempatamos”usando a ordem lex.

Exemplo 1.2 i) (1, 2, 3) >grlex (3, 2, 0) ja que |(1, 2, 3)| = 6 > 5 = |(3, 2, 0)|;

ii) (1, 2, 4) >grlex (1, 1, 5) ja que |(1, 2, 4)| = |(1, 1, 5)| e (1, 2, 4) >lex (1, 1, 5);

iii) As variaveis sao ordenadas de acordo com a ordem lex, isto e, x1 >grlex . . . >grlex xn.

Como no caso da ordem lex, existem n! ordens grlex sobre n variaveis, dependendo de comoas variaveis sao ordenadas.

Outra ordem, um tanto menos intuitiva, sobre monomios e a ordem lexicografica graduadareversa (ou ordem grevlex). Ainda que esta ordem de algum trabalho para que nos acostumemoscom ela, a ordem grevlex em algumas operacoes, e a mais eficiente para computacoes (oucalculos).

Definicao 4 (Ordem Grau-lex reversa): Seja α, β ∈ Zn≥0. Dizemos que α >grevlex β se:

|α| =n∑

i=1

αi > |β| =n∑

i=1

βi ou

|α| = |β| e a primeira entrada nao nula a partir da direita de α − β ∈ Zn≥0 e negativa.

Como na ordem grlex, a ordem grevlex e dada pelo grau total primeiro, porem, nesta ordem,o “desempate” se da de um jeito diferente.

Exemplo 1.3 i) (4, 7, 1) >grevlex (4, 2, 3) ja que |(4, 7, 1)| = 12 > 9 = |4, 2, 3|;

ii) (1, 5, 2) >grevlex (4, 1, 3) ja que |(1, 5, 2)| = |(4, 1, 3)| e (1, 5, 2) − (4, 1, 3) = (−3, 4,−1);

iii) A ordem grevlex da a mesma ordem sobre as variaveis que a ordem lex:

(1, 0, . . . , 0) >grevlex (0, 1, 0, . . . , 0) >grevlex . . . >grevlex (0, 0, . . . , 1),

entao x1 >grevlex x2 >grevlex . . . >grevlex xn

Igualmente as ordens lex e grlex, existem n! ordens grevlex, dependendo de como as variaveissao ordenadas.

Page 29: Numero 09 - Outubro de 2007

2 Ordenando Polinomios

Se f =∑

α aαxα e um polinomio em K [x1, . . . , xn] e escolhida uma ordem de monomios >,podemos entao ordenar os monomios de f sem ambiguidades com respeito a >.

Exemplo 2.1 Seja f = 4xy2z + 4z2 − 5x3 + 7x2z2 ∈ K [x, y, z]

a) Com respeito a ordem lex, reordenando os termos de f na ordem decrescente temos:

f = −5x3 + 7x2z2 + 4xy2z + 4z2

b) Com respeito a ordem grlex, temos:

f = 7x2z2 + 4xy2z − 5x3 + 4z2

c) Com respeito a ordem grevlex, temos:

f = 4xy2z + 7x2z2 − 5x3 + 4z2

Usaremos a seguinte terminologia:

Definicao 5 Sejam f =∑

α aαxα um polinomio nao nulo em K [x1, . . . , xn] e > uma ordemde monomios.

i) O multi-grau de f e:

multideg (f) = max(α ∈ Z

n≥0 : aα �= 0

)(o maximo e dado com respeito a >)

ii) O coeficiente lıder de f e:

LC (f) = amutideg(f) ∈ K

iii) O monomio lıder de f e:

LM (f) = xmultideg(f) (com coeficiente 1)

iv) O termo lıder de f e:

LT (f) = LC (f) · LM (f)

Exemplo 2.2 Seja f = −5x3+7x2z2+4xy2z+4z2 (como acima) e seja > a ordem lex. Entao:

multideg (f) = (3, 0, 0)

LC (f) = −5

LM (f) = x3

LT (f) = −5x3

Page 30: Numero 09 - Outubro de 2007

3 Algoritmo da Divisao em K [x1, . . . , xn]

Para estudar o problema da pertinencia de polinomios de varias variaveis no ideal, formularemosum algoritmo de divisao para polinomios em K [x1, . . . , xn] que estende o algoritmo para K [x].No caso geral, a meta e dividir f ∈ K [x1, . . . , xn] por f1, . . . , fs ∈ K [x1, . . . , xn]. Comoveremos, isto significa expressar f na forma:

f = a1f1 + . . . + asfs + r

onde os “quocientes” a1, . . . , as e o resto r estao em K [x1, . . . , xn]. Alguns cuidados seraonecessarios para caracterizar o resto e neste momento usaremos as ordens de monomios intro-duzidas.

Apos o algoritmo pronto veremos como aplica-lo ao problema da pertinencia.A ideia basica do algoritmo e a mesma que no caso de uma variavel: queremos cancelar o

termo lıder de f (com respeito a ordem de monomios fixada) multiplicando algum fi por ummonomio apropriado e subtraı-lo de f . Entao esse monomio torna-se um termo correspondenteai. Em vez de escrever o algoritmo no caso geral, primeiro trabalharemos com alguns exemplospara ver o que e envolvido.

Exemplo 3.1 Primeiro dividiremos f = xy2 + 1 por f1 = xy + 1 e f2 = y + 1 usando a ordemlex com x > y. Queremos empregar o mesmo esquema para divisao de polinomios de umavariavel, sendo que a diferenca e que existem varios divisores e quocientes.

xy2 + 1 | xy + 1; y + 1

Os termos lıderes LT (f1) = xy e LT (f2) = y ambos dividem o termo lıder LT (f) = xy2.Ja que f1 e listado primeiro, usaremos ele. Dividindo xy2 por xy, temos y e entao subtraimosyf1 de f .

xy2 + 1 |xy + 1; y + 1xy2 + y y ;−y + 1

Agora repetimos o mesmo processo sobre −y+1. Dessa vez usaremos f2 ja que LT (f1) = xynao divide LT (−y + 1) = −y. Assim obtemos:

xy2 + 1 |xy + 1; y + 1xy2 + y y ; (−1)−y + 1−y − 1

2

Ja que LT (f1) e LT (f2) nao dividem 2, o resto e r = 2 e concluimos a divisao. Entao,temos escrito f = xy2 + 1 na forma:

xy2 + 1 = y (xy + 1) + (−1) (y + 1) + 2

Exemplo 3.2 Neste exemplo, encontraremos uma sutileza inesperada que pode ocorrer quandoestamos trabalhando com polinomios de mais de uma variavel. Vamos dividir f = x2y+xy2+y2

por f1 = xy − 1 e f2 = y2 − 1. Como no exemplo anterior, usaremos a ordem lex com x > y.

Page 31: Numero 09 - Outubro de 2007

Os dois primeiros passos do algoritmo sao usuais, dando assim a seguinte divisao parcialmentecompletada.

x2y + xy2 + y2 |xy − 1; y2 − 1x2y − x x + y ;

xy2 + x + y2

xy2 − y

x + y2 + y

Note que nem LT (f1) = xy nem LT (f2) = y2 dividem LT (x + y2 + y) = x. Entretanto,x + y2 + y nao e o resto ja que LT (f2) divide y2. Entao, se movemos x para o resto, podemoscontinuar dividindo.

Observacao 3.1 Este e um problema que nunca acontece no caso de uma variavel: uma vezque o termo lıder do divisor nao divide mais o termo lıder que esta abaixo do radical, o algoritmotermina.

Para executar essa ideia, criamos uma coluna de resto r, do lado esquerdo do radical, ondecolocamos os termos que pertencem ao resto. E entao continuamos dividindo ate o dividendointermediario seja zero (chamamos o polinomio debaixo do radical de dividendo intermediario).Aqui esta o proximo passo, onde movemos x para a coluna do resto (como indicado pela seta):

r x2y + xy2 + y2 |xy − 1; y2 − 1x2y − x x + y ;

xy2 + x + y2

xy2 − y

x + y2 + y

x ←− y2 + y

Agora continuamos dividindo. Se podemos dividir pelo LT (f1) ou LT (f2), procedemoscomo usualmente, e se nenhum divide, movemos o termo lıder do dividendo intermediario paraa coluna do resto. Aqui esta o resto da divisao:

r x2y + xy2 + y2 |xy − 1; y2 − 1x2y − x x + y ; 1

xy2 + x + y2

xy2 − y

x + y2 + y

x ←− y2 + yy2 − 1y + 1

x + y ←− 1x + y + 1 ←− 0

Entao, o resto e x + y + 1, e obtemos:

x2y + xy2 + y2 = (x + y) (xy − 1) + 1(y2 − 1

)+ x + y + 1 (3)

Observe que o resto e a soma de monomios, nenhum dos quais e divisıvel pelos termos lıderesLT (f1) ou LT (f2).

Page 32: Numero 09 - Outubro de 2007

O exemplo acima e uma ilustracao bastante completa de como o algoritmo da divisao tra-balha. Este exemplo nos mostra tambem qual propriedade nos queremos que o resto tenha:nenhum dos termos pode ser divisıvel pelos termos lıderes dos polinomios que estao dividindo.

Podemos agora enunciar a forma geral do algortimo da divisao.

Teorema 3.1 (Algortimo da Divisao em K [x1, . . . , xn]): Fixe uma ordem de monomios > so-bre Z

n≥0 e seja F = (f1, . . . , fs) uma s-upla de polinomios ordenadas em K [x1, . . . , xn]. Entao

todo f ∈ K [x1, . . . , xn] pode ser escrito como:

f = a1f1 + . . . + asfs + r

onde ai,r ∈ K [x1, . . . , xn], e qualquer r = 0 ou r e uma combinacao linear, com coeficientesem K, de monomios, nenhum dos quais e divisıvel por nenhum dos LT (f1) , . . . , LT (fs). Noschamaremos r um resto de f na divisao por F . Alem disso, se aifi �= 0, entao temos:

multideg (f) ≥ multideg (aifi)

Demonstracao: Provemos a existencia de a1, . . . , as e r dando um algoritmo para aconstrucao deles e mostrando que ele opera corretamente sobre qualquer entrada dada. Vejamosa seguinte generalizacao:

Input : f1, . . . , fs, fOutput : a1, . . . , as, ra1 := 0, . . . , as := 0, r := 0p := fWhile p �= 0 Do

i := 1divisionocurred := falseWhile i ≤ s e divisionocurred = false Do

If LT (fi) divides LT (p) Thenai := ai + LT (p) / LT (fi)p := p − (LT (p) /LT (fi)) fi

divisionocurred = trueElse

i := i + 1If divisionocurred = false Then

r := r + LT (p)p := p − LT (p)

Relacionando este algoritmo com o exemplo anterior observamos que a variavel p representao dividendo intermediario para cada estagio, a variavel r representa a coluna do lado esquerdo, eas variaveis a1, . . . , as sao os quocientes. Finalmente, a variavel booleana “divisionocurred”nosfala quando algum LT (fi) divide o termo lıder do dividendo intermediario. Observe que cadavez que vamos atraves do laco principal While . . . Do, precisamente uma das duas coisas acon-tece:

i) (Passo da Divisao): Se algum LT (fi) divide LT (p), entao o algoritmo procede como ocaso de uma variavel;

ii) (Passo do Resto): Se nenhum LT (fi) divide LT (p), entao o algoritmo adiciona LT (p)para o resto.

Page 33: Numero 09 - Outubro de 2007

Estes passos correspondem exatamente ao que fizemos no Exemplo 3.2. Para provar que oalgoritmo funciona, primeiro mostraremos que:

f = a1f1 + . . . + asfs + r (4)

e valido para todos os estagios. Isto e claramente verdadeiro para os valores iniciais de a1, . . . ,as, p e r. Agora suponha que (4) e valido para um passo do algoritmo. Se o proximo passo forum Passo da Divisao, entao algum LT (fi) divide LT (p) e a igualdade

aifi + p = (ai + LT (p) / LT (fi)) fi + (p − (LT (p) / LT (fi)) fi)

mostra que aifi + p e inalterado. E como todas as outras variaveis nao sao afetadas, temos que(4) e verdadeira. Por outro lado, se o proximo passo for o Passo do Resto, entao p e r seraomudados, mas a soma p + r e inalterada ja que

p + r = (p − LT (p)) + (r + LT (p))

e como antes, a igualdade (4) e ainda preservada.A seguir, observe que o algoritmo para quando p = 0. Nesta situacao, (4) torna-se:

f = a1f1 + . . . + asfs

Ja que os termos sao adicionados a r somente quando eles nao sao divisıveis por nenhum dosLT (fi), isso segue que a1, . . . , as e r tem a propriedade desejada quando o algoritmo termina.

Finalmente, precisamos mostrar que o algoritmo eventualmente termina. A observacaochave e que cada vez que redefinimos a variavel p, qualquer um dos seus multi-graus diminui(relativo a nossa ordem de termos) ou se torna 0. Para ver isso, primeiro suponha que duranteum Passo da Divisao, p e redefinida por:

p′ = p − LT (p)

LT (fi)fi

Assim temos que:

LT

(LT (p)

LT (fi)fi

)=

LT (p)

LT (fi)LT (fi) = LT (p)

para que p e LT (p) / LT (fi) fi tenham o mesmo termo lıder. Entao, a diferenca deles, p′,tem o multi-grau estritamente menor quando p′ �= 0. A seguir, suponha que durante um Passodo Resto, p e redefinido por:

p′ = p − LT (p)

Aqui, e obvio que multideg (p′) < multideg (p) quando p′ �= 0. Entao, em qualquer umdos casos, o multi-grau cai. Se o algoritmo nunca terminasse, entao terıamos uma sequenciadecrescente infinita de multi-graus. A propriedade da boa ordenacao de >, como mostrado noLema 1.1, mostra que isto nao pode ocorrer. Entao p = 0 tem que ocorrer eventualmente, paraque o algoritmo termine depois de varios passos finalmente.

Resta estudar a relacao entre multideg (f) e multideg (aifi). Todo termo em ai e da formaLT (p) / LT (fi) para algum valor da variavel p. O algoritmo comeca com p = f e acabamosde provar que o multi-grau de p decresce. Isto mostra que LT (p) ≤ LT (f), e entao temos que:

LT (p) ≤ LT (f) =⇒ LT (p)

LT (fi)≤ LT (f)

LT (fi)=⇒ LT (p)

LT (fi)LT (fi) ≤ LT (f)

LT (fi)LT (fi) =⇒

=⇒ aiLT (fi) ≤ LT (f) =⇒ multideg (aifi) ≤ multideg (f)

Page 34: Numero 09 - Outubro de 2007

quando aifi �= 0. Isto prova o Teorema. �

A algebra por detras do algoritmo da divisao e muito simples (nao existe nada alem daalgebra que foi feita no colegial), o que surpreende e que esta forma de algoritmo foi isolada eexplorada somente nos ultimos 30 anos.

Infelizmente, esse algoritmo nao possui as mesmas propriedades agradaveis da versao deuma variavel.

A primeira propriedade importante do algoritmo da divisao em K [x] e que o resto nao eunicamente determinado. Para ver isto considere o seguinte exemplo:

Exemplo 3.3 Vamos dividir f = x2y + xy2 + y2 por f1 = y2 − 1 e f2 = xy − 1. Usaremos aordem lex com x > y. Este e o mesmo exemplo 3.2, exceto que mudamos a ordem dos divisores.

r x2y + xy2 + y2 |y2 − 1; xy − 1xy2 − x x + 1 ; x

x2y + x + y2

y2 − 1

x2y + x + 1x2y − x2x + 1

2x ←− 12x + 1 ←− 0

Isto mostra que:

x2y + xy2 + y2 = (x + 1)(y2 − 1

)+ x (xy − 1) + 2x + 1 (5)

Se compararmos esta equacao com a equacao (3), veremos que o resto e diferente do quevimos no Exemplo 3.2.

Isto mostra que o resto nao e unico, ou seja, para cada ordem F = (f1, . . . , fs), existe umresto na divisao de f por F .

Uma caracterıstica agradavel do Algortimo da Divisao em K [x] e o jeito dele resolver oproblema da pertinencia de polinomio de uma variavel no ideal. Nos temos alguma coisasimilar para varias variaveis? Uma consequencia e um simples corolario do Teorema 3.1: seapos a divisao de f por F = (f1, . . . , fs) obtermos um resto r = 0, entao f = a1f1 + . . . + asfs,de forma que f ∈ 〈f1, . . . , fs〉. Entao r = 0 e uma condicao suficiente para o problema dapertinencia. Contudo, como o seguinte exemplo mostra, r = 0 nao e uma condicao necessariapara estar no ideal.

Exemplo 3.4 Seja f1 = xy+1, f2 = y2−1 ∈ K [x, y] com a ordem lex. Dividindo f = xy2−xpor F = (f1, f2), o resultado e:

xy2 − x = y (xy + 1) + 0(y2 − 1

)+ (−x − y)

Com F = (f2, f1), entretanto, temos:

xy2 − x = x(y2 − 1

)+ 0 (xy + 1) + 0

O segundo calculo mostra que f ∈ 〈f1, f2〉. Entao o primeiro calculo mostra que ainda quef ∈ 〈f1, f2〉, e ainda possıvel obter um resto nao nulo na divisao por F = (f1, f2) .

Entao concluımos que o Algoritmo da Divisao dado e uma generalizacao imperfeita doequivalente de uma variavel. E para resolver essa imperfeicao para o problema da pertinencia,serao necessarias as Bases de Hilbert.

Page 35: Numero 09 - Outubro de 2007

4 O Teorema das Bases de Hilbert e as Bases

de Groebner

Definicao 6 Um ideal I ⊂ K [x1, . . . , xn] e um ideal de monomios se existe um conjuntoS ⊂ Z

n≥0 (possivelmente infinito) tal que I consiste de todos os polinomios que sao somas finitas

da forma Σα∈Ahαxα, onde hα ∈ K [x1, . . . , xn]. Neste caso, escrevemos I = 〈xα : α ∈ A〉

Lema 4.1 Seja I = 〈xα : α ∈ A〉 um ideal de monomios. Entao um monomio xβ pertence a Ise e somente se xβ e divisıvel por xα para algum α ∈ A.

Observe que xβ e divisıvel por xα exatamente quando xβ = xα · xγ para algum γ ∈ Zn≥0.

Lema 4.2 Seja I um ideal de monomios, e seja f ∈ K [x1, . . . , xn]. Entao as seguintescondicoes sao equivalentes:

i) f ∈ I

ii) Todo termo de f esta em I

iii) f e uma K-combinacao linear de monomios em I

Corolario 4.1 Dois ideais de monomios sao os mesmos se, e somente se, eles contem osmesmos monomios.

Teorema 4.1 (Lema de Dickson): Um ideal de monomios I = 〈xα : α ∈ A〉 ⊂ K [x1, . . . , xn]pode ser escrito sobre a forma I = 〈xα(1), . . . , xα(s)〉, onde α(1), . . . , α(s) ∈ A. Em particular,I tem uma base finita.

O Teorema 4.1 soluciona a descricao do ideal para ideais de monomios, por ele dizer quequalquer ideal tem uma base finita. Isto, por sua vez, nos permite resolver o problema dapertinencia para ideais de monomios. A saber, se I = 〈xα(1), . . . , xα(s)〉, entao podemos facil-mente mostrar que um polinomio f esta em I se, e somente se, o resto de f na divisao porxα(1), . . . , xα(s) e zero.

Definicao 7 Seja I ⊂ K [x1, . . . , xn] um ideal diferente de 0.

i) Denotamos por LT (I) o conjunto dos termos lıderes dos elementos de I. Entao,

LT (I) = cxα : existe f ∈ I com LT (f) = cxα

ii) Denotamos por 〈LT (I)〉 o ideal gerado pelos elementos de LT (I)

Ja vimos que os termos lıderes tem um importante papel no algoritmo da divisao. Comisso, surgi uma sutileza que deve ser mencionada: se damos um conjunto gerador finito para I,digamos I = 〈f1, . . . , fs〉, entao 〈LT (f1), . . . , LT (fs)〉 e 〈LT (I)〉 podem ser ideais diferentes. Everdade que 〈LT (fi)〉 ∈ LT (I) ⊂ 〈LT (I)〉 pela definicao, que implica 〈LT (f1), . . . , LT (fs)〉 ⊂〈LT (I)〉. Entretanto, 〈LT (I)〉 pode ser estritamente maior. Para ver isto, considere o exemploa seguir.

Page 36: Numero 09 - Outubro de 2007

Exemplo 4.1 Seja I = 〈f1, f2〉, onde f1 = x3 −2xy e f2 = x2y−2y2 +x e a ordem grlex sobremonomios em K [x, y]. Entao,

x · (x2y − 2y2 + x) − y(x3 − 2xy) = x2

e x2 ∈ I. Logo, x2 = LT (x2) ∈ 〈LT (I)〉. Entretanto, x2 ∈ I nao e divisıvel por LT (f1) = x3

ou LT (f2) = x2y, logo, x2 nao pertence 〈LT (f1), LT (f2)〉 pelo Lemma 4.1.

Agora mostraremos que 〈LT (I)〉 e um ideal monomial e isto nos permitira aplicar os resul-tados anteriores. Em particular, seguira que 〈LT (I)〉 e gerado por um numero finito de termoslıderes.

Proposicao 4.1 Seja I ⊂ K [x1, . . . , xn] um ideal.

i) 〈LT (I)〉 e um ideal monomial

ii) Existem g1, . . . , gt ∈ I tal que 〈LT (I)〉 = 〈LT (g1), . . . , LT (gt)〉

Demonstracao:

i) O monomio lıder LM(g) dos elementos g ∈ I − {0} gera o ideal monomial〈LM(g) : g ∈ I − {0}〉. Ja que LM(g) e LT (g) diferem apenas por uma constantenao nula, pelo Corolario 4.1 temos que 〈LM(g) : g ∈ I−{0}〉 = 〈LT (I)〉. Entao, 〈LT (I)〉e um ideal monomial.

ii) Ja que 〈LT (I)〉 e gerado pelos monomios LM(g) para g ∈ I−{0}, o Lema de Dickson nosdiz que 〈LT (I)〉 = 〈LM(g1), . . . , LM(gt)〉 para infinitos g1, . . . , gt ∈ I. Ja que LM(gi)difere de LT (gi)apenas por uma constante nao nula, novamente pelo Corolario 4.1, temosque 〈LT (I)〉 = 〈LT (g1), . . . , LT (gt)〉 e isto completa a prova.

Agora, podemos usar a Proposicao 4.1 e o Algoritmo da Divisao para provar a existencia deum conjunto gerador finito de todo ideal de polinomios e, entao dando uma resposta afirmativapara o problema da descricao. Seja I ⊂ K [x1, . . . , xn] um ideal qualquer e considere o idealassociado 〈LT (I)〉 como na Definicao 7. Como sempre, selecionamos uma ordem de monomioparticular para usar no algoritmo da divisao e na computacao dos termos lıderes.

Teorema 4.2 (Teorema da Base de Hilbert): Todo ideal I ⊂ K [x1, . . . , xn] tem um conjuntogerador finito. Isto, e, I = 〈g1, . . . , gt〉 para algum g1, . . . , gt ∈ I

Demonstracao:Se I = {0}, tomamos nosso conjunto gerador como {0}, que certamente e finito.Se I contem algum polinomio nao nulo, entao um conjunto gerador g1, . . . , gt para I pode

ser construido como a seguir. Pela Proposicao 4.1, existem g1, . . . , gt ∈ I tal que 〈LT (I)〉 =〈LT (g1), . . . , LT (gt)〉. Afirmamos que I = 〈g1, . . . , gt〉.

E claro que I = 〈g1, . . . , gt〉 ⊂ I, ja que cada gi ∈ I. Por outro lado, seja f ∈ I um polinomioqualquer. Se aplicarmos o algoritmo da divisao para dividir f por 〈g1, . . . , gt〉 entao chegamosnuma expressao da forma:

f = a1g1 + . . . + atgt + r

Page 37: Numero 09 - Outubro de 2007

onde nenhum termo de r e divisıvel por nenhum dos LT (g1), . . . , LT (gt).Afirmamos que r = 0. Para ver isto, observe que:

r = f − a1g1 + . . . + atgt ∈ I

Se r �= 0, entao LT (r) ∈ 〈LT (I)〉 = 〈LT (g1), . . . , LT (gt)〉, e pelo Lema 4.1, segue que LT (r)deve ser divisıvel por algum LT (gi). Isto contradiz o fato dele ser o resto e, consequentemente,r tem que ser zero. Entao,

f = a1g1 + . . . + atgt + 0 ∈ 〈LT (g1), . . . , LT (gt)〉que mostra que I ⊂ 〈g1, . . . , gt〉 e, portanto, I = 〈g1, . . . , gt〉

Alem de responder a questao da descricao do ideal, a base {g1, . . . , gt} usada na prova doTeorema 4.2 tem a propriedade especial 〈LT (I)〉 = 〈LT (g1), . . . , LT (gt)〉. Como nem todas asbases possuem essa propridade, como vimos no exemplo 3.2, as essas bases daremos o seguintenome.

Definicao 8 Fixe uma ordem de monomios. Um subconjunto finito G = {g1, . . . , gt} de umideal I e dito ser uma base de Groebner (ou base padrao) se

〈LT (g1), . . . , LT (gt)〉 = 〈LT (I)〉

Equivalentemente, mas mais informalmente, um conjunto {g1, . . . , gt} ⊂ I e uma base deGroebner de I se, e somente se, o termo lıder de algum elemento de I e divisıvel por um dosLT (gi). A prova do Teorema 4.2 tambem estabelece o seguinte resultado.

Corolario 4.2 Fixe uma ordem de monomios. Entao todo ideal I ⊂ K [x1, . . . , xn] diferentede {0} tem uma base de Groebner. Alem disso, qualquer base de Groebner para um ideal I euma base de I.

Definicao 9 Seja K um corpo e sejam f1, . . . , fs polinomios em K [x1, . . . , xn]. Denotamospor variedade afim definida por f1, . . . , fs o seguinte conjunto:

V (f1, . . . , fs) = {(a1, . . . , an) ∈ Kn : fi(a1, . . . , an) = 0, para todo 1 ≤ i ≤ s}

Exemplo 4.2 Seja J = 〈g1, g2〉 = 〈x + z, y − z〉. Temos que g1 e g2 formam uma base deGroebner usando a ordem lex em R[x, y, z].

Vamos mostrar que a forma inicial de todo elemento nao nulo de J implica no ideal〈LT (g1), LT (g1)〉 = 〈x, y〉. Pelo Lema 4.1, isto e equivalente mostrando que o termo lıder dequalquer elemento nao nulo de J e divisıvel por x ou y.

Para provar isto, considere algum f = Ag1 + Bg2 ∈ J . Suponha, por absurdo, que f e naonulo e LT (f) nao e divisıvel por x e nem por y. Entao pela definicao de ordem lex, f sera umpolinomio em z somente. Entretanto, f se anula no subespaco linear L = V (x + z, y − z) ⊂ R

3

ja que f ∈ J .Observe que (x, y, z) = (−t, t, t), para algum numero real t, ja que pela definicao de V ,

x + z = 0 ⇒ x = −z e y − z = 0 ⇒ y = z e assim fazendo z = t, temos (−t, t, t).O unico polinomio em z que anula nesses infinitos pontos e o polinomio nulo, o que e uma

contradicao. (De fato, pois caso contrario, terıamos que o polinomio possuindo grau igual a dpossuiria infinitas raizes, o que e um absurdo.)

Assim segue que 〈g1, g2〉 e uma base de Groebner.

Page 38: Numero 09 - Outubro de 2007

Referencias

[1] Cox, D. and Little, J. and O’Shea, D., Ideals, varieties, and algorithms, Springer, segundaedicao, 1991.

[2] Kreuzer, M. and Robbiano, L , Computational Commutative Algebra 1, Springer, 2000.

[3] CoCoA: a system for doing Computations in Commutative Algebra, disponıvel emhttp://cocoa.dima.unige.it

Page 39: Numero 09 - Outubro de 2007

Evolução do Número de Alunos Reprovados nas Disciplinas do Curso De Matemática Da UFU

Flávia Borges Arantes1, Ródney Silva Abreu2, Heyder Diniz Silva1,Rogério de Melo Costa Pinto1

Universidade Federal de Uberlândia Faculdade de Matemática1

Faculdade de Engenharia Elétrica2

Av. João Naves de Ávila, s/n Santa Mônica 38408-100 Uberlândia – MG – Brasil

Resumo:No presente trabalho, utilizou-se a estatística descritiva com o objetivo de

identificar se as reprovações e as evasões no Curso de Matemática da UFU estão

relacionadas com as Diretrizes Curriculares do MEC, com o Currículo/FAMAT ou com

a Grade Curricular/FAMAT. Observou-se que ocorre grande variação nos percentuais

de reprovação entre as diversas disciplinas, sendo que no início do curso os percentuais

de reprovação são mais elevados do que no final de curso. Nas disciplinas de início de

curso há uma predominância de reprovações por nota e por desistência, enquanto que,

nas de final de curso predominam as reprovações por falta.

Palavras-chave: Parâmetros estatísticos; Analise descritiva; Taxas de repetência; Conteúdos curriculares.

Introdução:Os sistemas de avaliações de curso da UFU apresentam algumas características:

Para cada disciplina são distribuídos 100(cem) pontos, em números inteiros. Para ser

aprovado, o aluno deve alcançar o mínimo de 60 (sessenta) pontos na soma das notas e

75% (setenta e cinco por cento) de freqüência às aulas e outras atividades curriculares

dadas. O plano de avaliação é parte integrante do Plano de Ensino e deve ser

apresentado pelo professor ao Colegiado de Curso, para aprovação, após a discussão

com sua turma, até 30(trinta) dias após o início do semestre ou ano letivo.

Page 40: Numero 09 - Outubro de 2007

A critério do Colegiado do Curso, ao aluno dos cursos de regime semestral de

matrícula por disciplina que atingir o mínimo de 45 e o máximo de 59 pontos de

aproveitamento, e no mínimo de 75% de freqüência, em uma determinada disciplina,

serão facultadas as matrículas nas disciplinas subseqüentes que tenha dependência de

pré-requisito. O Sistema de avaliação do Curso de Matemática da UFU segue o regime

semestral de matrícula por disciplina.

A Matemática é uma linguagem, uma ciência e uma arte. Abstraindo e

simplificando a partir dos dados dos sentidos, a Matemática põe o mundo da ciência e

da vida cotidiana em foco, para que seja acessível à compreensão humana, tornando

possível a descrição racional de experiências. A Estatística coleta organiza e analisa os

dados do fenômeno em questão, no qual o profissional da Estatística orienta e conduz

todo o levantamento dos dados referentes ao objeto de análise. No presente trabalho a

Estatística é de importância fundamental na apresentação da análise descritiva da

evolução dos alunos do Curso de Matemática da UFU de forma que se possam avaliar

as falhas curriculares e apresentar sugestões de enfrentamento da problemática

reprovação.

Assim, o objetivo deste trabalho foi identificar se as reprovações e as evasões no

Curso de Matemática da UFU estão relacionadas com as Diretrizes Curriculares do

MEC, com o Currículo/FAMAT ou com a Grade Curricular/FAMAT, com uma visão

estatística. Criar um modelo de analise de forma que se possam avaliar suas falhas

curriculares. Apresentar sugestões de enfrentamento da problemática reprovação e

evasão.

Material e Métodos

Foram utilizados os dados referentes ao número de alunos reprovados por nota,

falta e com RM em cada disciplina do curso de matemática do primeiro semestre de

2000 ao segundo semestre de 2004. Tais dados foram obtidos junto a Divisão de

Registro e Acompanhamento Acadêmico da UFU – DIRAC. As disciplinas Cálculo I

(MLI 11), Geometria Analítica (MLI 08), Cálculo3 (MLI 16), Álgebra Linear I (MLI

12) e Equações Diferenciais Ordinárias (DCE 10) não foram analisadas, porque, o

código destas disciplinas são os mesmos do curso Física, não podendo separar os alunos

da física com os da Matemática.

Page 41: Numero 09 - Outubro de 2007

Foi realizada a estatística descritiva dos dados. Os dados foram apresentados na

forma gráfica para facilitar sua visualização e análises.

Resultados e Discussão

Na Figura 1(a), encontram-se apresentados os resultados das reprovações dos

alunos na disciplina Fundamentos de Matemática Elementar 1 (MLI31) (FME1).

Observa-se nessa Figura que as reprovações variaram de 20 (01-02) a 65% (02-04). E

também que o número de reprovações apresentou uma tendência de aumento. Quanto ao

tipo de reprovação na Figura 1(b) verifica-se que em geral, a maior parte das

reprovações é por nota, exceção feita às turmas 01-01, 02-01 e 01-02 em que houve um

predomínio de reprovações por falta. Em todas as turmas avaliadas as reprovações com

RM foram bem inferiores as por nota. Na Figura 1(d) observa-se uma clara tendência de

aumento no número de alunos matriculados nesta disciplina, que esta associada a um

aumento no número de reprovações (Figura 1(c)).

Na Figura 2(a), encontram-se apresentados os resultados das reprovações dos

alunos em Fundamentos de Matemática Elementar 2 (MLI32) (FME2). Observa-se

nessa Figura que as reprovações variaram de 4 (01-02) a 75% (01-03), apresentando

uma amplitude de variação maior que FM1. Cabe salientar que os dois maiores índices

de reprovação nesta disciplina ocorreram em dois semestres consecutivos 66% (02-02) e

75 % (01-03) depois voltando para média. Quanto ao tipo da reprovação na Figura 2 (b)

verifica-se que as reprovações ocorrem predominantemente por desistência e por nota,

havendo um predomínio de desistência (faltas) o que não ocorreu com FM1. Apenas em

(01-02) onde houve uma reprovação maior por nota. Na Figura 2(c) e 2(d) verifica-se

que como vem aumentando o número de alunos também vem aumentando o número de

reprovações. E como (01-02) teve uma reprovação mínima então no semestre

consecutivo (02-02) o número de alunos matriculados foi o menor.

Page 42: Numero 09 - Outubro de 2007

(a) (b) REPROVAÇÕES FME1

0

10

20

30

40

50

60

70

01.2000

02.20

00

01.200

1

02.2001

01.2002

02.2002

01.20

03

02.200

3

01.2004

02.2004

TURMA

% D

E A

LUN

OS

TIPOS FME1

0%10%20%30%40%50%60%70%80%90%

100%

01.2000

02.20

00

01.2001

02.2001

01.2002

02.2002

01.20

03

02.2003

01.2004

02.2004

TURMA

% D

AS

REP

RO

VAÇ

ÕE

NOTA RM FREQÜÊNCIA

(c) (d) FME1

y = 1,5883x - 74,436R2 = 0,5213

0

10

20

30

40

50

60

35 45 55 65 75 85TOTAL DE MATRICULADOS

TOTA

L D

E R

EPR

OVA

DO

S

TOTAL ALUNOS

35404550556065707580

01.2000

02.2000

01.2001

02.2001

01.2002

02.2002

01.2003

02.2003

01.2004

02.2004

Figura 1. Disciplina FME1. Total de alunos reprovados por semestre (a), Tipo de reprovação (b); relação número de reprovados x número de matriculados (c) e total de alunos matriculados (d).

Na Figura 3(a), encontram-se apresentados os resultados das reprovações dos

alunos na disciplina Geometria Plana e Desenho Geométrico (MLI33). Observa-se

nessa Figura que as reprovações variaram de 23 (01-03) a 73% (02-04). E cabe salientar

que entre os semestres 02-01 e 01-03 foram bem menores os índices de reprovações. E

também que os índices de reprovações vêm aumentando a partir de 02-03. Quanto ao

tipo da reprovação na Figura 3(b) verifica-se que as reprovações ocorrem

predominantemente por desistência, e é interessante observar os 23% e os 73% da

Figura 3 (a) foram reprovados por desistência. As reprovações por RM foram mínimas

em relação aos outros tipos de reprovações. E em 02-03 as reprovações ocorreram 70%

por nota. Na Figura 3(d) verifica-se que tinha um número muito grande de alunos

matriculados até (02-01), como observamos na Figura 3 (a) o índice de reprovação foi

Page 43: Numero 09 - Outubro de 2007

5

mínimo neste e nos semestres consecutivos, então diminuiu o nº de alunos matriculados

e depois (02-03) vem aumentando este número, e na Figura 3(c) o número de

reprovações também aumenta.

(a) (b)

REPROVAÇÕES FME2

0

10

20

30

40

50

60

70

80

01.20

00

02.20

00

01.20

01

02.20

01

01.20

02

02.20

02

01.20

03

02.20

03

01.20

04

02.20

04

TURMA

% D

E A

LUN

OS

TIPOS FME2

0%10%20%30%40%50%60%70%80%90%

100%

01.2000

02.2000

01.2001

02.2001

01.2002

02.2002

01.2003

02.2003

01.2004

02.2004

TURMA%

DA

S R

EPR

OVA

ÇÕ

E

NOTA RM FREQÜÊNCIA

(c) (d)

FME2

y = 0,2495x + 17,834R2 = 0,0948

0

10

20

30

40

50

35 45 55 65 75 85TOTAL DE MATRICULADOS

TOTA

L D

E R

EPR

OVA

DO

S

TOTAL ALUNOS

3540455055606570758085

01.200

0

02.20

00

01.20

01

02.20

01

01.20

02

02.20

02

01.20

03

02.20

03

01.20

04

02.20

04

Figura 2. Disciplina FME2. Total de alunos reprovados por semestre (a), Tipo de

reprovação (b); relação número de reprovados x número de matriculados (c) e total de

alunos matriculados (d).

Na Figura 4(a), encontram-se apresentados os resultados das reprovações dos

alunos na disciplina Cálculo Diferencial Integral 2 (MLI64) (3º período). Observa-se

nessa Figura que as reprovações variaram de 35 (01-01) a 75% (02-04). E também que o

número de reprovações não tem uma amplitude de variação, exceto nos semestres 01-

00,02-00,01-02 e 02-04. O índice de reprovação médio é bem menor do que Geometria

Page 44: Numero 09 - Outubro de 2007

Plana e Desenho Geométrico (2º período). Quanto ao tipo da reprovação na Figura 4(b)

verifica-se que as reprovações ocorrem tanto por desistência quanto por nota, exceto, no

semestre (02-04) que houve uma reprovação por desistência maior. E a reprovação por

RM como nas disciplinas anteriores é muito pequena.

(a) (b)

REPROVAÇÕES GEOM. PLANA E DES. GEOMÉTRICO

01020304050607080

01.20

00

02.20

00

01.20

01

02.20

01

01.20

02

02.20

02

01.20

03

02.20

03

01.20

04

02.20

04

TURMA

% D

E A

LUN

OS

TIPOS GEOM. PLANA E DES. GEOMÉTRICO

0%10%20%30%40%50%60%70%80%90%

100%

01.2000

02.2000

01.2001

02.2001

01.2002

02.2002

01.2003

02.2003

01.2004

02.2004

TURMA

% D

AS

REP

RO

VAÇ

ÕE

NOTA RM FREQÜÊNCIA

(c) (d)

Geom. Plana e Desenho Geométrico

y = 0,6728x - 8,7057R2 = 0,5207

0

10

20

30

40

50

0 20 40 60 80 100TOTAL DE MATRICULADOS

TOTA

L D

E R

EPR

OVA

DO

S

TOTAL ALUNOS

0102030405060708090

01.200

0

02.20

00

01.20

01

02.20

01

01.2002

02.20

02

01.20

03

02.20

03

01.200

4

02.20

04

Figura 3. Disciplina GPDG. Total de alunos reprovados por semestre (a), Tipo de

reprovação (b); relação número de reprovados x número de matriculados (c) e total de

alunos matriculados (d).

Na Figura 4(c) e 4(d) verifica-se que não tem uma correlação entre os matriculados e os

reprovados. E o número de alunos matriculados vem diminuindo.

Page 45: Numero 09 - Outubro de 2007

(a) (b)

REPROVAÇÕES CÁLCULO II

0

10

20

30

40

50

60

70

80

01.20

00

02.20

00

01.20

01

02.20

01

01.20

02

02.20

02

01.20

03

02.20

03

01.20

04

02.20

04

TURMA

% D

E A

LUN

OS

TIPOS CÁLCULO II

0%

20%

40%

60%

80%

100%

01.2000

02.2000

01.2001

02.2001

01.2002

02.2002

01.2003

02.2003

01.2004

02.2004

TURMA

% D

AS

REP

RO

VAÇ

ÕE

NOTA RM FREQÜÊNCIA

(c) (d)

CÁLCULO II

y = 0,2004x + 14,343R2 = 0,1241

05

101520253035

0 20 40 60 80TOTAL DE MATRICULADOS

TOTA

L D

E R

EPR

OVA

DO

S

TOTAL ALUNOS

010203040506070

01.20

00

02.200

0

01.20

01

02.20

01

01.20

02

02.20

02

01.20

03

02.20

03

01.20

04

02.20

04

Figura 4. Disciplina Cálculo II. Total de alunos reprovados por semestre (a), Tipo de reprovação (b); relação número de reprovados x número de matriculados (c) e total de alunos matriculados (d).

Na Figura 5(a), encontram-se apresentados os resultados das reprovações dos

alunos na disciplina Introdução da Teoria dos Números (MLI35). Observa-se nessa

Figura que as reprovações variaram de 25 (02-03) a 74% (01-00). E também que o

número de reprovações não tem uma amplitude de variação como Cálculo II, com

exceção no semestre (01-04 e 02-04). Quanto ao tipo da reprovação na Figura 5(b)

verifica-se que as reprovações ocorrem tanto por desistência quanto por nota, com

exceções, nos semestres (01-04 e 02-04) que foram predominantes por desistência. Na

Figura 5(c) verifica-se que o número de alunos matriculados influencia pouco em

relação dos alunos reprovados. E na Figura 5(d) o número de alunos matriculados a

partir de 01-01 vem se mantendo em média de 50 alunos.

Page 46: Numero 09 - Outubro de 2007

(a) (b)

REPROVAÇÕES INT. TEORIA DOS NÚMEROS

01020304050607080

01.2000

02.2000

01.20

01

02.20

01

01.20

02

02.2002

01.2003

02.20

03

01.20

04

02.20

04

TURMA

% D

E A

LUN

OS

TIPOS INT.TEORIA DOS NÚMEROS

0%10%20%30%40%50%60%70%80%90%

100%

01.2000

02.20

00

01.200

1

02.2001

01.2002

02.20

02

01.200

3

02.2003

01.2004

02.2004

TURMA

% D

AS

REP

RO

VAÇ

ÕE

NOTA RM FREQÜÊNCIA

(c) (d)

INT. TEORIA DOS NÚMEROS

y = 0,5885x - 5,0834R2 = 0,3336

0

10

20

30

40

50

60

0 20 40 60 80TOTAL DE MATRICULADOS

TOTA

L D

E R

EPR

OVA

DO

S

TOTAL ALUNOS

01020304050607080

01.2000

02.2000

01.2001

02.2001

01.2002

02.2002

01.2003

02.2003

01.2004

02.2004

Figura 5. Disciplina Int. Teoria dos Números. Total de alunos reprovados por semestre (a), Tipo de reprovação (b); relação número de reprovados x número de matriculados (c) e total de alunos matriculados (d).

Na Figura 6(a), encontram-se apresentados os resultados das reprovações dos

alunos na disciplina Geometria Espacial (MLI34). Observa-se nessa Figura que as

reprovações variaram de 10 (02-02) a 47% (01-04). E também que o número de

reprovações é baixo tem uma pequena amplitude de variação, com exceções, nos

semestres (01-00) e (01-04). Quanto ao tipo da reprovação na Figura 6(b) verifica-se

que as reprovações ocorrem predominantemente por desistência, mas quando as

reprovações foram por notas foram iguais às reprovações com ou sem RM. Na Figura

6(c) verifica-se que o número de alunos matriculados não interfere no número de

reprovações como na disciplina Introdução a Matemática. E na Figura 6(d) o número de

alunos matriculados vem diminuindo, com exceção 01 -04.

Page 47: Numero 09 - Outubro de 2007

(a) (b)

REPROVAÇÕES GEOM. ESP.

05

101520253035404550

01.20

00

02.20

00

01.20

01

02.20

01

01.20

02

02.20

02

01.20

03

02.20

03

01.20

04

02.20

04

TURMA

% D

E A

LUN

OS

TIPOS GEOM. ESP.

0%

20%

40%

60%

80%

100%

01.2000

02.2000

01.2001

02.2001

01.2002

02.2002

01.2003

02.2003

01.2004

02.2004

TURMA

% D

AS

REP

RO

VAÇ

ÕE

NOTA RM FREQÜÊNCIA

(c) (d)

GEOM. ESP.

y = 0,2878x - 0,7853R2 = 0,1411

0

5

10

15

20

25

0 10 20 30 40 50TOTAL DE MATRICULADOS

TOTA

L D

E R

EPR

OVA

DO

S

TOTAL ALUNOS

05

101520253035404550

01.2000

02.2000

01.2001

02.2001

01.2002

02.2002

01.2003

02.2003

01.2004

02.2004

Figura 6. Disciplina Geometria Espacial. Total de alunos reprovados por semestre (a), Tipo de reprovação (b); relação número de reprovados x número de matriculados (c) e total de alunos matriculados (d).

Na Figura 7(a), encontram-se apresentados os resultados das reprovações dos

alunos na disciplina Estatística e Probabilidade (MLI54). Observa-se nessa Figura que

as reprovações variaram de 15 (01-04) a 40% (02-02). E também que tem uma pequena

amplitude variação como em Geometria Espacial. Quanto ao tipo da reprovação na

Figura 7(b) observa-se que as reprovações ocorreram por desistência nos dois últimos

semestres, os outros foram mais por notas. As reprovações com RM também foram

mínimas. Na Figura 7(c) verifica-se que o número de alunos matriculados não interfere

no número de reprovados. E na Figura 7(d) o número de alunos matriculados vem

diminuindo.

Page 48: Numero 09 - Outubro de 2007

(a) (b)

REPROVAÇÕES ESTATÍSTICA

05

1015202530354045

01.20

00

02.20

00

01.20

01

02.20

01

01.20

02

02.20

02

01.20

03

02.20

03

01.20

04

02.20

04

TURMA

% D

E A

LUN

OS

TIPOS ESTATÍSTICA

0%

20%

40%

60%

80%

100%

01.20

00

02.20

00

01.20

01

02.20

01

01.20

02

02.20

02

01.20

03

02.200

3

01.20

04

02.20

04

TURMA

% D

AS

REP

RO

VAÇ

ÕE

NOTA RM FREQÜÊNCIA

(c) (d)

ESTATÍSTICA

y = 0,2442x + 1,2922R2 = 0,3992

02468

10121416

0 10 20 30 40 50 60TOTAL DE MATRICULADOS

TOTA

L D

E R

EPR

OVA

DO

S

TOTAL ALUNOS

0

10

20

30

40

50

60

01.20

00

02.20

00

01.200

1

02.20

01

01.20

02

02.200

2

01.20

03

02.200

3

01.200

4

02.20

04

Figura 7. Disciplina Estatística. Total de alunos reprovados por semestre (a), Tipo de reprovação (b); relação número de reprovados x número de matriculados (c) e total de alunos matriculados (d).

Na Figura 8(a), encontram-se apresentados os resultados das reprovações dos

alunos na disciplina Estruturas Algébricas 1 (MLI53) (Est.Alg.1). Observa-se nessa

Figura que as reprovações variaram de 33 (02-02) a 90% (01-02). E também tem uma

maior amplitude variação em relação às disciplinas anteriores. Quanto ao tipo da

reprovação na Figura 8(b) verifica-se que as reprovações ocorrem por notas e

desistências. E as reprovações com RM são também menores como nas disciplinas

anteriores. Na Figura 8(c) observa-se que o número em média 60 alunos matriculados

tem uma reprovação maior. E na Figura 8(d) o número de alunos matriculados tem uma

maior variação em relação às outras disciplinas. Cabe salientar, no semestre (02-02)

teve o maior nº de alunos matriculados nesta disciplina e na Figura 8(a) verifica-se que

Page 49: Numero 09 - Outubro de 2007

foi o menor índice de reprovação, então no semestre consecutivo caiu o nº de alunos

matriculados.

(a) (b)

REPROVAÇÕES EST. ALG. 1

0102030405060708090

100

01.20

00

02.20

00

01.20

01

02.20

01

01.20

02

02.20

02

01.20

03

02.20

03

01.20

04

02.20

04

TURMA

% D

E A

LUN

OS

TIPOS EST. ALG. 1

0%10%20%30%40%50%60%70%80%90%

100%

01.20

00

02.20

00

01.20

01

02.20

01

01.20

02

02.20

02

01.20

03

02.20

03

01.20

04

02.20

04

TURMA%

DA

S R

EPR

OVA

ÇÕ

E

NOTA RM FREQÜÊNCIA

(c) (d)

EST. ALG. 1

y = 0,3246x + 16,51R2 = 0,1283

0

10

20

30

40

50

60

35 45 55 65 75 85TOTAL DE MATRICULADOS

TOTA

L D

E R

EPR

OVA

DO

S

TOTAL ALUNOS

3540455055606570758085

01.20

00

02.20

00

01.20

01

02.20

01

01.20

02

02.20

02

01.20

03

02.20

03

01.20

04

02.20

04

Figura 8. Disciplina Estruturas Algébricas 1. Total de alunos reprovados por semestre (a), Tipo de reprovação (b); relação número de reprovados x número de matriculados (c) e total de alunos matriculados (d).

Na Figura 9(a), encontram-se apresentados os resultados das reprovações dos

alunos na disciplina Cálculo Numérico (MLI55). Observa-se nessa Figura que as

reprovações variaram de 7 (01-02) a 58% (01-04). Cabe salientar que até (02-02) tinha

um índice de reprovação baixo e depois de (01-03) vinha aumentando este índice, até

que 02-04 voltou a diminuir. Quanto ao tipo da reprovação na Figura 9(b) verifica-se

que as reprovações ocorrem por notas e desistências, com exceções: em dois semestres

01-02 e 02-04 todas as reprovações foram por desistência, e no semestre 02-01 foi 60%

com RM e 20% por nota. A reprovação por RM é mínima também. Na Figura 9(d) e 9

Page 50: Numero 09 - Outubro de 2007

(c) observa-se que o número de alunos matriculados tem uma variação, e o nº de alunos

reprovados não tem uma correlação significativa com o nº de alunos matriculados.Cabe

salientar que apesar do nº de reprovações serem alto em 01-04 (Figura .9(a)) o nº de

alunos matriculados não aumentou Figura.9 (d).

(a) (b)

REPROVAÇÕES CALC NUMÉRICO

0

10

20

30

40

50

60

70

01.20

00

02.20

00

01.20

01

02.20

01

01.20

02

02.20

02

01.20

03

02.20

03

01.20

04

02.20

04

TURMA

% D

E A

LUN

OS

TIPOS CALC. NUMÉRICO

0%

20%

40%

60%

80%

100%

01.20

00

02.20

00

01.20

01

02.20

01

01.20

02

02.20

02

01.20

03

02.20

03

01.20

04

02.20

04

TURMA%

DA

S R

EPR

OVA

ÇÕ

E

NOTA RM FREQÜÊNCIA

(c) (d)

CALC. NUMÉRICO

y = 0,4276x - 4,7737R2 = 0,2421

0

5

10

15

20

0 10 20 30 40TOTAL DE MATRICULADOS

TOTA

L D

E R

EPR

OVA

DO

S

TOTAL ALUNOS

05

10152025303540

01.20

00

02.20

00

01.20

01

02.20

01

01.20

02

02.20

02

01.20

03

02.20

03

01.20

04

02.20

04

Figura 9. Disciplina Calculo Numérico. Total de alunos reprovados por semestre (a), Tipo de reprovação (b); relação número de reprovados x número de matriculados (c) e total de alunos matriculados (d).

Na Figura 10(a), encontram-se apresentados os resultados das reprovações dos

alunos na disciplina Cálculo 4 (MLI43) . Observa-se nessa Figura que as reprovações

variaram de 30 (01-02) a 69% (02-04). E também se observa que até o semestre (01-02)

as reprovações vinham diminuindo e depois em (02-02) as reprovações são maiores

quando a disciplina é ministrada no 2º semestre. Quanto ao tipo da reprovação na Figura

10(b) verifica-se que as reprovações ocorrem por notas e desistências, com exceção, no

semestre (02-02) que foi mais por desistência. As reprovações com RM são maiores

nesta disciplina do que as disciplinas anteriores.

Page 51: Numero 09 - Outubro de 2007

(a) (b)

REPROVAÇÕES CÁLCULO 4

0

10

20

30

40

50

60

70

80

01.20

00

02.20

00

01.2001

02.20

01

01.20

02

02.20

02

01.20

03

02.20

03

01.2004

02.20

04

TURMA

% D

E A

LUN

OS

TIPOS CÁLCULO 4

0%10%20%30%40%50%60%70%80%90%

100%

01.20

00

02.200

0

01.20

01

02.20

01

01.20

02

02.20

02

01.20

03

02.20

03

01.200

4

02.20

04

TURMA

% D

AS

REP

RO

VAÇ

ÕE

NOTA RM FREQÜÊNCIA

(c) (d)

CÁLCULO 4

y = 0,4216x + 3,5056R2 = 0,3665

0

5

10

15

20

25

30

0 10 20 30 40 50 60TOTAL DE MATRICULADOS

TOTA

L D

E R

EPR

OVA

DO

S

TOTAL ALUNOS

0

10

20

30

40

50

60

01.20

00

02.20

00

01.20

01

02.20

01

01.20

02

02.20

02

01.20

03

02.20

03

01.20

04

02.20

04

Figura 10. Disciplina Cálculo 4. Total de alunos reprovados por semestre (a), Tipo de reprovação (b); relação número de reprovados x número de matriculados (c) e total de alunos matriculados (d).

Na Figura 10(c) observa-se que o nº de alunos matriculados até 40 não

influenciam no nº de alunos reprovados, a partir daí maior nº de alunos matriculados

maior nº de alunos reprovados. Na Figura10(d) como na Figura 10(a) o número de

alunos matriculados no primeiro semestre 01-03 sempre aumenta porque tem uma

reprovação maior no segundo semestre.

Na Figura 11(a), encontram-se apresentados os resultados das reprovações dos

alunos na disciplina Álgebra linear 2 (MLI29) (AL2). Observa-se nessa Figura que as

reprovações variaram de 22 (02-03) a 100% (02-00).Cabe salientar que apesar de que só

nesta disciplina teve 100% de reprovação, a amplitude de variação de reprovação é

pequena. Quanto ao tipo da reprovação na Figura 11(b) verifica-se que as reprovações

ocorrem mais por desistência do que por nota, menos em (02-00) onde houve

Page 52: Numero 09 - Outubro de 2007

reprovação total por nota, isto é , o que se conclui que ocorreu um problema com esta

turma, não foi uma reprovação por freqüência como geralmente acontece, e a média das

reprovações tirando esta turma é de 40,9%. Observa-se também que em dois semestres

as reprovações quando não eram por faltas os alunos conseguiram o RM (01-03 e 02-

04). Na Figura 11(c) , como as turmas tem um nº pequeno de alunos não foi possível

relacionar os matriculados com os reprovados. Na Figura 11(d) o total de alunos

matriculados estão em torno de 3 a 8 com exceção da turma (02-01) com 18 alunos

matriculados.

Na Figura 12(a), encontram-se apresentados os resultados das reprovações dos

alunos na disciplina Funções Variáveis Complexas (DCE13). Observa-se nessa Figura

que as reprovações variaram de 14 (01-00) a 62% (02-02). Cabe salientar que as

reprovações vieram aumentando até (02-02) e depois diminuindo, e agora parece que

volta aumentar. Esta disciplina tem uma amplitude de variação bem diferente das

disciplinas anteriores. Quanto ao tipo da reprovação na Figura 12(b) verifica-se que as

reprovações ocorrem mais por nota do que desistência, com exceções, dos semestres 01-

01 e 02-01.As reprovações com RM são poucas como na maioria das disciplinas. Na

Figura 12(c) não tem uma relação entre os alunos matriculas e os reprovados. E a Figura

12(d) e 12(a) nos mostra que no semestre 02-03 teve o maior número de alunos

matriculados se o índice de reprovação foi baixo.

Page 53: Numero 09 - Outubro de 2007

(a) (b)

REPROVAÇÕES AL 2

0

20

40

60

80

100

120

01.20

00

02.20

00

01.20

01

02.20

01

01.20

02

02.20

02

01.20

03

02.20

03

01.20

04

02.20

04

TURMA

% D

E A

LUN

OS

TIPOS AL 2

0%10%20%30%40%50%60%70%80%90%

100%

01.20

00

02.20

00

01.20

01

02.20

01

01.20

02

02.20

02

01.20

03

02.20

03

01.20

04

02.20

04

TURMA

% D

AS

REP

RO

VAÇ

ÕE

NOTA RM FREQÜÊNCIA

(c) (d)

AL 2

y = 0,1928x + 1,5731R2 = 0,3994

0

1

2

3

4

5

6

0 5 10 15 20TOTAL DE MATRICULADOS

TOTA

L D

E R

EPR

OVA

DO

S

TOTAL ALUNOS

02468

101214161820

01.20

00

02.20

00

01.20

01

02.20

01

01.20

02

02.20

02

01.20

03

02.20

03

01.20

04

02.20

04

Figura 11. Disciplina Álgebra Linear 2. Total de alunos reprovados por semestre (a), Tipo de reprovação (b); relação número de reprovados x número de matriculados (c) e total de alunos matriculados (d).

Na Figura 13(a), encontram-se apresentados os resultados das reprovações dos

alunos na disciplina Análise1 (MLI 65). Observa-se nessa Figura que as reprovações

variaram de 20 (01-03) a 82% (02-00). E as reprovações eram em média de 60% , com

exceção em (02-00) depois diminui para uma média de 25%, com exceção (02-03).

Quanto ao tipo da reprovação na Figura 13(b) verifica-se que as reprovações ocorrem

predominantemente por nota do que por desistência, com exceções em 01-04 e 02-04, o

que não acontece nas disciplinas anteriores. E também se observa que tem uma

reprovação por RM mais expressiva. Na Figura 13(c) ,observa-se que não foi possível

relacionar os matriculados e reprovados. E a Figura 13(d) nos mostra que as turmas

Page 54: Numero 09 - Outubro de 2007

estavam aumentando depois que começaram a diminuir e agora esta aumentando de

novo. E ainda apesar de 02-03 ter tido um índice grande de reprovação não aumentou o

número de alunos matriculados no semestre consecutivo.

(a) (b)

REPROVAÇÕES FUN. VAR. COMPLEXA

0

10

20

30

40

50

60

70

01.20

00

02.20

00

01.20

01

02.20

01

01.20

02

02.20

02

01.20

03

02.20

03

01.20

04

02.20

04

TURMA

% D

E A

LUN

OS

TIPOS FUN.VAR.COMPLEXA

0%10%20%30%40%50%60%70%80%90%

100%

01.20

00

02.200

0

01.20

01

02.20

01

01.20

02

02.200

2

01.20

03

02.20

03

01.20

04

02.2004

TURMA

% D

AS

REP

RO

VAÇ

ÕE

NOTA RM FREQÜÊNCIA

(c) (d)

FUN.VAR.COMPLEXA

y = 0,366x - 1,8856R2 = 0,2489

0

5

10

15

20

10 20 30 40 50TOTAL DE MATRICULADOS

TOTA

L D

E R

EPR

OVA

DO

S

TOTAL ALUNOS

05

1015202530354045

01.20

00

02.20

00

01.20

01

02.20

01

01.20

02

02.20

02

01.20

03

02.20

03

01.20

04

02.20

04

Figura 12. Disciplina Fun. Variável Complexa. Total de alunos reprovados por semestre (a), Tipo de reprovação (b); relação número de reprovados x número de matriculados (c) e total de alunos matriculados (d).

Na Figura 14(a), encontram-se apresentados os resultados das reprovações dos

alunos na disciplina Estruturas Algébricas 2 (MLI24) (Est.Alg.2). Observa-se nessa

Figura que as reprovações variaram de 0 (02-02) a 80% (01-04). As reprovações estão

em torno de 40% exceto em 3 semestres que as reprovações foram acima de 60% 01-00,

02-03 e 01-04. Cabe salientar que o semestre consecutivo a um índice alto de

reprovação tem-se o um índice abaixo de 20%. Quanto ao tipo da reprovação na Figura

14(b) observa-se que as reprovações ocorrem mais por desistências, com exceção em

Page 55: Numero 09 - Outubro de 2007

(02-00) que foi totalmente por nota e RM. Na Figura 14(c) e 14(d) verifica-se que o

número de alunos matriculados é muito pequeno mesmo assim podemos observar que

aumenta o número de reprovados quando aumenta o número de matriculados.

(a) (b)

REPROVAÇÕES ANALISE 1

0102030405060708090

01.20

00

02.20

00

01.20

01

02.20

01

01.20

02

02.20

02

01.20

03

02.20

03

01.20

04

02.20

04

TURMA

% D

E A

LUN

OS

TIPOS ANALISE 1

0%

20%

40%

60%

80%

100%

01.20

00

02.20

00

01.20

01

02.20

01

01.20

02

02.20

02

01.20

03

02.20

03

01.20

04

02.20

04

TURMA

% D

AS

REP

RO

VAÇ

ÕE

NOTA RM FREQÜÊNCIA

(c) (d)

ANALISE 1

y = 0,239x + 9,4321R2 = 0,1577

05

101520253035

0 20 40 60 80TOTAL DE MATRICULADOS

TOTA

L D

E R

EPR

OVA

DO

S

TOTAL ALUNOS

010203040506070

01.20

00

02.200

0

01.20

01

02.20

01

01.200

2

02.20

02

01.20

03

02.200

3

01.20

04

02.20

04

Figura 13. Disciplina Análise 1. Total de alunos reprovados por semestre (a), Tipo de reprovação (b); relação número de reprovados x número de matriculados (c) e total de alunos matriculados (d).

Na Figura 15(a), encontram-se apresentados os resultados das reprovações dos

alunos na disciplina Introdução à Matemática (MLI 15) (Int. a Matemática). Observa-se

nessa Figura que as reprovações variaram de 0 (01-02),(02-02),(01-03) e (02-03) a 14%

(02-00). Quanto ao tipo da reprovação na Figura 15(b) observa-se que as reprovações

Page 56: Numero 09 - Outubro de 2007

ocorrem só por desistência. Na Figura 15(c) e 15(d) observa-se que o número de

reprovações é muito baixo., independente do número de alunos matriculados

(a) (b)

REPROVAÇÕES EST. ALG. 2

0102030405060708090

01.20

00

02.20

00

01.20

01

02.20

01

01.20

02

02.20

02

01.20

03

02.20

03

01.20

04

02.20

04

TURMA

% D

E A

LUN

OS

TIPOS EST. ALG. 2

0%10%20%30%40%50%60%70%80%90%

100%

01.20

00

02.20

00

01.20

01

02.20

01

01.20

02

02.20

02

01.20

03

02.20

03

01.20

04

02.20

04

TURMA

% D

AS

REP

RO

VAÇ

ÕE

NOTA RM FREQÜÊNCIA

(c) (d)

EST. ALG. 2

y = 0,6067x - 1,0255R2 = 0,652

0

2

4

6

8

10

0 5 10 15TOTAL DE MATRICULADOS

TOTA

L D

E R

EPR

OVA

DO

S

TOTAL ALUNOS

02468

101214

01.20

00

02.20

00

01.20

01

02.20

01

01.20

02

02.20

02

01.20

03

02.20

03

01.20

04

02.20

04

Figura 14. Disciplina Estrutura Algébrica 2. Total de alunos reprovados por semestre (a), Tipo de reprovação (b); relação número de reprovados x número de matriculados (c) e total de alunos matriculados (d).

Na Figura 16(a), encontram-se apresentados os resultados das reprovações dos

alunos na disciplina Metodologia do Ensino da Matemática (MLI 37) (Met. Ensino

Matemática). Observa-se nessa Figura que as reprovações variaram de 0 (01-02),(02-

02),(01-03) a 23% (01-00). Cabe salientar que ocorre da mesma forma que a disciplina

Introdução a Matemática. Quanto ao tipo da reprovação na Figura 16(b) observa-se que

as reprovações ocorrem predominantemente por desistência. Na Figura 16(c) e 16(d)

observa-se que o número de reprovações é muito baixo, independente do número de

alunos matriculados.

Page 57: Numero 09 - Outubro de 2007

(a) (b)

REPROVAÇÕES INT.A MATEMÁTICA

0

2

4

6

8

10

12

14

16

01.20

00

02.20

00

01.20

01

02.20

01

01.20

02

02.20

02

01.20

03

02.20

03

01.20

04

02.20

04

TURMA

% D

E A

LUN

OS

TIPOS INT.A MATEMÁTICA

0%10%20%30%40%50%60%70%80%90%

100%

01.20

00

02.20

00

01.20

01

02.20

01

01.20

02

02.20

02

01.20

03

02.20

03

01.20

04

02.20

04

TURMA

% D

AS

REP

RO

VAÇ

ÕE

NOTA RM FREQÜÊNCIA

(c) (d)

INT.A MATEMÁTICA

y = 0,0109x + 0,654R2 = 0,0048

00,5

11,5

22,5

33,5

0 10 20 30 40TOTAL DE MATRICULADOS

TOTA

L D

E R

EPR

OVA

DO

S

TOTAL ALUNOS

05

10152025303540

01.20

00

02.20

00

01.20

01

02.20

01

01.20

02

02.20

02

01.20

03

02.20

03

01.20

04

02.20

04

Figura 15. Disciplina Introdução a Matemática. Total de alunos reprovados por semestre (a), Tipo de reprovação (b); relação número de reprovados x número de matriculados (c) e total de alunos matriculados (d).

Na Figura 17(a), encontram-se apresentados os resultados das reprovações dos

alunos na disciplina Métodos Matemáticos (DCE 19) . Observa-se nessa Figura que as

reprovações variaram de 23 (02-02) a 67% (01-00). E não tem uma variação nos índices

de reprovação, ficando em média de 40%. Quanto ao tipo da reprovação na Figura 17(b)

verifica-se que as reprovações ocorrem mais por desistência e em dois semestres por

notas, (01-01) e (01-04). Não tendo em nenhum semestre, alunos reprovados com RM.

Na Figura 17(c) e 17(d) observa-se mesmo com um número pequeno de alunos

matriculados nesta disciplina que maior o nº de alunos matriculados maior é a

reprovação.(Esta disciplina é obrigatória somente para o bacharelado.)

Page 58: Numero 09 - Outubro de 2007

(a) (b)

REPROVAÇÕES MET. ENSINO MATEMÁTICA

0

5

10

15

20

25

01.20

00

02.20

00

01.20

01

02.20

01

01.20

02

02.20

02

01.20

03

02.20

03

01.20

04

02.20

04

TURMA

% D

E A

LUN

OS

TIPOS MET.ENSINO MATEMÁTICA

0%10%20%30%40%50%60%70%80%90%

100%

01.20

00

02.20

00

01.20

01

02.200

1

01.20

02

02.20

02

01.20

03

02.200

3

01.20

04

02.20

04

TURMA

% D

AS

REP

RO

VAÇ

ÕE

NOTA RM FREQÜÊNCIA

(c) (d)

MET.ENSINO MATEMÁTICA

y = -0,02x + 1,3364R2 = 0,0267

0

0,5

1

1,5

2

2,5

0 10 20 30 40TOTAL DE MATRICULADOS

TOTA

L D

E R

EPR

OVA

DO

S

TOTAL ALUNOS

05

101520253035

01.20

00

02.20

00

01.20

01

02.20

01

01.20

02

02.20

02

01.20

03

02.20

03

01.20

04

02.20

04

Figura 16. Disciplina Metodologia do Ensino da Matemática. Total de alunos reprovados por semestre (a), Tipo de reprovação (b); relação número de reprovados x número de matriculados (c) e total de alunos matriculados (d).

Na Figura 18(a), encontram-se apresentados os resultados das reprovações dos

alunos na disciplina Análise 2 (EXT 59) . Observa-se nessa Figura que as reprovações

variaram de 20 (01-04) a 84% (01-00). Cabe salientar que a amplitude de variação de

reprovação desta disciplina é maior do que Métodos Matemáticos. Quanto ao tipo da

reprovação na Figura 18(b) observa-se que as reprovações ocorrem predominantemente

por desistência, como na disciplina anterior. Nesta disciplina ocorreram reprovações por

RM. Na Figura 18(c) e 18(d) observa-se como o nº de alunos é muito pequeno não tem

uma relação entre matriculados e reprovados. (Esta disciplina é obrigatória somente

para o bacharelado.)

Page 59: Numero 09 - Outubro de 2007

(a) (b)

REPROVAÇÕES MÉTODOS MATEMÁTICOS

01020304050607080

01.20

00

02.20

00

01.20

01

02.20

01

01.20

02

02.20

02

01.20

03

02.20

03

01.20

04

02.20

04

TURMA

% D

E A

LUN

OS

TIPOS MÉTODOS MATEMÁTICOS

0%10%20%30%40%50%60%70%80%90%

100%

01.20

00

02.20

00

01.20

01

02.20

01

01.20

02

02.20

02

01.20

03

02.20

03

01.20

04

02.20

04

TURMA

% D

AS

REP

RO

VAÇ

ÕE

NOTA RM FREQÜÊNCIA

(c) (d)

MÉTODOS MATEMÁTICOS

y = 0,2934x + 0,4981R2 = 0,5718

0

1

2

3

4

5

6

0 5 10 15TOTAL DE MATRICULADOS

TOTA

L D

E R

EPR

OVA

DO

S

TOTAL ALUNOS

02468

101214

01.20

00

02.20

00

01.20

01

02.20

01

01.20

02

02.20

02

01.20

03

02.20

03

01.20

04

02.20

04

Figura 17. Disciplina Métodos Matemáticos. Total de alunos reprovados por semestre (a), Tipo de reprovação (b); relação número de reprovados x número de matriculados (c) e total de alunos matriculados (d).

Na Figura 19(a), encontram-se apresentados os resultados das reprovações dos

alunos na disciplina Topologia Espaços Métricos (MLI 50). Observa-se nessa Figura

que as reprovações variaram de 16 (02-01) a 63% (02-03), sendo que esta disciplina não

é ministrada em todos semestres. Cabe salientar que só em um semestre a reprovação

foi muito grande. Quanto ao tipo da reprovação na Figura 19(b) observa-se que as

reprovações ocorrem em 3 semestres por desistência e os outros 3 predominantemente

por notas e RM. Na Figura 19(c) e 19(d) verifica-se como o nº de alunos é muito

pequeno não tem uma relação entre matriculados e reprovados. (Esta disciplina é

obrigatória somente para o bacharelado.)

Page 60: Numero 09 - Outubro de 2007

(a) (b)

REPROVAÇÕES ANALISE 2

0102030405060708090

01.20

00

02.20

00

01.20

01

02.20

01

01.20

02

02.20

02

01.20

03

02.20

03

01.20

04

02.20

04

TURMA

% D

E A

LUN

OS

TIPOS ANALISE 2

0%

20%

40%

60%

80%

100%

01.20

00

02.20

00

01.20

01

02.20

01

01.20

02

02.20

02

01.20

03

02.20

03

01.20

04

02.20

04

TURMA

% D

AS

REP

RO

VAÇ

ÕE

NOTA RM FREQÜÊNCIA

(c) (d)

ANALISE 2

y = 0.4436x + 0.2623R2 = 0.4352

012345678

0 5 10 15TOTAL DE MATRICULADOS

TOTA

L D

E R

EPR

OVA

DO

S

TOTAL ALUNOS

02468

101214

01.20

00

02.20

00

01.20

01

02.20

01

01.20

02

02.20

02

01.20

03

02.20

03

01.20

04

02.20

04

Figura 18. Disciplina Análise 2. Total de alunos reprovados por semestre (a), Tipo de reprovação (b); relação número de reprovados x número de matriculados (c) e total de alunos matriculados (d).

Na Figura 20(a), encontram-se apresentados os resultados das reprovações dos

alunos na disciplina Geometria Não Euclidiana (DCE 14). Cabe salientar que ela tem

uma maior amplitude de variação de reprovação. Observa-se nessa Figura que as

reprovações variaram de 5(01-03) a 57% (02-03). Quanto ao tipo da reprovação na

Figura 20(b) observa-se que em alguns semestres a reprovação é total por desistência

(01-00) e (01-03) , e em (02-00) total por nota. Os outros semestres por desistência,

notas e RM. Esta disciplina tem também um número maior de reprovação com RM do

que as outras disciplinas. Na Figura 20(c) e 20(d) observa-se que não tem uma relação

entre matriculados e reprovados, e ainda que o número de alunos matriculados nesta

disciplina veio aumentando após 02-000. (Esta disciplina é obrigatória somente para a

licenciatura.)

Page 61: Numero 09 - Outubro de 2007

(a) (b)

REPROVAÇÕES TOP. DE ESP. METRIC

0

10

20

30

40

50

60

70

01.20

00

02.20

00

01.20

01

02.20

01

01.20

02

02.20

02

01.20

03

02.20

03

01.20

04

02.20

04

TURMA

% D

E A

LUN

OS

TIPOS TOP.DE ESP. METRIC.

0%10%20%30%40%50%60%70%80%90%

100%

01.20

00

02.20

00

01.20

01

02.20

01

01.20

02

02.20

02

01.20

03

02.20

03

01.20

04

02.20

04

TURMA

% D

AS

REP

RO

VAÇ

ÕE

NOTA RM FREQÜÊNCIA

(c) (d)

TOP. DE ESP. MÉTRIC

y = 0,295x + 0,107R2 = 0,6488

0

1

2

3

4

5

6

0 5 10 15TOTAL DE MATRICULADOS

TOTA

L D

E R

EPR

OVA

DO

S

TOTAL ALUNOS

02468

101214

01.20

00

02.20

00

01.20

01

02.20

01

01.20

02

02.20

02

01.20

03

02.20

03

01.20

04

02.20

04

Figura 19. Disciplina Topologia Espaços Métricos. Total de alunos reprovados por semestre (a), Tipo de reprovação (b); relação número de reprovados x número de matriculados (c) e total de alunos matriculados (d).

Na Figura 21(a), encontram-se apresentados os resultados das reprovações dos

alunos na disciplina o Ensino de Matemática Através de Problemas (MLI 62). Observa-

se nessa Figura, que são poucas as reprovações, da mesma forma que as disciplinas

Introdução a Matemática e Metodologia da Matemática. Quanto ao tipo da reprovação

na Figura 21(b) cabe salientar, que em alguns semestres a reprovação é total por

desistência e no 01-04 mais de 60% da reprovação foi por RM. Na Figura 21(c) e 21(d)

observa-se que como a reprovação é mínima então não tem relação entre matriculados e

reprovados (esta disciplina é obrigatória somente para a licenciatura).

Page 62: Numero 09 - Outubro de 2007

(a) (b)

REPROVAÇÕES GEOM. NÃO EUCLIDIANA.

0

10

20

30

40

50

60

01.20

00

02.20

00

01.20

01

02.20

01

01.20

02

02.20

02

01.20

03

02.20

03

01.20

04

02.20

04

TURMA

% D

E A

LUN

OS

TIPOS GEOM. NÃO EUCLIDIANA.

0%10%20%30%40%50%60%70%80%90%

100%

01.20

00

02.20

00

01.200

1

02.20

01

01.200

2

02.200

2

01.20

03

02.200

3

01.200

4

02.20

04

TURMA

% D

AS

REP

RO

VAÇ

ÕE

NOTA RM FREQÜÊNCIA

(c) (d)

GEOM. NÃO EUCLIDIANA

y = 0,2497x + 1,157R2 = 0,16

0

5

10

15

20

0 10 20 30 40 50TOTAL DE MATRICULADOS

TOTA

L D

E R

EPR

OVA

DO

S

TOTAL ALUNOS

05

1015202530354045

01.20

00

02.20

00

01.20

01

02.20

01

01.20

02

02.20

02

01.20

03

02.20

03

01.20

04

02.20

04

Figura 20. Disciplina Geometria Não Euclidiana. Total de alunos reprovados por semestre (a), Tipo de reprovação (b); relação número de reprovados x número de matriculados (c) e total de alunos matriculados (d).

Na Figura 22(a), encontram-se apresentados os resultados das reprovações dos

alunos na disciplina Geometria Diferencial (MLI 57). Observa-se nessa Figura que as

reprovações variaram de 0 (01-02) a 75% (02-04) e nos semestre (01-00), (02-02), e

(02-03) não foi oferecida esta disciplina. Cabe salientar que esta disciplina também tem

uma grande amplitude de variação nas reprovações. Quanto ao tipo da reprovação na

Figura 22(b) observa-se que as reprovações ocorrem predominantemente por

desistência. E quando ocorre a reprovação por notas e RM com a mesma porcentagem.

Na Figura 22(c) e 22(d) observa-se como o nº de alunos é muito pequeno não tem uma

relação entre matriculados e reprovados .(Esta disciplina é obrigatória somente para o

bacharelado.)

Page 63: Numero 09 - Outubro de 2007

(a) (b)

REPROVAÇÕES O ENS. MATEM. ATRAVÉS DE PROBLEMAS

0

5

10

1520

25

30

35

01.20

00

02.20

00

01.20

01

02.20

01

01.20

02

02.20

02

01.20

03

02.20

03

01.20

04

02.20

04

TURMA

% D

E A

LUN

OS

TIPOS O ENS. MATEM. ATRAVÉS DE PROBLEMAS

0%

20%

40%

60%

80%

100%

01.20

00

02.20

00

01.20

01

02.20

01

01.20

02

02.20

02

01.20

03

02.20

03

01.20

04

02.20

04

TURMA

% D

AS

REP

RO

VAÇ

ÕE

NOTA RM FREQÜÊNCIA

(c) (d)

O ENSINO DA MATEM. ATRAVÉS DE PROBLEMAS

y = 0,0795x - 0,2909R2 = 0,0794

0

2

4

6

8

0 10 20 30 40TOTAL DE MATRICULADOS

TOTA

L D

E R

EPR

OVA

DO

S

TOTAL ALUNOS

05

101520253035

01.200

0

02.20

00

01.20

01

02.200

1

01.20

02

02.20

02

01.200

3

02.20

03

01.20

04

02.200

4

Figura 21. Disciplina O Ensino da Matemática Através de Problemas. Total de alunos reprovados por semestre (a), Tipo de reprovação (b); relação número de reprovados x número de matriculados (c) e total de alunos matriculados (d).

Na Figura 23(a), encontram-se apresentados os resultados das reprovações dos

alunos na disciplina o História da Matemática (DCE 17). Observa-se nessa Figura que

são poucas as reprovações. Quanto ao tipo da reprovação na Figura 23(b) observa-se

que em alguns semestres a reprovação é total por desistência e no 02-01 foi total por

nota. Na Figura 23(c) e 23(d) observa-se que não tem uma relação entre matriculados e

reprovados. E o número de matriculados vem aumentando.

Na Figura 24(a), encontram-se apresentados os resultados das reprovações dos

alunos na disciplina Análise 3 (MLI 39) . Observa-se nessa Figura que as reprovações

variaram de 0 (02-02) (02-03) e (02-04) a 75% (01-01) . o semestre (01-02) não foi

oferecida esta disciplina. Cabe salientar como nas disciplinas Geometria Diferencial,

Topologia dos Espaços Métricos os índices de reprovações tem uma grande

variabilidade. Quanto ao tipo da reprovação na Figura 24(b) observa-se que as

Page 64: Numero 09 - Outubro de 2007

reprovações ocorrem em 3 semestres por desistência e um por RM 02-04. Na Figura

24(c) e 24(d) observa-se como o nº de alunos é muito pequeno não tem uma relação

entre matriculados e reprovados. (Esta disciplina é obrigatória somente para o

bacharelado.)

(a) (b)

REPROVAÇÕES GEOM. DIFERENCIAL

0

10

20

3040

50

60

70

80

01.20

00

02.20

00

01.20

01

02.20

01

01.20

02

02.20

02

01.20

03

02.20

03

01.20

04

02.20

04

TURMA

% D

E A

LUN

OS

TIPOS GEOMETRIA DIFERENCIAL

0%10%20%30%40%50%60%70%80%90%

100%

01.200

0

02.20

00

01.20

01

02.200

1

01.20

02

02.20

02

01.200

3

02.20

03

01.20

04

02.200

4

TURMA%

DA

S R

EPR

OVA

ÇÕ

E

NOTA RM FREQÜÊNCIA

(c) (d)

GEOM. DIFERENCIAL

y = 0,2587x + 0,3169R2 = 0,5089

0

1

2

3

4

5

0 5 10 15TOTAL DE MATRICULADOS

TOTA

L D

E R

EPR

OVA

DO

S

TOTAL ALUNOS

02468

101214

01.20

00

02.20

00

01.20

01

02.20

01

01.20

02

02.20

02

01.20

03

02.20

03

01.20

04

02.20

04

Figura 22. Disciplina Geometria Diferencial. Total de alunos reprovados por semestre (a), Tipo de reprovação (b); relação número de reprovados x número de matriculados (c) e total de alunos matriculados (d).

Page 65: Numero 09 - Outubro de 2007

(a) (b)

REPROVAÇÕES HIST. MATEMÁTICA

0

1

2

3

4

5

6

7

01.20

00

02.20

00

01.20

01

02.20

01

01.20

02

02.20

02

01.20

03

02.20

03

01.20

04

02.20

04

TURMA

% D

E A

LUN

OS

TIPOS HIST. MATEMÁTICA

0%10%20%30%40%50%60%70%80%90%

100%

01.20

00

02.20

00

01.200

1

02.20

01

01.200

2

02.200

2

01.2003

02.200

3

01.2004

02.2004

TURMA

% D

AS

REP

RO

VAÇ

ÕE

NOTA RM FREQÜÊNCIA

(c) (d)

HIST. MATEMÁTICA

y = 0,0684x - 0,8159R2 = 0,3233

-0,5

0

0,5

1

1,5

2

2,5

0 10 20 30 40

TOTAL DE MATRICULADOS

TOTA

L D

E R

EPR

OVA

DO

S

TOTAL ALUNOS

05

101520253035

01.20

00

02.20

00

01.20

01

02.20

01

01.20

02

02.20

02

01.20

03

02.20

03

01.20

04

02.20

04

Figura 23. Disciplina História da Matemática. Total de alunos reprovados por semestre (a), Tipo de reprovação (b); relação número de reprovados x número de matriculados (c) e total de alunos matriculados (d).

Page 66: Numero 09 - Outubro de 2007

(a) (b)

REPROVAÇÕES ANALISE 3

0

10

20

30

40

50

60

70

80

01.20

00

02.20

00

01.20

01

02.20

01

01.20

02

02.20

02

01.20

03

02.20

03

01.20

04

02.20

04

TURMA

% D

E A

LUN

OS

TIPOS ANALISE 3

0%10%20%30%40%50%60%70%80%90%

100%

01.20

00

02.200

0

01.20

01

02.20

01

01.20

02

02.20

02

01.20

03

02.20

03

01.20

04

02.200

4

TURMA

% D

AS

REP

RO

VAÇ

ÕE

NOTA RM FREQÜÊNCIA

(c) (d)

ANALISE 3

y = 0,439x - 0,4561R2 = 0,4924

-101234567

0 2 4 6 8 10TOTAL DE MATRICULADOS

TOTA

L D

E R

EPR

OVA

DO

S

TOTAL ALUNOS

0123456789

10

01.20

00

02.20

00

01.20

01

02.20

01

01.20

02

02.20

02

01.20

03

02.20

03

01.20

04

02.20

04

Figura 24. Disciplina Análise 3. Total de alunos reprovados por semestre (a), Tipo de reprovação (b); relação número de reprovados x número de matriculados (c) e total de alunos matriculados (d).

Conclusão:

De forma geral as reprovações nas disciplinas do curso de matemática variaram

de (0%) em disciplina/semestre a (100%) em disciplina/semestre. Nas disciplinas de

início de curso há uma predominância de reprovações por nota e por desistência (falta),

enquanto que, nas de final de curso predominam as reprovações por falta. São poucas as

disciplinas que tem uma reprovação expressiva de alunos com RM. No início do curso

os percentuais de reprovação são mais elevados do que os no final de curso. Assim

como o número de alunos matriculados. Mas a variabilidade de índices de reprovação

acontece mais no final do curso.

Page 67: Numero 09 - Outubro de 2007

Bibliografia:

BARDIN, Laurence. Análise de conteúdo. Lisboa, Edições 70, 1977.

CASTANHO, Sérgio Ensino com Pesquisa na Graduação. [palestra].IV Seminário de Qualidade Acadêmica: o currículo como expressão do projeto pedagógico.Uberlândia: UFU/PROGRAD/DIREN. Mimeo, 2003.

COELHO, Ildeu. O Saber, o Ensino e o Currículo em Questão. [palestra]. IV Seminário de Qualidade Acadêmica: o currículo como expressão do projeto pedagógico. Uberlândia: UFU/PROGRAD/DIREN. Mimeo, 2003.

COSTA NETO, P.L.O.2002. ESTÍSTICA. Edgard Blücher, São Paulo. 264p.

DE SORDI, Mara Regina Lemes. Avaliação Universitária: mecanismo de controle, de competição e exclusão ou caminho para construção da autonomia, da cooperação e da inclusão? [palestra]. III Seminário de Qualidade Acadêmica: o currículo como expressão do projeto pedagógico. Uberlândia: UFU/PROGRAD/DIREN. Mimeo, 2003

ForGRAD- Fórum de Pró- Reitores de Graduação das Universidades Brasileiras. Textosdas Oficinas de ForGRAD. Curitiba, PR, 2001.

ForGRAD- Plano Nacional de Graduação: um projeto em construção. Rio de Janeiro: UFF, 1999b

LEVINE, D.M. ; BERENSON,M.L. ; SETEPHAN,D. 2000.Estatística: teoria e aplicações ( usando Microsoft Excel em português).LCT EDITORA,812P.

MORETTIN,L. G. 1999.Estatística Básica – Inferência. V. 2. São Paulo. Makron Books.

TRIOLA, Mario F. Introdução à Estatística. LTC-Livros Técnicos e Científicos Editora S.A.Rio de Janeiro, RJ, 1999.

VEIGA NETO, Alfredo.Princípios norteadores para um novo paradigma curricular: interdisciplinaridade, contextualização e flexibilidade. [palestra]. IV Seminário de Qualidade Acadêmica: o currículo como expressão do projeto pedagógico. Uberlândia: UFU/PROGRAD/DIREN. Mimeo, 2003.

Page 68: Numero 09 - Outubro de 2007
Page 69: Numero 09 - Outubro de 2007

O Problema da Condução do Calor em Dimensão Maior que 1 e o Teorema Espectral

Karla Barbosa de Freitas1 e Valdair Bonfim2

Resumo

Na Álgebra Linear ensina-se o teorema espectral para operadores auto-adjuntos, mas devido aos objetivos específicos da disciplina e a limitação do tempo pouco se fala da sua importância. Quando isto é feito, naturalmente se restringe ao caso de operadores definidos em espaços vetoriais de dimensão finita. O objetivo deste trabalho é ilustrar uma importante aplicação do Teorema Espectral no caso de operadores auto-adjuntos definidos em espaços de dimensão infinita, ampliando a gama de exemplos práticos, bem como motivando o estudo futuro de tópicos avançados, como por exemplo a questão da compacidade de conjuntos e operadores, espaços funcionais, integral de Lebesgue, dentre outros. Para levar isso a termo iniciaremos tratando o problema da condução de calor numa barra unidimensional, na qual o método de separação de variáveis conduz à uma solução que pode ser explicitamente calculada. Ao passarmos para o problema da condução do calor em dimensão superior a 1, veremos que o método de separação de variáveis conduz a uma equação diferencial parcial cuja solução explícita é impossível, exceto em casos particulares em que o domínio apresenta simetria. Entretanto veremos que o Teorema Espectral para operadores compactos auto-adjuntos poderá fornecer a existência - pelo menos teórica - de tais soluções mesmo em casos em que o domínio não apresente qualquer tipo de simetria. Em razão do nosso entendimento de que complicações técnicas não se compatibilizam com um texto que propõe ser apenas motivador, adotaremos neste artigo uma postura pouco rigorosa, evitando demonstrações complexas e restringindo-nos a citar referências das mesmas.

1 – Preliminares:

Para posterior referência no texto enunciaremos o teorema espectral em dimensão finita, bem como alguns conceitos e notações.

Teorema 1 ( Teorema Espectral para Operadores Auto-Adjuntos ) :Sejam: • H um espaço vetorial de dimensão finita n munido de produto interno < , > . • HHT : linear. Se T é auto-adjunto ( isto é, vTuvuT ,, Hvu , ), então H possui uma base ortonormal nvvv ,...,, 21 constituída de autovetores de T.

Definição 1: Uma função Rlf ],0[: é dita ser seccionalmente contínua quando ela tiver um número finito de descontinuidades, de primeira espécie, em qualquer intervalo limitado.

1 Bolsista do Programa de Educação Tutorial – PET Acadêmica do Curso de Matemática da UFU. 2 Orientador; Professor da Faculdade de Matemática da UFU.

Page 70: Numero 09 - Outubro de 2007

Utilizaremos as seguintes notações para designar espaços funcionais:

• )],0[( lC é o conjunto das funções reais contínuas definidas em ],0[ l ; • ),0( lC k é o conjunto das funções cujas derivadas de ordem k são contínuas ;

Definição 2: Um espaço de Hilbert é um espaço vetorial V munido de produto interno < , > e completo na métrica vuvuvud ,),( .

2 – O problema da condução do calor: o caso unidimensional:

Nosso primeiro objetivo é obter um candidato ),( txu , solução do problema

( P1 ) )3(],0[,)()0,()2(0,0),(),0(

)1(0,),0(,2

2

lxxfxuttlutu

tlxxu

tu

o qual modela a condução do calor numa barra unidimensional de comprimento l, cujas extremidades são mantidas à temperatura nula, e cuja distribuição inicial de temperaturas

)0,(xu é uma função conhecida )(xf .

O método de separação de variáveis consiste em procurar solução não-nula na forma

( 4 ) )().(),( txtxu ,

onde Rl],0[: e R),0[: são funções reais de uma variável.

Levando (4) em (1) obtemos

)(.)()(.)( txtx ,

ou seja,

)()(

)()(

xx

tt .

Como o primeiro membro depende apenas de t e o segundo apenas de x, então ambos são iguais a uma constante , de onde segue que:

( 5 ) )(.)( tt , e

( 6 ) )(.)( xx .

Page 71: Numero 09 - Outubro de 2007

Mais ainda, impondo que ( 4 ) satisfaça às condições de contorno ( 2 ), obtemos:

( 7 ) 0)()0( l .

Entretanto, como veremos no que segue, o problema

( 8 ) 0)()0()(.)(

lxx

,

que nada mais é do que um problema de autovalores para o operador )(T , só admite solução 0 quando 0 .

De fato, se é uma solução não-nula de ( 8 ), então

lldxxdxxx

0

2

0))((.)(.)( ,

de onde segue, integrando por partes, que

l ll dxxdxxxx0 0

220

))((.))(()(.)( ,

que devido a ( 7 ) nos fornece

0))((

))((

0

2

0

2

l

l

dxx

dxx ,

de onde concluímos que 0 .

Assim, escrevendo

2 , com 0 ,

a equação ( 6 ) fica

0)(.)( 2 xx ,

cuja solução geral é

)(.)cos(.)( 21 xsencxcx .

Impondo a condição 0)0( obtemos 01c , e para conseguirmos uma solução não-

nula tomaremos 02c , por exemplo l

c 22 se desejarmos que

ldxx

0

2 1)( . Assim,

Page 72: Numero 09 - Outubro de 2007

)(.2)( xsenl

x , e para que tenhamos também 0)(l , deveremos impor 0)( lsen ,

ou seja:

Znl

n : .

Concluindo: para cada número natural n, o problema de autovalores ( 8 ) tem uma auto-função

lxnsen

lxsen

lx nn .2)(.2)( ,

associada ao auto-valor

2

222

ln

nn .

Determinando uma solução não-nula de ( 5 ) com n obtemos, por exemplo,

2

22

)( ltn

n et .

Conseqüentemente obtemos, para cada número natural n, uma função

lxnsen

letxu l

tn

n .2.),( 2

22

,

a qual satisfaz a equação diferencial ( 1 ) e também a condição de contorno ( 2 ).

É lógico que qualquer combinação linear finita

N

nnn txuatxu

1

),(.),(

das funções ),( txun ainda satisfaz ( 1 ) e ( 2 ), e portanto será uma solução de ( P1 ) desde que

N

nnn

N

nn xa

lxnsen

laxf

11)(..2.)( .

Entretanto, quando f não tem a forma acima, podemos partir para as combinações lineares infinitas das funções nun :{ }, o que ampliará enormemente o conjunto das funções f para as quais o problema ( P1 ) tem solução. Negligenciando a questão da

Page 73: Numero 09 - Outubro de 2007

convergência e da derivação termo a termo – pois já assumimos uma abordagem não rigorosa neste texto – obtemos uma função

( 9 )11

)(..),(.),( 2

22

nn

ltn

nn

nn xeatxuatxu ,

que satisfaz a equação (1) e a condição de contorno (2), quaisquer que sejam as escolhas das constantes na .

Entretanto, esta ),( txu somente será solução de ( P1 ) se tivermos também

( 10 ) )0,()( xuxf

ou seja,

( 11 )1

)(.)(n

nn xaxf .

No que segue veremos como devem ser escolhidas as constantes na para que (11) efetivamente ocorra. Para isso, introduziremos a notação

( 12 )l

dxxgxfgf0

)(.)(, ,

a qual está bem definida quando f e g são seccionalmente contínuas no intervalo ],0[ l , e observamos que

( 13 )1,1

,0,

nmsenmse

nm .

Assim, supondo que uma dada função f possa ser escrita na forma (11) e que a integração termo a termo também possa ser realizada, teremos:

1.

nnnaf

1,.,

nmnnm af

1,.

nmnna mmmm aa ,. ,

e conseqüentemente

( 14 )l

nnn dxxxffa0

)()(, , 1n .

Page 74: Numero 09 - Outubro de 2007

Conforme se pode ver em [1] ou [2], quando f e f são seccionalmente contínuas, então a série (11), cujos coeficientes na são dados por (14), converge para a média aritmética dos limites laterais de f no ponto x, isto é,

( 15 )1

)(.2

)0()0(n

nn xaxfxf ,

onde )0(xf e )0(xf denotam, respectivamente, os limites laterais de f à direita e à esquerda no ponto x. Ou seja, a série (11), também denominada Série de Fourier de Senos da função f , não “privilegia” nenhum dos limites laterais de f no ponto x, convergindo “democraticamente” para a média aritmética de ambos. Em particular, quando x é um ponto de continuidade de f , temos )()0()0( xfxfxf , e portanto

1 1.2.)()(

n nnnn l

xnsenl

axaxf .

Para ilustrar graficamente este resultado consideraremos 6L e a função Rf ]6,0[: definida por

]6,3[,3)3,0[,

)(xsexxsex

xf .

Em cada sistema de coordenadas abaixo vemos os gráficos de f em azul e o gráfico da

N-ésima soma parcial, N

nnN L

xnsenaxS1

.)( , para 5N , 10N e 20N , em

vermelho, onde se pode intuir que, de fato, 2

)0()0()( xfxfxS N quando N .

Page 75: Numero 09 - Outubro de 2007

Se exigirmos ainda que Rlf ],0[: seja contínua e 0)()0( lff , então a série em ( 9 ) converge uniformemente para uma função ),( txu que é contínua em ),0[],0[ l ,de classe C em ),0(),0( l , e resolve o problema ( P1 ), conforme demonstrado em [2].

Abaixo vemos duas vistas do gráfico da 10ª soma parcial ),(10 txS para 40 x e 20 t da série-solução do problema ( P1 )

11.2..),(.),( 2

22

n

ltn

nn

nn lxnsen

leatxuatxu ,

no caso em que o comprimento da barra é 4L e a distribuição inicial de temperaturas é 24)( xxxf .

Page 76: Numero 09 - Outubro de 2007

3 – Uma interpretação algébrica do parágrafo anterior:

Observe que as funções nn :{ }, obtidas no parágrafo anterior são auto-funções do operador

)],0[()],0[(: lClCDT ,

definido no subespaço

}0)()0(:),0()],0[({ 2 llClCD ,

e que associa a cada D a sua derivada segunda

)(T .

Page 77: Numero 09 - Outubro de 2007

Além disso, se considerarmos em )],0[( lC o produto interno

( 16 )l

dxxgxfgf0

)(.)(, ,

então podemos afirmar que

nn :{ } é ortonormal em relação ao produto interno < , >

conforme indica as relações obtidas em (13).

Mais ainda, dadas e D teremos, mediante integração por partes, que

llldxxxxxdxxxT

000)().()().()().(,,)( ,

e como D temos 0)()0( l , de onde segue que

( 17 )l

dxxxT0

)().(,)( .

De forma análoga, obtém-se

( 18 )l

dxxxT0

)().()(, .

o que nos leva a concluir que

)(,,)( TT , D, ,

ou seja:

O operador T é auto-adjunto com relação ao produto interno < , >

O que fizemos na sessão anterior nos permite afirmar que o espaço vetorial )],0[( lCpossui uma “base” ortonormal nn :{ } composta de auto-funções do operador auto-adjunto T. Ou seja, produzimos um exemplo de um operador linear auto-adjunto T definido num espaço vetorial de dimensão infinita para o qual a conclusão contida no Teorema Espectral da sessão 1 se verifica.

A pergunta natural que fica é a seguinte:

Page 78: Numero 09 - Outubro de 2007

Este exemplo é apenas uma coincidência, ou a conclusão do Teorema Espectral 1 é sempre verdadeira quando tivermos um operador linear auto-adjunto T definido num espaço com produto interno H ?

Sem nenhuma hipótese adicional a resposta é não. Entretanto, é possível provar o seguinte resultado:

Sejam H um espaço de Hilbert e T:H H um operador linear compacto. Então existem uma seqüência

Nkk de autovalores do operador T e uma

seqüência ortonormal Nkk H tal que kkkT .)( para todo natural k .

Este afirmação é parte de um resultado mais geral conhecido como

Teorema Espectral para Operadores Compactos Auto-Adjuntos em Espaços de Hilbert.

Observe entretanto os novos adjetivos que apareceram no nome do teorema: não se trata mais de qualquer operador linear auto-adjunto, mas os compactos. Mais ainda, não basta estarmos ambientados num espaço com produto interno, mas num espaço que seja completo na norma proveniente deste produto interno. Ou seja, questões topológicas que não apareciam no caso finito-dimensional agora entram em cena e, na maioria das vezes, constituem a parte mais difícil de ser contornada no problema. No que segue citaremos um exemplo concreto que põe em evidência a importância dos estudos abstratos realizados nas disciplinas Topologiados Espaços Métricos, Topologia Geral, Análise no Rn, Análise Funcional, Teoria da Medida– incluindo aí a integral de Lebesgue – , dentre outras.

4 – O problema da condução do calor em dimensão maior que 1:

O análogo do problema ( P1 ) consiste em determinar uma função real ),( txu , com ),...,( 1 nxxx Rn , 0t , satisfazendo:

( P2 )

xxfxu

txtxu

txtxutxtu

,)()0,(

0,,0),(

0,,),(),(

,

onde é o Laplaciano nas variáveis espaciais nxx ,...,1 , isto é, n

i ixuu

12

2

.

Procurando soluções não-nulas no formato

)(.)(),( txtxu ,

com R: e R),0[: obtemos

( 19 ) )(.)( tt , 0t

Page 79: Numero 09 - Outubro de 2007

e

( 20 )xxxxx

,0)(,)(.)(

.

Note que (20) é um problema de autovalores para o operador Laplaciano e, diferente de (8), trata-se de uma equação diferencial parcial com uma condição de contorno na fronteira de um domínio do Rn, e não na fronteira de um intervalo ],0[ l , que se reduz a dois pontos. Se o domínio não apresentar algum tipo de simetria, fica difícil achar soluções explícitas para (20), conforme fizemos na sessão 2 para o problema (8).

Entretanto, se para funções de quadrado integrável definidas em colocarmos

( 21 ) dxxgxfgf )().(,

então cabe perguntar se o operador )()(T é auto-adjunto em relação a < , > . Para isso consideraremos T no domínio }0:)()({ 12 CCD .Dadas D, e usando o Teorema da Divergência com o campo vetorial

xxxxF ,)().()( , obtemos

dSxn

xdxxxdxxx )().()().()().( ,

e como a última integral é nula ( pois se anula em ) ficamos com

dxxxdxxx )().()().( ,

ou seja,

dxxxT )().(,)( .

De maneira análoga, se considerarmos o campo de vetores )().()( xxxF ,obteremos

dxxxT )().()(, ,

o que nos leva a concluir que

DTT ,,)(,,)( ,

ou seja, que T é auto-adjunto com respeito ao produto interno (21).

Page 80: Numero 09 - Outubro de 2007

A questão da compacidade é um tanto mais complicada. Como o operador )()(T não é compacto, trabalha-se na tentativa de obter um domínio D contido num

espaço de Hilbert H de modo que o operador T : D H H fique inversível, com inverso 1 : H H compacto. Como o inverso é automaticamente auto-adjunto, encontramo-nos nas

condições do Teorema Espectral, e daí podemos afirmar que existirá uma seqüência Nkk )(de números reais e uma seqüência Nkk )( de funções do espaço H tais que kkk )(1

para todo k . Mais ainda, o conjunto Nkk : é ortonormal com relação ao produto interno < , >. Assim, para cada número natural k encontramos uma auto-função k do operador :

kkk )( , onde k

k1 para todo k .

Agora, considerando uma solução de (19) com k no lugar de obtemos

tk

ket)( ,

e conseqüentemente, para cada natural k, a função )(.),( xetxu kt

kk satisfará a equação do

calor, pois

)(.)()(.)(),( txtxt

txut kkkkk

),()().()(.)()(.)(. txutxtxtx kkkkkkkk ,

e também a condição de contorno

0)().(),( txtxu kkk , para todo x ,

já que todas as funções k se anulam na fronteira de .

A candidata natural a solução do problema ( P2 ) é a função

( 22 )1

)(..),(k

kt

k xeatxu k ,

onde as constantes ka são escolhidas de modo que )()0,( xfxu , ou seja, de modo que

( 23 )1

)(.)(k

kk xaxf .

Como o conjunto Nkk : é ortonormal, obtém-se

Page 81: Numero 09 - Outubro de 2007

( 24 ) kk fa , , para todo k.

É possível provar, veja [3], que:

Se 2Cf e 0f , então as séries (22) e (23) convergem uniformemente para f e para uma solução u(x,t) do problema

xxfxutxuu

CCu

t

,)()0,(),0(),(,

),0[),0(2

,

desde que R3 tenha fronteira de classe 2C .

5 – Considerações Finais :

Conforme pretendíamos, é possível observar que muitas das questões abstratas consideradas como objetos de estudo na Análise e na Topologia emergem naturalmente de problemas concretos. Vimos por exemplo que o Teorema Espectral, numa versão em dimensão infinita, rende soluções para o problema da condução do calor em domínios do espaço Rn cuja fronteira seja suficientemente regular. Convém observar também que, sendo o nosso texto apenas motivador, não detalhamos como são as funções do espaço H no qual procuramos as auto-funções :k R . Apenas “acenamos” com uma possibilidade de trabalhar com um espaço de funções que possuem quadrado integrável em , de modo que o produto interno (21) de duas tais funções estivesse bem definido. Entretanto, para que este espaço - usualmente denotado por )(2L - resulte completo, é necessário trabalhar com uma noção de integral mais geral que a de Riemann. Trata-se da integral de Lebesgue, da qual um estudo aprofundado consome boa parte de um curso de Teoria da Medida. A noção de compacidade e suas caracterizações ocupam, por sua vez, uma parte significativa de uma disciplina de Topologia. Mais ainda, a questão da compacidade de um operador definido entre espaços de funções demandam o estudo de desigualdades não triviais conhecidas como Desigualdades de Sobolev. Esperamos com este texto ter conscientizado o leitor da importância do estudo de tópicos abstratos, principalmente aqueles alunos de cursos de Matemática que estão em vias de fazer sua opção entre Licenciatura ou Bacharelado. Claro que esta é uma das várias motivações e, dependendo do gosto pessoal do leitor, ela pode até mesmo ser desmotivadora. O que nos interessa, entretanto, é fazer o leitor entender que os objetos de estudo da Matemática Pura não estão desvinculados dos problemas reais. Ainda que um tanto sofisticadas, as teorias matemáticas estão por trás de uma série de situações do cotidiano que um cidadão comum sequer pode imaginar. Essa não consciência por parte de uma maioria esmagadora não invalidam e nem devem desencorajar a pesquisa matemática. Se o texto serviu, pelo menos, para diminuir o preconceito de muitos para com a Matemática Pura, já nos damos por satisfeitos.

Page 82: Numero 09 - Outubro de 2007

6 – Bibliografia:

[1] Figueiredo, Djairo G. de; Análise de Fourier e Equações Diferenciais Parciais .

[2] Iório, Valéria; EDP: Um Curso de Graduação. Rio de Janeiro; Instituto de Matemática Pura e Aplicada; CNPq, 1991. Coleção Matemática Universitária.

[3] Iório Jr, R. J. & Iório, Valéria; Equações Diferenciais Parciais: Uma Introdução. Rio de Janeiro, Instituto de Matemática pura e Aplicada, CNPq, 1988. Projeto Euclides.

Page 83: Numero 09 - Outubro de 2007

PERFIL SÓCIO-ECONÔMICO DOS CANDIDATOS DO PAIES/UFU: SUBPROGRAMA 2002-20051

KÁTIA ALESSANDRA DE SOUZA CAETANO2, EDNALDO CARVALHO GUIMARÃES3, ROGÉRIO DE MELO COSTA PINTO4, MARCELO TAVARES4

RESUMO

A avaliação seriada para a admissão de alunos no ensino superior público sempre foi alvo de controversas em sua eficácia e como substituta dos vestibulares, e não é incomum o número de pesquisadores que criticam esta forma de processo seletivo que algumas universidades federais do país têm adotado. A Universidade Federal de Uberlândia é uma dentre estas universidades que utilizam este tipo de avaliação como forma de ingresso de seus candidatos, sendo o PAIES (Programa Alternativo de Ingresso ao Ensino Superior) o processo seletivo seriado da UFU que avalia os candidatos ao longo do Ensino Médio. A presente pesquisa propõe a exploração do banco de dados do PAIES da COPEVE/UFU (Comissão Permanente de Vestibular), objetivando analisar os dados socioeconômicos - culturais dos candidatos que foram aprovados e reprovados durante a etapa de 2002-2005, buscando analisar o perfil dos estudantes que obtém êxito no processo seletivo em questão, e verificar se existem diferenças sócio demográficas entre os candidatos aprovados e reprovados. Foi utilizada a estatística descritiva e o software SPSS 13.0 for Windows. Os resultados obtidos indicam diferenças entre os candidatos, sendo que a maioria dos alunos aprovados reside na cidade de Uberlândia; possuem maior renda econômica mensal familiar e acesso a computadores e internet; fizeram a maior parte ou toda formação de 2º grau em escolas particulares; e a escolaridades de seus pais é maior que a dos pais dos alunos reprovados. Já entre estes candidatos, há um número maior de estudantes que trabalham, e verificou-se também que há um número maior de estudantes das etnias negra, indígena e outras que não a branca, que são reprovados no processo seletivos seriado da UFU.

Palavras-chave: Avaliação seriada; dados sócio-demográficos; PAIES; estatística descritiva.

1. INTRODUÇÃO

Nos últimos anos as universidades federais têm buscado formas alternativas de ingresso ao ensino superior. Programas que consideram a nota recebida por alunos do Exame Nacional do Ensino Médio (ENEM) e os programas de exame seriado foram algumas alternativas apresentadas pelas universidades federais.

1 Pesquisa de iniciação científica (PROMAT/FAMAT/UFU) desenvolvida no período de novembro de 2006 a agosto de 2007. 2 Acadêmica do curso de Psicologia - Av. João Naves de Ávila, 2160, Bairro Santa Mônica, Uberlândia – MG, CEP: 38400-900- [email protected] 3 Prof. Orientador – FAMAT/UFU – Av. João Naves de Ávila, 2160, Bairro Santa Mônica, Uberlândia – MG, CEP: 38400-900 – [email protected] Prof. Colaborador – FAMAT/UFU

Page 84: Numero 09 - Outubro de 2007

O primeiro programa de exame seriado das universidades federais foi instituído pela Universidade de Brasília (UnB), no ano de 1996 (UnB, 2007).

O PAIES – Programa Alternativo de Ingresso ao Ensino Superior – aprovado pelo CONSEP – Conselho Superior de Ensino e Pesquisa da UFU – representa uma nova modalidade de avaliação gradual e sistemática, e constitui-se como processo alternativo para o aluno do Ensino Médio na conquista de uma vaga na UFU. No PAIES, em vez de fazer somente uma prova para conseguir uma vaga no ensino superior (como ocorre no vestibular), os estudantes do ensino médio interessados fazem uma prova ao fim de cada ano, somando o resultado das três no fim do 2º grau. Os que obtiverem notas mais altas são classificados e garantem assim, o direito a uma vaga na Universidade Federal de Uberlândia (UFU, 2007).

São destinados ao PAIES 25% do total das vagas anuais da Universidade; e 2007 marca o décimo ano no qual a prova é realizada, pois em 1.997 o PAIES executou a primeira etapa do programa, com a participação de 9.336 candidatos inscritos, oriundos de 190 diferentes municípios brasileiros e processou o credenciamento de 403 escolas do Ensino Médio. Nota-se que o programa possui grande importância na avaliação do ensino da região, devido á relevante amplitude das escolas credenciadas e diferentes municípios que participam do processo.

É notório, portanto, que os dados sócio-econômicos e o desempenho dos alunos nas mais diversas disciplinas, são uma importante fonte de análises estatísticas e inferências não só para a UFU, mas para a comunidade em geral, principalmente para aqueles preocupados com a qualidade do ensino médio, e porque não dizer, do ensino fundamental na região.

Autores como Pantaleo Junior (2005), Frois e Barreto (2005), Franco (2006), Rodrigues (2006), Sobral e Oliveira (2006), Veiga et al (2007) apresentam aspectos de descrição estatística de processos seletivos.

O presente trabalho visou analisar, através do banco de dados do PAIES da COPEVE/UFU, os dados sócio-demográficos dos candidatos do PAIES 2002/2005 e compará-los entre os candidatos que foram aprovados e reprovados, objetivando verificar se existem diferenças sócio-demográficas entre os sujeitos que obtiveram êxito no processo seletivo e os que não obtiveram. Verificar e identificar algumas características sócio-demográficas que poderiam estar relacionadas com o desempenho exitoso do aluno, e analisar e compreender sobre qual o perfil de estudante que tem sido aprovado pelo PAIES na Universidade, são também objetivos da pesquisa.

2. MATERIAL E MÉTODOS

A amostra utilizada na pesquisa foi retirada do banco de dados da Comissão Permanente de Vestibular (COPEVE/UFU), e é composta por 200 candidatos que responderam ao questionário socioeconômico-cultural durante a inscrição da terceira etapa do PAIES 2002-2005, sendo que 99 foram reprovados e 101 aprovados. As perguntas do questionário utilizadas na pesquisa foram: Qual o seu sexo? ; Qual sua idade em 31 de dezembro de 2004? ; Qual seu estado civil? ; Onde você reside? ; Você reside? ; Você se considera? ; Qual sua religião? ; Você exerce atividade remunerada? ; Em que faixa melhor se enquadra a renda bruta mensal (sem descontos) de seu grupo familiar (soma dos rendimentos dos seus pais, irmãos, conjugue, filhos, etc.)? ; Nível de instrução do seu pai? ; Nível de instrução da sua mãe? ; Indique o principal responsável pelo sustento da sua família? ; Possui computador em sua residência? ; Meio de transporte que você mais utiliza? ; Pretende trabalhar enquanto faz curso superior? ; Você mora? ; Qual é a ocupação principal exercida pelo seu pai? ;

Page 85: Numero 09 - Outubro de 2007

Qual é a ocupação principal exercida pela sua mãe? ; Qual a sua principal fonte de informações sobre os acontecimentos atuais? ; Dos tipos de revistas citadas abaixo, qual você mais lê? ; Onde você cursou o ensino fundamental? ; Onde você cursou, integralmente ou em sua maior parte, o Ensino Médio? ; O que o levou a escolher seu curso? ; O que você espera em primeiro lugar de um curso universitário? ; e Qual a razão principal que o levou a escolher a UFU? . Foi utilizado o software SPSS 13.0 for Windows, e a estatística escolhida para obter os resultados e comparações entre os candidatos aprovados e reprovados, foi a estatística descritiva conforme procedimentos encontrados em Arango (1999), Triola (1999), Bussab e Morettin (2002). Foi realizado também um teste de hipóteses para diferença entre duas proporções, objetivando a averiguação de diferenças entre freqüências. Outro software utilizado foi o Microsoft Office Excel 2003, para a montagem de gráficos.

3. RESULTADOS E DISCUSSÃO

A primeira variável analisada foi o sexo, e observou-se que a mesma não intervém na reprovação ou aprovação do candidato, pois não houve diferenças significativas entre o número de mulheres e homens reprovados e aprovados (do total de candidatos reprovados, 52,8% eram mulheres e 47,2% eram homens; e do total de candidatos aprovados, 52,6% eram mulheres e 47,4% eram homens).

Em relação ao estado de origem, 77,5% dos candidatos reprovados residem no estado de Minas Gerais; 15,7% no estado de São Paulo; 4,5% no estado de Goiás; 1,1% no Distrito Federal (Brasília) e 1,1% no Mato Grosso do Sul. Entre os aprovados, 89,7% residem em Minas Gerais e 9,3% no estado de São Paulo. Nota-se uma maior heterogeneidade em relação ao estado de origem entre os candidatos reprovados, do que entre os candidatos aprovados. É interessante notar que grande parte dos candidatos aprovados (72,2%) reside na cidade de Uberlândia.

Os estudantes têm em média 17 anos (62,9% dos reprovados e 73,2% dos aprovados), e nota-se que há um número maior de candidatos reprovados com idade superior a 18 anos (37%), do que os candidatos aprovados com essa faixa etária (26,8%). Enfim, observa-se em geral que entre os aprovados no processo seletivo, há uma maior correspondência entre a faixa etária e o nível de escolaridade adequado, fato que não ocorre tão frequentemente entre os candidatos reprovados, que possuem maior discrepância entre a faixa etária e o nível de escolaridade correto.

Em relação á etnia, 62,9% dos reprovados se considera brancos, contrastando com os 77,3% que se declaram pertencer a este grupo étnico, entre os candidatos aprovados. Foi realizado um teste de hipótese para diferença entre duas proporções onde se encontrou um p-valor (significância) de 0,0272. Nota-se, portanto que há diferença significativa entre o número de sujeitos que se declaram brancos e outras etnias (como pardo, negro, indígena e amarelo) que ingressam no processo seletivo. Entretanto não se deve considerar etnia como uma variável causal pelo ingresso ou não no processo seletivo em questão, e sim a outras variáveis intervenientes como renda mensal, tipo de ensino freqüentado durante o ensino médio, etc. Deve-se recordar também, que etnia é uma variável subjetiva, ficando à percepção individual, a “cor” que cada candidato possui, podendo ou não a mesma corresponder a realidade. O estado civil de praticamente 100% da amostra é solteiro.

Já em relação á religião, obteve-se resultados semelhantes entre os candidatos aprovados e reprovados: a maioria se declarou cristãos (71% dos reprovados e 80,4% dos aprovados).

Page 86: Numero 09 - Outubro de 2007

Sobre o exercício de atividade remunerada, observa-se que entre os candidatos reprovados, há um número maior de sujeitos que praticam algum tipo de trabalho, do que entre os candidatos aprovados (65% dos reprovados não praticam atividade remunerada, contra 85% dos aprovados, que alegam não exercer tal tipo de atividade).

A dinâmica familiar dos candidatos também foi analisada, e observou-se que em 58,4% dos lares dos candidatos reprovados, o pai é o único responsável pelo sustento familiar, contrastando com os 48, 5% dos alunos aprovados; indicando que nos lares dos estudantes que não obtiveram êxito no processo seletivo, o padrão familiar mais comum é o do chefe de família que sustenta a casa. Observou-se também que praticamente todos os candidatos moram com família, em imóvel próprio (82% dos reprovados e 79,4% dos aprovados residem em imóvel próprio).

Em relação à renda mensal familiar dos candidatos reprovados, obtiveram-se resultados distintos dos candidatos aprovados, pois 62,9% de tais candidatos possuem renda mensal de até R$ 1820 (estando a classe modal entre R$ 261 até R$ 780), e já entre os candidatos aprovados, apenas 41,2% tem renda familiar semelhante a esta quantia. Entre os aprovados, 58,7% têm renda mensal superior a R$ 1.821 e a classe modal encontra-se entre os valores de R$ 2.601 até R$ 5.200. O que se pode inferir através da análise frequencial dos dados e da classe modal, é que, no geral, os candidatos aprovados e seus familiares vivem com rendimentos mensais médio a alto, ganhando quantias entre 7 a 30 ou mais salários mínimos (em relação ao salário mínimo de R$ 260 do ano de 2004); e os candidatos reprovados e seus familiares vivem com renda mensal baixa a média, ganhando quantias entre menos de 1 e até 7 salários mínimos. Obviamente, a distribuição gráfica desta variável é distinta entre os dois tipos de candidatos, conforme pode ser observado na Figura 1:

Renda mensal familiar dos candidatos aprovados e reprovados

05

1015202530

Até R$260

EntreR$ 261

e R$780

EntreR$ 781

e R$1380

EntreR$

1381 eR$

1820

EntreR$

1821 eR$

2600

EntreR$

2601 eR$

5200

EntreR$

5201 eR$

7800

Acimade R$7800

Renda Mensal

Freq

uênc

ia

AprovadosReprovados

Figura 1. Renda familiar dos candidatos reprovados e aprovados.

Observou-se que a variável escolaridade dos pais também sofreu diferenças significativas ao se analisar os dados entre os aprovados e os reprovados. Em relação á estes, 77,5% dos pais e 66,3% das mães têm até o ensino médio completo, contrastando com a escolaridade dos 48,5% dos pais e 34% das mães dos aprovados que tem até o 2º grau completo. Entre estes candidatos (aprovados), nota-se que o nível de escolaridade mais freqüente entre os pais é de superior incompleto à pós-graduação completo (sendo

Page 87: Numero 09 - Outubro de 2007

51,5% dos pais e 66% das mães). Enfim, o que se observa é que entre os estudantes aprovados no processo seletivo, a escolaridade de seus pais é significativamente maior que a escolaridade dos pais dos reprovados no processo. Entretanto, um dado recorrente entre ambos os candidatos foi o maior nível de instrução das mães em relação aos pais, demonstrando que estas se preparam mais para o mercado de trabalho e tem maior formação educacional que seus parceiros. As Figuras 2 e 3 demonstram tanto a diferença entre o nível de instrução dos pais dos sujeitos aprovados e reprovados; quanto a sobrepujança do nível de escolaridade das mães sobre os

pais:

Nível de escolaridade entre os pais dos candidatos reprovados

05

10152025303540

Não

alfa

betiz

ado

Fund

amen

tal

inco

mpl

eto

Fund

amen

tal

com

plet

o

Méd

ioin

com

plet

o

Méd

ioco

mpl

eto

Sup

erio

rin

com

plet

o

Sup

erio

rco

mpl

eto

Pós

-gr

adua

ção

inco

mpl

eto

Pós

-gr

adua

ção

com

plet

o

Nível de instrução

Freq

uênc

ia

PaiMãe

Figura 2. Nível de escolaridade entre os pais dos candidatos reprovados.

Nível de escolaridade entre os pais dos candidatos aprovados

05

101520253035

Não

alfa

betiz

ado

Fund

amen

tal

inco

mpl

eto

Fund

amen

tal

com

plet

o

Méd

ioin

com

plet

o

Méd

ioco

mpl

eto

Supe

rior

inco

mpl

eto

Supe

rior

com

plet

o

Pós-

grad

uaçã

oin

com

plet

o

Pós-

grad

uaçã

oco

mpl

eto

Nível de instrução

Freq

uênc

ia

PaiMãe

Figura 3. Nível de escolaridade entre os pais dos candidatos aprovados.

Observam-se também diferenças entre as profissões e tipo de atividades remuneradas mais exercidas pelos pais dos candidatos reprovados e aprovados, estando entre os candidatos aprovados, maiores porcentagens em atividades que exigem formação superior e especializada, e entre os reprovados, freqüência maior em atividades mais subalternas, que não exigem um alto grau de formação acadêmica ou especializada. As Figuras 4 e 5 permitem a diferenciação entre o tipo de trabalho exercido pelos pais dos estudantes aprovados e reprovados, e a Tabela 1 permite a diferenciação entre as profissões:

Page 88: Numero 09 - Outubro de 2007

Atividade remunerada exercida pelos pais dos reprovados

0

10

20

30

40

50

1 2 3 4 5 6

Profissões

Freq

uênc

ia

PaiMãe

Atividade remunerada exercida pelos pais dos aprovados

0

10

20

30

40

50

1 2 3 4 5 6

Profissões

Freq

uênc

ia

PaiMãe

Figura 4. Atividade remunerada exercida pelos Figura 5. Atividade remunerada exercida pelospais dos reprovados pais dos aprovados

Tabela 1. Diferenciação das profissões.

Profissões1 Banqueiro; deputado; senador; diplomata; capitalista; grande industrial; grande

proprietário rural, (olhar no manual e completar as profissões que faltam)2 Profissionais liberais de nível universitário, cargo técnico-científico, cargo de chefia

ou gerência em empresa de porte médio, posto militar; grande comerciante; dono de propriedade rural de 200 a 2000 hectares e outras ocupações semelhantes.

3 Bancário; oficial de justiça, professor do Ensino Médio e Fundamental; despachante; representante comercial; auxiliar administrativo e de escritório; posto militar de sargento, subtenente e equivalentes; pequeno industrial; comerciante médio; proprietário rural de 20 a 200 hectares, e outras ocupações com características semelhantes.

4 Datilógrafo; telefonista; mecanógrafo (olhar no manual e completar as profissões que faltam)

5 Operário não-qualificado; servente; operador; (olhar no manual e completar as profissões que faltam)

6 Dona de casa .

Em relação a possuir ou não computador em casa, observa-se que o número de candidatos aprovados que possuem computador em suas residências é significativamente maior que o número de candidatos reprovados que possuem o aparelho (77,3% dos aprovados têm computador em casa, sendo que destes, 70,1% tem acesso á internet; contra 53,4% dos reprovados que possuem o aparelho em seus domicílios, sendo que destes, apenas 35,2% possuem acesso á internet.). Fica claro que os candidatos reprovados enfrentam maiores barreiras e dificuldades de acesso á informação digital, haja vista que, o universo virtual tem a primazia da fonte de informações do mundo contemporâneo globalizado. Portanto, sobre o adquirir informações e conhecimentos que possam ser úteis e facilitadores na aprovação do processo seletivo em questão, e posterior entrada no universo acadêmico, os candidatos reprovados ficam aquém dos candidatos aprovados.

Page 89: Numero 09 - Outubro de 2007

A fonte de informações prevalente sobre os acontecimentos atuais tanto entre aprovados, quanto em reprovados, é o telejornal, sendo lembrado por 76,4% dos reprovados e 53,6% dos aprovados. As revistas e a internet são as segundas fontes de informações, sendo que entre os aprovados, 17,5% se informam através de revistas e 17,5% através da internet; e entre os reprovados, 11,2% utilizam revistas para manter-se informados e 6,7% usam a internet. Outras fontes como, jornal escrito e o jornal falado (rádio) também foram citadas. Ainda em relação à fonte de conhecimentos atuais, observa-se que as revistas do tipo informativa (Veja, Exame, etc.) são as mais lidas, tanto por candidatos aprovados (71,1%), quanto por candidatos reprovados (68,5%).

Em relação ao tipo de ensino (se público ou privado) mais frequentemente cursado pelos candidatos, notou-se que entre os aprovados, tanto no que tange ao ensino fundamental, quanto ao ensino médio, há um número maior de candidatos que cursaram todo ou a maioria do ensino em escolas particulares; dado que não ocorreu entre os candidatos reprovados, que tiveram sua formação educacional realizada de forma mais recorrente em escolas públicas. O ensino fundamental entre os candidatos que não obtiveram êxito no processo seletivo, foi cursado em escolas públicas por 66,3% dos alunos, e entre os que obtiveram sucesso, por 50,5%. Já em relação ao 2º grau, 50,6% dos candidatos reprovados cursaram todo ou a maioria do ensino médio em escolas públicas, e entre os aprovados, somente 20,6% tiveram sua formação média realizada na rede pública de ensino. A Figura 6 deixa claro a discrepância existente entre o tipo de ensino mais freqüentado pelos estudantes aprovados e reprovados (é interessante notar a prevalência do setor privado principalmente na educação e formação média dos aprovados):

0102030405060708090

100

1º Grau -Reprovados

1º Grau -Aprovados

2º Grau -Reprovados

2º Grau -Aprovados

Tipo de ensino (se público ou privado) utilizado pelos candidatos

Escolas privadas (todo ou a maiorparte)Escolas públicas( todo ou a maiorparte)

Figura 6. Tipo de ensino (se público ou privado) utilizado pelos candidatos.

As expectativas dos candidatos em relação ao curso superior, vida acadêmica e a escolha da UFU também foram analisadas e constatou-se que a maioria dos estudantes escolheu o curso superior pelo interesse que o mesmo desperta, independente se eram candidatos aprovados ou reprovados. Já quando questionados sobre o que esperam obter em primeiro lugar de um curso superior, o item “formação profissional voltada para o trabalho” foi o mais lembrado pelos candidatos, embora entre os reprovados, o número de estudantes que escolheram este item tenha sido muito maior (71,9%) do que entre os candidatos aprovados (53,6%). Pode-se hipotetizar que entre os candidatos reprovados, a aquisição de um curso superior e a utilidade do papel do 3º grau está mais atrelada à formação técnica voltada exclusivamente para o mercado de trabalho, e não, por exemplo, correlacionada à aquisição de conhecimentos e cultura em geral. A Figura 7 permite a comparação entre as expectativas dos candidatos aprovados e reprovados

Page 90: Numero 09 - Outubro de 2007

sobre um curso superior e também, o levantamento de algumas hipóteses como a suscitada anteriormente:

Diferenças entre as expectativas sobre um curso universitário entre aprovados e reprovados

0 20 40 60 80

Aquisição de cultura geral

Formação profissional voltada parao trabalho

Formação teórica voltada para apesquisa

Formação acadêmica voltada pramelhorar a atividade prática

Aquisição de conhecimentos queme permitam compreender melhor o

mundo

Aquisição de conhecimentos queme permitam melhorar minha

instrução

Outro

AprovadosReprovados

Figura 7. Diferenças entre as expectativas de um curso universitário entre candidatos reprovados e aprovados.

Já em relação à principal razão que levou os candidatos a escolher a UFU, nota-se uma grande heterogeneidade de respostas, sendo a mais freqüente, (38,1% dos aprovados e 42,7% dos reprovados) devido à qualidade do curso oferecido pela instituição.

Page 91: Numero 09 - Outubro de 2007

4. CONCLUSÕES

1) A maioria dos candidatos aprovados reside em Uberlândia ou em outras cidades do estado de Minas Gerais. Pode-se levantar a hipótese de que a maioria dos aprovados é da própria cidade de Uberlândia pelo fato das escolas da cidade terem o PAIES e a aprovação de seus alunos neste processo seletivo, como uma de suas metas educacionais a serem atingidas. Escolas de São Paulo, ou Goiás, por exemplo, poderiam não ver o processo seletivo seriado da UFU como importante prova de admissão para o ensino superior público, buscando contemplar a preparação de seus alunos para outras universidades que não a Universidade Federal de Uberlândia.

2) Há um número maior de candidatos que se declaram brancos que são admitidos no processo seletivo, indicando que um número menor de alunos de outras etnias, como a negra e a indígena são aprovados no PAIES. È óbvio, entretanto, que não se deve concluir que negros, indígenas e outras etnias, são menos inteligentes e capazes de serem aprovados em uma universidade pública de qualidade. O resultado estatístico obtido somente espelha as desigualdades sociais, culturais e econômicas, e a exclusão existente no país, principalmente para aqueles de outras etnias que não a branca. O que se pode observar através desta sobrepujança de etnia, são as raízes históricas de um país escravocrata e excludente como o Brasil, que durante séculos não delegou aos seus cidadãos iguais oportunidades de ascensão social.

3) Há um número maior de candidatos reprovados que trabalham, e também entre estes candidatos encontram-se os menores índices de renda econômica mensal familiar. Observa-se que os estudantes que não foram aprovados no processo seletivo, no geral, são de classe econômica inferior, e muitas das vezes precisam trabalhar, além de estudar, para ajudar na economia familiar, o que obviamente diminui as horas e a qualidade dedicada á preparação para o PAIES.

4) A escolaridade dos pais dos candidatos reprovados foi inferior em relação aos aprovados. Entre estes, há um número maior de pais que estão concluindo ou concluíram a graduação em um curso superior e/ou especialização. Ainda sobre esta variável (escolaridade dos pais), observou-se que tanto entre aprovados, quanto em reprovados, as mães possuem maior nível de escolaridade, demonstrando e confirmando os dados obtidos nos últimos censos realizados pelo IBGE (Instituto Brasileiro de Geografia e Estatística), de que as mulheres têm buscado maior formação acadêmica e especialização em relação aos homens, conquistando maiores e melhores espaços no mercado de trabalho. Contudo, observou-se também na pesquisa que mesmo as mães possuindo, no geral, maior formação educacional, elas ainda exercem atividades econômicas mais subalternas e inferiores do que os pais, que, no geral, possuem menor escolaridade. Estes dados demonstram a discriminação e exclusão de gênero que ainda existe no país, refletindo as desigualdades existentes nas relações de trabalho entre homens e mulheres.

5) Os candidatos reprovados enfrentam maiores dificuldades de acesso á informação do mundo contemporâneo, haja vista que o universo virtual tem a primazia da fonte dos meios de comunicação sobre os acontecimentos atuais. É conspícuo, portanto, que os candidatos reprovados sofrem sim a “exclusão digital”, ficando aquém dos candidatos aprovados no ter acesso aos conhecimentos atuais e também curriculares, que são úteis na aprovação no PAIES.

Page 92: Numero 09 - Outubro de 2007

6) Entre os candidatos reprovados, há uma prevalência do setor público na formação básica e média educacional, e pode-se hipotetizar que esta variável influenciou na aprovação do candidato no processo seletivo, haja vista que no atual sistema público educacional de 1º e 2º graus, o ensino encontra-se defasado e as condições ofertadas pelo Estado, em suas mais variadas instâncias, são precárias e muito aquém do desejado e necessário para uma educação de qualidade. Praticamente 80% dos candidatos que foram aprovados cursaram todo ou a maior parte do Ensino Médio em escolas particulares e fica claro o quão importante é a qualidade e a formação de 2º grau na aprovação do aluno.

7) A maioria dos candidatos (aprovados e reprovados) escolheram o curso ao qual concorreram a uma vaga na universidade, pelo interesse que esse desperta nos mesmos. Observou-se que entre os candidatos reprovados, a formação superior correlaciona-se mais à obtenção de formação profissional específica voltada para o mercado de trabalho, demonstrando que possivelmente para esses candidatos, a graduação é vista como uma ferramenta para a ascensão social e admissão no mercado de trabalho. Já entre os estudantes aprovados, o curso superior está além de correlacionado á formação para o trabalho, atrelado também à aquisição de cultura em geral, por exemplo; e a formação superior não é vista como útil somente para a ascensão ao mercado de trabalho, mas também como um meio de crescimento e amadurecimento pessoal.

5. REFERÊNCIAS BIBLIOGRÁFICAS

ARANGO, H. G. Bioestatística: Teórica e Computacional. Rio de Janeiro: Guanabara Koogan, 235p., 2001.

BUSSAB, W. O.; Morettin, P. A. Estatística Básica. 5ª edição. São Paulo: Saraiva, 2002, 526 p.. FRANCO, H. F. S. Avaliação do desempenho dos alunos aprovados e reprovados durante as três etapas. Monografia. Especialização em Estatística Aplicada, Faculdade de Matemática, Universidade Federal de Uberlândia. Uberlândia, 22 p., 2006.

FROIS, L. A.; BARRETO, C. L. Perfil sócio-econômico dos candidatos a ingresso em 2004 no mestrado profissionalizante em Ensino de Ciências Naturais e Matemática da UFRN. In: Simpósio Nacional de Ensino de Física. Anais do..., 16, 2005, 3 p..

PANTALEO JUNIOR, M.; TAKEUCHI, M. Y.; TEIXEIRA, R. R. P. Perfil dos alunos ingressantes em Licenciatura em Física do Centro Federal de Educação Tecnológica de São Paulo. In: Simpósio Nacional de Ensino de Física. Anais do..., 16, 2005, 3 p..

RODRIGUES, A. Análise dos resultados do PAIES em suas três etapas. Monografia. Especialização em Estatística Aplicada, Faculdade de Matemática, Universidade Federal de Uberlândia. Uberlândia, 17 p., 2006.

SOBRAL, D. T.; OLIVEIRA, P. G. Avaliação seriada versus exame vestibular: semelhanças e diferenças entre os coortes no curso de medicina da Universidade de Brasília. Revista Brasileira de Educação Médica. V. 30, n. 3, p. 181-191, 2006.

TRIOLA. M. F. Introdução à Estatística. 7. ed. Rio de Janeiro. 1999.

Page 93: Numero 09 - Outubro de 2007

UNIVERSIDADE DE BRASÍLIA - CESPE. Programa de Avaliação Seriada (PAS).Disponível em: www.cespe.unb.br/pas/oquepas/principios/principiospas.htm [Acesso em: 14/05/2007].

UNIVERSIDADE FEDERAL DE UBERLÂNDIA – COPEVE. Manual do PAIES.Disponível: http://www.ingresso.ufu.br/Paies2007/pdf/Manual_candidato_3etapa_2004-2007.pdf [Acesso em; 16/05/2007]

VEIGA, R. D.; LANCHOTE, L. N.; POMPEU, P. V.; SILVA, A. L. L. Relação entre desempenho no vestibular e em cursos de graduação – Universidade Federal de Lavras. In: Reunião Anual da Região Brasileira da Sociedade Internacional de Biometria (RBRAS) e Simpósio de Estatística Aplicada à Experimentação Agronômica. Anaisda..., 52, CD-ROOM, 2007, 5 p. (2007)

Page 94: Numero 09 - Outubro de 2007
Page 95: Numero 09 - Outubro de 2007

Um Texto Sobre Superfıcies ParametrizadasRegulares

Laıs Bassame Rodrigues∗ Edson Agustini†

Faculdade de Matematica - Famat

Universidade Federal de Uberlandia - Ufu - MG

Setembro de 2007

Resumo

Este trabalho e um texto sobre superfıcies parametrizadas regulares, assunto abor-dado nos cursos de Geometria Diferencial O texto avanca ate os Teoremas Egregiumde Gauss e Fundamental das Superfıcies, sendo que o primeiro esta demonstrado.

1 Superfıcies

1.1 Uma Pequena Revisao de Analise no Rn

SejaF : U ⊂ R

2 −→ R3

(u, v) �−→ F (u, v) = (F1(u, v) , F2(u, v), F3 (u, v))

sendo Fi : U ⊂ R2 −→ R; i = 1, 2, 3.

Dizemos que F e contınua quando cada Fi e contınua.Dizemos que F e de classe Ck quando cada Fi for de classe Ck, ou seja, quando existiremtodas as derivadas parciais de Fi ate a ordem k e todas forem contınuas.Dizemos que F : U ⊂ R

2 −→ R3 e diferenciavel em P = (u, v) ∈ U quando existir uma

aplicacao lineardFP : R

2 −→ R3

tal que, para todo w ∈ R2, temos F (w + P) = F (P)+dFP (w)+R (w) , sendo lim

w→0

R (w)

|w|=

0.

A demonstracao da Proposicao 1 abaixo pode ser encontrada em livros de Analise Realcomo, por exemplo, [5].

Proposicao 1. Se F : U ⊂ R2 −→ R

3 e diferenciavel em P ∈ U, entao

dFP (w) = limt→0

F (P + tw) − F (P)

t

[email protected] Orientanda Pet Faculdade de Matematica - jan/06 a dez/06.†[email protected] Professor orientador.

Page 96: Numero 09 - Outubro de 2007

para todo w ∈ R2.

Seja B = {e1, e2} base canonica do R2. Temos:

dFP (e1) = limt→0

F (P + te1) − F (P)

t

= limt→0

F (u0 + t, v0) − F (u0, v0)

t

=

(∂F1

∂u(P) ,

∂F2

∂u(P) ,

∂F3

∂u(P)

)= Fu (P)

sendo P = (u0, v0) .

Analogamente:

dFP (e2) =

(∂F1

∂v(P) ,

∂F2

∂v(P) ,

∂F3

∂v(P)

)= Fv (P) .

Assim, a matriz de dFP em relacao a base B e dada por:

JF (P) =

⎡⎢⎢⎢⎢⎢⎢⎢⎢⎣

∂F1

∂u(P)

∂F1

∂v(P)

∂F2

∂u(P)

∂F2

∂v(P)

∂F3

∂u(P)

∂F3

∂v(P)

⎤⎥⎥⎥⎥⎥⎥⎥⎥⎦e e a chamada matriz jacobiana de F em P.

1.2 Superfıcies Parametrizadas

Uma superfıcie parametrizada diferenciavel e uma aplicacao

S : U ⊂ab

R2 −→ R

3

(u, v) �−→ (x (u, v) , y (u, v) , z (u, v))

diferenciavel, sendo U um subconjunto aberto e conexo do R2.

(u,v)

S(u,v)

S

S(u,v) = (x(u,v),y(u,v),z(u,v))

U

R2

R3

u

v

x

y

z

Page 97: Numero 09 - Outubro de 2007

Alem disso, quando dSP e injetora para qualquer P = (u, v) ∈ U, dizemos que S e regular.

Como S (u, v) = (x (u, v) , y (u, v) , z (u, v)) , temos x, y, z : U ⊂ R2 −→ R como funcoes

componentes (ou coordenadas) de S.

Seja

Su : R2 −→ R

3

P �−→ (∂x

∂u(P) ,

∂y

∂u(P) ,

∂z

∂u(P)

).

Indicaremos Su =

(∂x

∂u,∂y

∂u,∂z

∂u

). Analogamente, Sv =

(∂x

∂v,∂y

∂v,∂z

∂v

).

Proposicao 2. Seja S superfıcie parametrizada diferenciavel. Entao:

dSP e injetora(1)⇐⇒ os vetores Su (P) e Sv (P) sao linearmente independentes

(2)⇐⇒ Su (P)×Sv (P) �= −→

0(3)⇐⇒ JS (P) tem posto 2.

Demonstracao.

De (1):

Recordemos que Su (P) = dSP (e1) e Sv (P) = dSP (e2) .

=⇒) Suponhamos que ∃α1, α2 ∈ R tais que α1Su (P)+α2Sv (P) = 0. Seja w = (α1, α2) =

α1e1 + α2e2 ∈ R2. Logo,

dSP (w) = dSP (α1e1 + α2e2) = α1dSP (e1) + α2dSP (e2) = α1Su (P) + α2Sv (P) = 0.

Como dSP e injetora temos que

w = 0 =⇒ α1e1 + α2e2 = 0 =⇒ α1 = α2 = 0,

ou seja, Su (P) e Sv (P) sao linearmente independentes.⇐=) Seja w ∈ ker (dSP) . Entao:

dSP (w) = 0 =⇒α1dSP (e1) + α2dSP (e2) = 0 =⇒

α1Su (P) + α2Sv (P) = 0 =⇒α1 = α2 = 0

(pois Su (P) e Sv (P) sao linearmente independentes)

Assim, w = 0. Logo, ker (dSP) = {0} , ou seja, dSP e injetora.

Demonstracao de (2) e (3):

Page 98: Numero 09 - Outubro de 2007

Temos:

Su (P) × Sv (P) ≡ det

⎡⎢⎢⎢⎢⎢⎣e1 e2 e3

∂x

∂u(P)

∂y

∂u(P)

∂z

∂u(P)

∂x

∂v(P)

∂y

∂v(P)

∂z

∂v(P)

⎤⎥⎥⎥⎥⎥⎦

=

⎛⎜⎜⎜⎜⎜⎜⎜⎜⎝det

⎡⎢⎢⎢⎣∂y

∂u(P)

∂z

∂u(P)

∂y

∂v(P)

∂z

∂v(P)

⎤⎥⎥⎥⎦︸ ︷︷ ︸

,

D1

det

⎡⎢⎢⎢⎣∂z

∂u(P)

∂x

∂u(P)

∂z

∂v(P)

∂x

∂v(P)

⎤⎥⎥⎥⎦︸ ︷︷ ︸

D2

, det

⎡⎢⎢⎢⎣∂x

∂u(P)

∂y

∂u(P)

∂x

∂v(P)

∂y

∂v(P)

⎤⎥⎥⎥⎦︸ ︷︷ ︸

D3

⎞⎟⎟⎟⎟⎟⎟⎟⎟⎠e

JS (P) =

⎡⎢⎢⎢⎢⎢⎢⎢⎢⎣

∂x

∂u(P)

∂x

∂v(P)

∂y

∂u(P)

∂y

∂v(P)

∂z

∂u(P)

∂z

∂v(P)

⎤⎥⎥⎥⎥⎥⎥⎥⎥⎦Assim, JS (P) tem posto 2 ⇐⇒ Di �= 0 para algum i = 1, 2, 3 ⇐⇒ Su (P) × Sv (P) �=(0, 0, 0) ⇐⇒ Su (P) e Sv (P) sao linearmente independentes. �

Definimos o traco da superfıcie regular S como sendo a imagem da aplicacao S.

Podemos ampliar nossa definicao de superfıcie regular como segue.

SejaS : U ⊂

abR

2 −→ R3

(u, v) �−→ (x (u, v) , y (u, v) , z (u, v))

uma aplicacao diferenciavel e U aberto conexo do R2. Dizemos que S e uma superfıcie

parametrizada diferenciavel regular quando qualquer uma das afirmacoes a seguir foremverdadeiras:(i) dSP e injetora para ∀P = (u, v) ∈ U;

(ii) JS (P) tem posto 2 para ∀P = (u, v) ∈ U;

(iii) Su (P) =

(∂x

∂u(P) ,

∂y

∂u(P) ,

∂z

∂u(P)

)e Sv (P) =

(∂x

∂v(P) ,

∂y

∂v(P) ,

∂z

∂v(P)

)sao lin-

earmente independentes para ∀P = (u, v) ∈ U;

(iv) Su (P) × Sv (P) �= (0, 0, 0) , ∀P = (u, v) ∈ U.

Daqui em diante diremos superfıcie regular para designar uma superfıcie parametrizadadiferenciavel regular.

Page 99: Numero 09 - Outubro de 2007

Exemplos:

(1) SejaS : R

2 −→ R3

(u, v) �−→ (u, v, u2 + v2

)Se u = 0 =⇒ S (0, v) =

(0, v, v2

)=⇒ parabola.

Se v = 0 =⇒ S (u, 0) =(u, 0, u2

)=⇒ parabol.a

Se u2+v2 = k > 0 temos circunferencias de raio√

k (interseccao de S com o plano z = k).

x = u

y = v

z = k

z

k

S

S e diferenciavel e

Su (u, v) = (1, 0, 2u) ;

Sv (u, v) = (0, 1, 2v) .

Logo, JS (u, v) =

⎡⎣ 1 0

0 1

2u 2v

⎤⎦ tem posto 2 pois det

[1 0

0 1

]= 1 �= 0.

Portanto, S e regular.

(2) Seja f : U ⊂ R2 −→ R de classe C∞ e consideremos

S : U ⊂ R2 −→ R

3

(u, v) �−→ (u, v, f (u, v))

S(u,v)

Tr(S) = Im(S) = Graf(f)

(u,v) U

x

z

y

O grafico de f e dado por {(u, v, f (u, v)) | (u, v) ∈ U} = Im (S) .

S e uma superfıcie regular:

JS (P) =

⎡⎢⎣ 1 0

0 1∂f

∂u(P)

∂f

∂v(P)

⎤⎥⎦

Page 100: Numero 09 - Outubro de 2007

e

det

[1 0

0 1

]= 1 �= 0 =⇒ JS (P) tem posto 2.

(3) Consideremos a superfıcie

S : R2 −→ R

3

(u, v) �−→ (u, v,

√u2 + v2

)Neste caso, f (u, v) =

√u2 + v2;

∂f

∂u(u, v) =

u√u2 + v2

e∂f

∂v(u, v) =

v√u2 + v2

.

Logo, S e diferenciavel em R2 − {(0, 0)} .

Consequentemente, pelo Exemplo 2, S e regular em R2 − {(0, 0)} .

y

z

xS não é regularem (0,0)

S

(4) Consideremos

S : R2 −→ R

3

(u, v) �−→ (a1u + b1v + c1, a2u + b2v + c2, a3u + b3v + c3)

sendo (a1, a2, a3) �= k (b1, b2, b3) .

Observacao: se (a1, a2, a3) = k (b1, b2, b3) entao, fazendo t = ku + v, temos S (u, v) =

S (t) = (b1t + c1, b2t + c2, b3t + c3) , ou seja, S se degeneraria em um uma reta ou emum ponto.

Temos que Traco (S) e um plano no espaco. S e diferenciavel, Su (u, v) = (a1, a2, a3) eSv (u, v) = (b1, b2, b3) . Logo, Su e Sv sao linearmente independentes. Assim, S e regular.

y

z

x

a

a � �a1 ,a2 ,a3 �

b

b � �b1 ,b2 ,b3 �

Page 101: Numero 09 - Outubro de 2007

Equacao vetorial do plano:

X (u, v) = (c1, c2, c3) + u (a1, a2, a3) + v (b1, b2, b3) = S (u, v) .

(5) ConsideremosS : R

2 −→ R3

(u, v) �−→ (cos (u) , sen (u) , v)

y

z = v

xS1

� R : cilindro circular reto

S u,v( )

u

S e diferenciavel, Su (u, v) = (− sen (u) , cos (u) , 0) e Sv (u, v) = (0, 0, 1) .

Su (u, v) × Sv (u, v) ≡ det

⎡⎣ e1 e2 e3

− sen (u) cos (u) 0

0 0 1

⎤⎦ ≡ (cos (u) , sen (u) , 0) �= (0, 0, 0) .

Portanto, S e regular.

(6) Consideremos

S : R2 −→ R

3

(u, v) �−→ (cos (u) cos (v) , cos (u) sen (v) , sen (u))

Temos:

Traco (S) ={(x, y, z) ∈ R

3 | x2 + y2 + z2 = 1}

;

Su (u, v) = (− sen (u) cos (v) ,− sen (v) sen (u) , cos (u)) ;

Sv (u, v) = (− sen (v) cos (u) , cos (u) cos (v) , 0) ;

Su (u, v) × Sv (u, v) =(− cos2 (u) cos (v) ,− cos2 (u) sen (v) ,− cos (u) sen (u)

);

|Su (u, v) × Sv (u, v)|2

= cos4 (u) cos2 (v) + cos4 (u) sen2 (v) + cos2 (u) sen2 (u)

= cos4 (u) + cos2 (u) sen2 (v)

= cos2 (u) .

Assim, S e regular se cos2 (u) �= 0. Mas:

cos2 (u) = 0 ⇐⇒ cos (u) = 0 ⇐⇒ u =π

2+ kπ, k ∈ Z.

Se v ∈ R e u ∈]−

π

2,π

2

[obtemos a esfera menos os polos.

Page 102: Numero 09 - Outubro de 2007

y

z

xesfera

v

S u,v( )

u

1.3 Superfıcies de Revolucao

Superfıcies regulares obtidas por revolucao de curvas em torno de eixos sao muito im-portantes em Geometria Diferencial. Alem disso, elas possuem parametrizacoes bastantesimples, conforme veremos abaixo.

Em torno do eixo z.

Seja α : ]a, b[ ⊂ R −→ R3 uma curva regular tal que α (u) = (f (u) , 0, g (u)) com f, g :

]a, b[−→ R diferenciaveis e f (u) �= 0, ∀u ∈ ]a, b[ .

A matriz de rotacao de α por um angulo v em torno do eixo z e dada por:⎡⎣cos (v) − sen (v) 0

sen (v) cos (v) 0

0 0 1

⎤⎦ .

Para obter a superfıcie descrita pela curva α em torno do eixo z fazemos⎡⎣cos (v) − sen (v) 0

sen (v) cos (v) 0

0 0 1

⎤⎦ .

⎡⎣f (u)

0

g (u)

⎤⎦ =

⎡⎣f (u) cos (v)

f (u) sen (v)

g (u)

⎤⎦com v variando em R, ou seja:

S : ]a, b[ × R −→ R3

(u, v) �−→ (f (u) cos (v) , f (u) sen (v) , g (u))

e chamada de superfıcie de revolucao da curva α em torno do eixo z.

As curvas S (u, v0) , com v0 fixo e u ∈ ]a, b[ sao chamadas de meridianos da superfıcie derevolucao S.

As curvas S (u0, v) , com u0 fixo e u ∈ R sao chamadas de paralelos da superfıcie derevolucao S.

y

z

x

curva�

meridiano

paralelo

Page 103: Numero 09 - Outubro de 2007

Observacoes:

(i) Fazendo v variar em um intervalo de R de comprimento menor do que 2π, obtemosparte de uma superfıcie de revolucao. Por exemplo, fazendo v ∈ ]0, 2π[ obtemos a su-perfıcie S menos um meridiano (o que corresponderia a v0 = 0 ou v0 = 2π).

(ii) Se permitıssemos f (u0) = 0 para algum u0 ∈ ]a, b[ , terıamos que α (u0) = (0, 0, g (u0))

seria um ponto do eixo z e S (u0, v) = (0, 0, g (u0)) , ∀v ∈ R, ou seja, terıamos um paralelodegenerado em um ponto do eixo z fazendo com que S nao seja regular nesse ponto.

Exemplos:

(1) Seja α (u) = (1, 0, u) , u ∈ R, uma reta do plano xz perpendicular ao eixo x.

Temos a superfıcie de revolucao de α em torno de z dada por

S (u, v) = (cos (v) , sen (v) , u) ,

que e um cilindro circular reto de raio 1.

y

z

x

(2) Seja α (u) = (a + r cos (u) , 0, r sen (u)) , 0 < r < a, uma circunferencia de centro(a, 0, 0) e raio r no plano xz. De fato, fazendo{

x = a + r cos (u)

z = r sen (u)=⇒ {

(x − a)2

= r2 cos2 (u)

z2 = r2 sen2 (u)=⇒ (x − a)

2+ z2 = r2.

Temosf (u) = a + r cos (u) = 0 ⇐⇒ cos (u) = −

a

r< −1,

ou seja, f (u) �= 0, ∀u ∈ R.

Temos a superfıcie de revolucao de α em torno de z dada por

S (u, v) = ((a + r cos (u)) cos (v) , (a + r cos (u)) sen (v) , r sen (u)) ,

que e um toro circular de raios a e r.

Circunferênciade centro ( ,0,0)

e raioa

r x

y

z

“Toro Circular”

Page 104: Numero 09 - Outubro de 2007

Em torno do eixo x.

Seja α : ]a, b[ ⊂ R −→ R3 uma curva regular tal que α (u) = (g (u) , f (u) , 0) com

f, g : ]a, b[−→ R diferenciaveis e f (u) �= 0, ∀u ∈ ]a, b[ .

A matriz de rotacao de α por um angulo v em torno do eixo x e dada por:⎡⎣1 0 0

0 cos (v) − sen (v)

0 sen (v) cos (v)

⎤⎦ .

Para obter a superfıcie descrita pela curva α em torno do eixo x fazemos⎡⎣1 0 0

0 cos (v) − sen (v)

0 sen (v) cos (v)

⎤⎦ .

⎡⎣g (u)

f (u)

0

⎤⎦ =

⎡⎣ g (u)

f (u) cos (v)

f (u) sen (v)

⎤⎦com v variando em R, ou seja:

S : ]a, b[ × R −→ R3

(u, v) �−→ (g (u) , f (u) cos (v) , f (u) sen (v))

e chamada de superfıcie de revolucao da curva α em torno do eixo x.

y

z

x

Meridianos e paralelos de S sao definidos de modo analogo ao caso anterior.Observacoes analogas ao caso anterior tambem sao validas.

Exemplo: Seja α (u) = (u, cosh (u) , 0) , u ∈ R, uma catenaria no plano xy.

Temos g (u) = u e f (u) = cosh (u) �= 0, ∀u ∈ R. Logo,

S (u, v) = (u, cosh (u) cos (v) , cosh (u) sen (v)) , u, v ∈ R,

que e chamada de catenoide.

y

z

x �é uma catenária

S é um catenóide

Page 105: Numero 09 - Outubro de 2007

Em torno do eixo y.

Seja α : ]a, b[ ⊂ R −→ R3 uma curva regular tal que α (u) = (0, g (u) , f (u)) com f, g :

]a, b[−→ R diferenciaveis e f (u) �= 0, ∀u ∈ ]a, b[ .

A matriz de rotacao de α por um angulo v em torno do eixo y e dada por:⎡⎣cos (v) 0 − sen (v)

0 1 0

sen (v) 0 cos (v)

⎤⎦ .

Para obter a superfıcie descrita pela curva α em torno do eixo x fazemos⎡⎣cos (v) 0 − sen (v)

0 1 0

sen (v) 0 cos (v)

⎤⎦ .

⎡⎣ 0

g (u)

f (u)

⎤⎦ =

⎡⎣ g (u)

f (u) cos (v)

⎤⎦com v variando em R, ou seja:

S : ]a, b[ × R −→ R3

(u, v) �−→ (−f (u) sen (v) , g (u) , f (u) cos (v))

e chamada de superfıcie de revolucao da curva α em torno do eixo y.

y

z

x

Meridianos e paralelos de S sao definidos de modo analogo ao caso anterior.Observacoes analogas ao caso anterior tambem sao validas.

1.4 Curvas Coordenadas

Sejam S : U ⊂ R2 −→ R

3 uma superfıcie regular e P = (u0, v0) ∈ U. As curvas

αu0(v) = S (u0, v) , (u0, v) ∈ U

e

αv0(u) = S (u, v0) , (u, v0) ∈ U

sao chamadas curvas (ou linhas) coordenadas da superfıcie S.

Observemos que meridianos e paralelos de superfıcies de revolucao sao curvas coordenadasda superfıcie.

Exemplos:

(1) Seja S (u, v) = (cos (u) , sen (u) , v) , (u, v) ∈ R2, um cilindro.

Fazendo P = (u0, v0) temos αu0(v) = S (u0, v) = (cos (u0) , sen (u0) , v) uma reta e

αv0(u) = S (u, v0) = (cos (u) , sen (u) , v0) uma circunferencia como curvas coordenadas

passando por P.

Page 106: Numero 09 - Outubro de 2007

y

z

x

�v0�u�

�u 0�v �

S�u0 ,v 0 �

(2) Seja S (u, v) =(u, v, u2 + v2

), u, v ∈ R, um paraboloide circular.

Fazendo P = (u0, v0) temos αu0(v) =

(u0, v, u

20 + v2

)uma parabola e αv0

(u) =(u, v0, u

2 + v20

),

tambem um parabola como curvas coordenadas passando por P.

x

y

z

S

(paralelo ao plano )yz

(paralelo ao plano )xz

S(u ,v )0 0

�u 0�v �

�v0�u�

1.5 Plano Tangente

Sejam S : U ⊂ R2 −→ R

3 uma superfıcie regular e P ∈ U.

O plano tangente a S em P e o plano, denotado por TPS, que passa por S (P) e e paraleloaos vetores Su (P) e Sv (P) .

S(P)

Su�P� � Sv�P�

Su�P�

TPS

S

Sv �P�

Observacao: E comum considerarmos TPS passando por (0, 0, 0) , isto e:

TPS = {λSu (P) + μSv(P), λ, μ ∈ R} .

Exemplo: Seja S (u, v) =(u, v, u2 + v2

), u, v ∈ R um paraboloide circular. Temos

Su (u, v) = (1, 0, 2u) e Sv (u, v) = (0, 1, 2v) . Em P = (0, 0) temos: Su (P) = (1, 0, 0) eSv (P) = (0, 1, 0) , Su (P) × Sv (P) = (0, 0, 1) e S (P) = (0, 0, 0) . Logo:

TPS = {λ (1, 0, 0) + μ (0, 1, 0) , λ, μ ∈ R} = {(λ, μ, 0) , λ, μ ∈ R} ,

ou seja, o plano xy (de equacao z = 0).

Page 107: Numero 09 - Outubro de 2007

x

y

z = 0

z

S

S(P)S (P)u

S (P)v

T SP

S (P) S (P)u v�

1.6 Mudanca de Parametros

SejamS : U ⊂ R

2 −→ R3

uma superfıcie regular eh : V ⊂ R

2 −→ R2

aplicacao diferenciavel tal que

h (V) = U e |Jh (P)| = det Jh (P) �= 0,∀P ∈ V.

A aplicacao S = S ◦ h : V ⊂ R2 −→ R

3 e uma superfıcie regular que tem o mesmo tracode S.

U

R2

R3

V

R2

h

u

v

t

w

�S �

S �h

S

x

y

z

De fato: JS (P) = J (S ◦ h) (P) = JS (h (P)) Jh (P) . Como JS (h (P)) tem posto 2, pois S e

regular e Jh (P) tem posto 2 por hipotese, entao JS (P) tem posto 2. Logo, S e regular.

A aplicacao h e chamada mudanca de parametros para S.

Observacao: S e diferenciavel pois e composta de aplicacoes diferenciaveis. Quanto aotraco:

Seja Q ∈ Im S, entao ∃P ∈ U tal que Q = S (P) . Como h (V) = U, entao ∃R ∈ V tal que

h (R) = P. Logo, Q = S (h (R)) = S (R) , ou seja, Q ∈ Im S. Assim, Im S ⊂ Im S.

Page 108: Numero 09 - Outubro de 2007

Seja Q ∈ Im S, entao ∃R ∈ V tal que S (R) = Q. Mas S = S ◦ h. Logo, S ◦ h (R) = Q ⇒Q = S (h (R)) , ou seja, Q ∈ Im S. Assim, Im S ⊂ Im S.

Conclusao: Im S = Im S.

P

R2

R3

R

R2

h

u

v

t

w

S

x

y

z

V

U

S

Q

Im S = Im S

Observemos que a aplicacao h nao precisa ser, necessariamente, injetora.

Exemplos:

(1) h (w, t) = (ew cos (t) , ew sen (t)) .

Temos:

Jh (w, t) =

[ew cos (t) −ew sen (t)

ew sen (t) ew cos (t)

]=⇒ |Jh (w, t)| = ew �= 0, ∀ (w, t) ∈ R

2.

No entanto, h (w, 0) = h (w, 2π) = (ew, 0) , ou seja, h nao e injetora.

(2) Sejam S (u, v) = (u + v, u − v, 4uv) ; (u, v) ∈ R2 e S (w, t) =

(w, t, w2 − t2

); (w, t) ∈

R2. Mostremos que S e uma reparametrizacao de S.

Queremos determinar

h : R2 −→ R

2

(w, t) �−→ (h1 (w, t) , h2 (w, t))

tal que S = S ◦ h e |Jh (w, t)| �= 0, ∀ (w, t) ∈ R2. Assim:

S (w, t) = S (h (w, t)) =⇒(w, t, w2 − t2

)= S (h1 (w, t) , h2 (w, t)) =⇒(

w, t, w2 − t2)

= (h1 (w, t) + h2 (w, t) , h1 (w, t) − h2 (w, t) , 4h1 (w, t) h2 (w, t)) .

Logo: ⎧⎨⎩

h1 + h2 = w

h1 − h2 = t

4h1h2 = w2 − t2

=⇒ h1 (w, t) =w + t

2e h2 (w, t) =

w − t

2,

Page 109: Numero 09 - Outubro de 2007

ou seja:

h (w, t) =

(w + t

2,w − t

2

).

Temos que h e diferenciavel C∞ , h e sobrejetora e

|Jh (w, t)| = det

[12

12

12

−12

]= −

1

2�= 0; ∀ (w, t) ∈ R

2.

u

v

x

y

z

SR2

Parabolóide hiperbólico

1.7 Forma Local das Superfıcies Regulares

Consideremos a esfera unitaria com centro na origem:

C ={(x, y, z) ∈ R

3 | x2 + y2 + z2 = 1}

,

que e uma superfıcie regular.

C

y

z

x

Temos: {se z > 0, z =

√1 − x2 − y2, ou seja, z = f1 (x, y)

se z < 0, z = −√

1 − x2 − y2, ou seja, z = f2 (x, y).

Isto significa que, sob certas condicoes podemos enxergar partes da superfıcie C comografico de funcoes reais de duas variaveis.Este exemplo pode ser generalizado, ou seja, toda superfıcie regular pode ser vista lo-calmente como grafico de uma funcao real diferenciavel de duas variaveis reais. Este e oconteudo da proposicao abaixo:

Proposicao 3. Seja S : U −→ R3 uma superfıcie regular e seja P = (u0, v0) . Entao,

existem um conjunto aberto V ⊂ U, com P ∈ V e uma mudanca de parametros h : W −→V tal que o traco de S = S ◦ h e o grafico de uma aplicacao diferenciavel.

Demonstracao.

De acordo com as hipoteses da proposicao acima podemos considerar o seguinte diagrama:

Page 110: Numero 09 - Outubro de 2007

P

R2

R3

R2

h

u

v

t

w

S

x

y

z

V U

S

S(P)Im S

W

Sendo S (u, v) = (x (u, v) , y (u, v) , z (u, v)) e S regular, entao JS (P) tem posto 2.

Vamos supor:

det

⎡⎢⎢⎢⎣∂x

∂u(P)

∂x

∂v(P)

∂y

∂u(P)

∂y

∂v(P)

⎤⎥⎥⎥⎦ �= 0.

Seja F (u, v) = (x (u, v) , y (u, v)) . Logo, |JF (P)| = D �= 0.

Pelo Teorema da Funcao Inversa (aplicado a F), existe um aberto V ⊂ R2; V ⊂ U; P ∈ V

tal que F|V : V −→ F (V) = W tem inversa h = F−1 diferenciavel:

h : W −→ V

(w, t) �−→ F−1 (w, t).

Seja h (Q) = P.

Temos que Jh (Q) tem posto 2.

De fato: F ◦ h = F ◦ F−1 = Id =⇒ JF ◦ h (Q) = JId (Q) = Id. Logo, |JF ◦ h (Q)| =

|JF (P)| . |Jh (Q)| = 1. Temos |Jh (Q)| �= 0, ou seja, Jh (Q) tem posto 2. Temos, portanto,

que S = S ◦ h : W −→ R3 e regular.

Observemos que:

F ◦ h (w, t) = Id (w, t) =⇒ F (h (w, t)) = (w, t) =⇒ (x (h (w, t)) , y (h (w, t))) = (w, t) =⇒{x ◦ h (w, t) = w

y ◦ h (w, t) = t.

Logo,

S (w, t) = S ◦ h (w, t) = (x (h (w, t)) , y (h (w, t)) , z (h (w, t))) = (w, t, z ◦ h (w, t)) ,

ou seja, o traco de S e o grafico da funcao diferenciavel:

z ◦ h : W −→ R

(w, t) �−→ z ◦ h (w, t).

Page 111: Numero 09 - Outubro de 2007

1.8 Aplicacao Normal de Gauss

Seja S : U ⊂ R2 −→ R

3 uma superfıcie regular e P ∈ U. Vamos denotar vetor unitarionormal a S em P o vetor:

η (P) =Su (P) × Sv (P)

|Su (P) × Sv (P)|.

S

U

Su�P�

Sv�P�

u

v

x

y

z

S(P)

�(P)

A aplicacao:η : U ⊂ R

2 −→ R3

(u, v) �−→ Su × Sv

|Su × Sv|(u, v)

chamada Aplicacao Normal de Gauss de S.

Observando que |η (u, v)| = 1, ∀ (u, v) ∈ U, temos que a imagem da Aplicacao Normalde Gauss esta contida na esfera unitaria

S2 =

{(x, y, z) ∈ R

3 | |(x, y, z)| = 1}

={(x, y, z) ∈ R

3 | x2 + y2 + z2 = 1}

.

u

v S

x

y

z

U

y

z

x

C

Colocando todosos vetores

“na origem”:

Exemplo: S (u, v) =(u, v, u2 + v2

); (u, v) ∈ R

2. (paraboloide circular)Temos:

Su (u, v) = (1, 0, 2u) ;

Sv (u, v) = (0, 1, 2v) ;

(Su × Sv) (u, v) = det

⎡⎣e1 e2 e3

1 0 2u

0 1 2v

⎤⎦ = (−2u,−2v, 1) .

Logo, |(Su × Sv) (u, v)| =√

1 + 4u2 + 4v2.

Page 112: Numero 09 - Outubro de 2007

Assim:

η : R2 −→ R

3

(u, v) �−→ (−2u,−2v, 1)√1 + 4u2 + 4v2

.

1.9 Curvas sobre uma Superfıcie Regular

Sejam α : I ⊂ R → U ⊂ R2 uma curva regular e S : U ⊂ R

2 −→ R3 uma superfıcie

regular.

A curva β = S ◦ α e uma curva cujo traco esta contido na Im S. Alem disso, β e difer-enciavel, pois e composta de aplicacoes diferenciaveis.

R2

R3

u

v

S

x

y

zU

I

� � S � �

A curva β e regular.

De fato:

β′ (t) = (S ◦ α)′= JS (α (t)) α′ (t) = Su (α (t)) x′

1 (t) + Sv (α (t)) x′2 (t) ,

sendo α (t) = (x1 (t) , x2 (t)) .

Como Su (α (t)) e Sv (α (t)) sao linearmente independentes:

β′ (t) = (0, 0, 0) ⇐⇒ {x′

1 (t) = 0

x′2 (t) = 0

.

Mas α e regular. Logo, α′ (t) = (x′1 (t) , x′

2 (t)) �= (0, 0) .

Conclusao: β′ (t) �= (0, 0, 0) , ∀t ∈ I, ou seja, β e regular.

Reciprocamente: seja β : I ⊂ R −→ R3 uma curva regular tal que Im β ⊂ Im S sendo

S : U ⊂ R2 −→ R

3 uma superfıcie regular. Sera que existe α : I ⊂ R −→ R2 tal que

β = S ◦ α?

Page 113: Numero 09 - Outubro de 2007

R2

R3

����

u

v

S

x

y

zU

I

� � S � �

Temos β (t) ∈ Im S, ∀t ∈ I. Logo, para cada t existem u = u (t) e v = v (t) tais queβ (t) = S (u (t) , v (t)) . Definindo

α : I ⊂ R −→ R2

t �−→ α (t) = (u (t) , v (t))

temos S ◦ α (t) = β (t) .

Observemos que α e diferenciavel pois S e β sao diferenciaveis.

Consideremos as duas curvas coordenadas de S passando por S (P) = S (u0, v0):

αu0(v) = S (u0, v) e αv0

(u) = S (u, v0) .

Temos

α′u0

(v) = Sv (u0, v)

e

α′v0

(u) = Su (u, v0) ,

ou seja, os vetores tangentes as curvas coordenadas αu0e αv0

no ponto S (u0, v0) per-tencem ao plano tangente a superfıcie S em S (u0, v0) .

TPS

�v0

� �u0 � � Su�u0 ,v 0 �

�u 0

� �v 0 � � Sv�u0 , v 0 �

�v0�u0 � � �u0

�v 0 � � S�u0 ,v 0 � � S�P�

�u 0

�v0

S(P)

Na verdade, um vetor β′ (t0) tangente a uma curva regular β qualquer de uma su-perfıcie sera paralelo ao plano tangente a superfıcie no ponto β (t0) . Este e o conteudoda proposicao abaixo.

Page 114: Numero 09 - Outubro de 2007

Proposicao 4. Seja β : I ⊂ R −→ R3 uma curva regular na superfıcie S : U ⊂ R

2 −→ R3.

Seja t0 ∈ I e β (t0) = S (P) para algum P ∈ U. Entao, β′ (t0) ∈ TPS.

Demonstracao.

Vimos que ∃α : I ⊂ R −→ U ⊂ R2 tal que β = S◦α, sendo α (t) = (u (t) , v (t)) . Tambem

vimos que:β′ (t) = x′

1 (t)Su (α (t)) + x′2 (t) Sv (α (t)) .

Fazendo t = t0:β′ (t0) = λSu (α (t0)) + μSv (α (t0)) ,

sendo λ = x′1 (t0) e μ = x′

2 (t0) , ou seja, β′ (t0) ∈ Tα(t0)S = TPS.

Conclusao: Os vetores tangentes as curvas na superfıcie S passando por S (P) sao paralelosa TPS, ou seja,

A = {β′ (t0) | β e uma curva S com β (t0) = S (P)} ⊂ TPS,

como querıamos. �

Tomando w ∈ TPS; w = aSu (P) + bSv (P) ; e possıvel mostrar que existem α : I ⊂R −→ R

2 e β = S ◦ α tais que β (t0) = S (P) e β′ (t0) = w, ou seja, TPS ⊂ A.

Assim podemos concluir que TPS = A, ou seja:

TPS ={w ∈ R

3 | w = β′ (t0) onde β e uma curva em S tal que β (t0) = S (P)}

.

1.10 Primeira Forma Quadratica

Seja S : U ⊂ R2 −→ R

3 uma superfıcie regular e seja P ∈ U. A aplicacao

IP : TPS −→ R

w �−→ 〈w,w〉 = |w|2

e chamada de Primeira Forma Quadratica (ou Fundamental) de S em P.

Temosw ∈ TPS =⇒ w = aSu (P) + bSv (P) .

Logo,

IP (w) = 〈aSu (P) + bSv (P) , aSu (P) + bSv (P)〉= a2 〈Su (P) , Su (P)〉 + 2ab 〈Su (P) , Sv (P)〉 + b2 〈Sv (P) , Sv (P)〉

Facamos ⎧⎨⎩

E (P) = 〈Su (P) , Su (P)〉F (P) = 〈Su (P) , Sv (P)〉G (P) = 〈Sv (P) , Sv (P)〉

,

que sao chamados de coeficientes da Primeira Forma Quadratica.Logo,

IP (w) = a2E (P) + 2abF (P) + b2G (P) .

Observacao: E (P) , G (P) > 0.

Page 115: Numero 09 - Outubro de 2007

Exemplo: Consideremos S (u, v) = (cos (u) , sen (u) , v) , (u, v) ∈ R2. Seja P ∈ R

2.

Temos:

Su (P) = (− sen (u) , cos (u) , 0) ;

Sv (P) = (0, 0, 1) .

Logo:

E (P) = 〈Su (P) , Su (P)〉 = cos2 (u) + sen2 (u) + 02 = 1

F (P) = 〈Su (P) , Sv (P)〉 = 0 + 0 + 0 = 0

G (P) = 〈Sv (P) , Sv (P)〉 = 02 + 02 + 12 = 1

Assim:IP (w) = a2 + b2.

1.11 Comprimento de Curvas em Superfıcies

Consideremos uma curva β sobre a superfıcie S:

R2

R3

u

v

S

x

y

zU

I

� � S � �

P = (t )�0

S(P) ’(t )0

t0

Temos:β′ (t) = u′ (t)Su (α (t)) + v′ (t) Sv (α (t)) ,

sendo α (t) = (u (t) , v (t)) .

Seja l o comprimento de β entre os pontos a e b, a < b e a, b ∈ I:

l =

∫b

a

|β′ (t)| dt

Temos:

|β′ (t)|2 = 〈u′ (t)Su (α (t)) + v′ (t) Sv (α (t)) , u′ (t) Su (α (t)) + v′ (t) Sv (α (t))〉= (u′ (t))2 〈Su, Su〉 (α (t)) + 2u′ (t) v′ (t) 〈Su, Sv〉 (α (t)) + (v′ (t))2 〈Sv, Sv〉 (α (t))

= (u′ (t))2E (α (t)) + 2u′ (t) v′ (t) F (α (t)) + (v′ (t))2

G (α (t))

= Iα(t) (β′ (t))

Page 116: Numero 09 - Outubro de 2007

Logo:

l =

∫b

a

√Iα(t) (β′ (t))dt,

ou seja, o comprimento de β entre a e b so depende da primeira forma quadratica.

Exemplo: Seja S (u, v) = (cos (u) , sen (u) , v) , (u, v) ∈ R2. Calculemos o comprimento

da curva β = S ◦ α, sendo α (t) = (t, t) , t ∈ [0, 2π] .

y

z

x

2�0

2�

2�v

u

S�

Se w = aSu (P) + bSv (P) , vimos que IP (w) = a2 + b2.

Temos P = α (t) , α′ (t) = (u′ (t) , v′ (t)) = (1, 1) e β′ (t) = u′ (t) Su (P) + v′ (t)Sv (P) .

Logo,Iα(t) (β′ (t)) = 12 + 12 = 2.

Assim,

l =

∫2π

0

√2dt = 2

√2π.

Observacao: β (t) = S ◦ α (t) = S (t, t) = (cos (t) , sen (t) , t) , ou seja, β e uma helice.

y

z

x

0

2�

1.12 Area de uma Superfıcie

Seja S : U ⊂ R2 −→ R

3 uma superfıcie regular. Considere V uma regiao do R2 tal que

V ⊂ U, sendo V compacto, conexo, homeomorfo a um disco do R2 e S| ◦

Vinjetora. (

◦V e o

interior de V)Do Calculo Diferencial e Integral:

S (P)u

S (P)v

|Su�P� � Sv�P�|

S(P)

S

Page 117: Numero 09 - Outubro de 2007

A area de S (V) e dada por:

A (S (V)) =

∫∫V

|Su (P) × Sv (P)| dudv.

Mas vimos que

Su (P)×Sv (P) =

⎛⎝det

⎡⎣ ∂y∂u

(P) ∂z∂u

(P)

∂y∂v

(P) ∂z∂v

(P)

⎤⎦ , det

⎡⎣ ∂z∂u

(P) ∂x∂u

(P)

∂z∂v

(P) ∂x∂v

(P)

⎤⎦ , det

⎡⎣ ∂x∂u

(P) ∂y∂u

(P)

∂x∂v

(P) ∂y∂v

(P)

⎤⎦⎞⎠ .

Dessa forma, no ponto P:

|Su × Sv|2

=(

∂y∂u

∂z∂v

− ∂y∂v

∂z∂u

)2+

(∂z∂u

∂x∂v

− ∂z∂v

∂x∂u

)2+

(∂x∂u

∂y∂v

− ∂x∂v

∂y∂u

)2

=(

∂y∂u

∂z∂v

)2− 2

(∂y∂u

∂z∂v

) (∂y∂v

∂z∂u

)+

(∂y∂v

∂z∂u

)2

+(

∂z∂u

∂x∂v

)2− 2

(∂z∂u

∂x∂v

) (∂z∂v

∂x∂u

)+

(∂z∂v

∂x∂u

)2

+(

∂x∂u

∂y∂v

)2− 2

(∂x∂u

∂y∂v

) (∂x∂v

∂y∂u

)+

(∂x∂v

∂y∂u

)2

=(

∂x∂u

)2((

∂z∂v

)2+

(∂y∂v

)2+

(∂x∂v

)2)

−(

∂x∂u

)2 (∂x∂v

)2

+(

∂y∂u

)2((

∂z∂v

)2+

(∂y∂v

)2+

(∂x∂v

)2)

−(

∂y∂u

)2 (∂y∂v

)2

+(

∂z∂u

)2((

∂z∂v

)2+

(∂y∂v

)2+

(∂x∂v

)2)

−(

∂z∂u

)2 (∂z∂v

)2

− 2(

∂y∂u

∂z∂v

) (∂y∂v

∂z∂u

)− 2

(∂z∂u

∂x∂v

) (∂z∂v

∂x∂u

)− 2

(∂x∂u

∂y∂v

) (∂x∂v

∂y∂u

)=

((∂z∂u

)2+

(∂y∂u

)2+

(∂x∂u

)2)((

∂z∂v

)2+

(∂y∂v

)2+

(∂x∂v

)2)

−(

∂x∂u

∂x∂v

) (∂x∂v

∂x∂u

+ ∂y∂v

∂y∂u

+ ∂z∂v

∂z∂u

)−

(∂y∂u

∂y∂v

) (∂x∂v

∂x∂u

+ ∂y∂v

∂y∂u

+ ∂z∂v

∂z∂u

)−

(∂z∂u

∂z∂v

) (∂x∂v

∂x∂u

+ ∂y∂v

∂y∂u

+ ∂z∂v

∂z∂u

)= 〈Su, Su〉 〈Sv, Sv〉 −

(∂x∂v

∂x∂u

+ ∂y∂v

∂y∂u

+ ∂z∂v

∂z∂u

)2

= 〈Su, Su〉 〈Sv, Sv〉 − 〈Su, Sv〉2= EG − F2.

Logo,

A (S (V)) =

∫∫V

√E (P) G (P) − F (P)

2dudv.

Exemplo: Consideremos S (u, v) = (cos (u) , sen (u) , v) , (u, v) ∈ R2.

Seja V = [0, 2π] × [0, 1] .

Vimos que E (P) = 1; F (P) = 0 e G (P) = 1.

Logo,

A (S (V)) =

∫2π

0

∫1

0

√1.1 − 02dudv = 2π.

Page 118: Numero 09 - Outubro de 2007

y

z

x

“Anel de raio 1

e altura 1.”

2�

1

v

u

S

0

1.13 Segunda Forma Quadratica

Consideremos uma curva β sobre a superfıcie S:

u

v

SU

� � S � �

S(P)

TPS

S

w

a

b

t0

P = (t )�0

(t ) = S(P)

(t ) = w0

0’

Definimos a Segunda Forma Quadratica (ou Fundamental) da superfıcie S em P comosendo a aplicacao:

IIP : TPS −→ R

w �−→ 〈β′′ (t0) , η (P)〉sendo η (P) o vetor normal unitario a S em P:

η (P) =Su (P) × Sv (P)

|Su (P) × Sv (P)|

e w = β′ (t0) para alguma curva β sobre S satisfazendo β (t0) = P.

A expressao da Segunda Forma Quadratica nao depende da curva β escolhida.De fato: seja β = S ◦ α e α (t) = (u (t) , v (t)) .

Temos β′ (t) = u′ (t) Su (α (t)) + v′ (t) Sv (α (t)) .

Logo,

β′′ (t) = u′′ (t)Su (α (t)) + u′ (t) [Suu (α (t))u′ (t) + Suv (α (t)) v′ (t)]

+ v′′ (t)Sv (α (t)) + v′ (t) [Svu (α (t)) u′ (t) + Svv (α (t)) v′ (t)]

= u′′ (t)Su (α (t)) + v′′ (t) Sv (α (t)) + u′ (t)2Suu (α (t))

+ 2u′ (t) v′ (t) Suv (α (t)) + v′ (t)2Svv (α (t))

Page 119: Numero 09 - Outubro de 2007

Suv = Svu pois as funcoes componentes sao C∞ .

Facamos a = u′ (t0) e b = v′ (t0) ; P = α (t0) .

Temos:

β′ (t0) = u′ (t0)Su (P) + v′ (t0) Sv = aSu (P) + bSv (P) .

Logo,

IIP (w) = 〈β′′ (t0) , η (P)〉= u′ (t0)

2 〈Suu, η〉 (P) + 2u′ (t0) v′ (t0) 〈Suv, η〉 (P) + v′ (t0)2 〈Svv, η〉 (P)

= a2 〈Suu, η〉 (P) + 2ab 〈Suv, η〉 (P) + b2 〈Svv, η〉 (P)

Sejam:

e (P) = 〈Suu (P) , η (P)〉 ;

f (P) = 〈Suv (P) , η (P)〉 ;

g (P) = 〈Svv (P) , η (P)〉 .

Logo,

IIP (w) = a2e (P) + 2abf (P) + b2g (P) ,

sendo w = aSu (P) + bSv (P) , ou seja, IIP (w) nao depende de β.

Exemplo: Seja S (u, v) = (cos (u) , sen (u) , v) , (u, v) ∈ R.

Seja P = (u0, v0) qualquer e consideremos o plano tangente TPS.

Temos:

Su (P) = (− sen (u0) , cos (u0) , 0) ;

Sv (P) = (0, 0, 1) ;

Su (P) × Sv (P) = det

⎡⎣ e1 e2 e3

− sen (u0) cos (u0) 0

0 0 1

⎤⎦ = (cos (u0) , sen (u0) , 0) ;

Suu (P) = (− cos (u0) ,− sen (u0) , 0) ;

Suv (P) = (0, 0, 0) ;

Svv (P) = (0, 0, 0) ;

e (P) = 〈Suu (P) , η (P)〉

=

⟨(− cos (u0) ,− sen (u0) , 0) ,

(cos (u0) , sen (u0) , 0)√cos2 (u0) + sen2 (u0) + 02

⟩= − cos2 (u0) − sen2 (u0) + 02 = −1;

f (P) = 〈Suv (P) , η (P)〉 = 0

g (P) = 〈Svv (P) , η (P)〉 = 0

Assim, se w = aSu (P) + bSv (P) , temos IIP (w) = −a2.

Page 120: Numero 09 - Outubro de 2007

1.14 Superfıcies Isometricas

Uma superfıcie S : U ⊂ R2 −→ R

3 e dita simples quando S e injetora.

Observacao: Se S : U ⊂ R2 −→ R

3 e regular, entao ∃V ⊂ U tal que S|V e injetora.

Duas superfıcies simples S, S : U ⊂ R2 −→ R

3 sao ditas isometricas quando os coeficientesdas Primeiras Formas Quadraticas de S e S coincidem, isto e, E (P) = E (P) , F (P) = F (P)

e G (P) = G (P) , ∀P ∈ U.

Exemplo: As superfıcies

S : ]0, 2π[ × R −→ R3

(u, v) �−→ (u, v, 0)e

S : ]0, 2π[ × R −→ R3

(u, v) �−→ (cos (u) , sen (u) , v)

sao superfıcies isometricas pois E (u, v) = E (u, v) = 1, F (u, v) = F (u, v) = 0 e G (u, v) =

G (u, v) = 1, ∀ (u, v) ∈ ]0, 2π[ × R.

Observacao: se S, S : U ⊂ R2 −→ R

3 sao superfıcies regulares simples, entao existe umabijecao φ : S (U) −→ S (U) .

De fato, S : U −→ S (U) e S : U −→ S (U) sao bijetoras. Logo, a composta φ = S ◦ S−1 :

S (U) −→ S (U) e bijetora.

R2

R3

u

v S

x

y

z

U

R3

x

y

z

S(U)

S(U)

� � S � S�1

S

1.15 Distancia Intrınseca e Isometrias

Seja S : U ⊂ R2 −→ R

3 uma superfıcie regular e sejam x1, x2 ∈ S (U) . Considere umacurva D : [a, b] −→ S ⊂ R

3 tal que D (a) = x1 e D (b) = x2. Seja l (D) o comprimentode D.

Definimos a distancia intrınseca entre x1 e x2 sobre S como sendo:

d (x1, x2) = inf {l (D) : D e uma curva na superfıcie S ligando x1 a x2} .

x = D( )1 a

x = D( )2 b

l(D)

Page 121: Numero 09 - Outubro de 2007

Sejam S, S : U ⊂ R2 −→ R

3 superfıcies regulares. Uma aplicacao φ : S (U) −→ S (U) edita uma isometria entre S e S quando preserva distancias intrınsecas, ou seja, quandod (x1, x2) = d (φ (x1) , φ (x2)) , ∀x1, x2 ∈ S (U) , sendo d a distancia intrınseca em S e d adistancia intrınseca em S.

Proposicao 5. Sejam S, S : U ⊂ R2 −→ R

3 superfıcies isometricas. Se β : I −→ R3 e

uma curva na superfıcie S, entao o comprimento de β entre a, b ∈ I, a < b e igual aocomprimento de φ ◦ β entre a, b ∈ I, onde φ = S ◦ S−1.

Demonstracao.

Temos:

β = S ◦ α;

φ ◦ β = S ◦ S−1 ◦ S ◦ α = S ◦ α.

Seja α (t) = (u (t) , v (t)) .

S

x

y

z

y

z

S(U)

S(U)

� � S � S�1

S

��

u

v

U

�a

bI

� � S � �

� � � � S � �

� � �

x

Temos:

l (β) =

∫ t1

t0

|β′ (t)| dt =

∫ t1

t0

|u′ (t) Su (α (t)) + v′ (t) Sv (α (t))| dt

=

∫ t1

t0

√u′ (t)2

E (α (t)) + 2u′ (t) v′ (t) F (α (t)) + v′ (t)2G (α (t))dt

Analogamente:

l (φ ◦ β) =

∫ t1

t0

∣∣(φ ◦ β)′(t)

∣∣dt =

∫ t1

t0

∣∣u′ (t) Su (α (t)) + v′ (t)Sv (α (t))∣∣dt

=

∫ t1

t0

√u′ (t)2

E (α (t)) + 2u′ (t) v′ (t) F (α (t)) + v′ (t)2G (α (t))dt

Mas S e S sao isometricas, ou seja, E (α (t)) = E (α (t)) , F (α (t)) = F (α (t)) e G (α (t)) =

G (α (t)) . Logo, l (β) = l (φ ◦ β) . �

Page 122: Numero 09 - Outubro de 2007

Proposicao 6. Se S, S : U ⊂ R2 −→ R

3 sao superfıcies isometricas, entao φ = S ◦ S−1 euma isometria entre S e S.

Demonstracao.

Sejam d distancia intrınseca em S (U) e d distancia intrınseca em S (U) . Vamos mostrarque d (x1, x2) = d (φ (x1) , φ (x2)) .

S

x2

��1

� �

x1

�( )x2

S

�( )x1� � �

Vimos na proposicao anterior que A = B sendo

A = {l (β) : β e curva em S ligando x1 a x2}

B ={l (φ ◦ β) : φ ◦ β e uma curva em S ligando φ (x1) a φ (x2)

}.

Logo, inf A = inf B e, portanto, d (x1, x2) = d (φ (x1) , φ (x2)) . Como x1 e x2 sao ar-bitrarios, temos que φ e isometria. �

Proposicao 7. Sejam S, S : U ⊂ R2 −→ R

3 superfıcies simples. Se a aplicacao φ :

S (U) −→ S (U) , tal que φ = S◦S−1, preserva comprimento de curvas, entao as superfıciesS e S sao isometricas.

Demonstracao.

Se β = S ◦ α e uma curva em S, entao, por hipotese, l (β) = l (φ ◦ β) .

S

x2

��1

� �

x1

�( )x2

S

�( )x1� � �

Seja α (t) = P + t−→v , sendo P = (u0, v0) ∈ U, −→v = (m,n) um vetor nao nulo e α (t) ∈ U

para ∀t ∈ [0, b] .

u

v

U

v

P = (u ,v )0 0

Page 123: Numero 09 - Outubro de 2007

Logo, α (t) = (u0 + tm, v0 + tn) . Sejam β = S ◦α e β = φ ◦β. As funcoes comprimentode arco para β e β sao dadas por:

S (t) =

∫ t

0

|β′ (u)| du; S (b) = l (β)

e

S (t) =

∫ t

0

∣∣∣β′(u)

∣∣∣du; S (b) = l(β)

Como φ preserva comprimento de curvas temos S (t) = S (t) ; ∀t ∈ [0, b] . Logo,

S′ (t) = S′(t) =⇒ |β′ (t)| =

∣∣∣β′(t)

∣∣∣ , ∀t.

Mas α (0) = P = (u0, v0) e α′ (0) = (m,n) = −→v . Alem disso,

|β′ (0)|2

= Iα(0) (β′ (0))

e ∣∣∣β′(0)

∣∣∣2 = Iα(0)

(β′(0)

).

Assim,

IP (β′ (0)) = IP

(β′(0)

)=⇒

m2E (P) + 2mnF (P) + n2G (P) = m2E (P) + 2mnF (P) + n2G (P) .

Como (m,n) e um vetor qualquer tal que m �= 0 ou n �= 0, vamos supor que m = 0 en �= 0. Assim:

n2G (P) = n2G (P) =⇒ G (P) = G (P) .

Se n = 0 e m �= 0 temos:

m2E (P) = m2E (P) =⇒ E (P) = E (P) .

Se n �= 0 e m �= 0 temos

F (P) = F (P) .

Logo, S e S sao isometricas. �

Exemplo: Vimos na subsecao anterior que as superfıcies:

S : ]0, 2π[ × R −→ S (]0, 2π[ × R) ⊂ R3

(u, v) �−→ (u, v, 0)

e S : ]0, 2π[ × R −→ S (]0, 2π[ × R) ⊂ R3

(u, v) �−→ (cos (u) , sen (u) , v)

sao isometricas. Logo, a aplicacao φ : S (]0, 2π[ × R) −→ S (]0, 2π[ × R) dada por

φ (u, v, 0) = S ◦ S−1 (u, v, 0) = S (u, v) = (cos (u) , sen (u) , v)

e uma isometria entre S (]0, 2π[ × R) (parte do plano) e S (]0, 2π[ × R) (parte do cilindro).

Page 124: Numero 09 - Outubro de 2007

1.16 Funcao Curvatura Normal

Sejam S : U ⊂ R2 −→ R

3 uma superfıcie regular e P ∈ U. A funcao curvatura normal kN

de S em P e definida por:

kN : TPS − {0} −→ R

w �−→ IIP (w)

IP (w)

.

Mostra-se que kN nao depende do modulo de w e, portanto, podemos falar de funcaocurvatura normal segundo uma direcao no plano tangente.

Interpretacao Geometrica da Curvatura Normal

Seja w ∈ TPS − {0} tal que |w| = 1.

Existe uma curva β : I ⊂ R −→ R3; (β = S ◦ α) ; tal que β′ (t0) = w e β (t0) = S (P) .

Vamos supor β parametrizada pelo comprimento de arco.Temos:

kN (w) =IIP (w)

IP (w)

=〈β′′ (t0) , η (P)〉

|w|2

= 〈β′′ (t0) , η (P)〉= 〈T ′ (t0) , η (P)〉= 〈k (t0)N (t0) , η (P)〉= k (t0) 〈N (t0) , η (P)〉 ,

sendo k (t0) a curvatura de β em t0, T (t0) = β′ (t0) vetor velocidade de β em t0, N (t0)

o vetor normal a curva β em t0 e η (P) =Su × Sv

|Su × Sv|(P) vetor normal unitario a superfıcie

S em P.

Seja θ o angulo entre N (t0) e η (P) . Temos:

cos (θ) =〈N (t0) , η (P)〉|N (t0)| . |η (P)|

= 〈N (t0) , η (P)〉 .

Logo, kN = k (t0) cos (θ) .

S(P)

�(P)

N(t )0

’(t )0

S

Seja w ∈ TPS. Consideremos a curva β determinada pela interseccao de S com o planodeterminado por η (P) , w e S (P) . Esta curva e denominada seccao normal da superfıcieS determinada por w ∈ TPS.

Page 125: Numero 09 - Outubro de 2007

S(P)

�(P)

S

w

Pode-se provar que existe t0 ∈ I tal que β (t0) = S (P) , β′ (t0) = w e N (t0) = ±η (P) .

Desta forma, kN (w) = ±k (t0) , pois o angulo entre η (P) e N (t0) e 0 ou π radianos.Logo,

|kN (w)| = k (t0) .

Conclusao: o modulo da curvatura normal de S em P na direcao de w e a curvatura emt0 da curva obtida pela interseccao de S com o plano determinado por S (P) , w e η (P) .

Com isso, |kN (w)| mede o quanto a superfıcie S “se afasta” da direcao tangente determi-nada por w.

Observacoes:(1) Os pontos X do traco de β sao os pontos β (t) = S (u (t) , v (t)) que satisfazem aequacao

〈X − S (P) , η (P) × w〉 = 0.

O primeiro vetor esta no plano normal a S em t0 que determina β e o segundo e perpen-dicular a esse plano.

(2) Se β e uma curva na superfıcie S, a curvatura normal de S em β (t) na direcao dew = β′ (t) e dada por:

kN =IIα(t) (β′ (t))Iα(t) (β′ (t))

,

sendo:

β (t) = S ◦ α (t) ;

α (t) = (x (t) , y (t)) ;

β′ (t) = x′ (t) Su (α (t)) + y′ (t) Sv (α (t)) .

Exemplo 1:

(i) Seja S (u, v) = (u, v, 1) , (u, v) ∈ R2. (S e um plano)

y

z

x

wS(P)

�(P)

S ( )�2

P

R2

S

Page 126: Numero 09 - Outubro de 2007

Temos TPS = S(R

2).

Seja w ∈ TPS. O plano determinado por w, S (P) e η (P) determina uma reta β em S, ouseja, as seccoes normais determinadas por qualquer w sao retas, cuja curvatura e nula.Logo:

|kN (w)| = k (t0) = 0 =⇒ kN (w) = 0, ∀w ∈ TPS.

(ii) Seja S (u, v) = (a sen (v) cos (u) , a sen (v) sen (u) , a cos (v)) , (u, v) ∈ R2; a > 0.

As seccoes normais na esfera sao cırculos maximos β, cuja curvatura e constante e igual

a1

a. Logo:

|kN (w)| = k (t0) =1

a=⇒ kN (w) = ± 1

a.

O sinal depende da parametrizacao de S.

}

a

�(P)

S(P)

w

Existem direcoes determinadas por w onde a curvatura normal e maxima ou mınima.Vejamos, primeiramente, um exemplo.

Exemplo 2: Sejam S (u, v) = (cos (u) , sen (u) , v) , (u, v) ∈ R2 e P = (u, v) .

Temos:

E (P) = 1, F (P) = 0, G (P) = 1.

e (P) = −1, f (P) = 0, g (P) = 0.

Seja:w = aSu (P) + bSv (P) ∈ TPS − {0} .

Logo:

kN =IIP (w)

Ip (w)=

−a2

a2 + b2=⇒ −1 ≤ kN (w) ≤ 0, ∀w ∈ TPS − {0} .

Seja w1 = 1Su (P)+0Sv (P) = (− sen (u) , cos (u) , 0) . Temos kN (w1) = −1 (valor mınimode kN).Seja w2 = 0Su (P) + 1Sv (P) = (0, 0, 1) . Temos kN (w2) = 0 (valor maximo de kN). Ascurvas normais que correspondem aos valores mınimos sao cırculos de raio 1. As curvasnormais que correspondem aos valores maximos sao retas.

A demonstracao da proposicao abaixo pode ser encontrada em [6], paginas 164, 165, 166

e 167.

Proposicao 8. Sejam S : U ⊂ R2 −→ R

3 uma superfıcie regular, P ∈ U e kN :

TPS − {0} −→ R a funcao curvatura normal de S em P. Entao, existem vetores ortogonais

Page 127: Numero 09 - Outubro de 2007

e unitarios w1 e w2 em TPS tais que k1 = kN (w1) e valor mınimo de kN e k2 = kN (w2)

e valor maximo de kN.

Os valores k1 e k2 da proposicao acima sao chamados de curvaturas principais de S emP e os vetores w1 e w2 sao os vetores principais de S em P. As direcoes determinadas porw1 e w2 sao chamadas de direcoes principais de S em P.

A demonstracao da proposicao abaixo pode ser encontrada em [6], paginas 169 e 170.

Proposicao 9. (Formula de Euler) Sejam S : U ⊂ R2 −→ R

3 superfıcie regular; P ∈ U,

w1, w2 os vetores principais de S em P e k1, k2 as curvaturas principais de S em P. Sejaw ∈ TPS − {0} tal que |w| = 1. Se w = w1 cos (θ) + w2 sen (θ) , entao

kN (w) = k1 cos2 (θ) + k2 sen2 (θ) .

1.17 Curvatura Gaussiana e Curvatura Media de S em P

Seja S : U ⊂ R2 −→ R

3 superfıcie regular com curvaturas principais k1 e k2 em P.

DefinimosK (P) = k1k2

como sendo a curvatura gaussiana de S em P e

H (P) =k1 + k2

2

como sendo a curvatura media de S em P.

Observacao: Conhecendo-se K (P) e H (P) , podemos encontrar as curvaturas principaisresolvendo a equacao do segundo grau:

x2 − 2H (P) x + K (P) = 0.

A demostracao da proposicao abaixo pode ser encontrada em [6] , paginas 171 e 172.

Proposicao 10. Sejam S : U ⊂ R2 −→ R

3 superfıcie regular e p ∈ U. Entao, ascurvaturas gaussiana e media sao dadas por:

K (P) =e (P)g (P) − f (P)

2

E (P)G (P) − F (P)2

e

H (P) =1

2.e (P) G (P) − 2f (P) F (P) + E (P) g (P)

E (P) G (P) − F (P)2

,

sendo E (P) , F (P) e G (P) coeficientes de da Primeira Forma Quadratica de S em P ee (P) , f (P) e g (P) os coeficientes da Segunda Forma Quadratica de S em P.

Exemplos:

(1) Seja S (u, v) = (cos (u) , sen (u) , v) , (u, v) ∈ R2 a parametrizacao de um cilindro.

Page 128: Numero 09 - Outubro de 2007

Temos:

E (P) = 1, F (P) = 0, G (P) = 1

e (P) = −1, f (P) = 0, g (P) = 0

Assim, K (P) =0

1= 0, ∀P ∈ U e H (P) =

1

2

(−

1

1

)= −

1

2�= 0, ∀P ∈ U.

(2) Seja S (u, v) =(u, v, v2 − u2

), (u, v) ∈ R

2. (paraboloide hiperbolico)Temos:

E (0, 0) = 1, F (0, 0) = 0, G (0, 0) = 1

e (0, 0) = −2, f (0, 0) = 0, g (0, 0) = 2

Assim, K (0, 0) = −4 e H (0, 0) = 0. As curvaturas principais em P = (0, 0) sao k1 = −2

e k2 = 2.

(3) Sejam S (u, v) =((

1 − u3)cos (v) , u,

(1 − u3

)sen (v) + 1

), −1 < u < 1 e v ∈ R.

Esta superfıcie e obtida pela rotacao da curva α (u) =(0, u, u3

), −1 < u < 1, em torno

da reta z = 1 contida no plano yz. Seu nome e Chapeu de Sherlock. Temos:

E (0, v) = 1, F (0, v) = 0, G (0, v) = 1

e (0, v) = 0, f (0, v) = 0, g (0, v) = −1

para qualquer (0, v) ∈ R2.

S(0,v)

x

y

z

Logo, K (0, v) = 0 e H (0, v) = −1

2para todo v ∈ R. As curvaturas principais em (0, v)

sao k1 = −1 e k2 = 0.

Dizemos que S : U ⊂ R2 −→ R

3 regular e uma superfıcie mınima se H (P) = 0; ∀P ∈ U.

Dentre as superfıcies regulares em R3, duas classes se destacam:

(1) As superfıcies de curvatura gaussiana constante (para qualquer ponto), como porexemplo:(1 − i) plano e cilindro, no qual K (P) = 0, ∀P.

(1 − ii) esfera de raio r > 0, na qual K (P) =1

r2, ∀P.

Page 129: Numero 09 - Outubro de 2007

(1 − iii) pseudo-esfera, na qual K (P) = −1, ∀P. A pseudo-esfera pode ser obtida pelarotacao da tratriz:

α (t) =

(sen (t) , 0, cos (t) + ln

(tan

(t

2

))), t ∈

]0,

π

2

[,

em torno do eixo z.

(2) As superfıcies mınimas, como por exemplo:(2 − i) plano, no qual H (P) = 0, ∀P.

(2 − ii) catenoide no qual H (P) = 0, ∀P.

(2 − iii) helicoide, no qual H (P) = 0, ∀P. (superfıcie gerada por todos os segmentosparalelos a xy ligando o eixo z a uma helice cilındrica de eixo z.

Algumas propriedades geometricas interessantes em superfıcies com K (P) = c ou H (P) =

0, ∀P, sao:

(1) Se S e S possuem mesma curvatura gaussiana constante, entao e possıvel restringir osdomınios de S e S de tal modo que exista uma isometria entre S e S (homogeneidade).

(2) Se S e superfıcie mınima, entao considerando uma regiao suficientemente pequena emS, a area dessa regiao e menor do que ou igual a area de qualquer superfıcie que tenha amesma fronteira da regiao (e a superfıcie da “pelıcula de sabao”).

1.18 Classificacao de Pontos em uma Superfıcie

Seja S : U ⊂ R2 −→ R

3 superfıcie regular e P ∈ U. Dizemos que o ponto P e:(i) elıptico quando K (P) > 0.

(ii) hiperbolico quando K (P) < 0.

(iii) parabolico quando K (P) = 0 e H (P) �= 0.

(iv) planar quando K (P) = 0 e H (P) = 0.

Exemplo 1:

(i) Todos os pontos de um plano sao planares, pois K (P) = H (P) = 0, ∀P.

(ii) Todos os pontos de uma esfera de raio r sao elıpticos pois K (P) =1

r2, ∀P.

(iii) O ponto P = (0, 0) do paraboloide hiperbolico S (u, v) =(u, v, v2 − u2

), (u, v) ∈

R2 e hiperbolico pois K (0, 0) = −4. Na verdade, todos os pontos de um paraboloide

hiperbolico sao hiperbolicos.

(iv) Todos os pontos do cilindro S (u, v) = (cos (u) , sen (u) , v) , (u, v) ∈ R2 sao parabolicos,

pois K (P) = 0 e H (P) = −1

2�= 0, ∀P ∈ U.

Exemplo 2:

(i) No “Chapeu de Sherlock” S (u, v) =((

1 − u3)cos (v) , u,

(1 − u3

)sen (v) + 1

), −1 <

u < 1 e v ∈ R, os pontos da forma S (u, v) com u = 0 sao parabolicos; com −1 < u < 0

sao hiperbolicos e; com 0 < u < 1 sao elıpticos.

Page 130: Numero 09 - Outubro de 2007

(ii) No toro S (u, v) = ((a + r cos (u)) cos (v) , (a + r cos (u)) sen (v) , r sen (u)) , (u, v) ∈R

2, 0 < r < a temos:(a) Se −

π

2+ 2hπ < u <

π

2+ 2hπ, h ∈ Z, entao os pontos P = (u, v) sao elıpticos.

(b) Se u =π

2+ hπ, h ∈ Z, entao os pontos P = (u, v) sao pontos parabolicos.

(c) Seπ

2+ 2hπ < u <

2+ 2hπ, h ∈ Z, entao os pontos P = (u, v) sao hiperbolicos.

Observacao: Em uma superfıcie mınima temos k1 = −k2. Logo:

K (P) = − (k2)2 ≤ 0, ∀P ∈ U.

Conclusao: Em uma superfıcie mınima todos os pontos sao hiperbolicos ou planares.

Planos Tangentes e Classificacao de Pontos

(1) Sejam S : U ⊂ R2 −→ R

3 superfıcie regular e P ∈ U. Vimos que P e elıptico quandoK (P) > 0. Mas, K (P) = k1k2 > 0 =⇒ k1, k2 > 0 ou k1, k2 < 0. Lembrando que acurvatura normal

kN : TPS − {0} −→ R

w �−→ kN (w)

e tal que k1 ≤ kN (w) ≤ k2 para todo w, temos:

(1 − i) Se k1, k2 > 0, entao:

0 < k1 ≤ kN ≤ k2 =⇒ kN (w) > 0, ∀w.

Vimos que |kN (w)| = k (t0) sendo k (t0) a curvatura da curva β que e a seccao normalda superfıcie S determinada por w, (β (t0) = S (P)).

Como kN (w) > 0 para qualquer w ∈ TPS − {0} , entao k (t0) > 0 para qualquer seccaonormal da superfıcie S passando por S (P) .

Vimos em curvas planas que se k (t0) > 0, entao a curva β tem concavidade voltada parao sentido do vetor β′′ (t0) . Mas

N (t0) =β′′ (t0)

|β′′ (t0)|

e o vetor normal unitario a curva β em β (t0) e vimos

kN (w) = k (t0) cos (θ)

Page 131: Numero 09 - Outubro de 2007

sendo θ o angulo entre os vetores N (t0) e η (P) , com η (P) vetor unitario normal a S emP. Como, neste caso:

cos (θ) = 1 =⇒ θ = 0 =⇒ N (t0) = η (P)

para qualquer seccao normal de S passando por S (P) , ou seja, todas as seccoes normaisβ possuem concavidades voltadas para o sentido do vetor η (P) .

Conclusao: Existe uma vizinhanca V de P tal que S (V) esta contido em apenas um dossemiespacos determinados pelo plano tangente a S em S (P) .

T SP

S(P) = (t )

N(t ) = (P)

= S

� ���

0

0

N(t )0

S

�S(P)

(1 − ii) Se k1, k2 < 0, entao:

k2 ≤ kN (w) ≤ k1 < 0 =⇒ kN (w) = 0, ∀w ∈ TPS − {0} .

Como kN (w) = k (t0) cos (θ) e k (t0) > 0 (as parametrizacoes de β sao tais que a cur-vatura k (t0) seja positiva), temos que

cos (θ) = −1 =⇒ θ = π,

ou seja, η (P) e N (t0) possuem sentidos opostos. Assim, para qualquer seccao normal deS passando por S (P) temos a concavidade voltada para o sentido do vetor −η (P) .

Conclusao: Existe uma vizinhaca V de P tal que S (V) esta contido em apenas um dossemiespacos determinado pelo plano tangente a S em S (P) .

S(P)

TPS

S

�(S)

N(t )0

(2) Seja S : U ⊂ R2 −→ R

3 superfıcie regular e P ∈ U. Vimos que P e hiperbolico quandoK (P) < 0. Mas K (P) = k1k2 < 0 =⇒ k1 e k2 possuem sinais opostos. Neste caso temosseccoes normais a S passando por S (P) com concavidades locais voltadas para os doissemiespacos determinados por TPS.

S(P)

�(P)

T SP

Page 132: Numero 09 - Outubro de 2007

(3) Seja S : U ⊂ R2 −→ R

3 superfıcie regular e P ∈ U.

Vimos que P e parabolico quando K (P) > 0 e H (P) �= 0.

Mas K (P) = k1k2 = 0 ⇐⇒ k1 = 0 ou k2 = 0.

Como H (P) =k1 + k2

2�= 0 =⇒ k1 e k2 nao sao nulos ao mesmo tempo.

Assim, se k1 = 0 temos:

0 = k1 ≤ kN (w) ≤ k2 =⇒ 0 ≤ kN (w) .

Se k2 = 0 temos:

k1 ≤ kN (w) ≤ k2 = 0 =⇒ kN (w) ≤ 0.

Geometricamente temos que ha uma seccao normal de S passando por S (P) na qual acurvatura k (t0) e nula e todas as outras seccoes normais a S passando por S (P) estaocom concavidades locais voltadas para um mesmo semiespaco determinado por TPS. Noentanto, isso nao quer dizer que, localmente, todas as secoes normais estao com concavi-dades voltadas para um mesmo lado de TpS. A secao normal que possui curvatura nulapode ter comportamento do tipo da curva y = x3 no plano. No “Chapeu de Scherlock”ha exemplos desse tipo de comportamento.

(4) Seja S : U ⊂ R2 −→ R

3 superfıcie regular e P ∈ U. Vimos que P e planar quando

K (P) = H (P) = 0. Como K (P) = K1K2 e H (P) =K1 + K2

2, entao:

K1 = K2 = 0 =⇒ KN (w) = 0; ∀w ∈ TpS.

Geometricamente, toda seccao normal a S passando por S (P) possui curvatura nula, ouseja, localmente (em torno de S (P)) a superfıcie S se comporta como um plano.

Assim como no caso dos pontos parabolicos, nada se pode afirmar sobre a concavidadedas seccoes normais em P ∈ S em relacao ao plano tangente TPS. Dois exemplos docomportamento erratico dos pontos planares com relacao ao plano tangente sao dadospelas superfıcies S (u, v) =

(u, v, u3 − 3uv2

), (u, v) ∈ R

2 (Sela de Macaco) e S (u, v) =(v4 cos (u) , v4 sen (u) , v

), (u, v) ∈ R

2 (rotacao de z = x4 em torno do eixo z).

1.19 Pontos Umbılicos

Seja S : U ⊂ R2 −→ R

3 superfıcie regular e P ∈ U. Dizemos que um ponto P e umbılicode S quando k1 = k2 em P, ou seja, a curvatura normal kN e constante em P.

Exemplo 1: Os pontos umbılicos de uma superfıcie mınima sao planares.

De fato: como em uma superfıcie mınima k1 = −k2, se P e umbılico, entao k1 = k2, ouseja,

k1 = k2 = 0 =⇒ K (P) = H (P) = 0.

Proposicao 11. Se P e um ponto umbılico de uma superfıcie S, entao H (P)2

= K (P) .

Demonstracao.

Page 133: Numero 09 - Outubro de 2007

Temos:

H (P)2− K (P) =

(k1 + k2

2

)2

− k1k2

=k2

1 + 2k1k2 + k22 − 4k1k2

4

=k2

1 − 2k1k2 + k22

4

=(k1 − k2)

2

2

≥ 0.

Como P e umbılico: k1 = k2. Logo:

H (P)2− K (P) = 0 =⇒ H (P)

2= K (P) ,

como querıamos. �

Exemplo 2:

(i) Na esfera todos os pontos sao umbılicos pois k1 = k2 =1

r.

(ii) No plano todos os pontos sao umbılicos pois k1 = k2 = 0.

Proposicao 12. Sejam S : U ⊂ R2 −→ R

3 superfıcie regular e P ∈ U. Entao, P eumbılico se, e somente se, ∃λ ∈ R tal que e (P) = λE (P) ; f (P) = λF (P) e g (P) = λG (P) .

Demonstracao.

=⇒) Se P e umbılico, entao:

k1 = k2 = λ =⇒ kN (w) =IIP (w)

IP (w)= λ (pois k1 ≤ kN (w) ≤ k2).

Se w = aSu (P) + bSv (P) temos:

kN =a2e (P) + 2abf (P) + b2g (P)

a2E (P) + 2abF (P) + b2G (P)= λ =⇒

a2e (P) + 2abf (P) + b2g (P) = a2λE (P) + 2abλF (P) + b2λG (P)

para quaisquer (a, b) ∈ R2 − {(0, 0)} .

Fazendo: ⎧⎨⎩

a = 1 e b = 0 temos e (P) = λE (P)

a = 0 e b = 1 temos g (P) = λG (P)

a = 1 e b = 1 temos f (P) = λF (P).

⇐=) Se e (P) = λE (P) , f (P) = λF (P) e g (P) = λG (P) temos:

IIP (w) = λIP (w) =⇒ IIP (w)

IP (w)= λ =⇒ kN (w) = λ =⇒ P e umbılico.

Page 134: Numero 09 - Outubro de 2007

Exemplo 3. Determinacao de todos os pontos umbılicos de um paraboloide circular

S (u, v) =(u, v, u2 + v2

), (u, v) ∈ R

2.

x

y

z

Temos:

Su (u, v) = (1, 0, 2u) e Sv (u, v) = (0, 1, 2v) ;

(Su × Sv) (u, v) = (−2u,−2v, 1) ;

Suu (u, v) = (0, 0, 2) , Svv (u, v) = (0, 0, 2) e Suv (u, v) = (0, 0, 0) .

Temos:

η (u, v) =(−2u,−2v, 1)√4u2 + 4v2 + 1

.

Assim:

E (u, v) = 1 + 4u2, F (u, v) = 4uv e G (u, v) = 1 + 4v2;

e (u, v) =2√

4u2 + 4v2 + 1, f (u, v) = 0 e g (u, v) =

2√4u2 + 4v2 + 1

.

Vimos que P = (u, v) e umbılico se, e somente se, e (P) = λE (P) , f (P) = λF (P) eg (P) = λG (P) , sendo λ = k1 = k2 = kN (w) , ∀w ∈ TPS − {0} .

Seja λ ∈ R tal que

e (u, v) = λE (u, v) =⇒ 2√4u2 + 4v2 + 1

= λ(1 + 4u2

)(1)

f (u, v) = λF (u, v) =⇒ 0 = λ4uv (2)

g (u, v) = λG (u, v) =⇒ 2√4u2 + 4v2 + 1

= λ(1 + 4v2

)(3)

De (1) temos λ �= 0. De (2) temos u = 0 ou v = 0. De (1) e (3) temos u2 = v2. Logo,u = v = 0 e P = (0, 0) e o unico ponto umbılico de S.

A demonstracao da proposicao abaixo pode ser encontrada em [6] , paginas 189, 190 e191.

Proposicao 13. Seja S : U ⊂ R2 −→ R

3 uma superfıcie regular, sendo U um abertoconexo do R

2. Se todo P ∈ U e um ponto umbılico de S, entao a curvatura gaussiana K econstante e K (P) ≥ 0, ∀P ∈ U.

Se K (P) = 0, entao S (U) esta contida em um plano.

Se K (P) > 0, entao S (U) esta contida numa esfera de raio1√

K (P).

Page 135: Numero 09 - Outubro de 2007

1.20 Linhas de Curvatura

Sejam S : U ⊂ R2 −→ R

3 uma superfıcie regular e α : I ⊂ R −→ R3 curva regular

tal que α (t) ∈ S (U) , ou seja, ∃β (t) = (u (t) , v (t)) ∈ U tal que α (t) = S ◦ β (t) =

S (u (t) , v (t)) .

Dizemos que α e uma linha de curvatura em S quando α′ (t) e uma direcao principal deS para todo t ∈ I.

S

x

y

z

S

��

u

v

U

I

(t)�(t)

t

�’(t)

�’(t) é vetor principal

Exemplo 1:

(1 − i) Plano. Temos que kN (w) = 0 para todo w ∈ TPS − {0} . Logo, qualquer vetor noplano e vetor principal e, portanto, qualquer direcao no plano e direcao principal. Destemodo, qualquer curva regular no plano e uma linha de curvatura.

(1 − ii) Esfera. Temos que kN (w) =1

r, ∀w ∈ TPS − {0} . Logo, qualquer w determina

uma direcao principal na esfera. Deste modo, qualquer curva regular na esfera e umalinha de curvatura.

(1 − iii) Cilindro. As linhas de curvatura no cilindro sao os meridianos (retas) e osparalelos (circunferencias).

linhas de curvatura

As demonstracoes das proposicoes abaixo podem ser encontradas em [6] , paginas 195,

197, 198 e 199.

Page 136: Numero 09 - Outubro de 2007

Proposicao 14. Sejam S : U ⊂ R2 −→ R

3 uma superfıcie regular e α (t) = S (u (t) , v (t))

uma curva regular sobre S. Entao, α e uma linha de curvatura se, e somente se, as funcoesu (t) e v (t) satisfazem

det

⎡⎣ v′ (t)2−u′ (t) v′ (t) u′ (t)2

E (u (t) , v (t)) F (u (t) , v (t)) G (u (t) , v (t))

e (u (t) , v (t)) f (u (t) , v (t)) g (u (t) , v (t))

⎤⎦ = 0,

sendo E, F, G, e, f, g os coeficientes da primeira e segunda formas quadraticas de S em(u (t) , v (t)) .

Proposicao 15. (Olinde Rodrigues) Sejam S : U ⊂ R2 −→ R

3 uma superfıcie regular eα (t) = S (u (t) , v (t)) , t ∈ I, uma curva regular em S. Entao, α e uma linha de curvaturaem S se, e somente se, ∃λ : I −→ R diferenciavel tal que η′ (t) + λ (t) α′ (t) = 0, ∀t ∈ I,

sendo η (t) = η (u (t) , v (t)) . Neste caso, λ (t) = kN (α′ (t)) .

Proposicao 16. Sejam S : U ⊂ R2 −→ R

3 uma superfıcie regular e P ∈ U um pontonao umbılico de S. Entao, existe uma vizinhanca V ⊂ U, P ∈ V tal que ∀Q ∈ V, Q eum ponto nao umbılico de S. Alem disso, se Q ∈ V, existem exatamente duas linhas decurvatura em P, α1 e α2, tais que α1 (t) = S (u1 (t) , v1 (t)) e α2 (t) = S (u2 (t) , v2 (t))

com (u1 (0) , v1 (0)) = Q = (u2 (0) , v2 (0)) .

�1 �2

S(Q)

S

Q não é umbílico

Observacao: Se o ponto P for umbılico na proposicao acima, nada podemos afirmarsobre a existencia de linhas de curvatura. Considere os seguintes exemplos.

Exemplo 2:

(2 − i) Seja S (u, v) =(u, v, u2 + v2

), (u, v) ∈ R

2 (paraboloide circular). Vimos queP = (0, 0) e o unico ponto umbılico de S. Nesse caso, E (P) = 1, F (P) = 0, G (P) = 1,

e (P) = 2, f (P) = 0 e g (P) = 2. Seja α (t) = S (u (t) , v (t)) uma linha de curvatura. PelaProposicao 14, as funcoes u (t) e v (t) devem satisfazer:

δ = det

⎡⎣ v′ (t)2−u′ (t) v′ (t) u′ (t)2

E (u (t) , v (t)) F (u (t) , v (t)) G (u (t) , v (t))

e (u (t) , v (t)) f (u (t) , v (t)) g (u (t) , v (t))

⎤⎦ = 0.

No entanto, quaisquer funcoes u (t) , v (t) satisfazem δ = 0. Logo, temos infinitas linhasde curvatura em P = (0, 0) .

(2 − ii) Pelos pontos umbılicos do elipsoide nao existem linhas de curvatura.

Page 137: Numero 09 - Outubro de 2007

Exemplo 3: Se S e superfıcie de rotacao entao os meridianos e paralelos sao linhas decurvatura. De fato:

(1) α (t) = S (u0, v (t)) e tal que α′ (t) = Su (u0, v (t)) 0 + Sv (u0, v (t)) v′ (t) ;

(2) β (t) = S (u (t) , v0) e tal que β′ (t) = Su (u (t) , v0) u′ (t) + Sv (u (t) , v0) 0.

Em ambos os casos, u′ (t) = 0 e v′ (t) = 0.

Por outro lado, e facil verificar que F (u (t) , v (t)) = f (u (t) , v (t)) = 0 em uma superfıciede rotacao.\

Logo, o determinante da Proposicao 14 sempre se anula para os meridianos e paralelos,ou seja, os mesmos sao linhas de curvatura.

1.21 Linhas Assintoticas

Seja S : U ⊂ R2 −→ R

3 uma superfıcie regular e P ∈ U. Uma direcao assintotica em P euma direcao determinada por um vetor tangente w ∈ TPS − {0} tal que kN (w) = 0.

Exemplo 1:

(1 − i) No plano, todo w ∈ TPS − {0} e tal que kN (w) = 0. Logo, qualquer direcao noplano e uma direcao assintotica.

(1 − ii) Na esfera, kN (w) =1

r�= 0, ∀w ∈ TPS − {0} . Logo, nao existem direcoes

assintoticas na esfera.

(1 − iii) Seja o paraboloide hiperbolico S (u, v) =(u, v, u2 − v2

), (u, v) ∈ R

2. Seja P =

(0, 0) .

Se w = aSu (0, 0) + bSv (0, 0) , temos

kN (w) =−2a2 + 2b2

a2 + b2.

Assim:

kN (w) = 0 ⇐⇒ a = ±b.

Como Su (0, 0) = (1, 0, 0) e Sv (0, 0) = (0, 1, 0) temos

w = a (1, 0, 0) + b (0, 1, 0) =⇒ w = (a, a, 0) ou w = (a,−a, 0)

determinam as direcoes assintoticas em P = (0, 0) .

direções assintóticas

S(P)

S

Page 138: Numero 09 - Outubro de 2007

A demonstracao da proposicao abaixo pode ser encontrada em [6] , paginas 200 e 201.

Proposicao 17: Seja S : U ⊂ R2 −→ R

3 uma superfıcie regular e seja P ∈ U.

(a) Se P e elıptico, entao nao existem direcoes assintoticas em P.

(b) Se P e hiperbolico, entao existem exatamente duas direcoes assintoticas em P.

(b) Se P e parabolico, entao existe exatamente uma direcao assintotica em P, que tambeme principal.(d) Se P e planar, entao todas as direcoes sao assintoticas em P.

Seja S : U ⊂ R2 −→ R

3 uma superfıcie regular. Uma curva regular α (t) = S (u (t) , v (t)) ,

t ∈ I ⊂ R, e uma linha assintotica de S, se para todo t ∈ I, α′ (t) determina uma direcaoassintotica de S em (u (t) , v (t)) .

Exemplo 2:

(2 − i) Toda curva regular do plano e uma linha assintotica.

(2 − ii) Na esfera nao ha linhas assintoticas.

(2 − iii) No cilindro os meridianos (retas) sao linhas assintoticas.

linhas assintóticas

(2 − iv) Se α (t) = S (u (t) , v (t)) e uma reta em S, entao α e uma linha assintotica.De fato: kN (α′ (t)) = k (t) cos (θ) , sendo θ o angulo entre N (t) e η (u (t) , v (t)) .

Logo, kN (α′ (t)) = 0, ∀t ∈ I, isto e, α e linha assintotica.Desta forma, no paraboloide hiperbolico S (u, v) =

(u, v, u2 − v2

), (u, v) ∈ R

2 temos queas retas α1 (t) = (t, t, 0) e α2 (t) = (t,−t, 0) sao linhas assintoticas.

Observacao: Seja α (t) = S (β (t)) uma linha assintotica em S, sendo β (t) = (u (t) , v (t))

Entao:

kN (α′ (t)) = 0 ⇐⇒ IIβ(t) (α′ (t))Iβ(t) (α′ (t))

= 0 ⇐⇒ IIβ(t) (α′ (t)) = 0. (4)

Temos α′ (t) = Su (β (t))u′ (t) + Sv (β (t)) v′ (t) , isto e, a = u′ (t) e b = v′ (t). Logo, de(4):

u′ (t)2e (β (t)) + 2u′ (t) v′ (t) f (β (t)) + v′ (t)2

g (β (t)) = 0

que e uma equacao diferencial ordinaria que permite encontrar as linhas assintoticas. (ue v sao as solucoes)

Exemplo 3:

Page 139: Numero 09 - Outubro de 2007

(3 − i) Consideremos S (u, v) = (cos (u) , sen (u) , v) , (u, v) ∈ R2.

Temos:

Su (u, v) = (− sen (u) , cos (u) , 0) e Sv (u, v) = (0, 0, 1) ;

η (u, v) = (cos (u) , sen (u) , 0) ;

Suu (u, v) = (− cos (u) ,− sen (u) , 0) , Suv (u, v) = (0, 0, 0) e Svv (u, v) = (0, 0, 0) .

Logo, e (u, v) = 1, f (u, v) = 0 e g (u, v) = 0. A equacao diferencial ordinaria acima fica

u′ (t)21 = 0 =⇒ u (t) = a (constante).

Assim:α (t) = (cos (a) , sen (a) , v (t)) ,

com v (t) qualquer.Logo, as linhas assintoticas de um cilindro sao os meridianos.

Os meridianossão as únicas

linhas assintóticasdo cilindro.

(3 − ii) Consideremos S (u, v) = (u, v, uv) , (u, v) ∈ R2.

Temos

Su (u, v) = (1, 0, v) e Sv (u, v) = (0, 1, u) ;

η (u, v) =(−v, −u, 1)

|(Su × Sv) (u, v)|;

Suu (u, v) = (0, 0, 0) , Suv (u, v) = (0, 0, 1) e Svv (u, v) = (0, 0, 0) .

Logo, e (u, v) = 0, f (u, v) =1

|(Su × Sv) (u, v)|e g (u, v) = 0. A equacao diferencial

ordinaria acima fica

2u′ (t) v′ (t)1

|(Su × Sv) (u, v)|= 0 =⇒

u′ (t) v′ (t) = 0 =⇒{u′ (t) = 0

v′ (t) = 0=⇒

u (t) = a (constante) ou v (t) = b (constante).

Assim,α (t) = (a, v (t) , av (t))

com v (t) qualquer (reta contida no plano x = a) ou

α (t) = (u (t) , b, b.u (t))

Page 140: Numero 09 - Outubro de 2007

com u (t) qualquer (reta contida no plano y = b).

(3 − iii) Consideremos S (u, v) =(u, v, u2 + v2

); (u, v) ∈ R

2. (paraboloide circular)Temos:

Su (u, v) = (1, 0, 2u) e Sv (u, v) = (0, 1, 2v) ;

η (u, v) =(−2u,−2v, 1)

|(Su × Sv) (u, v)|;

Suu (u, v) = (0, 0, 2) , Suv (u, v) = (0, 0, 0) e Svv (u, v) = (0, 0, 2) .

Logo, e (u, v) =2

|(Su × Sv) (u, v)|, f (u, v) = 0 e g (u, v) =

2

|(Su × Sv) (u, v)|. A equacao

diferencial ordinaria acima fica:

2

|(Su × Sv) (u, v)|u′ (t)2

+2

|(Su × Sv) (u, v)|v′ (t)2

= 0 =⇒u′ (t)2

+ v′ (t)2= 0 =⇒

u′ (t) = 0 e v′ (t) = 0 =⇒u (t) = a (constante) e v (t) = b (constante).

Logo, nao existem linhas assintoticas em S (alem disso, α (t) deveria ser regular =⇒α′ (t) �= 0 =⇒ u′ (t) �= 0 ou v′ (t) �= 0.

1.22 Geodesicas

Uma curva α parametrizada pelo comprimento de arco na superfıcie regular S : U ⊂R

2 −→ R3, α (t) = S (u (t) , v (t)) , t ∈ I e uma geodesica quando α′′ (t) e paralelo a

η (u (t) , v (t)) para qualquer t ∈ I, sendo que η (u (t) , v (t)) denota o vetor normal asuperfıcie S.

Observacao: Se α′′ (t) �= 0, ∀t ∈ I, N (t) =α′′ (t)|α′′ (t)|

e o vetor normal a curva α em t. Se

α e uma geodesica, entao N (t) // η (u (t) , v (t)) .

�(t)S

N�t� � ���u�t�,v�t��

N�t� � ��u�t�, v�t��

Exemplo 1:

(1 − i) Todas as retas em uma superfıcie regular S sao geodesicas.De fato, se α (t) e uma reta em S, entao:

α′′ (t) = 0 =⇒ α′′ (t) // η (u (t) , v (t)) , (α′′ (t) = 0.η (u (t) , v (t))) .

Page 141: Numero 09 - Outubro de 2007

Observacao: Seja α (t) uma seccao normal de S em P = α (t0) ∈ U. Embora N (t0) //

η (P) , nem sempre a seccao α e geodesica de S, pois os vetores N (t) e η (u (t) , v (t))

podem ser paralelos apenas em α (t0) .

(1 − ii) Cilindro: S (u, v) = (cos (u) , sen (u) , v) , (u, v) ∈ R2.

Os meridianos de um cilindro sao geodesicas (pois sao retas).Seja α (t) = (cos (t) , sen (t) , c) , c constante (um cırculo contido no cilindro).Temos α′′ (t) = (− cos (t) ,− sen (t) , 0) e

η (u, v) = (cos (u) , sen (u) , 0) =⇒ η (u (t) , v (t)) = η (t, c) = (cos (t) , sen (t) , 0)

(observemos que α (t) = S (t, c)).

Logo, α′′ (t) // η (t, c) , ou seja, α (t) e uma geodesica em S.

Seja α (t) = (cos (t) , sen (t) , t) (uma helice circular contida no cilindro).Temos α′′ (t) = (− cos (t) ,− sen (t) , 0) e

η (u, v) = (cos (u) , sen (u) , 0) =⇒ η (u (t) , v (t)) = η (t, t) = (cos (t) , sen (t) , 0)

(observemos que α (t) = S (t, t)).

Logo, α′′ (t) // η (t, t) , ou seja, α (t) e uma geodesica em S.

Círculos

Meridiano

Hélice

A demonstracao da proposicao abaixo pode ser encontrada em [6] , paginas 203 e 204.

Proposicao 18. Seja S : U ⊂ R2 −→ R

3 superfıcie regular. Se α (t) = S (u (t) , v (t)) ,

t ∈ I ⊂ R, e uma geodesica, entao:(a) |α′ (t)| e constante;

(b) ± d

dt(η (u (t) , v (t))) = −k (t) T (t) − τ (t) B (t) , quando α for parametrizada por

comprimento de arco e α′′ (t) �= 0. (k e curvatura e τ e a torcao)

Com o auxılio da proposicao acima, e facil provar que de uma curva α na esfera e umageodesica se, e somente se, α e um cırculo maximo.

Observacao: Considere o plano π gerado por η (u (t) , v (t)) e α′ (t) passando por α (t)

em uma superfıcie regular S. Vamos supor α parametrizada pelo comprimento de arco.Se α e uma geodesica, entao N (t) = ±η (u (t) , v (t)) . Logo, o plano π e gerado por N (t)

e T (t) = α′ (t) , ou seja, π e o plano osculador de α em t.

Page 142: Numero 09 - Outubro de 2007

Conclusao: Se α e uma geodesica em S, entao o plano osculador de α em t e perpendicularao plano tangente a superfıcie S em α (t) .

Seja S : U ⊂ R2 −→ R

3 uma superfıcie regular. Temos que Su (u, v) , Sv (u, v) e η (u, v) =Su × Sv

|Su × Sv|(u, v) sao linearmente independentes. Logo, {Su, Sv, η} forma uma base de R

3

para cada (u, v) ∈ U.

Deste modo, os vetores Suu (u, v) , Suv (u, v) e Svv (u, v) podem ser escritos como com-binacoes lineares de Su, Sv e η, ou seja:⎧⎨

⎩Suu (u, v) = Γ1

11Su (u, v) + Γ211Sv (u, v) + a11η (u, v)

Suv (u, v) = Γ112Su (u, v) + Γ2

12Sv (u, v) + a12η (u, v)

Svv (u, v) = Γ122Su (u, v) + Γ2

22Sv (u, v) + a22η (u, v)(5)

sendo Γkij = Γk

ij (u, v) ∈ R e aij = aij (u, v) ∈ R.

Sabemos que 〈ηu, η〉 (u, v) = 0 e 〈ηv, η〉 (u, v) = 0. Portanto, ηu e ηv sao vetores do planotangente a S em P = (u, v) . Logo:{

ηu (u, v) = b11Su (u, v) + b12Sv (u, v)

ηv (u, v) = b21Su (u, v) + b22Sv (u, v)(6)

sendo bij = bij (u, v) ∈ R.

Apos algumas contas:

Γ111 (u, v) =

GEu − 2FFu + FEv

2 (EG − F2)(u, v) Γ2

11 (u, v) =2EFu − EEv − FEu

2 (EG − F2)(u, v)

Γ112 (u, v) =

GEv − FGu

2 (EG − F2)(u, v) Γ2

12 (u, v) =EGu − FEv

2 (EG − F2)(u, v)

Γ122 (u, v) =

2GFv − GGu − FGv

2 (EG − F2)(u, v) Γ2

22 (u, v) =EGv − 2FFv + FGu

2 (EG − F2)(u, v)

b11 (u, v) =fF − eG

EG − F2(u, v) b12 (u, v) =

eF − fE

EG − F2(u, v)

b21 (u, v) =gF − fG

EG − F2(u, v) b22 (u, v) =

fF − gE

EG − F2(u, v)

(7)

a11 (u, v) = e (u, v) ; a12 (u, v) = f (u, v) ; a22 (u, v) = g (u, v)

Os numeros reais Γkij; k, i, j = 1, 2; sao chamados Sımbolos de Christoffel da superfıcie S.

A demonstracao da proposicao abaixo pode ser encontrada em [6] , paginas 207 e 208.

Proposicao 19. Sejam S : U ⊂ R2 −→ R

3 superfıcie regular e α : I ⊂ R −→ R3,

α (t) = S (u (t) , v (t)) curva regular sobre S. Entao, α e uma geodesica de S se, e somentese, as funcoes u = u (t) e v = v (t) satisfazem o sistema de equacoes diferenciais ordinarias:{

u′′ + (u′)2Γ111 + 2u′v′Γ1

12 + (v′)2Γ122 = 0

v′′ + (u′)2Γ211 + 2u′v′Γ2

12 + (v′)2Γ222 = 0

Page 143: Numero 09 - Outubro de 2007

Exemplo 2:

(2 − i) Plano: S (u, v) = P + uw1 + vw2, u, v ∈ R.

Tomemos Su (P) = w1 e Sv (P) = w2.

Temos E = |w1|2, F = 〈w1, w2〉 , G = |w2|

2, Eu = Ev = 0, Fu = Fv = 0 e Gu = Gv = 0.

Substituindo no sistema de equacoes diferenciais ordinarias acima:{u′′ = 0

v′′ = 0=⇒ {

u′ = b

v′ = d=⇒ {

u = a + bt

v = c + dt.

Logo:

α (t) = S (a + bt, c + dt) = P + (a + bt) w1 + (c + dt) w2 =⇒α (t) = P + aw1 + cw2 + t (bw1 + dw2) .

Logo, α e geodesica do plano ⇐⇒ α e uma reta.

(2 − ii) Vimos que S, S : U ⊂ R2 −→ R

3 sao superfıcies isometricas se, e somente se,E = E, F = F e G = G. Sejam

S (u, v) = (u, v, 0) ; 0 < u < 2π; v ∈ R. (plano)

S (u, v) = (cos (u) , sen (u) , v) ; 0 < u < 2π; v ∈ R. (cilindro)

Assim, E = 1 = E, F = 0 = F e G = 1 = G.

Observemos que em superfıcies isometricas os Sımbolos de Christoffel sao iguais. Logo,se α (t) = S (u (t) , v (t)) e uma geodesica em S, entao α (t) = S (u (t) , v (t)) e umageodesica em S.

Como vimos no exemplo anterior, u (t) = a + bt e v (t) = c + dt. Logo, as geodesicas docilindro sao

α (t) = S (a + bt, c + dt) =⇒ α (t) = (cos (a + bt) , sen (a + bt) , c + dt) .

Se b = 0 e d �= 0, entao α (t) = (cos (a) , sen (a) , c + dt) e um meridiano do cilindro.

Se b �= 0 e d = 0, entao α (t) = (cos (a + bt) , sen (a + bt) , c) e um cırculo do cilindro.

Se b �= 0 e d �= 0, entao α (t) = (cos (a + bt) , sen (a + bt) , c + dt) e uma helice docilindro.

E estes tres tipos de curvas sao as unicas geodesicas possıveis em um cilindro. Vejailustracao acima.

Outra consequencia importante da proposicao acima:

Se S, S : U ⊂ R2 −→ R

3 sao superfıcies isometricas, φ : S (U) −→ S (U) e uma isome-tria entre S e S e α (t) = S (u (t) , v (t)) e uma geodesica em S, entao φ ◦ α (t) =

φ (S (u (t) , v (t))) e uma geodesica em S.

Page 144: Numero 09 - Outubro de 2007

S

S

u

v

U

I

� S�

S

��

Temos que α = S ◦ β e α = φ ◦ α. De fato:

φ ◦ α (t) = φ (S (u (t) , v (t))) = φ (S (β (t))) = S (β (t)) =⇒ φ ◦ α = S ◦ β.

1.23 Teorema Egregium de Gauss

Proposicao 20. (Teorema Egregium de Gauss) A curvatura gaussiana so depende daprimeira forma quadratica.

Demonstracao:

Inicialmente, lembramos que se S : U ⊂ R2 → R

3 e uma superfıcie e η e a AplicacaoNormal de Gauss a ela associada, entao como vimos na secao anterior, Suu, Suv, Svv saocombinacoes lineares de Su, Sv e η. Alem disso, ηu, ηv, por serem tangentes a superfıcie,sao combinacoes lineares de Su e Sv. Os coeficientes destas combinacoes lineares, queforam obtidos em (7) , nao sao independentes, pois devem satisfazer as relacoes:

(Suu)v = (Suv)u ,

(Svv)u = (Suv)v , (8)

ηuv = ηvu.

Substituindo (5) e (6) em (8) , cada equacao de (8) se reduz a anular uma combinacaolinear de Su, Sv e η, que sao vetores linearmente independentes de R

3. Portanto, anulandoos coeficientes destas combinacoes lineares obtemos nove relacoes.Vejamos com detalhes as relacoes que resultam da primeira equacao de (8) .

Substituindo (7) na primeira equacao de (8) temos

∂v

(Γ111Su + Γ2

11Sv + eη)

=∂

∂u

(Γ112Su + Γ2

12Sv + fη).

Efetuando as derivadas parciais acima e substituindo Suu, Suv, Svv, ηu e ηv em funcao deSu, Sv e η, pelas relacoes (7) , obtemos as seguintes equacoes:(

Γ111

)v+ Γ1

11Γ112 + Γ2

11Γ122 + e

gF − fG

EG − F2=

(Γ112

)u

+ Γ112Γ

111 + Γ2

12Γ112 + f

fF − eG

EG − F2,(

Γ211

)v+ Γ1

11Γ212 + Γ2

11Γ222 + e

fF − gE

EG − F2=

(Γ212

)u

+ Γ112Γ

211 + Γ2

12Γ212 + f

eF − fE

EG − F2,

fΓ111 + gΓ2

11 + ev = eΓ112 + fΓ2

12 + fu,

Page 145: Numero 09 - Outubro de 2007

onde nas equacoes acima substituimos os coeficientes bij de ηu e ηv por suas expressoesdadas em (7) .

As tres equacoes acima podem ser escritas na forma

Feg − f2

EG − F2=

(Γ112

)u

−(Γ111

)v+ Γ2

12Γ112 − Γ2

11Γ122,

−Eeg − f2

EG − F2=

(Γ212

)u

−(Γ211

)v+ Γ1

12Γ211 − Γ1

11Γ212 + Γ2

12Γ212 − Γ2

11Γ222, (9)

ev − fu = eΓ112 + f

(Γ212 − Γ1

11

)− gΓ2

11. (10)

De modo analogo, considerando os coeficientes de Su, Sv e η das duas ultimas equacoesde (8) obtemos outras seis relacoes, dais quais destacamos

fv − gu = eΓ122 + f.

(Γ222 − Γ1

12

)− gΓ2

12 (11)

A equacao (9) e precisamente

−EK =(Γ212

)u

−(Γ211

)v+ Γ1

12Γ211 − Γ1

11Γ212 +

(Γ212

)2− Γ2

11Γ222 (12)

e como os sımbolos de Christoffel so dependem da primeira forma quadratica, concluimosque a curvatura gaussiana depende apenas da primeira forma quadratica. �

A equacao (12) da demonstracao do Teorema Egregium de Gauss:

−EK =(Γ212

)u

−(Γ211

)v+ Γ1

12Γ211 − Γ1

11Γ212 +

(Γ212

)2− Γ2

11Γ222

e dita Equacao de Gauss.As equacoes (10) e (11)

ev − fu = eΓ112 + f

(Γ212 − Γ1

11

)− gΓ2

11

fv − gu = eΓ122 + f

(Γ222 − Γ1

12

)− gΓ2

12

sao chamadas Equacoes de Codazzi-Mainardi.As equacoes equacoes de Gauss e de Codazzi-Mainardi sao ditas Equacoes de compatibil-idade.

Observacoes:

(1) Como vimos, a curvatura gaussiana e definida a partir da primeira e segunda formasquadraticas, mas o teorema acima afirma que, na verdade, a curvatura gaussiana de-pende apenas da primeira forma quadratica. Como consequencia, superfıcies isometricaspossuem mesma curvatura gaussiana.

(2) A recıproca da propriedade descrita em (1) nao e verdadeira, isto e, duas superfıciespodem possuir mesma curvatura gaussiana e nao serem isometricas.

(3) A recıproca descrita em (2) e verdadeira se a curvatura gaussiana das superfıcies foremiguais e constantes.

(4) Como as curvaturas gaussianas do plano e da esfera sao diferentes, temos que seuscoeficientes da primeira forma fundamental serao diferentes e, consequentemente, o planoe a esfera nao sao isometricos, mesmo restringindo seus domınios. O mesmo com o toroe a esfera, toro e cilindro, esfera e cilindro, plano e toro, esfera e cone, etc.

Page 146: Numero 09 - Outubro de 2007

1.24 Teorema Fundamental das Superfıcies

Proposicao 21. (Teorema Fundamental das Superfıcies) Sejam E, F,G, e, f, g funcoesreais diferenciaveis definidas em um aberto conexo U ⊂ R

2, tais que E, F, EG − F2 > 0. SeE, F,G, e, f, g satisfazem as Equacoes de Compatibilidade, entao:(a) Existe uma superfıcie parametrizada regular S : U ⊂ R

2 −→ R3 tal que E, F,G sao

os coeficientes da primeira forma quadratica de S e e, f, g sao os coeficientes da segundaforma quadratica de S.

(b) Se S e S sao duas superfıcies satisfazendo (a) , entao existe um movimento rıgido M

de R3 (uma isometria de R

3) tal que S = M ◦ S.

A demonstracao do Teorema Fundamental das Superfıcies envolve conceitos de equacoesdiferenciais parciais e esta alem dos objetivos deste trabalho. No entanto, sua demon-stracao pode ser encontrada em [2] , paginas de 375 a 379.

2 Referencias Bibliograficas

[1] Araujo, P. V. Geometria Diferencial. Rio de Janeiro: SBM - Sociedade Brasileirade Matematica. (Colecao Matematica Universitaria). 1998.

[2] Carmo, M. P. do. Geometria Diferencial de Curvas e Superfıcies. Rio de Janeiro:SBM - Sociedade Brasileira de Matematica. (Colecao Textos Universitarios). 2005.

[3] Lima, E. L. Algebra Linear. 3a. ed. Rio de Janeiro: SBM - Sociedade Brasileira deMatematica. (Colecao Matematica Universitaria). 1999.

[4] Lima, E. L. Curso de Analise. Vol. 1. 12a. ed. Rio de Janeiro: SBM - SociedadeBrasileira de Matematica. (Colecao Matematica Universitaria). 2000.

[5] Lima, E. L. Curso de Analise. Vol. 2. 9a. ed. Rio de Janeiro: SBM - SociedadeBrasileira de Matematica. (Colecao Matematica Universitaria). 2000.

[6] Tenemblat, K. Introducao a Geometria Diferencial. Brasılia: Editora da UnB. 1988.

[7] www-groups.dcs.st-and.ac.uk/˜history/. Site de Historia da Matematica da Universidadede Saint Andrews-UK.

Page 147: Numero 09 - Outubro de 2007

O Teorema de Barlotti

Luciana Yoshie Tsuchiya∗ Gabriela Aparecida dos Reis†

Edson Agustini‡

Faculdade de Matematica - Famat

Universidade Federal de Uberlandia - Ufu - MG

Setembro de 2007

1 Introducao

Este trabalho de iniciacao cientıfica esta baseado na dissertacao de mestrado “Complexi-dade em Geometria Plana Euclidiana”, de S. M. R. Lopes, ref. [4] , e tivemos por objetivodemonstrar o Teorema de Barlotti:

“Se Pn e um n-agono regular afim do plano complexo, entao o n-agono Bn cujos verticessao os baricentros dos n-agonos regulares construıdos (todos externamente ou todos in-ternamente) sobre cada um dos n lados de Pn e regular.”

utilizando o conceito de complexidade algebrica associado as demonstracoes de GeometriaEuclidiana Plana. Este conceito e oriundo do fato de que em algumas demonstracoesde geometria e possıvel utilizar determinados polinomios cujas raızes representam casosparticulares no qual o teorema que se deseja provar se torna verdadeiro e, a partir dessescasos particulares, e possıvel demonstrar o caso geral. Considerando p como sendo opolinomio de menor grau que podemos deduzir em uma determinada demonstracao deum teorema, podemos definir a complexidade algebrica do teorema como sendo o grau dep.

No Teorema de Barlotti enunciado acima, um n-agono e dito regular afim do planocomplexo quando for imagem de um n-agono regular por uma transformacao que e com-posta de uma translacao com um operador linear do plano complexo. Como todo trianguloe regular afim, temos como corolario imediato do Teorema de Barlotti o Teorema deNapoleao:

“Dado um triangulo ABC qualquer, sejam os triangulos equilateros apoiados externa-mente (ou internamente) sobre cada um de seus lados. Entao, os baricentros X, Y eZ destes triangulos equilateros formam um triangulo XYZ tambem equilatero, chamadoTriangulo de Napoleao Externo (ou Interno).”

Para empenhar o estudo acima, introduzimos uma secao de resultados preliminaresque se mostraram bastante uteis para a familiarizacao dos conceitos que utilizamos na

[email protected] - Pet - Programa de Educacao Tutorial - Famat - Ufu.†[email protected] - Pet - Programa de Educacao Tutorial - Famat - Ufu.‡[email protected] Professor orientador.

Page 148: Numero 09 - Outubro de 2007

demonstracao do Teorema de Barlotti. Trata-se de uma breve introducao a alguns con-ceitos de Algebra Linear, Numeros Complexos e Isometrias no Plano Euclidiano.

2 Preliminares

2.1 Algebra Linear

2.1.1 Espacos Vetoriais

Seja o conjunto V nao-vazio sobre o qual estao definidas as operacoes de adicao e multi-plicacao por escalar, isto e:(i) ∀u, v ∈ V, u + v ∈ V ;

(ii) ∀λ ∈ R, ∀u ∈ V, λu ∈ V.

O conjunto V munido dessas duas operacoes e chamado espaco vetorial sobre R se foremverificados os seguintes axiomas:(A) Em relalcao a adicao, para quaisquer u, v, w ∈ V temos:

(A1) (u + v) + w = u + (v + w) ;

(A2) u + v = v + u;

(A3) ∃0 ∈ V tal que u + 0 = u;

(A4)∀u ∈ V, ∃ (−u) ∈ V tal que u + (−u) = 0.

(M) Em relacao a multiplicacao por escalar, para quaisquer u, v ∈ V e α,β ∈ R temos:(M1) (αβ)u = α(βu);

(M2) (α + β)u = αu + βu;

(M3) α(u + v) = αu + av;

(M4) 1u = u.

Os elementos do espaco vetorial V sao chamados vetores, independentemente de suanatureza.

Sejam V um espaco vetorial e S um subconjunto nao-vazio de V. O subconjunto S eum subespaco vetorial de V quando S e um espaco vetorial em relacao a adicao e amultiplicacao por escalar.

Seja V um espaco vetorial. Um conjunto B = {v1, ..., vn} ⊂ V e uma base de V quando:(i) B e linearmente independente, ou seja, um vetor de B nao pode ser escrito comocombinacao linear dos demais vetores de B;

(ii) B gera V, ou seja, qualquer vetor de V pode ser escrito como combinacao linear devetores de B.

Seja V um espaco vetorial.(i) Quando V possui uma base com n vetores, dizemos que V tem dimensao n e denotamosdim V = n;

(ii) Quando V nao possui base, dizemos que V e o espaco vetorial nulo e denotamosdim V = 0.

2.1.2 Transformacoes Lineares

Para dizer que T e uma transformacao do espaco vetorial V no espaco vetorial W, escreve-se T : V −→ W. Sendo T uma funcao, cada vetor v ∈ V tem um so vetor imagem w ∈ W,

que sera indicado por w = T (v) .

Page 149: Numero 09 - Outubro de 2007

Sejam V e W espacos vetoriais. Uma aplicacao:

T : V −→ W

v �−→ T (v) = w

e chamada transformacao linear de V em W quando para quaisquer u, v ∈ V e α ∈ R:(i) T(u + v) = T (u) + T (v) ;

(ii) T (αu) = αT (u) .

Sejam T1 : V −→ W e T2 : V −→ W transformacoes lineares. Chama-se soma dastranformacoes lineares T1 e T2 a transformacao linear:

T1 + T2 : V −→ W

u �−→ (T1 + T2)(u) = T1(u) + T2(u).

Sejam T : V −→ W uma transformacao linear e α ∈ R. Chama-se produto de T peloescalar α a transformacao linear:

αT : V −→ W

u �−→ (αT)(u) = αT(u).

Sejam T1 : U −→ V e T2 : V −→ W transformacoes lineares. Chama-se aplicacao compostade T1 com T2, e representa-se por T2 ◦ T1, a transformacao linear:

T2 ◦ T1 : U −→ W

u �−→ (T2 ◦ T1)(u) = T2(T1(u)).

Dada uma aplicacao T : V −→ W, diremos que T e injetora quando: dados u, v ∈ V taisque u �= v, entao T(u) �= T(v).

A aplicacao T : V −→ W sera sobrejetora quando: dado w ∈ W, existir v ∈ V tal queT(v) = w.

Chama-se nucleo de uma transformacao linear T : V −→ W ao conjunto de todos osvetores v ∈ V que sao transformados em 0 ∈ W pela transformacao T. Indica-se esseconjunto por N(T) ou ker(T). Assim, N(T) = {v ∈ V | T(v) = 0}.

Chama-se imagem de uma transformacao linear T : V −→ W ao conjunto dos vetoresw ∈ W que sao imagens de pelo menos um vetor v ∈ V. Indica-se esse conjunto por Im(T)

ou T(V).

Propriedades:Sejam V e W espacos vetoriais e T : V → W transformacao linear. Entao:I) O nucleo de T e um subespaco vetorial de V.

II) T e injetora se, e somente se, N (T) = {0}.

III) A imagem de T e um subespaco vetorial de W.

IV) Se dim V = dim W, entao T e injetora se, e somente se, T e sobrejetora.V) Se dim V = dim W e T e injetora, entao T transforma base em base, isto e, se B =

{v1, ..., vn} e base de V, entao T(B) = {T(v1), ..., T(vn)} e base de W.

Chama-se isomorfismo do espaco vetorial V no espaco vetorial W a uma transformacaolinear T : V −→ W, que e bijetora. Nesse caso, os espacos vetoriais V e W sao ditosisomorfos.

Page 150: Numero 09 - Outubro de 2007

2.1.3 Operadores Lineares

Uma transformacao linear de V em V (ou seja, V = W) e chamada operador linear sobreV.

As propriedades gerais das transformacoe lineares sao validas para os operadores lineares.

Propriedades:Seja T : V −→ V um operador linear.I) Se T e inversıvel e T−1 e sua inversa, entao T ◦ T−1 = T−1 ◦ T = I (identidade).II) T e inversıvel se, e somente se, N(T) = {0}.

III) Se T e inversıvel, entao T transforma base em base, isto e, se B e uma base de V, T(B)

tambem e base de V.

IV) Se T e inversıvel e B uma base de V, entao T−1 : V −→ V e linear e[T−1

]B

= ([T ]B)−1

,

isto e, a matriz do operador linear inverso numa certa base B e a inversa da matriz dooperador T nessa mesma base. Daı temos que: T e inversıvel se, e somente se, det [T ] �= 0.

Seja V um espaco vetorial euclidiano (espaco vetorial real, de dimensao finita, com umproduto interno definido). Um operador linear T : V −→ V e ortogonal quando preservao modulo de cada vetor, isto e, quando para qualquer v ∈ V temos |T(v)| = |v| .

Observacoes:1) Tendo em vista que o modulo de um vetor e calculado por meio de um produto in-terno (|v| =

√〈v, v〉), entao os operadores ortogonais sao definidos nos espacos vetoriaiseuclidianos.2) Nos operadores ortogonais, serao consideradas somente bases ortogonais em V.

Propriedades:I) Seja T : V −→ V um operador ortogonal sobre o espaco euclidiano V. Entao, a inversada matriz de T coincide com a sua transposta, isto e,

[T−1

]= [T ]

t.

II) O determinante da matriz de um operador ortogonal e +1 ou −1.

III) Todo operador ortogonal T : V −→ V preserva o produto interno de vetores, isto e,para quaisquer vetores u, v ∈ V, tem-se 〈u, v〉 = 〈T (u) , T (v)〉 ;

IV) A composta de dois operadores ortogonais e um operador ortogonal ou, equivalente-mente, o produto de duas matrizes ortogonais e uma matriz ortogonal.V) As colunas (ou linhas) de uma matriz ortogonal formam um conjunto de vetoresortonormais.

2.1.4 Transformacoes Lineares no Plano

Iremos trabalhar com o espaco vetorial R2 = {(x1, x2) : x1, x2 ∈ R} munido das operacoes

de adicao e multiplicacao por escalar usuais sobre o corpo dos numeros reais R, ou seja:se u = (x1, x2), v = (y1, y2) e λ ∈ R, entao:(i) u + v = (x1 + y1, x2 + y2) ;

(ii) λu = (λx1, λx2) .

Para efeitos geometricos, iremos identificar o plano euclidiano com o espaco vetorial R2.

Entende-se por transformacoes lineares no plano as transformacoes lineares de R2 em R

2.

Page 151: Numero 09 - Outubro de 2007

Reflexoes

(i) Reflexao em torno do eixo dos x.

Essa transformacao linear leva cada ponto (x, y) em (x, −y) , simetrico em relacao ao eixodos x, ou seja:

T : R2 −→ R

2

(x, y) �−→ (x, −y).

x

y

o

(x,y)

(x,-y)

T

(ii) Reflexao em torno do eixo dos y:

T : R2 −→ R

2

(x, y) �−→ (−x, y).

x

y

O

(x,y)(-x,y) T

(iii) Reflexao em torno da origem:

T : R2 −→ R

2

(x, y) �−→ (−x, −y).

x

y

o

(x,y)

(-x,-y)

T

Page 152: Numero 09 - Outubro de 2007

(iv) Reflexao em torno da reta y = x:

T : R2 −→ R

2

(x, y) �−→ (y, x).

x

y

o

(x,y)

(y,x)

y=x

T

(v) Reflexao em torno da reta y = −x:

T : R2 −→ R

2

(x, y) �−→ (−y,−x).

x

y

o(x,y)

(-y,-x)

y=-x

T

Dilatacoes e Contracoes

(i) Dilatacao ou contracao na direcao do vetor v = (x, y):

T : R2 −→ R

2

(x, y) �−→ α (x, y), α ∈ R

∗.

x

y

o

v

T(v)

Page 153: Numero 09 - Outubro de 2007

se |α| > 1, entao T “dilata” o vetor v;se |α| < 1, entao T “contrai” o vetor v;se α = 1, entao T e a identidade I;

se α < 0, entao T muda o sentido (sinal) do vetor v.

(ii) Dilatacao ou contracao na direcao do eixo dos x:

T : R2 −→ R

2

(x, y) �−→ (αx, y), α ∈ R

∗.

(x,y) (3x,y)

x

y

o

se |α| > 1, entao T “dilata” o plano real na direcao do eixo dos x a partir do eixo dos y.

se 0 �= |α| < 1, entao T “contrai” o plano real na direcao do eixo dos x a partir do eixodos y.

(iii) Dilatacao ou contracao na direcao do eixo dos y:

T : R2 −→ R

2

(x, y) �−→ (x, αy), α ∈ R

∗.

x

y

(x,y)

(x,2y)

(x, y)y

y

2y

o

1 12 2__ __

Rotacao

A rotacao no plano, que faz cada ponto descrever um angulo θ no sentido anti-horario emtorno da origem, determina uma transformacao linear:

Tθ : R2 −→ R

2

(x, y) �−→ (x cos (θ) − y sen (θ) , x sen (θ) + y cos (θ)),

ou seja, a matriz de Tθ em relacao as bases canonicas de R2 e dada por:

[Tθ] =

[cos (θ) − sen (θ)

sen (θ) cos (θ)

].

Page 154: Numero 09 - Outubro de 2007

�v

T�

T (v)�

e1

e2

o x

y

Cisalhamentos

(i) Cisalhamento na direcao do eixo dos x:

T : R2 −→ R

2

(x, y) �−→ (x + αy, y).

B

o A

P

T

B’ P’

Aox x

y y

O efeito do cisalhamento pode ser visto na transformacao do retangulo OAPB no parale-logramo OA′P′B, de mesma base e mesma altura. Observemos que, por esse cisalhamento,cada ponto (x, y) se desloca paralelamente ao eixo dos x ate chegar em (x + αy, y), comexcecao dos pontos do proprio eixo dos x, que permanecem em sua posicao, pois para elesy = 0.

(ii) Cisalhemento na direcao do eixo dos y:

T : R2 −→ R

2

(x, y) �−→ (x, y + αx).

2.2 Numeros Complexos

2.2.1 Definicoes Basicas

Numeros complexos sao numeros da forma x + yi, onde i =√

−1 e chamado unidadeimaginaria; x e chamado de parte real e y de parte imaginaria do numero complexo.

A igualdade e as operacoes de adicao e multiplicacao de numeros complexos sao definidasde maneira que permanecam validas as propriedades associativa, comutativa e distributivausuais.

Page 155: Numero 09 - Outubro de 2007

Assim os numeros complexos sao determinados pelas seguintes definicoes:

i2 = −1;

ai = ia;

a + ib = c + id ⇐⇒ a = c e b = d;

(a + ib) + (c + id) = (a + c) + i (b + d) ;

(a + ib) (c + id) = (ac − bd) + i (ad + bc) .

A subtracao de numeros complexos e definida em termos da adicao e do oposto de umnumero. O oposto de z = x + yi e o numero −z = (−x) + i (−y) . Dados z1 = x1 + iy1 ez2 = x2 + iy2, definimos:

z1 − z2 = z1 + (−z2),

isto e,z1 − z2 = (x1 − x2) + i (y1 − y2) .

Observemos que os numeros complexos da forma x + i0 se comportam, com relacao aadicao e a multiplicacao, do mesmo modo que os numeros reais.

Dado o numero complexo z = x + yi, sua parte real x e designada por Re z e sua parteimaginaria y por Im z. O plano complexo consiste na representacao de todos os numeroscomplexos z = x + yi pelos pontos P = (x, y) do plano.

Re

Im

x

y z x yi= +

De acordo com essa representacao temos as seguintes definicoes:

(a, b) = (c, d) ⇐⇒ a = c e b = d;

(a, b) + (c, d) = (a + c, b + d) ;

(a, b) (c, d) = (ac − bd, ad + bc) .

As conhecidas regras do paralelogramo para soma e subtracao de vetores se aplicam nocaso de soma e subtracao de numeros complexos quando representados no plano.

Re

Im

z1

z2

z z1 2+

O modulo ou valor absoluto de um numero complexo z = x + iy e a distancia do ponto z

a origem e e definido como sendo o numero real nao-negativo |z| =√

x2 + y2.

Page 156: Numero 09 - Outubro de 2007

Propriedades:i) |z| ≥ 0 e |z| = 0 ⇐⇒ z = 0;

ii) |Re z| ≤ |z| e |Im z| ≤ |z| ;

iii) |z| = |−z| ;

iv) |z1z2| = |z1| |z2| ;

v) |z1 + z2| ≤ |z1| + |z2| , designada como desigualdade do triangulo;vi) |z1 − z2| ≤ |z1| + |z2| .

O complexo conjugado de z = x+iy e definido como sendo o numero complexo z = x−iy.

Re

Im

z x yi= +

z x yi= -

Assim, temos as seguintes propriedades:i) zz = |z|

2.

Com esta propriedade podemos calcular o quociente z = z1

z2de dois numeros complexos

z1 = x1 + iy1 e z2 = x2 + iy2, z2 �= 0, da seguinte forma:

z =z1

z2

=z1z2

z2z2

=(x1 + iy1) (x2 − iy2)

(x2 + iy2) (x2 − iy2)=

x1x2 + y1y2

x22 + y2

2

+ i−x1y2 + y1x2

x22 + y2

2

.

ii) |z| = |z| ;

iii) Re z = z+z2

;

iv) Im z = z−z2

;

v) z1 + z2 = z1 + z2;

vi) z1z2 = z1z2;

vii)(

z1

z2

)= z1

z2.

2.2.2 Representacao Polar

Considerando a representacao geometrica de um numero complexo z = (a, b) , chama-sede argumento de z o angulo θ formado pelo vetor (1, 0) e o vetor (a, b) .

Re

Im

z = (a,b)

z

(1,0)O

Page 157: Numero 09 - Outubro de 2007

Os angulos sao orientados de Ox para Oz e consideraremos positivo o sentido anti-horario.O argumento de z so pode ser definido quando z �= 0, mesmo nesta hipotese o argumentoso fica determinado a menos de multiplos inteiros de 2π. Como x = |z| cos (θ) e y =

|z| sen (θ) , temos a representacao polar de z:

z = r (cos (θ) + i sen (θ)) , r = |z| ;

r e θ sao designados as coordenadas polares de z.

O produto de dois numeros complexos na forma polar e o numero cujo modulo e o produtodos modulos dos fatores e, cujo argumento e a soma dos argumentos dos fatores.De fato, sejam dois numeros complexos

z1 = r1 (cos (θ1) + i sen (θ1))

ez2 = r2 (cos (θ2) + i sen (θ2)) .

Multiplicando-os temos:

z1z2 = r1r2 (cos (θ1) + i sen (θ1)) (cos (θ2) + i sen (θ2))

= r1r2 (cos (θ1) cos (θ2) − sen (θ1) sen (θ2)) + i (sen (θ1) cos (θ2) + cos (θ1) sen (θ2))

= r1r2 (cos (θ1 + θ2) + i sen (θ1 + θ2)) .

Fica facil perceber entao que para fazermos a rotacao de um numero complexo em torno daorigem, basta multiplica-lo por uma constante complexa de modulo 1. Usando o formulaacima, fazendo z2 ter modulo um, ou seja r2 = 1 temos:

z1z2 = r1.1. (cos (θ1 + θ2) + i sen (θ1 + θ2))

= r1 (cos (θ1 + θ2) + i sen (θ1 + θ2))

Observe que o produto dos dois numeros complexos acima tem o mesmo modulo de z1

e o argumento de z1 foi somado com o argumento de z2, ou seja, z1 foi rotacionado, nosentido anti-horario, de um angulo de medida θ2 em torno da origem.

Re

Im

z1

z2

����

���� ��

z z1 2

z1 ��

De forma analoga a deducao da formula para multiplicacao obtemos o resultado para adivisao. Daı:

z1

z2

=r1

r2

(cos (θ1 − θ2) + i sen (θ1 − θ2)) ,

ou seja, para dividir numeros complexos na forma polar, basta fazer o quociente dosmodulos e a diferenca dos argumentos.

Page 158: Numero 09 - Outubro de 2007

A formula de multiplicacao acima se estende para um numero qualquer de fatores. Sendo

zj = rj (cos (θj) + i sen (θj)) , j = 1, 2, ..., n,

temos

z1z2...zn = r1r2...rn (cos (θ1 + θ2 + ... + θn) + i sen (θ1 + θ2 + ... + θn)) .

Em particular quando todos os fatores sao iguais e de modulo unitario, obtemos a Formulade De Moivre:

(cos (θ) + i sen (θ))n

= cos (nθ) + i sen (nθ) .

As raızes n-esimas n√

a = a1n = z de um numero complexo a �= 0 sao obtidas como as

solucoes da equacao zn = a. Pondo

a = r (cos (θ) + i sen (θ)) ,

z = ρ (cos (φ) + i sen (φ))

e usando a Formula de De Moivre, obtemos

ρn (cos (nφ) + i sen (nφ)) = r (cos (θ) + i sen (θ)) .

Daı temos,ρn cos (nφ) = r cos (θ) e ρn sen (nφ) = r sen (θ)

donde concluımos que:

ρn = r, nφ = θ + 2kπ, com k ∈ Z.

Segue daqui que ρ e uma raiz n-esima positiva de r e:

z = n√

r

(cos

(θ + 2kπ

n

)+ i sen

(θ + 2kπ

n

)).

Esta formula produz n raızes distintas zk, quando k varia de 0 a n− 1, todas com mesmomodulo ρ = n

√|a| e com argumentos φk = θ+2kπ

n, k = 0, 1, ..., n − 1.

��n��

��

zn-2

zn-1

z0

z1

z2

z3

��� 2�n

No caso particular a = 1, obtemos as raızes n-esimas da unidade: 1,ω, ω2, ..., ωn−1, onde

ω = cos

(2π

n

)+ i sen

(2π

n

).

Page 159: Numero 09 - Outubro de 2007

Definicao de Exponencial

Desenvolvendo a funcao exponencial f (x) = ex e as funcoes trigonometricas sen (x) ecos (x) em series de potencias para x ∈ R temos:

ex =∞∑

n=0

xn

n!= 1 + x +

x2

2!+

x3

3!+ · · · ;

cos (x) =∞∑

n=0

(−1)n

x2n

(2n) != 1 −

x2

2!+

x4

4!−

x6

6!+ · · · ;

sen (x) =∞∑

n=0

(−1)n

x2n+1

(2n + 1) != x −

x3

3!+

x5

5!−

x7

7!+ · · ·

Fazendo x = iθ para a funcao exponencial temos:

eiθ = 1 + iθ −θ2

2!−

iθ3

3!+ · · ·

Fazendo x = θ para as funcoes trigonometricas temos:

cos (x) = 1 −θ2

2!+

θ4

4!−

θ6

6!+ · · · ;

sen (x) = θ −θ3

3!+

θ5

5!−

θ7

7!+ · · ·

Reescrevendo a funcao eiθ, separando a parte real e a imaginaria temos:

eiθ = 1 −θ2

2!+

θ4

4!−

θ6

6!+ · · · + i

(θ −

θ3

3!+

θ5

5!−

θ7

7!+ · · ·

)ou seja,

eiθ = cos (θ) + i sen (θ) .

Essas consideracoes, que sao puramente informais, nao estabelecem a relacao acima, masservem como motivacao para definirmos a funcao exponencial.Tomando z = x + iy como um expoente qualquer, a definicao da exponencial e feita demaneira a manter a propriedade aditiva da exponencial real:

ex1+x2 = ex1ex2 .

Definimos, entao, a exponencial ez como:

ez = ex+iy = ex (cos (y) + i sen (y)) .

2.3 Isometrias Planas

Isometrias no plano π sao transformacoes T : π −→ π que tem a propriedade de preservardistancias.Admitiremos fixada uma unidade de comprimento e indicaremos por AB a distancia doponto A ao ponto B no plano, ou seja, o comprimento do segmento de reta AB.

Page 160: Numero 09 - Outubro de 2007

Se T e uma isometria, entao para quaisquer pontos X, Y ∈ π, sendo X′ = T(X) e Y′ = T(Y),

tem-se X′Y′ = XY.

Toda isometria T : π −→ π e uma transformacao injetiva, pois:

X �= Y =⇒ XY > 0 =⇒ X′Y′ = XY > 0 =⇒ X′ �= Y′.

E e tambem sobrejetiva, logo ela e uma bijecao cuja a inversa T−1 : π −→ π e ainda umaisometria.Consequentemente, toda isometria T : π −→ π transforma retas em retas.O exemplo mais obvio de isometria e a transformacao identidade Id : π −→ π.

Outros exemplos de isometrias sao dados na proxima subsecao.

2.3.1 Tipos de Isometrias Planas

Simetria em torno de um ponto.

Tomemos um ponto A no plano π. A simetria em torno de A e a transformacao SA :

π −→ π assim definida: SA(A) = A e, para X �= A, SA(X) = X′, sendo X′ o simetrico deX relativamente a A. Em outras palavras A e o ponto medio do segmento XX′.

X Y

X’

A

Y’

Reflexao em torno de uma reta.

Seja r uma reta no plano π. A reflexao em torno da reta r e a transformacao Rr : π −→ π

assim definida: Rr(X) = X para todo X ∈ r e, para X /∈ r, Rr(X) = X′, sendo X′ tal que amediatriz do segmento XX′ e a reta r. Em outras palavras, se Y e o pe da perpendicularbaixada de X sobre r, entao Y e o ponto medio do segmento XX′.

X

Y

r

X’

Um fato geometrico importante a respeito da reflexao Rr : π −→ π e que ela transformao triangulo ABC num triangulo A′B′C′ no qual o sentido da rotacao dos vertices A′ −→B′ −→ C′ e o oposto do sentido A −→ B −→ C, isto significa que a reflexao em torno deuma reta e uma isometria “impropria”, que inverte a orientacao no plano.

Page 161: Numero 09 - Outubro de 2007

C

B

A A

C

B

B’

C’

A’A’

B’C’

r

Translacao.

Sejam A,B pontos distintos do plano π. A translacao TAB : π −→ π e a transformacaoassim definida:(1) Dado X ∈ π, com X, A e B nao colineares, sua imagem X′ = TAB(X) e o quarto verticedo paralelogramo que tem AB e XX′ como lados.

A

X X’

B’

M

(2) Dado X ∈ ←→AB ⊂ π sua imagem X′ por T e definida na propria reta

←→AB tal que

XX′ = AB.

Qualquer que seja a posicao de X no plano π, sua imagem X′ = TAB(X) fica inteiramentecaracterizada pelo fato de que o segmento de reta AX′ e BX tem o mesmo ponto medioM.

E importante observar que na definicao de TAB e essencial levar em conta a ordem emque sao mencionados os pontos A e B. A translacao TBA e diferente de TAB. Na realidade,como se ve facilmente, tem-se TBA = (TAB)−1. Podemos dizer, tambem, que o ponto X foi

transladado segundo um vetor−→AB.

A translacao TAB nao possui pontos fixos, pois para todo ponto X ∈ π, com T(X) = X′,tem-se XX′ = AB. Como A �= B temos AB �= 0, ou seja, XX′ �= 0 que implica X �= X′.

Rotacao.

Sejam O um ponto no plano π e α = AOB um angulo de vertice O. A rotacao de anguloα em torno do ponto O e a transformacao ρO,α : π → π assim definida: ρO,α (O) = O e,para todo ponto X �= O em π, ρO,α (X) = X′, sendo X′ o ponto do plano π tal que:

XO = X′O e XOX′ = α,

e o sentido de rotacao de A para B e o mesmo de X para X′.

Page 162: Numero 09 - Outubro de 2007

O

X’

X

B

A��

Reflexao com deslizamento.

Sejam v =−→AB um vetor nao-nulo e r uma reta paralela a v no plano π. A reflexao com

deslizamento, determinada pelo vetor v e pela reta r, e a isometria T = Tv ◦ Rr : π → π,

obtida fazendo a translacao Tv seguida da reflexao Rr. A reflexao com deslizamento naopossui pontos fixos.

v

X

Rr(X) X’

A

C

B

A’

B’

C’

r

2.3.2 Classificacao de Isometrias Planas

Para o desenvolvimento seguinte, utilizaremos um resultado da teoria de isometrias planasdado abaixo:

“Se duas isometrias T : π → π e T ′ : π → π coincidem em tres pontos distintos, entaoT = T ′.”

Existem apenas quatro tipos de isometrias T : π → π alem da identidade, a saber:translacao, rotacao, reflexao e reflexao com deslizamento.

Com efeito, seja T : π → π uma isometria diferente da identidade. Existe um ponto A ∈ π

tal que A′ = T (A) �= A. Seja A′′ = T (A′) . Evidentemente A′A′′ = AA′ > 0. Ha trescasos a considerar.

Primeiro caso: A, A′ e A′′ sao nao-colineares.

A

A’

A"

B

B

Page 163: Numero 09 - Outubro de 2007

A imagem do triangulo pela isometria T e um triangulo que tem A′ e A′′ como verticese os seus lados tem medidas iguais as dos lados de AA′A′′. Assim, existem duas posicoespossıveis para o seu terceiro vertice B.

(i) A e B estao do mesmo lado da reta←−→A′A′′.

Nesse caso o ponto B = T (A′′) forma com A, A′ e A′′ o quadrilatero convexo AA′A′′B, no

qual os lados AA′, A′A′′ e A′′B tem a mesma medida e os angulos A′e A′′ sao congruentes.Consequentemente, os angulos A e B no quadrilatero tambem sao congruentes e, portanto,suplementares aos seus angulos opostos A′′e A′, respectivamente.Assim, o quadriatero AA′A′′B pode ser inscrito em um cırculo de raio OA, cujo o centroO e o ponto de encontro das mediatrizes dos segmentos AA′, A′A′′ e A′′B.

A

A’ A’’

B

O

Seja O′ = T (O) .

Entao, como OA ≡ OA′ ≡ OA′′, temos O′A′ ≡ O′A′′ ≡ O′B, logo O′ pertence asmediatrizes dos segmentos A′A′′ e A′′B e portanto O = O′.Assim, temos que OA ≡ OA′ ≡ OA′′ ≡ OB e tambem AA′ ≡ A′A′′ ≡ A′′B, entao ostriangulos AOA′, A′OA′′ e A′′OB sao congruentes (caso LLL), isto significa que os angulos

AOA′, A′OA′′ e A′′OB tambem sao congruentes.Portanto, se considerarmos a rotacao ρ de centro O e angulo AOA′, teremos ρ (A) =

A′ = T (A) , ρ (A′) = A′′ = T (A′) e ρ (A′′) = B = T (A′′) . Daı temos que T = ρ e umarotacao.

(ii) A e B nao estao do mesmo lado da reta AA′.

Neste caso B forma com os pontos A, A′ e A′′ um paralelogramo no qual AA′ e A′′B saolados opostos e A′A′′ e uma diagonal.

A

M

A’

N

A’’

B

Pr

Seja M, N e P os pontos medios dos segmentos AA′, A′A′′ e A′′B respectivamente.Temos que M, N e P estao sobre uma mesma reta r.

Page 164: Numero 09 - Outubro de 2007

Se considerarmos a isometria S = TMN ◦ Rr, composta da translacao TMN com a reflexaoem torno de r, veremos que S e T coincidem nos pontos nao-colineares A, A′ e A′′, logoT = S. Concluımos entao que T e uma reflexao com deslizamento.

Segundo Caso: A, A′ e A′′ sao pontos distintos e colineares.

Note que neste caso A′ e ponto medio do segmento AA′′ pois AA′ = A′A′′.A reta r que contem os tres pontos dados, e transformada em si mesma pela isometria T.

Alem disso T coincide nos pontos A e A′ com a translacao TAA′ : π → π. Entao, em todosos pontos de r, T coincide com esta translacao.

Consideremos um ponto B fora da reta r.

A

B

A’A’’

B’

B’’

O triangulo AA′B e transformado pela isometria T em outro triangulo que tem A′ e A′′

como vertices e lados com as mesmas medidas que os de AA′B.

Assim existem duas posicoes possıveis B′ e B′′ para o terceiro vertice.

(i) B e B′ estao do mesmo lado da reta r.

Neste caso AB e A′B′ sao lados opostos de um paralelogramo logo,considerando a translacaoTAA′ : π → π, vemos que ela coincide com a isometria T nos pontos nao-colineares A, A′

e B. Logo T = TAA′, portanto, T e uma translacao.

(ii) B e B′′ estao de lados opostos da reta r.

Neste caso B′′ e o simetrico de B′ em relacao a reta r, considerando a reflexao comdeslizamento S = TAA′ ◦ Rr : π → π, vemos que S (A) = T (A) = A′, S (A′) = T (A′) = A′′

e S (B) = T (B) = B′′. Logo, S = T.

Portanto, T e uma reflexao com deslizamento.

Terceiro caso: A′′ = A.

Neste caso, a isometria T transforma o segmento de reta AA′ em si mesmo. Logo T (M) =

M se M e o ponto medio de AA′. A mediatriz s desse segmento e entao transformada emsi mesmo por T.

Page 165: Numero 09 - Outubro de 2007

A M A’

B

B’

r

s

Seja B um ponto dessa mediatriz diferente de M. Ha entao duas possibilidades: T (B) = B

ou T (B) = B′, ponto simetrico de B relativamente a reta r =←→AA.

(i) T (B) = B

Neste caso T coincide com a reflexao Rs : π → π nos pontos A, A′ e B, logo T = Rs.

(ii) T (B) = B′.

Neste caso T coincide com a a rotacao ρ : π → π em torno do ponto M, com angulo de180◦, nos pontos nao-colineares A,B e M, logo T = ρ.

Portanto, neste terceiro caso, T e uma translacao ou uma rotacao de 180◦.

2.3.3 Centro da composicao de duas rotacoes

Rotacao como composta de duas reflexoes

Seja ρO,α uma rotacao no sentido anti-horario de um angulo α e de centro O e r e s duasretas concorrentes em O que formam entre si um angulo β, tal que β = α

2.

Afirmamos que ρO,α = Rs ◦ Rr, ou seja, a composta das duas reflexoes Rs e Rr (nestaordem) e a rotacao de centro O e angulo igual ao dobro do angulo de r para s.

De fato, Rs ◦ Rr coincide com a rotacao ρO,α em todos os pontos X ∈ r e em todos ospontos Y ∈ s′, onde s′ = Rr (s) .

X

X’

O

Y’

Y

s

r

s’

Page 166: Numero 09 - Outubro de 2007

Composta de duas rotacoes.

Se duas rotacoes ρO,α e ρO,β tem o mesmo centro O, e claro que,

ρO,α ◦ ρO,β = ρO,β ◦ ρO,α = ρO,α+β.

Onde ρo,α+β e a identidade,quando α + β = 360◦ e e a simetria em torno de O seα + β = 180◦.Consideremos entao, duas rotacoes ρO,α e ρO′,β de centros distintos O e O′. Seja r a retaque passa pelos dois centros. Tomemos s, passando por O, tal que o angulo de r para s

sejaα

2, e a reta t, pasando por O′, e concorrente com a reta s no ponto O′′ e de modo

que o angulo de t para r sejaβ

2.

O

O’’

O’

��� ��

���

s

r

t

Assim podemos escrever:

ρO,α = Rs ◦ Rr, ρO,β = Rr ◦ Rt,

dondeρO,α ◦ ρO,β = Rs ◦ Rr ◦ Rr ◦ Rt = Rs ◦ Rt.

Mas Rs ◦ Rt e a rotacao em torno de O′′com angulo igual ao dobro do angulo da reta t

para a reta s, ou seja, com angulo igual a 2

(α + β

2

)= α + β.

Logo,ρO,α ◦ ρO,β = ρO′′,α+β.

Portanto, O′′ e o centro de ρO,α ◦ ρO,β.

Analogamente se ve queρO,β ◦ ρO,α = ρO′′′,α+β.

3 O Teorema de Barlotti

Nosso objetivo nesta secao e demonstrar o chamado Teorema de Barlotti, que e uma ge-neralizacao do famoso Teorema de Napoleao (corolario abaixo). Para tanto, consideremosC como espaco vetorial sobre R com as operacoes usuais.

Lema 1. Seja Qn um n-agono regular no plano complexo. Entao, o centro de Qn e mediaaritmetica de seus vertices.

Page 167: Numero 09 - Outubro de 2007

Demonstracao

(i) Considere um n-agono regular Rn de centro c = 0 e vertices 1,ω1, . . . , ωn−1 (raızesn-esimas da unidade).Daı temos:

1 + ω1 + · · · + ωn−1

n= c = 0 (1)

De fato: observemos que S = 1 + ω1 + · · · + ωn−1 + ωn e a soma dos termos de umaprogressao geometrica com 1o. termo a1 = 1 e razao r = ω. Assim, S sera:

S =a1(1 − rn+1)

1 − r=

1 − ωn+1

1 − ω.

Daı,

1 + ω1 + · · · + ωn−1 + ωn =1 − ωn+1

1 − ω⇒

1 + ω1 + · · · + ωn−1 =1 − ωn+1

1 − ω− ωn

=1 − ωn+1 − ωn(1 − ω)

1 − ω

=1 − ωn+1 − ωn + ωn+1

1 − ω

Mas ωn = 1 (pois c = 0). Logo:

1 + ω1 + · · · + ωn−1 =1 − 1

1 − ω= 0.

Portanto, a equacao (1) fica verificada.

Um n-agono regular Qn qualquer no plano complexo pode ser obtido de Rn por meio daaplicacao de uma composta F = T ◦ h ◦ ρ : C → C, sendo ρ uma rotacao:

ρ : C −→ C

z �−→ zeiα , 0 ≤ α < 2π,

h uma homotetia:h : C −→ C

z �−→ kz, k ∈ R

∗,

e T uma translacao:T : C −→ C

z �−→ T(z) = z + z0, z0 ∈ C.

Assim, o centro de Qn = F (Rn) e z0 e seus vertices sao dados por k1eiα + z0, kωeiα + z0,

. . . , kωn−1eiα + z0. Logo:(k1eiα + z0

)+

(kωeiα + z0

)+ · · · + (

kωn−1eiα + z0

)n

= keiα1 + ω + · · · + ωn−1

n+

nz0

n

=keiα0

n+ z0

= z0.

Page 168: Numero 09 - Outubro de 2007

Daı, concluımos que o centro de um n-agono regular Qn qualquer no plano complexo e amedia aritmetica de seus vertices. �

Lema 2. Sejam z ∈ C, V1 = a1 + b1z e V2 = a2 + b2z, sendo a1, b1, a2, b2 ∈ C taisque V1 �= V2. Seja Q um n-agono regular no plano complexo tendo V1 e V2 como verticesconsecutivos. Entao, os vertices de Q sao da forma ak + bkz com ak, bk ∈ C, isto e, osvertices de Q sao expressoes afins em z.

Demonstracao.

Sejam V1 = a1 + b1z e V2 = a2 + b2z pontos distintos do plano complexo.Seja n ∈ N, n ≥ 3.

Definamos V3 = ((a1 + b1z) − (a2 + b2z)) e−iα + (a2 + b2z), sendo α =(n − 2)π

n. Logo,

fazendo a3 = (a1 − a2) e−iα+a2 ∈ C e b3 = (b1 − b2) e−iα+b2 ∈ C, temos V3 = a3+b3z

uma expressao afim em z.

Geometricamente V3 foi obtido girando-se V1 de um angulo α no sentido horario em tornode V2 (ver figura).

V =a +b1 1 1

zV =a +b

3 3 3z

V =a +b2 2 2

z

V =a +bn n n

z

(a +b ) (a +b )1 1 2 2

z z�

Procedendo de modo analogo,

Vk = ((ak−2 + bk−2z) − (ak−1 + bk−1z)) e−iα + (ak−1 + bk−1z) ,

k = 3, ..., n, sao todos expressoes afins em z.

Finalmente, Q e um polıgono regular de n lados que possui os vertices todos como ex-pressoes afim em z. �

Chamaremos F : C → C de transformacao afim quando F = T ◦ L, sendo L : C → C umatransformacao linear (C espaco vetorial sobre R com operacoes usuais) e T : C → C umatranslacao.

Chamaremos um n-agono Pn do plano complexo de regular afim quando Pn = F (Rn) ,

sendo Rn um n-agono regular com centro na origem e vertices nas raızes n-esimas daunidade e F uma transformacao afim bijetiva.

Teorema (Barlotti). Se Pn e um n-agono regular afim do plano complexo, entao on-agono Bn cujos vertices sao os baricentros dos n-agonos regulares construıdos (todosexternamente ou todos internamente) sobre cada um dos n lados de Pn e regular.

Page 169: Numero 09 - Outubro de 2007

Rn

F( )=R Pn n

Bn

Demonstracao

Faremos a demonstracao desse teorema usando o conceito de complexidade algebrica.

Temos Pn = F (Rn) = T ◦ L (Rn) , sendo T translacao, L transformacao linear bijetiva eRn um n-agono regular com centro na origem e vertices nas raızes n-esimas da unidade.Como a natureza (ser regular) de Bn nao e alterada por translacoes, homotetias e rotacoesaplicadas a Pn, podemos considerar, sem perda de generalidade, que Pn e a imagem deRn por L.

Sejam L (1) = z0 = |z0| eiα um vertice de Pn,

h : C −→ C

z �−→ h(z) = z|z0 |

homotetiaρ : C −→ C

z �−→ ρ(z) = ze−iα

rotacao ef : C −→ C

z �−→ f(z) = ρ ◦ h ◦ L (z)

transformacao linear bijetiva.

L �

� o o =h L f

0 00 01 1

��

z0 _ _z

0

z0 z

0

z e0

i�

=

h

Assim, f (1) = 1 e, mais uma vez, como a natureza de Bn nao e alterada por homotetiase rotacoes aplicadas a Pn, podemos supor, sem perda de generalidade que 1 e vertice dePn.

Toda transformacao linear f : C → C fica completamente determinada pelos seus valoresna base {1, i} de C, ou seja, pelos valores f (1) = 1 e f(i) = z, sendo que estes saolinearmente independentes, pois f e bijetiva.Os vertices de Rn serao as raızes n-esimas da unidade: 1,ω1, . . . , ωn−1. Logo, os verticesde Pn sao f (1) = 1, . . . , f

(ωn−1

).

Page 170: Numero 09 - Outubro de 2007

Sendo:ωj = aj + bji, com aj e bj ∈ R,

entao:f(ωj) = f(aj + bji) = f(1aj) + f(ibj) = ajf(1) + bjf(i) = aj + bjz, (2)

ou seja, todo vertice de Pn e uma expressao afim em z.

Tomemos sobre os lados de Pn = f(Rn), n-agonos regulares, dos quais obtemos Bn a partirda uniao dos baricentros desses n-agonos.Queremos mostrar que Bn e um n-agono regular.

Sejam Q0, . . . , Qn−1, n-agonos regulares construidos sobre os lados de Pn de modo queQk tenha como vertices consecutivos f

(ωk

)e f

(ωk+1

). Pelo Lema 2, os vertices de cada

Qk sao expressoes afins em z.

Sejam q0, . . . , qn−1 os centros dos n-agonos Q0, . . . , Qn−1 e Bn o n-agono de verticesnesses centros. Pelo Lema 1, cada qk e media aritmetica dos vertices de Qk. Logo, qk e,tambem, expressao afim em z.

Re

Im

1

�n-1

�2�1

O

Re

Im

f( )�1

f( )�n-1

f( )=11

f( )�2

q0

q1Rn

q2

qn-1

O

f

f(1)=1

f i z( )=

Para verificar que Bn e regular, vamos mostrar que ao rotacionarmos um vertice de Bn

de um angulo de2π

n, no sentido anti-horario, em torno da origem, esse vertice coincidira

com o proximo. Isso e verdade para um n-agono regular, pois ele pode ser inscrito emuma circunferencia de tal modo que todos os seus vertices estejam sobre essa circun-ferencia. Logo, seus angulos centrais formados com seus vertices dois a dois consecutivos

sao congruentes e de medida2π

n.

Mas sabemos que no plano complexo fazer a rotacao de um numero no sentido anti-horarioem torno da origem de um angulo α equivale a multiplica-lo pela constante complexa eαi,

de modulo 1.

Assim, para mostrar que Bn e regular precisamos verificar que a equacao

qk+1 = qke2πn

i, k = 0, . . . , n − 1. (3)

Page 171: Numero 09 - Outubro de 2007

e sempre verdadeira para qualquer z = f (i) .

Para isto, basta encontrarmos dois valores de z que sao raızes da equacao afim (3) , ouseja, duas transformacoes lineares f para as quais o Teorema de Barlotti e verdadeiro, poisduas raızes sao suficientes para mostrar que uma expressao afim em z e uma identidadee vale para qualquer z.

Vamos tomar, por exemplo, z1 = i.

Assim, temos f(1) = 1 e f(i) = i, ou seja, f sera a identidade e Pn = Rn.

Geometricamente:

Re

Im

�0

�n-1

�2

�1

O

f( )�0

f( )�n-1

f( )�1

f( )�2

f i i( )=

f( )1 =1

q0q1

q2 qn-1

Re

Im

O

Assim, q0, . . . , qn−1 estarao a mesma distancia da origem 0, centro de Pn. Alem disso, o

angulo entre eles e2π

n, pois Pn e regular.

q0

q1

q2 q

n-1

O

2�n

2�n

2�n

2�n

2�n

Logo, a equacao afim (3) fica satisfeita para z = i.

Outra raiz que satisfaz a equacao afim (3) e z2 = −i.

Assim, f(1) = 1 e f(i) = −i. Dessa forma f e a reflexao com relacao ao eixo real. De fato,f(ωj) = T(aj + bji) = aj − bji em (2) .

Page 172: Numero 09 - Outubro de 2007

Re

Im

Re

Im

�0

�n-1

�2

�1

O

f i i( )=

f( )1 =1

f( )�0

f( )�n-1

f( )�1

f( )�2

q0q1

q2 qn-1

Entao, neste caso, os vertices de Pn terao a orientacao invertida em relacao aos verticesde Rn e, portanto, Pn e regular.

Deste modo, a demonstracao de que Bn e regular, para z2 = −i, segue de maneira analogaao caso z1 = i.

Logo, a equacao afim (3) fica satisfeita para z = −i.

Conclusao: a equacao (3) e uma identidade e, portanto, vale para qualquer z, ou seja,vale para qualquer n-agono Pn regular afim. �

4 O Teorema de Napoleao

Dado um triangulo ABC qualquer, sejam os triangulos equilateros apoiados externamente(ou internamente) sobre cada um de seus lados. Unindo-se os baricentros X, Y e Z dostriangulos equilateros obtemos o chamado Triangulo Externo (ou Interno) de Napoleao.

Corolario (Teorema de Napoleao) O Triangulo Externo (ou Interno) de Napoleao XYZ

de qualquer triangulo ABC e equilatero.

Page 173: Numero 09 - Outubro de 2007

R

A B

C

Q

P

X

YZ

R'

B

C

A

Q'

P'

X'

Y'

Z'

Demonstracao.

Basta mostrar que todo triangulo ABC com vertices a1+b1i, a2+b2i e a3+b3i e regularafim, ou seja ABC = P3 = F (R3) = T ◦L (R3) , sendo T translacao, L transformacao linearbijetiva e R3 o triangulo regular com centro na origem e vertices nas raızes cubicas da

unidade: 1, −1

2+

√3

2i e −

1

2−

√3

2i.

Sejam L (1) = α + βi, L (i) = γ + δi e T (z) = z + (a + bi) , sendo α,β, γ, δ, a, b ∈ R ez ∈ C.

Logo:

F (x + yi) = L (x + yi) + a + bi

= xL (1) + yL (i) + a + bi

= xα + xβi + yγ + yδi + a + bi

= (xα + yγ + a) + (yδ + xβ + b) i.

Fazendo:

F (1) = a1 + b1i

F

(−

1

2+

√3

2i

)= a2 + b2i

F

(−

1

2−

√3

2i

)= a3 + b3i

Encontramos:

F (x + iy) =

(x2a1 − a2 − a3

3+ y

a2 − a3√3

+a1 + a2 + a3

3

)+

(x2b1 − b2 − b3

3+ y

b2 − b3√3

+b1 + b2 + b3

3

)i,

que e uma transformacao afim.Conclusao: todo triangulo ABC e regular afim e, pelo Teorema de Barlotti, o Triangulode Napoleao e equilatero. �

Page 174: Numero 09 - Outubro de 2007

5 Referencias Bibliograficas

[1] Avila, G. Variaveis Complexas e Aplicacoes. Rio de Janeiro: LTC - Livros Tecnicose Cientıficos Editora. 1990.

[2] Callioli, C. A., Domingues, H. H. & Costa, R. C. F. Algebra Linear eAplicacoes. Sao Paulo: Atual Editora. 1983.

[3] Lima, E. L. Isometrias. Rio de Janeiro: SBM - Sociedade Brasileira de Matematica(Colecao do Professor de Matematica). 1996.

[4] Lopes, S. M. R. Complexidade em Geometria Plana Euclidiana. (Dissertacao deMestrado). Rio de Janeiro: PUC - Pontifıcia Universidade Catolica. 2002.

Page 175: Numero 09 - Outubro de 2007

.

Sobre a necessidade das hipóteses

no Teorema do Ponto Fixo de Banach

Marcelo Lopes Vieira 1

Valdair Bonfim 2

1. Introdução:

O Teorema do Ponto Fixo de Banach é crucial na demonstração de vários

resultados importantes da Matemática. Na teoria das equações diferenciais ordinárias,

por exemplo, ele é utilizado para demonstrar que se o campo vetorial nn RRDf :

é lipschitziano, então o problema de valor inicial

0)0())(()('

)(xx

txftxP

possui uma única solução nRRIx : definida num intervalo maximal I contendo a

origem. ( ver p.ex.[1] ou [3] ). Na demonstração do Teorema de Stampacchia, o qual é

útil na teoria das equações diferenciais parciais elípticas, o Teorema do Ponto Fixo de

Banach desempenha um papel crucial, conforme se pode ver à página 82 de [2].

Além destes dois exemplos, vale citar que a existência de solução )(xf para a

equação integral b

axgdyyfyxKxf )()().,(.)( ,

onde ),( yxK e )(yg são funções contínuas dadas, também pode ser estabelecida com o

auxílio do Teorema do Ponto Fixo de Banach, desde que seja suficientemente

pequeno. ( ver p. ex. [3], à página 19 ).

Dada a importância deste teorema de ponto fixo é natural perguntar se as

hipóteses do mesmo podem ser enfraquecidas, o que levaria a eventuais generalizações

dos teoremas que dele dependem. O que faremos neste trabalho é discutir a necessidade

das hipóteses do referido teorema, mostrando por meio de exemplos que elas são

realmente essenciais.

1 Acadêmico do Curso de Matemática da Universidade Federal de Uberlândia Projeto de Iniciação Científica – PROMAT – FAMAT - UFU. 2 Professor da Faculdade de Matemática – Universidade Federal de Uberlândia Orientador de Projeto de Iniciação Científica no âmbito do PROMAT.

Page 176: Numero 09 - Outubro de 2007

.

2. Preliminares:

Definição 1 ( Contração ):

Sejam ),( dM e ),( N espaços métricos. Uma aplicação ),(),(: NdMf é

dita ser uma contração quando existe uma constante )1,0(c tal que

Myxyxdcyfxf ,,),(.))(,)(( .

Definição 2 ( Sequência de Cauchy ):

Uma seqüência )( nx num espaço métrico ),( dM é denominada Sequência de Cauchy

quando para cada 0 dado, existe Nn0 tal que:

0, nnm ),( nm xxd .

Definição 3 ( Espaço Métrico Completo ) :

Dizemos que o espaço métrico ),( dM é completo quando toda seqüência de Cauchy

)( nx em M converge para um ponto Mp na métrica d, isto é, 0),( pxd n quando

n .

Definição 4 ( Ponto Fixo ):

Dizemos que Mp é um ponto fixo da aplicação MMT : se ppT )( .

3. O Teorema Principal e a necessidade de suas hipóteses

Teorema do Ponto Fixo de Banach: Seja M um espaço métrico completo e seja

MMT : uma contração. Então T possui um único ponto fixo, isto é, existe um

único Mp tal que ppT )( .

Demonstração: Seja 0x um ponto qualquer de M e considere a seqüência 1)( nnx

construída da seguinte forma:

...),(...,),(),( 11201 nn xTxxTxxTx

Observe que

),(.))(),((),( 101021 xxdcxTxTdxxd

),(.),(.))(),((),( 102

212132 xxdcxxdcxTxTdxxd

Page 177: Numero 09 - Outubro de 2007

.

Em geral temos que ),(.),( 101 xxdcxxd nnn para todo inteiro positivo n.

Segue, então, que para todos os números naturais pn , temos:

),(.1

),(.]...1[),(]....[

),(...),(),(),(),(

10101

10121

132211

xxdc

cxxdcccxxdcccc

xxdxxdxxdxxdxxdn

pnpnnnn

pnpnnnnnnnpnn

e como 10 c , segue que 0nc quando n , de onde concluímos que )( nx é

uma seqüência de Cauchy em ),( dM . Sendo M completo, )( nx converge para um ponto

Mp . Assim, como T é contínua (pois sendo contração, é lipschitziana), T

transformará seqüência convergente em seqüência convergente, ou seja:

pxxTxTpT nnn 1lim)(lim)(lim)( .

Fica demonstrada, portanto, a existência de ponto fixo de T .

Provemos agora a unicidade. Para isso, suponhamos que existam Mba , tais que

)(aTa e )(bTb . Então,

0),().1(),(.))(),((),( badcbadcbTaTdbad

e como 01 c , concluímos que 0),( bad , ou seja, ba .

Um fato que chama a atenção neste teorema é a presença de apenas duas hipóteses,

suficientes para demonstrá-lo. Veremos agora alguns exemplos que mostrarão ser estas

hipóteses também necessárias. Precisamente, veremos que a conclusão do teorema fica

prejudicada com a falta de qualquer uma delas.

Exemplo1: Uma das hipóteses do Teorema do Ponto Fixo de Banach é que o espaço

métrico seja completo. Para mostrar que esta hipótese é essencial consideremos o

espaço métrico )1,0(M , o qual não é completo, e a função )1,0()1,0(:f definida

por21

21)( xxf . É fácil ver que f é uma contração e que f não possui ponto fixo

p no intervalo )1,0( , pois

121

21)( pppppf .

Este exemplo mostra que, mesmo tendo uma contração, é impossível obter as

conclusões do Teorema do Ponto Fixo de Banach caso o espaço métrico em questão não

seja completo. A próxima figura ilustra o comportamento da seqüência nx construída

Page 178: Numero 09 - Outubro de 2007

.

iterativamente na demonstração do Teorema do Ponto Fixo de Banach. Observe que

apesar de ser de Cauchy, ela não converge para um ponto do domínio da função f .

Observe também que, se estendermos f continuamente no completamento do espaço

)1,0( , isto é, no domínio ]1,0[ , então a seqüência nx convergirá, de fato, para o único

ponto fixo de f , a saber, o ponto 1p .

Exemplo 2:

Quanto à outra hipótese do teorema, basta tomarmos o espaço métrico completo dos

números reais com a métrica usual e a função RRf : definida primeiramente por

1)( 2xxf . É fácil ver que esta função não é uma contração no domínio R, e que não

possui ponto fixo pois 01)( 2 xxxxf , que não possui solução real. Logo, f

não possui nenhum ponto fixo. Por outro lado, se definirmos 1)( 2xxf , notamos

facilmente que esta função também não é uma contração, como no exemplo acima, mas

agora perdemos a unicidade pois f possui dois pontos fixos, a saber:

251 e

251 .

0x 1x 2x 3x

21

21)( xxfy

xy

Page 179: Numero 09 - Outubro de 2007

.

De fato,

251011)( 22 xxxxxxxf .

Exemplo 3: Não-expansões admitem ponto fixo?

Não necessariamente. Neste caso tudo pode ocorrer. As não-expansões são aplicações

),(),(: dMdMf tais que Myxyxdyfxfd ,,),())(,)(( , e, a menos que

se consiga obter uma desigualdade análoga com uma constante )1,0(c , não dá pra

afirmar que f terá ponto fixo, ou então que f terá um único ponto fixo. Os exemplos

simples que seguem ilustram essa afirmação. Um exemplo é a função RRf :

definida por 1)( xxf , que é uma não-expansão. Neste caso claramente f não possui

ponto fixo, caso contrário, teríamos a igualdade 10 .

Outro exemplo é a função RRf : definida por xxf )( , que é uma não-expansão.

Observe que, em oposição ao exemplo anterior, neste caso todos os pontos do domínio

são pontos fixos.

Bibliografia

[1] Lima, Elon Lages;

Espaços Métricos. Rio de Janeiro, IMPA, CNPq, 1977 ( Projeto Euclides )

[2] Brezis, H.; Analyse Fonctionnelle, Theorie et applications;

Collection Mathématiques Appliquées pour la maitrise.

[3] Goffman, C. & Pedrick, G.; First Course in Functional Analysis.

Prentice-Hall – Series in Modern Analysis

Page 180: Numero 09 - Outubro de 2007
Page 181: Numero 09 - Outubro de 2007

HEURÍSTICAS E EQUAÇÕES DIOFANTINAS

Michelle Crescêncio de Miranda Programa Institucional de Iniciação Científica e Monitoria

da Faculdade de Matemática – PROMAT [email protected]

Luiz Alberto Duran Salomão Professor orientador

[email protected]

Faculdade de Matemática – FAMAT Universidade Federal de Uberlândia – UFU

Resolver problemas é uma habilidade prática como nadar, esquiar ou tocar piano; você pode aprendê-la por meio de imitação e prática. (...)se você quer aprender a nadar, você tem que entrar na água e se você quer se tornar um bom “resolvedor de problemas”, tem que resolver problemas.

George Polya

Introdução

Neste artigo, desenvolvemos um breve estudo sobre equações diofantinas. No entanto, não menos importante que o tema desenvolvido é a oportunidade de nos exercitarmos em diversas heurísticas especialmente adequadas para tratar problemas da natureza que permeia o assunto em tela. Uma heurística é uma sugestão ou estratégia geral, independente de algum tópico particular ou do assunto em questão, que ajude os “resolvedores de problemas” a abordar e entender um problema e a dirigir eficientemente seus recursos para resolvê-lo. Neste breve estudo, destacamos o emprego de algumas dessas heurísticas, a saber, a redução de um problema a uma situação mais simples, o argumento por contradição, o método da descida infinita e o Princípio de Dirichlet.

1. A equação pitagórica

Um dos mais antigos problemas da teoria dos números é a determinação de todas as soluções

inteiras da equação 222 zyx . (I)

Page 182: Numero 09 - Outubro de 2007

Como veremos, a solução dessa equação pode ser obtida através de propriedades elementares

de números inteiros.

Um terno (x, y, z) de inteiros que satisfaz (I) é dito um terno pitagórico. Obviamente, vamos

omitir qualquer caso onde uma das coordenadas do terno (x, y, z) seja zero. Inicialmente,

notemos que se (x, y, z) é um terno pitagórico, então qualquer terno (kx, ky, kz) também o será,

onde k é um inteiro diferente de zero. É claro, ainda, que a recíproca da afirmação que

acabamos de fazer também é verdadeira. Portanto, vamos restringir nossa busca ao caso em

que as coordenadas x, y, e z do terno não têm nenhum fator comum maior do que 1. Nesse

caso, dizemos que uma tal solução (x, y, z) de (I) é primitiva. Por exemplo, (3, 4, 5) é uma

solução primitiva de (I) mas (6, 8, 10), embora seja solução de (I), não é primitiva. Podemos,

na verdade, dizer que se (x, y, z) é uma solução primitiva de (I) não há duas de suas

coordenadas que não sejam inteiros primos entre si. Em outras palavras, se (x, y, z) é uma

solução primitiva de (I), então mdc(x, y) = mdc(x, z) = mdc(y, z) = 1. De fato, se p é um

número primo divisor comum de x e y, então é claro que p também é divisor de

e, conseqüentemente de z (pois p é primo), o que contraria o fato de (x, y, z) ser solução

primitiva de (I). Portanto, mdc (x, y) = 1. Claramente, o mesmo argumento mostra que

mdc(x,z) = 1 e mdc(y, z) = 1. Como conseqüência do fato que acabamos de justificar, x e y

não podem ser ambos pares, se (x, y, z) for uma solução primitiva de (I). Porém, podemos

ainda fazer uma outra afirmação: x e y não podem ser ambos ímpares. De fato, se x = 2a +1 e

y = 2b+1, onde a, b , então x + y = (2a +1) + (2b +1) = 2 + 4( a + a 2 + b + b ),

ou seja, z é divisível por 2 mas não por 4. Ora, isto não é possível pois, se z é divisível

por 2, z também o é; daí, z é divisível por 4. Concluímos, assim, que se (x, y, z) é um terno

pitagórico primitivo, exatamente um dos inteiros x ou y é par e z é ímpar. Vamos assumir

daqui em diante, sem perda de generalidade, que x é par.

222 zyx

2 2 2 2 2

2 2

2

Vamos, agora, determinar todas as soluções primitivas (x, y, z) de (I), reduzindo o problema a

uma situação mais simples. Observe que, uma condição necessária para (x, y, z) ser uma

solução de (I) é que

x = z - y = (z – y) (z + y). (II). 2 2 2

No caso da solução ser primitiva, z – y e z + y são inteiros pares. Daí, podemos dividir por 4

os membros extremos de (II), obtendo

yzyzx41

21 2

.

Page 183: Numero 09 - Outubro de 2007

Chamando yzm21

1 e yzn21

1 , obtemos

11

2

21 nmx (III)

e podemos afirmar que e são primos entre si - de fato, se p é um divisor comum de

e , p divide e p divide

1m 1n 1m

1n znm 11 ynm 11 o que não é possível pois, como vimos,

mdc(y, z) = 1. Além disso, de (III) concluímos que e são quadrados perfeitos, já que

mdc = 1 e o produto é um quadrado perfeito. Portanto, existem inteiros positivos

m e n tais que = m , e = n e mdc(m, n) = 1. Logo,

1m 1n

11, nm 11nm

1m 21n 2 yzmm

21

21

12 ,

yznn21

21

12 e m n =2 2 2

41 x . Decorre daí que

x = 2mn, y = m - n e z = m2 2 2 n . (IV) 2

Observe que m e n, em (IV), têm paridades opostas, pois z e y são ímpares. É imediato

verificar que se x, y, e z são da forma dada em (IV), então (x, y, z) satisfaz a equação

pitagórica (I).

As considerações feitas acima permitem-nos enunciar a seguinte proposição..

Proposição 1: A condição necessária e suficiente para que (x, y, z) seja um terno pitagórico

primitivo, com coordenadas positivas, é que existam inteiros positivos m e n, primos entre si,

de paridades opostas, com m > n, de modo que x = 2mn, y = m - n e z = m + n .2 2 2 2

A tabela a seguir ilustra parcialmente uma representação dos ternos pitagóricos primitivos, conforme a proposição acima.

m

n

2 3 4 5 6 7

1 (4,3,5) (8,15,17) (12,35,37)

2 (12,5,13) (20,21,29) (28,45,53)

3 (24,7,25)

4 (40,9,41) (56,33,65)

5 (60,11,61)

6 (84,13,85)

Page 184: Numero 09 - Outubro de 2007

2. Inexistência de soluções não triviais de algumas equações diofantinas

Ao contrário do que vimos no parágrafo anterior, algumas equações diofantinas podem não ter soluções, além das triviais. Uma ferramenta poderosa para provar a inexistência de soluções de algumas dessas equações é o método da descida infinita, cuja criação é atribuída ao matemático francês Pierre de Fermat (1601 – 1665). Basicamente, esse método consiste em supor a existência e uma solução não trivial que seja, em algum sentido, mínima. Em seguida, deve-se encontrar uma solução que, de alguma forma, venha a contrariar a minimalidade da tal solução, advindo daí uma contradição. A seguir, veremos algumas aplicações desse método.

Proposição 2: A equação x2 + y2 =3z2 não tem soluções inteiras não nulas. Demonstração: Suponha que a equação dada tenha soluções (x, y, z) em inteiros positivos não nulos. Assim, seja (a, b, c) a solução que tenha a coordenada z = c mínima. Sabemos que, se um número inteiro n não for múltiplo de 3, então seu quadrado n2 deixa resto 1 quando dividido por 3. Daí, a e b têm que ser ambos múltiplos de 3, ou seja, existem inteiros r e s tais que a = 3r e b = 3s. Assim, 9r2 + 9s2 =3c2, o que acarreta 3(r2 + s2) = c2. Portanto, c2 é múltiplo de 3 e, conseqüentemente, c é múltiplo de 3. Logo, existe um inteiro t de modo que c=3t. Por fim, temos 3(r2 + s2) =9t2 e, daí, r2 + s2 =3t2, o que quer dizer que o terno (r, s, t) é

solução da equação dada, com cct3

. Contradição com o fato da coordenada c ser

mínima.

A proposição a seguir emprega uma pequena variação do método utilizado na Proposição 1.

Proposição 3: A equação x2 + y2 + z2 = 2xyz não tem soluções inteiras não nulas. Demonstração: Observe que, no membro da esquerda da equação dada, exatamente um dos termos é par ou todos os três são pares. Todavia, na primeira situação, o membro da esquerda seria múltiplo de 2 mas não de 4, enquanto o da direita seria múltiplo de 4. Isso reduz o problema ao caso em que x, y e z são todos pares. Dessa forma, se (x, y, z) satisfaz a equação dada, existem inteiros x1, y1 e z1 tais que x =2x1, y =2y1 e z = 2z1 e, daí,

1112

12

12

1 4 zyxzyx .

Usando o mesmo argumento, temos que existem inteiros x1, y1 e z1 tais que x1 =2x2, y1 =2y2 e z1 = 2z2 e, por conseguinte,

2222

22

22

2 8 zyxzyx .Este argumento pode ser repetido indefinidamente e, assim, teremos que

...2...222

...2...222

...2...222

33

22

1

33

22

1

33

22

1

nn

nn

nn

zzzzzyyyyyxxxxx

o que mostra que x, y e z são divisíveis por 2n, para todo inteiro n. Ora, isso só seria possível para x = y = z = 0.

O resultado a seguir é um caso particular do célebre Último Teorema de Fermat.

Proposição 4: A equação diofantina xn + yn = zn não tem soluções em inteiros não nulos, se nfor um inteiro positivo múltiplo de 4.

Page 185: Numero 09 - Outubro de 2007

Demonstração: Suponha que n = 4k, onde k é um inteiro positivo. Se xn + yn = zn , então temos que (xk)4 + (yk)4 = (z2k)2, ou seja, (xk, yk, z2k) será uma solução da equação a4 + b4 = c2.Assim, o problema fica reduzido a se mostrar que essa última equação não tem soluções além das triviais. Suponha, por absurdo, que a, b e c sejam inteiros positivos que satisfaçam a equação a4 + b4 = c2. Além disso, para aplicarmos o método da descida infinita de Fermat,vamos incluir a hipótese adicional de que c seja mínima, isto é, que não exista uma outra solução (a’, b’, c’), em inteiros positivos, com c’ < c. Então, a e b são primos entre si e, pela Proposição 1, existem inteiros positivos primos entre si u e v tais que a2 = u2 – v2, b2 = 2uve c = u2 + v2. Como a2 + v2 = u2, novamente pela Proposição 1, temos que existem inteiros positivos primos entre si p e q tais que a = p2 – q2, v = 2pq e u = p2 + q2. Daí, segue que b2=2uv = 4pq(p2 + q2). Como p e q são relativamente primos, ambos são também relativamente primos com p2 + q2. Agora, sendo 4pq(p2 + q2) um quadrado perfeito, deveremos ter p, q e p2 + q2 também quadrados perfeitos; portanto, existem inteiros positivos

, e de modo que e . Daí segue que ,sendo Isso contradiz a minimalidade de c.

22 , qp 222 qp 244

.22222 qpuvuc

3. A equação de Pell

Se d é um inteiro positivo que não é um quadrado perfeito, sabemos que d é um número irracional. A equação , onde m representa um inteiro qualquer, é conhecida como a equação de Pell. É claro que, no caso m=0, a equação de Pell não tem solução além

da trivial (x = y = 0) pois, caso contrário, teríamos

mdyx 22

yxd , o que iria contradizer a

irracionalidade de d . Neste parágrafo, desenvolveremos um breve estudo sobre a determinação das soluções da equação de Pell.

A proposição que veremos a seguir é um resultado clássico devido a P.G. Lejeune Dirichlet (1805 – 1859).

Proposição 5: Dado um número irracional , existem infinitos racionais qp , com p e q

inteiros não nulos primos entre si, tais que 2

1qq

p .

Demonstração: Dado um inteiro positivo N qualquer, consideremos os N+1 elementos do intervalo [0, 1) da forma jj , com Nj0 , onde x representa o maior inteiro

que não supera x. Como 1

0

1,1,0N

k Nk

Nk , pelo Princípio de Dirichlet, existem dois

desses elementos, digamos 11 jj e 22 jj pertencentes a um mesmo intervalo

Nk

Nk 1, . Supondo, sem perda de generalidade, que j1 < j2 e chamando q = j2 – j1 e

12 jjp , temos que N

pq 10 e, daí, 2

11qqNq

p . Por fim, podemos

supor que p e q são primos entre si. De fato, se p = p1c e q = q1c, para algum inteiro c>1,

então 21

21

1 11qqq

p.

Page 186: Numero 09 - Outubro de 2007

O resultado a seguir mostra a existência de valores de m para os quais a equação de Pell tem infinitas soluções nos inteiros.

Proposição 6: Se d é um inteiro positivo que não é um quadrado perfeito, existe um inteiro mtal que a equação x2 – dy2 = m admite infinitas soluções inteiras. Demonstração: Como d é irracional, segue pela Proposição 5, que existem infinitos pares

(x, y) de inteiros primos entre si tais que 2

1y

dyx (*). Agora, se x e y são inteiros

satisfazendo essa desigualdade, temos que

122112122 dydyy

ydydxy

ydxydxdyx .

Segue, daí, que algum inteiro não nulo m entre 12 d e 12 d repete-se um número infinito de vezes dentre os valores de x2 – dy2, para x e y satisfazendo a condição (*), ou seja, a equação x2 – dy2 = m admite infinitas soluções inteiras, para um tal m.

Proposição 7: A equação x2 – dy2 = 1, onde d é um inteiro positivo que não é um quadrado perfeito, admite soluções. Demonstração: Conforme a Proposição 6, podemos tomar um inteiro não nulo m de modo que a equação x2 – dy2 = m admite infinitas soluções inteiras. Podemos escolher duas dessas soluções (x1, y1) e (x2, y2) de modo que 21 xx , mas )(mod21 mxx e .Assim,

)(mod21 myy

dyxyxydyxxdyxdyx 211221212211 . (**) Mas, e 2121 ydyxx )(mod02

12

1 mdyx )(mod2112 myxyx e, daí, existem inteiros u e vtais que = mu e 2121 ydyxx mvyxyx 2112 . Segue, então, de (**) que

dyxdyx 2211 = dvum e, daí, dyxdyx 2211 = dvum .

Multiplicando, membro a membro, as duas igualdades acima, obtemos 2222

22

22

12

12 dvumdyxdyxm , ou seja, .122 dvu

Assim, a demonstração estará concluída se mostrarmos que u e v não são nulos. De fato, se u= 0, teríamos –dv2 = 1, o que é um absurdo. Se v = 0, teríamos u = 1 ou -1. De (**), viria

dyxdyx 2211 = e, conseqüentemente, m dyxdyx 2211 e, ainda,

21 xx , o que contraria nossa hipótese sobre as soluções (x1, y1) e (x2, y2).

Proposição 8: Se d é um inteiro positivo que não é um quadrado perfeito então existe uma solução (x0, y0) da equação x2 – dy2 = 1, onde x0 e y0 são inteiros positivos, de modo que todas

as demais soluções (xn, yn) dessa equação satisfazem a condição n

nn dyxdyx 00 ,para algum inteiro n.Demonstração: Mais uma vez, teremos uma aplicação do método da descida infinita. Consideremos a solução (x0, y0) da equação dada, com coordenadas inteiras positivas, de modo que, dentre todas as soluções da equação, o valor dyx 00 seja o menor possível.

Vamos identificar cada solução (x, y) da equação com o número dyx . Pela igualdade dyxdyxdyx 22 , é fácil ver que o produto de duas soluções da equação

também é uma solução, no sentido da identificação acima. Vamos mostrar que todas as

Page 187: Numero 09 - Outubro de 2007

soluções da equação dada são da forma n

dyx 00 , para algum inteiro n. Suponha que

(u, v) seja uma solução da equação em tela e que dvu não seja uma potência com expoente inteiro de dyx 00 . Assim, para algum n, temos

ndyx 00 < dvu <

1

00

ndyx .

Multiplicando cada membro da expressão acima pela solução n

dyx 00 , obtemos

1 < ( dvu )n

dyx 00 < )( 00 dyxo que é um absurdo pois o termo intermediário é uma solução, o que contraria a minimalidade da solução dyx 00 .

Referências bibliográficas

[1] ANDERSON, J. A. e BELL, J. M. – Number Theory with applications – Prentice Hall – 1997[2] ENGEL, A. – Problem-Solving Strategies – Springer – 1997 [3] MOREIRA, C. G. – Propriedades estatísticas de frações contínuas e aproximações diofantinas – Matemática Universitária – Sociedade Brasileira de Matemática – nº 29 – 2000 [4] MUNIZ NETO, A. C. – Equações Diofantinas – EUREKA! – Sociedade Brasileira de Matemática – nº 7 - 2000

Page 188: Numero 09 - Outubro de 2007
Page 189: Numero 09 - Outubro de 2007

Obtenção dos projetos ótimos de gráficos de X utilizando o Matlab.

Robson Silva Rossi 1

[email protected]

Aurélia Aparecida de Araújo Rodrigues2

[email protected]

ResumoFoi desenvolvido um programa em MATLAB para a seleção de parâmetros (tamanho de amostra, intervalo de tempo entre amostras e largura dos limites de controle) de gráficos de X para controle estatístico de processos. Esse programa fornece soluções ótimas em termos de rapidez de detecção do descontrole. Apresenta-se um procedimento para selecionar o projeto ótimo através das tabelas geradas pelo programa. A simplicidade do programa o torna viável para implementação em qualquer ambiente.

Palavras-chave: gráficos de X , controle estatístico de qualidade, projeto de gráfico de controle.

1. Introdução Os processos de produção devem ser permanentemente monitorados, para detectar a presença de causas especiais (que aumentam sua dispersão e/ou tiram sua média do valor-alvo). Detectada essa presença, deve-se proceder a uma investigação para identificar a(s) causa(s) especial(is) e intervir para eliminá-las. A principal ferramenta utilizada para monitorar os processos de produção é o gráfico de controle e foi desenvolvido em 1924 por Walter A. Shewhart. Os gráficos de controle de X , também conhecidos como gráficos da média, servem para monitorar processos cuja característica de qualidade de interesse X é uma grandeza mensurável: o diâmetro de um eixo, volume de uma lata de óleo, o teor de carbono em uma liga metálica etc. O monitoramento é realizado através da análise periódica de amostras: a cada intervalo de tempo h retira-se uma amostra de n itens para análise. Por exemplo, a cada uma hora de produção (h = 60 min), selecionam-se aleatoriamente, cinco garrafas (n = 5), cujos volumes são medidos. Para cada amostra, é calculada a média X dos valores medidos. Os valores de X das diversas amostras são marcados no gráfico. A implementação e o desempenho satisfatório do gráfico de controle de Xdependem da identificação do projeto (design) do gráfico, ou seja, da seleção dos valores para os parâmetros: n (tamanho de amostra), h (intervalo de tempo entre amostra consecutivas), e k (fator de abertura dos limites de controle). A dificuldade em obter valores precisos para esses parâmetros é grande. A estabilidade do processo está associada à freqüência com que ele tem tendência de sair de controle, a qual pode ser medida pelo tempo médio que opera isento de causas especiais, ou seja, em controle. O prejuízo de operar um processo sob efeito de uma ou mais causas especiais está associado ao prejuízo devido ao aumento da proporção de

1 Aluno de graduação em Engenharia Mecânica (UFU) e de Iniciação Científica (PROMAT). 2 Orientadora. Professora Adjunto da Faculdade de Matemática (UFU).

Page 190: Numero 09 - Outubro de 2007

unidades produzidas fora das especificações quando a média do processo desloca-se e/ou a variabilidade aumenta. Esse prejuízo é função da magnitude do aumento da variância. Na prática quase geral, o valor de h acaba sendo escolhido arbitrariamente. Ao arbitrar o valor de h, devemos ter sempre em mente o seguinte: valores pequenos de h implicam custos elevados com amostragens e maior incidência de alarmes falsos. Exemplo: se k = 3, n = 4 e h = 1 hora, então, inspecionam-se quatro itens por hora e, em média, há um alarme falso a cada 370,4 horas. Por outro lado, se h = 30 minutos, então inspecionam-se oito itens por hora e, em média há um alarme falso a cada 185,2 horas (Costa et al. 2004).

O objetivo deste trabalho foi desenvolver um programa em Matlab (2002), o qual fornece o projeto ótimo do gráfico de X e auxilia o usuário na escolha dos parâmetros desse projeto.

O presente trabalho baseia-se no procedimento de Costa et. al. (2004), que utiliza uma planilha do Excel, para obter a melhor combinação de n, h e k para a determinação dos valores ótimos para os parâmetros do gráfico X . Aqui é abordado apenas o gráfico de X . No entanto, o programa apresentado pode ser facilmente adaptado a outros gráficos, ou conjunto de gráficos.

2. Modelo matemático Se a característica de qualidade de interesse for representada por variáveis

contínuas (mensuráveis), por exemplo, volume de refrigerante em uma garrafa, então, o tipo de gráfico indicado para monitorar o processo são os gráficos de controle por variáveis. Neste caso, o gráfico da média (de X ) é o mais usado para controlar a média de um processo, enquanto que o gráfico da amplitude (de R) ou o gráfico dodesvio-padrão (S) controla a variabilidade do processo. Assim, é recomendável e usual a implementação simultânea dos gráficos de X e R ou (S) para controlar a média e a variabilidade do processo. Aqui, será descrito apenas o gráfico de X ; o desenvolvimento dos gráficos de R ou de S pode ser encontrado na literatura especializada. Ver, por exemplo, Montgomery (2004) e Costa et al. (2004). Suponha que a característica de qualidade X de um processo seja uma variável contínua. Considere que, com o processo em controle, X seja normalmente distribuída com média 0 e variância 0

2; toma-se uma amostra de tamanho n, então a estatística

1

n

ii

XX

n (1)

é normalmente distribuída com média e variância dadas, respectivamente, por: 0E X (2)

20Var X n (3)

Logo, os limites de controle do gráfico de X sãoLSC = 0 + k 0/ n (4) LM = 0 (5)

LIC = 0 - k 0/ n (6) Os pontos no gráfico são as médias de cada amostra dos dados. Tipicamente, as médias são registradas nos gráficos durante algum tempo, por exemplo, durante dias ou semanas. Se os valores de 0 e 0 não forem conhecidos, precisam ser estimados a partir de amostras iniciais. Recomenda-se vinte amostras, no mínimo, para construir gráficos que forneçam boas estimativas estatísticas.

Page 191: Numero 09 - Outubro de 2007

Suspeita-se que um processo esteja fora de controle quando um ponto se encontra acima do limite de controle superior ou abaixo do limite de controle inferior, ou ainda se for identificado um comportamento não aleatório dos pontos, ou seja, os pontos apresentam um padrão; por exemplo, pontos consecutivos crescendo ou decrescendo. Ver a figura 1. Portanto, outras regras que identificam indícios de processos fora de controle foram desenvolvidas; são as chamadas regras suplementares.A inclusão de novas regras de decisão implica sempre um aumento na incidência dealarmes falsos. Uma boa discussão sobre uso de regras suplementares é feita por Costa et al. (2004).

LSC Variação devida a causas especiais

LM Variação devida a causas comuns

LIC Variação devida a causas especiais

Figura 1: Ilustração esquemática de um gráfico de controle

Quando for localizado um ponto fora dos limites de controle ou identificado um comportamento não aleatório dos pontos no gráfico, passa-se então ao processo de investigação para descobrir se de fato o processo está fora de controle; se ele de fato estiver, as causas especiais devem ser removidas. O sensato é não parar o desenvolvimento da qualidade quando o sistema está em controle. É necessário que se busque o aperfeiçoamento contínuo da qualidade, por exemplo, através do uso de outras ferramentas como fluxogramas, gráficos de causa e efeito, gráficos de Pareto e projeto e análise de experimentos. Descrições dessas ferramentas são encontradas, em Montgomery (2004).

2.1 Medidas de desempenho de gráficos de controle Existe uma conexão estreita entre os gráficos de controle e os testes de hipótese.

Basicamente, um gráfico de controle é um teste de hipótese em que a hipótese nula é a de que o processo está em controle:

H0: = 0

H1: 0Este teste é realizado para cada nova amostra, pois o estado do processo pode

mudar.

Page 192: Numero 09 - Outubro de 2007

Região de

rejeição de H0 LSC

Região de LM

aceitação de H0

Região de LIC

rejeição de H0

Figura 2: Relação entre gráficos de controle e testes de hipóteses

Quando o ponto amostral se situa entre limites de controle, LIC e LSC, aceita-se a hipótese nula; caso contrário, deve-se rejeitá-la. Como em qualquer teste de hipótese, podem ocorrer erros do tipo I e tipo II; caso, respectivamente, se rejeite a hipótese nula de que o processo está em controle quando ele na realidade está em controle e caso se aceite a hipótese nula de que o processo está em controle quando ele, na realidade, não está. As probabilidades dos erros do tipo I e II são denotadas por e ,respectivamente. O NMA (ou ARL, do inglês Average Run Length) representa o número médio de amostras retiradas até que seja emitido um sinal pelo gráfico. Quando o processo está em controle, o NMA é denotado por NMA0 e quando o processo está fora de controle, o NMA é denotado por NMA1. A variável aleatória NA, que representa o número de amostras até um sinal, é distribuída geometricamente, com probabilidade de sucesso p, onde p é igual a quando o processo está em controle e igual a (1 - ) quando o processo está fora de controle.Então,

01NMA (7)

11

1NMA 1

poder (8)

Lembrando que é a probabilidade do erro do tipo I e é a probabilidade do erro tipo II; logo, aqui no caso, (1 - ) representa o poder do gráfico. Para um gráfico com limites de 3 , = 0,0027 é a probabilidade nominal (supondo distribuição normal da estatística amostral, e 0 e 0 estimados sem erro) de que um ponto caia fora dos limites de controle, quando o processo está em controle. Assim:

NMA0 = 10,0027

= 370 amostras. (9)

Page 193: Numero 09 - Outubro de 2007

Ou seja, mesmo que o processo esteja em controle, um sinal de alarme – alarme falso – será gerado, em média, a cada 370 amostras. Por outro lado, quando o processo está fora de controle e um alarme é gerado, esse alarme é um alarme verdadeiro. Portanto, são desejáveis valores grandes para NMA0 e valores pequenos para NMA1.

2.2. Tempo esperado até o sinal Suponha que o processo está sujeito a descontroles, na forma de deslocamentos

bruscos (shifts) em sua média. Um ponto iX fora dos limites de controle é interpretado como sinal de que o processo está fora de controle. Seja 1 o valor da média após o deslocamento, defina-se

1 0

0

d (10)

Essa expressão representa o deslocamento em unidades de desvio padrão. O intervalo de tempo entre a alteração do parâmetro do processo (média, por

exemplo) que está sendo monitorado por um gráfico de controle e a ocorrência do alarme é uma medida de eficiência do gráfico. O valor esperado desse intervalo de tempo, que será chamado tempo esperado até o sinal (TES), depende da magnitude da alteração do parâmetro do processo (d), do tamanho das amostras (n), do intervalo de tempo entre as amostras (h) e do fator de abertura dos limites de controle (k).

descontrole SINAL

t t + h t +2h t +(NA)hh

S TS

(NA)h

Figura 3: Tempo até o sinal, sendo intervalo de tempo entre amostras fixo

Suponha-se que a alteração da média seja brusca e mantida (ocorre instantaneamente e permanece até que se intervenha no processo), definam-se as seguintes variáveis aleatórias (ver Figura 3): S, tempo decorrido entre a extração da última amostra anterior ao descontrole (alteração) e o descontrole (0 S h); NA, o número de amostras extraídas entre a ocorrência do descontrole e o sinal emitido pelo gráfico, e TS, o tempo entre descontrole e o sinal.

Então,TS = (NA)h – S. (11)

O tempo esperado entre o descontrole e o sinal, TES, é: TES = E(TS) = hE(NA) – E(S). (12)

Page 194: Numero 09 - Outubro de 2007

Dado que E(NA) é o número médio de amostras até o sinal (NMA), já definido anteriormente, e como S tem uma distribuição aproximadamente uniforme no intervalo [0, h], (Reynolds,1988). Então,

2hE S . (13)

Portanto,

TES h(NMA – 0,5). (14)

2.3. Formulação do problema Considere o seguinte exemplo: se amostras de tamanho 2 são retiradas do processo a cada meia hora e os limites adotados são de 3 (n = 2; h = 0,5; k = 3,00). Se a média do processo se deslocar de 1,5 desvios-padrão ( d = 1,5), então, em média, serão necessárias 5,26 amostras até o sinal de descontrole (NMA1= 5,26 amostras) e o TES = 2,38 horas (143 minutos). Ao aumentar o tamanho de n, por exemplo, se n = 4, o NMA1 e o TES serão reduzidos, ou seja, NMA1= 2 amostras e TES = 0,75 horas (45 minutos). Contudo, a taxa de amostragem, r = n/h, passa de 4 itens por hora para 8 itens por hora. Em ambos os casos haverá, em média, um alarme falso a cada 370,4 amostras ou a cada 185,2 horas, pois, o tempo médio entre alarmes falsos é TMAF = NMA0 h = h/ . Veja situação 1 na Tabela 1.

Tabela 1: Exemplos de projetos para o gráfico de controle X , d = 1,5 e TMAF = 500 Situação 1 Situação 2

n 2 amostras 4 amostras 2 amostras h 0,5 hora 0,5 hora 0,25 hora k 3,00 3,00 3,00r 4 amostras/hora 8 amostras/hora 8 amostras/hora

NMA1 5,26 amostras 2 amostras 5,26TES 2,38 horas 0,75 horas 1,19 amostras

MNA0 370,4 amostras 370,4 amostras 370,4 amostras TMAF 185,2 horas 185,2 horas 92,6 horas

Outra alternativa para reduzir o TES consiste em retirar as amostras com maior freqüência; por exemplo, uma amostra de tamanho n = 2 a cada 15 minutos (0,25 horas). Veja situação 2 na Tabela 1. O valor de TES cai para a metade, TES =1,19 minutos. Agora, além de a taxa de amostragem r dobrar, a freqüência de alarmes falso também dobra, ou seja passa-se a ter em média um alarme falso a cada 370,4 0,25 = 92,6 horas. Nesse exemplo, a primeira alternativa é melhor, pois leva a um menor TES e a um maior TMAF. Mas pode ser que haja alguma alternativa ainda melhor, mantendo a taxa de amostragem r = 8, como, por exemplo. Retirar amostras de tamanho n = 8 de hora em hora. Seria preciso calcular o TES dessa alternativa para dizer se ela é ou não mais vantajosa que a primeira.

Percebe-se pelo exemplo dado que, para determinar os parâmetros do gráfico de controle X , é conveniente usar o TES como medida de rapidez de detecção, mas é preciso definir primeiro a freqüência admissível de alarmes falsos (TMAF) e a taxa de amostragem (r) . Existe uma infinidade de combinações de n, h e k, que resultam em uma mesma taxa de amostragem r e num mesmo TMAF pré-especificados. No entanto, para cada valor hipotético do deslocamento, d, há uma combinação que minimiza o

Page 195: Numero 09 - Outubro de 2007

TES. Por meio de um programa simples em MATLAB, ver Apêndice, é possível obter o TES para diferentes combinações de n, h, TMAF e do deslocamento d contra o qual se deseja maior proteção (isto é, para o qual se quer minimizar o TES). O parâmetro nassume valores inteiros (1, 2, ...) e o parâmetro h, valores práticos ( 1/4 de hora, ½ hora, 2 horas). Portanto, através da tabela gerada pelo programa, podemos rapidamente encontrar a melhor combinação de n, h e k, ou seja, aquela que minimiza o TES.

Para cada valor de h, o valor de k é determinado por: 1

2( )hk

TMAF

1Poder

, (15)

pois = 2 (-k), ou seja, k = - -1( /2) = - -1(h / (2TMAF)). Após obtenção do valor de k, utiliza-se a expressão para calcular o poder do

gráfico, ou simplesmente, Poder := Pr PrZ k n Z k n (16)

Utilizando o valor do Poder, calcula-se o NMA1 (na equação 8) e finalmente o TES (na equação 14)

3. Soluções ótimas Utilizando o programa em MATLAB, ver no Apêndice, é possível obter o

projeto ótimo do gráfico X , ou seja, combinação de valores para n, h e k que possui o menor TES. O programa consiste em implementar as equações 8, 15 e 16.

Aqui, os valores de TMAF, r, d e n serão fixos, veja tabelas 2, 3 e 4: TMAF = 500 horas; r = 6, 8 e 10 n = 1 a 20; d = 0,5 a 3,5 Caso seja de interesse do usuário, ele pode atribuir outros valores para TMAF, r,

d e n no programa em MATLAB. Conforme tabelas 2 a 4, é possível concluir que: a) d 1,0: é melhor utilizar amostras grandes (n 16) e intervalo entre amostragens

longo (h 96 minutos). b) 1,25 d 1,5: é melhor utilizar amostras médias (8 n 16) e intervalos entre

amostragens moderados (48 minutos h 120 minutos). c) 1,75 d 2,25: é melhor utilizar amostras pequenas (4 n 6) e intervalos entre

amostragens curtos (24 minutos h 60 minutos). d) d 2,5: é melhor utilizar amostras muito pequenas (2 n 3) e intervalos entre

amostragens muito curtos (12 minutos h 30 minutos).

Page 196: Numero 09 - Outubro de 2007

Tabela 2: Tempo esperado até o sinal, em horas ( r = n/h = 6 e TMAF = 500 minutos)

n = 1 2 3 4 5 6 7 8 9 10 11 12 13 14 15 16 17 18 19 20h* = 10,0 20,0 30,0 40,0 50,0 60,0 70,0 80,0 90,0 100,0 110,0 120,0 130,0 140,0 150,0 160,0 170,0 180,0 190,0 200,0k =3,59 3,40 3,29 3,21 3,14 3,09 3,04 3,00 2,97 2,94 2,91 2,88 2,85 2,83 2,81 2,79 2,77 2,75 2,73 2,71

0,50 161,79 94,25 64,85 48,65 38,53 31,69 26,80 23,15 20,35 18,14 16,35 14,89 13,67 12,64 11,77 11,02 10,37 9,80 9,30 8,860,75 73,09 34,60 21,29 14,90 11,29 9,02 7,48 6,40 5,59 4,98 4,51 4,13 3,83 3,59 3,39 3,23 3,10 2,99 2,91 2,841,00 34,42 14,10 8,14 5,55 4,16 3,33 2,80 2,43 2,17 1,99 1,86 1,77 1,72 1,68 1,67 1,67 1,69 1,72 1,76 1,801,25 17,11 6,37 3,59 2,45 1,88 1,55 1,35 1,23 1,17 1,14 1,14 1,16 1,20 1,24 1,30 1,37 1,44 1,52 1,59 1,671,50 8,97 3,17 1,80 1,26 1,01 0,89 0,84 0,83 0,85 0,89 0,95 1,02 1,09 1,17 1,25 1,34 1,42 1,50 1,58 1,671,75 4,96 1,72 1,01 0,75 0,65 0,63 0,65 0,70 0,77 0,84 0,92 1,00 1,08 1,17 1,25 1,33 1,42 1,50 1,58 1,672,00 2,88 1,01 0,63 0,52 0,50 0,54 0,60 0,67 0,75 0,83 0,92 1,00 1,08 1,17 1,25 1,33 1,42 1,50 1,58 1,672,25 1,76 0,64 0,44 0,41 0,44 0,51 0,59 0,67 0,75 0,83 0,92 1,00 1,08 1,17 1,25 1,33 1,42 1,50 1,58 1,672,50 1,12 0,44 0,34 0,36 0,42 0,50 0,58 0,67 0,75 0,83 0,92 1,00 1,08 1,17 1,25 1,33 1,42 1,50 1,58 1,672,60 0,95 0,38 0,31 0,35 0,42 0,50 0,58 0,67 0,75 0,83 0,92 1,00 1,08 1,17 1,25 1,33 1,42 1,50 1,58 1,672,70 0,81 0,34 0,30 0,34 0,42 0,50 0,58 0,67 0,75 0,83 0,92 1,00 1,08 1,17 1,25 1,33 1,42 1,50 1,58 1,672,80 0,69 0,30 0,28 0,34 0,42 0,50 0,58 0,67 0,75 0,83 0,92 1,00 1,08 1,17 1,25 1,33 1,42 1,50 1,58 1,672,90 0,59 0,27 0,27 0,34 0,42 0,50 0,58 0,67 0,75 0,83 0,92 1,00 1,08 1,17 1,25 1,33 1,42 1,50 1,58 1,673,00 0,52 0,25 0,26 0,34 0,42 0,50 0,58 0,67 0,75 0,83 0,92 1,00 1,08 1,17 1,25 1,33 1,42 1,50 1,58 1,673,10 0,45 0,23 0,26 0,33 0,42 0,50 0,58 0,67 0,75 0,83 0,92 1,00 1,08 1,17 1,25 1,33 1,42 1,50 1,58 1,673,20 0,39 0,22 0,26 0,33 0,42 0,50 0,58 0,67 0,75 0,83 0,92 1,00 1,08 1,17 1,25 1,33 1,42 1,50 1,58 1,673,30 0,35 0,20 0,25 0,33 0,42 0,50 0,58 0,67 0,75 0,83 0,92 1,00 1,08 1,17 1,25 1,33 1,42 1,50 1,58 1,673,40 0,31 0,20 0,25 0,33 0,42 0,50 0,58 0,67 0,75 0,83 0,92 1,00 1,08 1,17 1,25 1,33 1,42 1,50 1,58 1,673,50 0,28 0,19 0,25 0,33 0,42 0,50 0,58 0,67 0,75 0,83 0,92 1,00 1,08 1,17 1,25 1,33 1,42 1,50 1,58 1,67

* em minutos

d

Page 197: Numero 09 - Outubro de 2007

Tabela 3: Tempo esperado até o sinal, em horas ( r = n/h = 8 e TMAF = 500 minutos)

n = 1 2 3 4 5 6 7 8 9 10 11 12 13 14 15 16 17 18 19 20h* = 7,5 15,0 22,5 30,0 37,5 45,0 52,5 60,0 67,5 75,0 82,5 90,0 97,5 105,0 112,5 120,0 127,5 135,0 142,5 150,0k =3,66 3,48 3,37 3,29 3,23 3,17 3,13 3,09 3,06 3,02 2,99 2,97 2,94 2,92 2,90 2,88 2,86 2,84 2,82 2,81

0,50 156,48 89,62 60,88 45,19 35,46 28,93 24,28 20,84 18,20 16,13 14,46 13,10 11,97 11,03 10,22 9,53 8,93 8,41 7,96 7,550,75 69,41 32,09 19,39 13,38 10,01 7,91 6,51 5,51 4,78 4,23 3,80 3,47 3,19 2,97 2,79 2,65 2,53 2,43 2,35 2,281,00 32,13 12,77 7,22 4,83 3,57 2,83 2,35 2,02 1,79 1,63 1,51 1,42 1,37 1,33 1,31 1,30 1,31 1,32 1,35 1,381,25 15,71 5,64 3,10 2,08 1,57 1,27 1,10 0,99 0,92 0,89 0,88 0,89 0,91 0,95 0,99 1,03 1,09 1,14 1,20 1,261,50 8,11 2,75 1,52 1,05 0,82 0,71 0,66 0,64 0,65 0,68 0,72 0,77 0,82 0,88 0,94 1,00 1,06 1,13 1,19 1,251,75 4,41 1,46 0,83 0,61 0,52 0,49 0,50 0,53 0,58 0,63 0,69 0,75 0,81 0,88 0,94 1,00 1,06 1,13 1,19 1,252,00 2,53 0,85 0,51 0,41 0,39 0,41 0,45 0,51 0,56 0,63 0,69 0,75 0,81 0,88 0,94 1,00 1,06 1,13 1,19 1,252,25 1,52 0,53 0,35 0,31 0,34 0,38 0,44 0,50 0,56 0,63 0,69 0,75 0,81 0,88 0,94 1,00 1,06 1,13 1,19 1,252,50 0,96 0,35 0,26 0,27 0,32 0,38 0,44 0,50 0,56 0,63 0,69 0,75 0,81 0,88 0,94 1,00 1,06 1,13 1,19 1,252,60 0,81 0,31 0,24 0,26 0,32 0,38 0,44 0,50 0,56 0,63 0,69 0,75 0,81 0,88 0,94 1,00 1,06 1,13 1,19 1,252,70 0,68 0,27 0,23 0,26 0,31 0,38 0,44 0,50 0,56 0,63 0,69 0,75 0,81 0,88 0,94 1,00 1,06 1,13 1,19 1,252,80 0,58 0,24 0,22 0,26 0,31 0,38 0,44 0,50 0,56 0,63 0,69 0,75 0,81 0,88 0,94 1,00 1,06 1,13 1,19 1,252,90 0,50 0,22 0,21 0,25 0,31 0,38 0,44 0,50 0,56 0,63 0,69 0,75 0,81 0,88 0,94 1,00 1,06 1,13 1,19 1,253,00 0,43 0,20 0,20 0,25 0,31 0,38 0,44 0,50 0,56 0,63 0,69 0,75 0,81 0,88 0,94 1,00 1,06 1,13 1,19 1,253,10 0,37 0,18 0,20 0,25 0,31 0,38 0,44 0,50 0,56 0,63 0,69 0,75 0,81 0,88 0,94 1,00 1,06 1,13 1,19 1,253,20 0,33 0,17 0,19 0,25 0,31 0,38 0,44 0,50 0,56 0,63 0,69 0,75 0,81 0,88 0,94 1,00 1,06 1,13 1,19 1,253,30 0,29 0,16 0,19 0,25 0,31 0,38 0,44 0,50 0,56 0,63 0,69 0,75 0,81 0,88 0,94 1,00 1,06 1,13 1,19 1,253,40 0,25 0,15 0,19 0,25 0,31 0,38 0,44 0,50 0,56 0,63 0,69 0,75 0,81 0,88 0,94 1,00 1,06 1,13 1,19 1,253,50 0,22 0,14 0,19 0,25 0,31 0,38 0,44 0,50 0,56 0,63 0,69 0,75 0,81 0,88 0,94 1,00 1,06 1,13 1,19 1,25

* em minutos

d

Page 198: Numero 09 - Outubro de 2007

Tabela 4: Tempo esperado até o sinal, em horas ( r = n/h =10 e TMAF = 500 minutos)

n = 1 2 3 4 5 6 7 8 9 10 11 12 13 14 15 16 17 18 19 20h* = 6,0 12,0 18,0 24,0 30,0 36,0 42,0 48,0 54,0 60,0 66,0 72,0 78,0 84,0 90,0 96,0 102,0 108,0 114,0 120,0k =3,72 3,54 3,43 3,35 3,29 3,24 3,19 3,16 3,12 3,09 3,06 3,04 3,01 2,99 2,97 2,95 2,93 2,91 2,89 2,88

0,50 152,52 86,23 58,00 42,70 33,27 26,97 22,51 19,22 16,70 14,73 13,16 11,88 10,81 9,92 9,17 8,52 7,97 7,48 7,06 6,680,75 66,72 30,29 18,05 12,32 9,13 7,15 5,84 4,92 4,24 3,73 3,34 3,03 2,78 2,57 2,41 2,27 2,16 2,07 1,99 1,931,00 30,48 11,84 6,58 4,34 3,18 2,49 2,05 1,75 1,54 1,39 1,28 1,20 1,15 1,11 1,09 1,07 1,07 1,08 1,10 1,121,25 14,71 5,14 2,77 1,83 1,36 1,10 0,93 0,83 0,77 0,74 0,73 0,73 0,74 0,77 0,80 0,83 0,87 0,92 0,96 1,011,50 7,50 2,46 1,33 0,90 0,70 0,60 0,55 0,53 0,53 0,55 0,58 0,62 0,66 0,71 0,75 0,80 0,85 0,90 0,95 1,001,75 4,04 1,29 0,72 0,52 0,43 0,40 0,41 0,43 0,47 0,51 0,55 0,60 0,65 0,70 0,75 0,80 0,85 0,90 0,95 1,002,00 2,29 0,74 0,43 0,34 0,32 0,33 0,36 0,40 0,45 0,50 0,55 0,60 0,65 0,70 0,75 0,80 0,85 0,90 0,95 1,002,25 1,36 0,46 0,29 0,26 0,27 0,31 0,35 0,40 0,45 0,50 0,55 0,60 0,65 0,70 0,75 0,80 0,85 0,90 0,95 1,002,50 0,85 0,30 0,22 0,22 0,26 0,30 0,35 0,40 0,45 0,50 0,55 0,60 0,65 0,70 0,75 0,80 0,85 0,90 0,95 1,002,60 0,71 0,26 0,20 0,21 0,25 0,30 0,35 0,40 0,45 0,50 0,55 0,60 0,65 0,70 0,75 0,80 0,85 0,90 0,95 1,002,70 0,60 0,23 0,19 0,21 0,25 0,30 0,35 0,40 0,45 0,50 0,55 0,60 0,65 0,70 0,75 0,80 0,85 0,90 0,95 1,002,80 0,51 0,20 0,18 0,20 0,25 0,30 0,35 0,40 0,45 0,50 0,55 0,60 0,65 0,70 0,75 0,80 0,85 0,90 0,95 1,002,90 0,43 0,18 0,17 0,20 0,25 0,30 0,35 0,40 0,45 0,50 0,55 0,60 0,65 0,70 0,75 0,80 0,85 0,90 0,95 1,003,00 0,37 0,16 0,16 0,20 0,25 0,30 0,35 0,40 0,45 0,50 0,55 0,60 0,65 0,70 0,75 0,80 0,85 0,90 0,95 1,003,10 0,32 0,15 0,16 0,20 0,25 0,30 0,35 0,40 0,45 0,50 0,55 0,60 0,65 0,70 0,75 0,80 0,85 0,90 0,95 1,003,20 0,28 0,14 0,16 0,20 0,25 0,30 0,35 0,40 0,45 0,50 0,55 0,60 0,65 0,70 0,75 0,80 0,85 0,90 0,95 1,003,30 0,25 0,13 0,15 0,20 0,25 0,30 0,35 0,40 0,45 0,50 0,55 0,60 0,65 0,70 0,75 0,80 0,85 0,90 0,95 1,003,40 0,22 0,12 0,15 0,20 0,25 0,30 0,35 0,40 0,45 0,50 0,55 0,60 0,65 0,70 0,75 0,80 0,85 0,90 0,95 1,003,50 0,19 0,12 0,15 0,20 0,25 0,30 0,35 0,40 0,45 0,50 0,55 0,60 0,65 0,70 0,75 0,80 0,85 0,90 0,95 1,00

* em minutos

d

Page 199: Numero 09 - Outubro de 2007

4. Procedimento para obtenção do projeto ótimo do gráfico XOs valores ótimos de n, h e k podem ser facilmente obtidos utilizando as tabelas 2 a 4

ou o programa em MATLAB, ver apêndice. Veja o procedimento a seguir. 1°) Fixar TMAF e r . Exemplo: TMAF = 500 minutos e r = 6 amostras por hora. 2°) Localizar a tabela que contempla a situação do passo anterior. Exemplo: tabela 2, para TMAF = 500 minutos e r = 6 amostras por hora . Se for o caso, utilizar o programa em MATLAB para gerar outra tabela com outros valores de interesse para TMAF e r. Outros valores também podem ser atribuídos a n e d.3°) Fixar o valor de d (primeira coluna da tabela gerada). Exemplo: d = 1,75. 4°) Na linha de d fixo, identificar o TESmínimo e o respectivo projeto ótimo. Exemplo: na tabela 2, para d = 1,75, obtém-se TESmínimo = 0, 63 horas. 5°) Identificar os valores ótimos para n, h e k. Exemplo: na tabela 2, para d = 1,75 e TESmínimo = 0, 63; tem-se nótimo = 6; h ótimo =60 minutos e k ótimo = 3,09.

5. Conclusão O presente trabalho apresenta um programa em MATLAB que gera tabelas para

auxiliar o usuário na escolha dos parâmetros ótimos (tamanho de amostra n, intervalo de tempo entre amostras h e fator de abertura do limites de controle k) do gráfico de X .

É comum o uso do gráfico X em conjunto com o gráfico de R ou S para monitoramento da variabilidade. O procedimento e programa aqui apresentados podem ser adaptados para outros gráficos ou conjuntos de gráficos

Os gráficos de controle de X e outros gráficos de controle de Shewhart são relativamente insensíveis para pequenos desvios no processo, ou seja, para d < 1,5. Neste caso, outros gráficos podem ser usados, por exemplo o gráfico da soma acumulada ( CUSUM) e o gráfico da média móvel ponderada exponencialmente ( EWMA). Esses gráficos são descritos por Montgomery (2004) e Costa et al. (2004).

5. Referências bibliográficas

COSTA, A. F. B.; EPPRECHT, E. K.; CARPINETTI, L. C. R. Controle Estatístico de Qualidade. São Paulo: Editora Atlas, 2004.

HANSELMAN, D.; LITTLEFIELD, D. B. Matlab 6: curso completo. São Paulo: Prentice Hall, 2003.

MATLAB 6.5 Release 13, The Mathworks. 2002.

MONTGOMERY, D. C. Introduction to Statistical Quality Control. 5. ed. New York: John Wiley, 2004.

REYNOLDS, M. R.; AMIN, R. W.; ARNOLD, J. C.; NACHLAS, J. A. x charts with variable sampling intervals. Technometrics, v.30, n.2, p.181-192, 1988.

Page 200: Numero 09 - Outubro de 2007

Apêndice: Programa em MATLAB

clear all; TMAF=500 ; %tempo medio ate a ocorrencia de um alarme falso. r=6; %taxa de amostragem. i=0;for d=[.5:.25:2.25 2.5:.1:3.5]; i=i+1; j=0; for n= [1:1:20] %tamanho da amostra. L=length(d); m=length(n); j=j+1; h=n/r ; %intervalo de tempo entre as amostras, em horas. k=(norminv(h/(2*TMAF)))*(-1) %fator de abertura do limite de controle. vetk(:,j)= k poder=normcdf(-k+(d*sqrt(n)),0,1)+ normcdf(-k-(d*sqrt(n)),0,1); % poder do grafico de controle NMA1=1/poder; %Numero medio de amostras ate detecçao de descontrole TES(L,m)=(h*(NMA1-0.5)); A(i,j)= TES(L,m); end end

Page 201: Numero 09 - Outubro de 2007

Formas Quadráticas e Cônicas

Stela Zumerle Soares1 Antônio Carlos Nogueira2

([email protected]) ([email protected])

Faculdade de Matemática, UFU, MG

1 Bolsista do PET -Matemática da Universidade Federal de Uberlândia 2 Docente da Faculdade de Matemática da Universidade Federal de Uberlândia

1. Resumo

Nesse trabalho pretendemos apresentar alguns resultados da álgebra linear. Nosso objetivo é exibir os conceitos de formas bilineares e formas quadráticas. Além disso, faremos a classificação das cônicas no plano.

2 - Formas Bilineares

Definição 2.1 - Seja V um espaço vetorial sobre o corpo . Uma forma bilinear sobre V é uma função

Ff , que associa a cada par ordenado de vetores , em V , um escalar ( , )f

em , e que satisfaz F1 2 1 2

1 2 1 2

( , ) ( , ) ( ,( , ) ( , ) ( , )

f c cf ff c cf f

) .

A função nula de V é também uma forma bilinear. Além disso, toda combinação linear de formas bilineares sobre V é uma forma bilinear.

V

Assim, o conjunto das formas bilineares sobre V é um subespaço vetorial do espaço das funções de V em .V F

Exemplo 2.1 – Seja V um espaço vetorial sobre o corpo e sejam e funcionais lineares sobre V . Definamos

F 1L 2Lf por

1 2( , ) ( ) ( )f L L .

Fixando e considerando f como uma função de , então temos simplesmente um múltiplo escalar do funcional linear .1LCom fixo, f é um múltiplo escalar de .2LAssim, é evidente que f é uma forma bilinear sobre V .

Definição 2.2 – Seja V um espaço vetorial de dimensão finita e seja 1{ , , }n uma base ordenada de V . Se f é uma forma bilinear sobre V , a matriz de f em relação à base

Page 202: Numero 09 - Outubro de 2007

ordenada é a matriz n n A com elementos ( , )ij i jA f . Às vezes indicaremos esta matriz por [ ]f .

Teorema 2.1 – Seja V um espaço vetorial de dimensão finita sobre o corpo . Para cada base ordenada

F de V , a função que associa a cada forma bilinear sobre V sua matriz em

relação à base ordenada é um isomorfismo do espaço ( , , )L V V F no espaço das matrizes sobre o corpo .n n F

Demonstração: Observamos anteriormente que f f é uma correspondência bijetora

entre os conjuntos das formas bilineares sobre V e o conjunto de todas as matrizes n nsobre . E isso é uma transformação linear, pois F

, ,i j i j i jcf g cf g ,

Para todos e i j . Isto diz simplesmente que

cf g c f g .

Corolário – Se 1, , n é uma base ordenada de V e *1, , nL L é a base dual

de , então as formas bilineares *V 2n

,ij i jf L L , 1 i n , 1 j n

formam uma base do espaço ( , , )L V V F . Em particular, a dimensão de ( , , )L V V F é .2n

Demonstração: A base dual 1, , nL L é definida essencialmente pelo fato de que iL é a i-ésima coordenada de em relação à base ordenada (para todo em V ). Ora, as funções

ijf definidas por

,ij i jf L L

são formas bilineares do tipo considerado no exemplo 1. Se

1 1 n nx x e 1 1 n ny y ,então

,ij i jf x y .

Seja f uma forma arbitrária sobre V e seja A a matriz de f em relação à base ordenada .Então

Page 203: Numero 09 - Outubro de 2007

,, ij i j

i jf A x y

o que diz simplesmente que

,,ij ij

i jf A f .

Agora é evidente que as formas 2n ijf formam uma base de ( , , )L V V F .

Outra maneira de demonstrar o corolário: A matriz da forma bilinear ijf em relação à base ordenada é a matriz “unitária” ,

cujo único elemento não-nulo é um 1 na linha i e coluna

,i jE

j . Como estas matrizes constituem uma base do espaço das matrizes

,i jEn n , as formas ijf constituem uma base do

espaço das formas bilineares.

Definição 2.3 – Uma forma bilinear f sobre um espaço vetorial V é dita não-degenerada (ou não-singular) se sua matriz em relação a alguma (toda) base ordenada de V é uma matriz não-singular, ou seja, se .( )Posto f n

2.1 - Formas Bilineares Simétricas e Formas Quadráticas

Nesta seção descreveremos um tipo especial de forma bilinear, as chamadas formas bilineares simétricas.

Definição 2.4 - Seja f uma forma bilinear sobre o espaço vetorial V . Dizemos que f ésimétrica se ( , ) ( , )f f , para quaisquer vetores , em V .

Se V é de dimensão finita, a forma bilinear f é simétrica se, e somente se, sua matriz A em relação a alguma ou (toda) base ordenada é simétrica, isto é, tA A . Para ver isto, perguntamos quando é que a forma bilinear

, tf X Y X AYé simétrica. Isto acontece se, e somente se, t tX AY Y AX para todas matrizes-colunas X e Y .Como tX AY é uma 1 matriz, temos 1 t t tX AY Y A X . Assim, f é simétrica se, e somente se, t t tY A X Y AX para todas . Evidentemente, isto significa apenas que

. Em particular, deve-se notar que se existir uma base ordenada de V em relação à qual

,X YtA Af seja representada por uma matriz diagonal, então f é simétrica, pois qualquer matriz

diagonal é uma matriz simétrica.

Page 204: Numero 09 - Outubro de 2007

Se f é uma forma bilinear simétrica, a forma quadrática associada a f é a função deem definida por

qV F

( ) ( , )q f .

Se é um subcorpo do corpo dos números complexos, a forma bilinear simétrica F f é completamente determinada por sua forma quadrática associada, de acordo com a seguinte identidade, conhecida por identidade de polarização:

1 1( , ) ( ) ( )4 4

f q q .

Demonstração:

Temos que: ( )( , )( , ) ( , )( , ) ( , ) ( , ) ( , )( , ) 2 ( , ) ( , )

qff ff f f ff f f( ) 2 ( , ) ( ).q f q (1)

Temos também que:

( )( , )( , ) ( , )( , ) ( , ) ( , ) ( , )( , ) 2 ( , ) ( , )

qff ff f f ff f f( ) 2 ( , ) ( ).q f q (2)

Fazendo (1) – (2), obtemos:

( ) ( )( ) 2 ( , ) ( ) ( ) 2 ( , ) ( )

4 ( , )

q qq f q q f q

f

Page 205: Numero 09 - Outubro de 2007

E então, 1( , ) ( ( ) ( ))4

f q q

1 1( , ) ( ) ( )).4 4

f q q

))

(3)

Observe que, fazendo (1)+(2), obtemos a identidade do paralelogramo

( ) ( ) 2( ( ) (q q q q . (4)

Uma classe importante de formas bilineares simétricas consiste dos produtos internos sobre espaços vetoriais reais. Se V é um espaço vetorial real, um produto interno sobre V é um a forma bilinear simétrica f sobre V que satisfaz

( , ) 0f , se 0 . (5)

Se f é uma forma bilinear dada pelo produto escalar, então a forma quadrática associada é

2 21 2 1 2( , , , )n nq x x x x x x2 .

Em outras palavras, ( )q é o quadrado do comprimento de .Para a forma bilinear ( , ) t

Af X Y X AY , a forma quadrática associada é

,( ) t

A ij i ji j

q X X AX A x x .

Uma forma bilinear que satisfaz a equação (5) é dita positiva definida. Assim, um produto interno sobre um espaço vetorial real é uma forma bilinear simétrica positiva definida sobre aquele espaço. Note que, um produto interno é não degenerado.

Dois vetores , são ditos ortogonais em relação ao produto interno f se . A

forma quadrática

, 0f

,q f toma apenas valores não-negativos e q é usualmente considerado como o quadrado do comprimento de .

Observe que se f é uma forma bilinear simétrica sobre um espaço vetorial V , é conveniente dizer que e são ortogonais em relação à f se ,f 0 . Mas não é aconselhável

considerar ,f como sendo o quadrado do comprimento de . Por exemplo, se V é

Page 206: Numero 09 - Outubro de 2007

um espaço vetorial complexo, podemos ter , 1f i , ou num espaço vetorial real

., 2f

Teorema 2.2 – Seja V um espaço vetorial de dimensão finita sobre um corpo de característica zero, e seja f uma forma bilinear simétrica sobre V . Então, existe uma base ordenada de V em relação à qual f é representada por uma matriz diagonal.

Demonstração: O que precisamos encontrar é uma base ordenada

1 2, , , n

tal que ,i jf 0 j

*

para , ou seja i

11 0 * 0

0 0nn

f

fSe ou , o teorema é verdadeiro, pois a matriz 10f 1n 1 é uma matriz diagonal. Assim, podemos supor e . Se 0f 1n ,f 0 para todo em V , a forma quadrática é identicamente 0 e a identidade de polarização mostra que , pois q 0f

1 1( , ) ( ) ( )4 4

f q q .

Assim, existe um vetor em V tal que , 0f q .Seja W o subespaço unidimensional de V que é gerado por e seja W ( W ortogonal) o conjunto de vetores em V tais que ,f 0 . Afirmamos agora, que V W W .

Certamente os subespaços W e W são independentes. Um vetor típico em W é c , onde c é um escalar. Se c está, também, em W , então 2, ,f c c c f 0 .

Mas, , logo . Além disso, todo vetor em V é a soma de um vetor em W eum em W . De fato, seja

,f 0 0cum vetor arbitrário em V e coloquemos:

,,

ff

.

Então

,, ,

,f

f f ff

,

Page 207: Numero 09 - Outubro de 2007

E como f é simétrica, , (pois ,f 0 f é diagonal e ).Portanto, está no subespaço W . A expressão

,,

ff

nos mostra que V W .W

A restrição de f a W é uma forma bilinear simétrica sobre W . Como W tem dimensão (pois tem ), podemos supor, por indução, que W possua uma base 1n W dim 1

2, , n tal que

,i jf 0 , 2, 2i j i j

Colocando 1 , obtemos uma base 1, , n de V tal que ,i jf 0 j para i .

Obs: Em termos das coordenadas dos vetores 1 1 2 2 n nx x x e

1 1 2 2 n ny y y relativamente à base 1, , n do teorema 2.2 a forma

bilinear f se expressa como , i i if x y .Em particular, a forma quadrática associada a q f é dada por uma combinação linear de quadrados:

2 21 1 2 2 n nq x x 2x .

Os escalares 1 2, , , n são os autovalores da matriz da forma bilinear.

2.2 – Formas Quadráticas no plano

De acordo com o teorema 1, uma forma quadrática no plano pode ser representada por uma

matriz simétrica . Isto é feito da seguinte maneira: a matriz simétrica real

associa ao vetor

a cA

c ba c

Ac b

2( , )sv x y R , referido à base canônica ,

( e ), o polinômio que é um polinômio homogêneo do 2º grau em

1 2{ , }S e e

1 (1,0)e 2 (0,1)e 2 2ax bxy cy2

x e y chamado forma quadrática no plano.

Na forma matricial, este polinômio é representado por:

Page 208: Numero 09 - Outubro de 2007

ts s

a c xv Av x y

c b y,

sendo a matriz simétrica A a matriz da forma quadrática. Assim, a cada vetor sv corresponde um número real:

2 22p ax bxy cy .

2.2.1 – Redução da Forma Quadrática à Forma Canônica.

A forma quadrática no plano ts sv Av pode ser reduzida através de mudanças de coordenadas à

forma:

2 21 2' 'x y

onde 1 e 2 são os autovalores da matriz A , e 'x e as componentes do vetor v na base , isto é,

'y

1 2{ , }P u u ( ', ')pv x y , sendo e os autovetores associados a 1u 2u 1 e 2 .

Demonstração:Temos que a matriz é a matriz mudança de base de para , pois: P P S

1P

SI S P IP P

E, portanto:

s pv Pv

logo,

tts s pv Av Pv A Pvp

ou,

t t tS S P Pv Av v P AP v .

Como diagonaliza P A ortogonalmente

Page 209: Numero 09 - Outubro de 2007

1

2

00

tP AP D ;

conclui-se que,

t tS S P Pv Av v Dv ,

ou,

1

2

0 '' '

0 'a c x x

x y x yc b y y

2'

2'

ou ainda,

2 2 21 22 'ax bxy cy x y .

A forma 21 2'x y é denominada forma canônica da forma quadrática no plano, ou

também, forma quadrática diagonalizada. O que na verdade acabamos de fazer foi uma mudança de base ou uma mudança de referencial. Essa mudança de referencial corresponde a uma rotação de um ângulo do sistema xOyaté o sistema ' 'x Oy . A matriz responsável por essa rotação é a matriz ortogonal , cujas colunas são os autovetores e de

P

1u 2u A .

3 – Cônicas.

Chama-se cônica a todo conjunto de pontos M do plano cujas coordenadas x e , em relação à base canônica, satisfazem a equação do 2º grau:

y

2 22 0ax bxy cy dx ey f

onde não são todos nulos. , ,a b c

3.1- Equação reduzida de uma Cônica.

Dada a cônica C de equação

Page 210: Numero 09 - Outubro de 2007

2 22 0ax bxy cy dx ey f (6)

queremos, através de mudanças de coordenadas, reduzí-la a uma equação de uma forma mais simples, chamada equação reduzida da cônica. Para isto seguimos as seguintes etapas.

1ª etapa: Eliminação do termo em xy :

1º passo: escrever a equação na forma matricial

0a c x x

x y d e fc b y y

(7)

ou,0t

s s sv Av Nv f .

2º passo: calcular os autovalores 1 e 2 e os autovetores unitários e da matriz simétrica

1 11 12( , )u x x

2 21 2( ,u x x 2 ) A .

3º passo: substituir na equação (7) a forma quadrática:

pela forma canônica ts s

a c xv Av x y

c b y

1

2

0 '' '

0 'tP P

xv Dv x y

y, e

s

xv

y por 11 21

12 22

''P

x x xPv

x x y

tendo o cuidado para que de , a fim de que essa transformação seja uma rotação. t( ) 1PAssim, a equação (7) se transforma em:

1 11 21

2 12 22

0 ' '' ' 0

0 ' 'x xx x

x y d e fx xy y

ou,2 2

1 2' ' ' 'x y px qy f 0

'

(8)

que é a equação da cônica dada em (7), porém referida ao sistema 'x Oy , cujos eixos são determinados pela base .1 2{ , }P u uObserve que enquanto a equação (7) apresenta o termo misto xy , a equação (8) é desprovida dele.

Page 211: Numero 09 - Outubro de 2007

Portanto da equação (7) para a (8) ocorreu uma simplificação.

2ª etapa: Translação de eixos:

Conhecida a equação da cônica

2 21 2' ' ' 'x y px qy f 0

'

. (9)

Para se obter a equação reduzida efetua-se uma nova mudança de coordenadas, que consiste na translação do último referencial 'x Oy para o novo, o qual denominaremos 'xO y . A seguir é feita a análise das duas possibilidades:

(I) Supondo 1 e 2 diferentes de zero, podemos escrever:

2 21 2

1 2

' ' ' 'p qx x y y f 0

2 2 22 2

1 22 21 1 2 2 1 2

' ' ' '4 4 4

p p q q p qx x y y f2

04

2 2 2 2

1 21 2 1

' '2 2 4 4p q px y f

2

0q .

Fazendo:

2 2

1 24 4p qf F

e por meio das fórmulas de translação:

1

'2pX x e

2

'2qY y

vem,

2 21 2 0X Y F

2 21 2 .X Y F (10)

A equação (10) é a equação reduzida de uma cônica de centro, e como se vê, o 1º membro é a forma canônica da forma quadrática do plano.

(II) Se um dos autovalores for igual a zero, 1 0 , por exemplo, a equação (9) fica:

Page 212: Numero 09 - Outubro de 2007

22 ' ' 'y px qy f 0

ou seja,

22

2

' ' 'qy y px f 0

2 22

2 22 2 2

' ' '4 4

q q qy y px f 0

2 2

22 2

' '2 4q f qy p x

p p0 .

Fazendo, por meio de uma translação:

2

2

'4

f qX xp p

e2

'2qY y

vem,

22 0Y pX . (11)

A equação (11) é a equação reduzida de uma cônica sem centro.

Se 2 0 , a equação (9) fica: 2

1 ' ' 'x px qy f 0

21

1

' ' 'px x qy f 0

2 22

1 21 1 1

' ' '4 4

p p px x qy f 0

2 2

11 1

' '2 4p f px q y

q q0 .

Fazendo por meio de uma translação:

2

1

'4

f pY yp q

e1

'2pX x

vem,

Page 213: Numero 09 - Outubro de 2007

21 0X qY .

3.2- Classificação das Cônicas.

I) A equação reduzida de uma cônica de centro é:

2 21 2X Y F .

Se 1 e 2 forem de mesmo sinal, a cônica será do gênero elipse. Se 1 e 2 forem de sinais contrários, a cônica será do gênero hipérbole.

II) A equação de uma cônica sem centro é:

22 0Y pX ou 2

1 0X qY .

Uma cônica representada por qualquer uma dessas equações é do gênero parábola.É usada a mesma classificação para as formas quadráticas.

Exemplo 3.1:a) Para a cônica de equação 2 22 2 2 7 2 5 2 10x y xy x y 0

1

, a matriz A é dada

por e seus autovalores são 2 11 2

A 1 23 e . Portanto, pela classificação de

cônicas, como os sinais dos autovalores são iguais, a cônica em questão é uma elipse.

Page 214: Numero 09 - Outubro de 2007

b) Para a cônica de equação , a matriz A é dada por e

como um de seus autovalores é nulo, concluímos que esta cônica é uma parábola.

2 22 8 4x xy y x 01 11 1

A

c) A equação 2 24 3 24 156 0x y xy , representa uma hipérbole, pois a matriz

apresenta autovalores de sinais opostos (4 12

12 3A 1 212 13e ).

3. Referências bibliográficas

[1] HOOFMAN, K. & KUNZE, R. Álgebra Linear. São Paulo: Polígono, Editora da Universidade de São Paulo,1971.

Page 215: Numero 09 - Outubro de 2007

[2] GREUB, W. Linear Algebra. 4ª ed. Nova York: Springer-Verlag, 1974.

[3] STEINBRUCH, A. & WINTERLE, P. Álgebra Linear. 2ª ed. São Paulo: Makron Books, 1987.

[4] LIMA, E. L. Álgebra Linear. 2ª ed. Instituto de Matemática Pura e Aplicada, 1996 (Coleção Matemática Universitária).

Page 216: Numero 09 - Outubro de 2007
Page 217: Numero 09 - Outubro de 2007

Soma de Quadrados

Stela Zumerle Soares1 Antônio Carlos Nogueira2

([email protected]) ([email protected])

Faculdade de Matemática, UFU, MG

1 Bolsista do PET -Matemática da Universidade Federal de Uberlândia 2 Docente da Faculdade de Matemática da Universidade Federal de Uberlândia

1. Resultados Preliminares

Historicamente, um problema que tem recebido uma atenção considerável é a representação de números como soma de quadrados. Por exemplo:

2

2 2

2 2 2

2

2 2

2 2 2

2 2 2 2

1 12 1 13 1 1 14 25 2 16 2 1 17 2 1 1 1

Nos preocuparemos, neste trabalho, em descrever os inteiros positivos que podem ser representados como soma de dois quadrados. Observamos inicialmente que, a solução do problema proposto depende do conhecimento de alguns resultados pertinentes às congruências quadráticas, ou seja, congruências do tipo

.2 0(mod )ax bx c nComeçamos então, considerando a congruência , onde é um primo ímpar e .

2 0(mod )ax bx c p p0(mod )a p

Definição 1.1 – Seja um primo ímpar e p ,mdc a p 1. Se a congruência quadrática 2 (mod )x a p tem uma solução, então a é dito ser um resíduo quadrático de p .

Exemplo 1.1 – Considere o primo 13p . Para encontrar quais dos inteiros 1, são resíduos quadráticos de 13, precisamos saber quais das congruências

2, 3, , 12

2 mod13x a

são solúveis com percorrendo a série a 1, 2, , 12 . Módulo 13, as equações dos inteiros são 1, 2, 3, , 12

Page 218: Numero 09 - Outubro de 2007

2 2

2 2

2 2

2 2

2 2

2 2

1 12 12 11 43 10 94 9 35 8 126 7 10

.

Consequentemente, os resíduos quadráticos de 13 são 1, , e os não resíduos quadráticos são .

3, 4, 9, 10, 122, 5, 6, 7, 8, 11

Critério de Euler – Seja p um primo ímpar e ,mdc a p 1. Então é um resíduo

quadrático se, e somente se,

a1

2 1 modp

a p .

Demonstração:Seja um não resíduo quadrático de a p e seja um dos inteiros 1, . Pela teoria das congruências lineares, existe uma solução de

c 2, , 1p'c modcx a p , com 'c também no

conjunto {1, 2, , 1}p . Note que 'c c ; caso contrário teremos que c a , o que contradiz o que assumimos na definição de resíduo quadrático. Assim, os inteiros entre 1 e

podem ser divididos em

2 d pmo

1p 12

p pares, , onde , 'c c ' modcc a p . Isto leva às 12

p

congruências1 1

2 2

1 12 2

' mod

' mod

' modp p

c c a p

c c a p

c c a p

.

Multiplicando-os e observando o produto 1 1 2 2 1 1

2 2

' ' ' .p pc c c c c c

É simplesmente um rearranjo de 1 2 , nós obtemos 3 ( 1)p1

21 ! modp

p a p .

Neste ponto o Teorema de Wilson entra em cena; para 1 ! 1 modp p , vem que 1

2 1 modp

a pque é o Critério de Euler quando a é um não resíduo quadrático de . p

Page 219: Numero 09 - Outubro de 2007

Posteriormente, examinamos o caso em que a é um resíduo quadrático de . Estabelecendo que a congruência

p2 modx a p admite duas soluções 1x x e 1x p x , para algum 1x

satisfazendo 11 1x p . Se 1x e 1p x são tirados do conjunto {1 , então sobram inteiros que podem ser agrupados em pares (onde ) tal que

. Para estas

, 2, , 1}p3p , 'c c 'c c

' modcc a p3

2p

congruências, adicionamos a congruência 2

1 1 1 modx p x x a p .Tomando o produto de todas as congruências envolvidas, chegamos à relação

121 ! mod

p

p a p .

Do Teorema de Wilson temos que 1 ! 1 modp p e daí segue que1

2 1 modp

a p .

Juntando tudo, mostramos que 1

2 1 modp

a p ou 1

2 1 modp

a p , caso seja um resíduo quadrático ou não resíduo quadrático de

ap .

Corolário - Seja um primo ímpar e p ,mdc a p 1. Então a é um resíduo quadrático (resp.

não resíduo quadrático) de se p1

2 1 modp

a p (resp. 1

2 1 modp

a p ).

Exemplo 1.2 – No caso onde , nós encontramos que 13p13 1

622 2 64 12 1 mod13 .Assim, em virtude do último corolário, o inteiro 2 é um não resíduo quadrático de 13. Por outro lado,

13 16 2 223 3 27 1 1 mod13 ,

e isto indica que 3 é um resíduo quadrático de 13 e assim a congruência 2 3 mod13x é

solúvel, de fato, as duas soluções não congruentes são 4 2 mod13x e .

Definição 1.2 – Seja p um primo ímpar e seja ,mdc a p 1. O símbolo de Legendre /a pé definido por

1/

1se a e um residuo quadratico de p

a pse a e um nao residuo quadratico de p

.

Page 220: Numero 09 - Outubro de 2007

Exemplo 1.3 – Tomemos o primo 13p , em particular. Tomando o símbolo de Legendre, o resultado do exemplo 1.1 pode ser expresso como

1/13 3/13 4/13 9/13 10/13 12/13 1

2/13 5/13 6/13 7/13 8/13 11/13 1e .

Teorema 1.1 – Seja p um primo ímpar e sejam e inteiros relativamente primos a a b p .Então o símbolo de Legendre tem as seguintes propriedades:

a) Se , então moda b p / /a p b p

b) 2 / 1a p

c)1

2/ mp

a p a dod

d) / / /a p a p b p

e) e 1/ 1p1

21/ 1p

p .

Observe que a propriedade c é simplesmente o corolário reformulado nos termos dos Símbolos de Legendre.

Corolário – Se p é um primo ímpar, então: 1 1 mo

1/1 3 mo

se pp

se p

d 4

d4.

Este corolário pode ser considerado como uma afirmação de que a congruência quadrática 2 1 modx p tem uma solução para um primo ímpar se, e somente se, é da forma

.p p

4 1k

Exemplo 1.4 - Vamos verificar se a congruência 2 46 mod17x é solúvel. Isto acontece

pela avaliação do símbolo de Legendre 46/17 . Primeiramente, recorremos às propriedades (d) e (e) do teorema 1.1 para escrever

46/17 1/17 16/17 46/17 .

Como 46 12 mod17 , segue do item (a) do teorema 1.1 que

46/17 12/17 .Agora da propriedade (f) segue que

212/17 3 2 /17 3/17 .Mas,

Page 221: Numero 09 - Outubro de 2007

17 12 2823/17 3 3 81 4 1 mod17

onde usamos a propriedade (c) do teorema 1.1; daí, 3/17 1.

Visto que 46/17 1, a congruência quadrática 2 46 mod17x não admite solução.

2. Inteiros que são escritos como soma de dois quadrados

Consideremos o seguinte lema.

Lema 2.1 - Se m e são cada um uma soma de dois quadrados, então seu produto mntambém o é.

n

Demonstração:Se e para inteiros então2 2m a b 2n c d2

2)

2

, , ,a b c d

2 2 2 2 2( )( ) ( ) (mn a b c d ac bd ad bc .

É claro que nem todo primo pode ser escrito como uma soma de dois quadrados, por exemplo, não tem solução para inteiros e b . No caso mais geral, podemos provar o teorema 1.1.

23 a b a

Teorema 2.1 - Nenhum primo da forma p 4k 3 é uma soma de dois quadrados.

Demonstração:Dado qualquer inteiro , temos que a 0, 1, 2, 3(mod4)a ou ; consequentemente,

. Daí, segue que, para inteiros arbitrários e ,2 0 1 mod4a ou

2

a b

2 2 0, 1, 2(mod 4)a b ou

Como , a equação 3(mod4)p 2p a b é impossível.

De outro modo, todo primo que é congruente a 1 módulo 4 é expresso como a soma de dois quadrados. A demonstração deste fato, utiliza um teorema de congruência devido ao matemático norueguês Axel Thue. Este, por sua vez, fez uso do Princípio da Casa dos Pombos de Dirichlet.

Princípio da Casa dos Pombos – Se n objetos são dispostos em caixas (ou casa de pombos), e se , então alguma caixa conterá ao menos dois objetos.

mn m

Exprimindo em termos mais matemáticos, este simples princípio afirma que se um conjunto com elementos está na união de de seus subconjuntos, e se , então algum de seus subconjuntos têm mais que um elemento.

n m n m

Page 222: Numero 09 - Outubro de 2007

Lema de Thue - Seja um número primo e p ( , ) 1mdc a p . Então a congruência

(mod )ax y p

admite uma solução ,o ox y , onde

0 ox p e 0 oy p

Demonstração:Seja 1k p , e considere o conjunto de inteiros

| 0 1,0 1S ax y x k y k

Como ax y tem valores possíveis, o Princípio da Casa dos Pombos garante que ao menos dois membros de são congruentes módulo ; chamá-lo-emos de

2k p

1S p 1ax y e , onde 2ax y2 21x x ou 1 2y y . Então podemos escrever

1 2 1 2 moda x x y y p

Fazendo 0 1 2x x x e 0 1 2y y y , segue que 0x e 0y fornece uma solução para a congruência . Se modax y p 0x ou 0y é igual a zero, então o fato de que ( , ) 1mdc a ppode ser usado para mostrar que o outro também pode ser zero, contrariando a hipótese. Daí, 0 1ox k p e 0 1oy k p .

Agora deduziremos um teorema devido a Fermat que diz que todo primo da forma 4 1kpode ser expresso como a soma dos quadrados de dois inteiros. (Em termos precedentes, Albert Girard reconheceu este fato vários anos antes e o resultado é referido somente como o teorema de Girard). Fermat citou seu teorema numa carta para Mersenne, datada de 25 de dezembro de 1640, declarando que ele possuía uma demonstração irrefutável. Contudo, a primeira demonstração publicada foi dada por Euler em 1754, que nos acréscimos sucessivos mostrou que a representação é única.

Teorema (Fermat) - Um primo ímpar é expresso como uma soma de dois quadrados se, e só se, .

p1 mod4p

Demonstração:

Page 223: Numero 09 - Outubro de 2007

Suponha que possa ser escrito como a soma de dois quadrados, digamos p 2 2p a b .Como é um primo, temos que não divide a e não divide (Se divide a , então divide , e assim, divide b , levando à contradição de que

p p p b p p2b p 2p divide ). Assim, pela

teoria de congruências lineares, existe um inteiro tal que p

c 1 modbc p . Módulo , a relação

p

2 2 2ac bc pc

torna-se

2 1 modac p ,

e daí, fazendo um resíduo quadrático de 1 p .

Para a recíproca, seja . Como 1 mod4p 1 é um resíduo quadrático de , podemos

encontrar um inteiro a satisfazendo

p2 p1 moda ; de fato, 1 / 2 !a p é um tal

inteiro. Agora , assim, a congruência admite uma solução ( , )p 1mdc a 2

,1 modac p

o ox y devido ao lema de Thue. Segue que,

22 2 2 20 0 0 0 modx a x ax y p

ou

2 20 0x y kp

para algum inteiro . Visto que 1k 0 ox p e 0 oy p , obtemos 2 20 00 2x y p , o

que implica que . Consequentemente, 1k 2 20 0x y p , e terminamos.

Calculando e como uma soma, temos o corolário seguinte. 2a 2a

Corolário – Todo primo da forma p 4k 1 pode ser representado de forma única (exceto da ordem das parcelas) como uma soma de dois quadrados.

Demonstração:

Para estabelecer a afirmação única, suponhamos que

2 2 2p a b c d

Page 224: Numero 09 - Outubro de 2007

onde são todos inteiros positivos. Então , , ,a b c d

2 2 2 2 2 2 0 moda d b c p d b p

donde modad bc p ou modad bc p . Como são todos menores que , , ,a b c d p ,estas relações implicam que

0ad bc ou ad bc p .

Se a segunda igualdade é assegurada, então temos que ac bd ; pois

2 22 2 2 2 2 2 2p a b c d ad bc ac bd p ac bd

e então . Segue que 0ad bc

ad bc ou ac bd .

Suponha, por exemplo, que . Então , com ad bc |a bc ,mdc a b 1, o que força que ;

seja c . A condição

|a c

ka ad bc b ka então reduz a d bk . Mas

2 2 2 2 2 2p c d k a b

implica que . Neste caso, adquirimos a1k c e b d . Por um argumento similar, a condição leva a e bac bd a d c . O que é importante é que, num outro evento, suas duas representações do primo tornam-se idênticas.p

Exemplo 2.1 - Para o passo seguinte utilizaremos o primo 13p . Uma escolha para o inteiro é 6 . Uma solução para a congruência a ! 720 720 mod13x y , ou, melhor

dizendo,5 é obtido considerando o conjunto mod13x y

5 | 0 ,S x y x y 4

Os elementos de S são justamente os inteiros

0 5 10 11 4 9 12 3 8 13 2 7 12

543

que, módulo 13, tornam-se

Page 225: Numero 09 - Outubro de 2007

0 5 1012 4 9 111 3 8 010 2 7 12

2

.

Entre as várias possibilidades, temos

5 1 3 2 5 3 0 mod13

ou,

5 1 3 3 mod13 .

Assim, nós podemos tomar e 0 2x 0 3y para obter

2 2 20 013 2 3x y 2 .

Observe que alguns autores alegam que um primo 1 mod4p pode ser escrito como uma soma de dois quadrados de oito maneiras. Para 13p , temos

2 2 22 2 2 2

2 2 22 2 2 2

13 2 3 2 3 2 3 2 3

3 2 3 2 3 2 3 2 .

2

2

Como todas as oito representações podem ser obtidas de algumas delas pela troca dos sinais de 2 e 3 ou pela troca das parcelas, existe “essencialmente” somente uma maneira de fazer isto. Assim, do nosso ponto de vista, 13 é representado de forma única como a soma de dois quadrados.

Mostramos que todo primo tal que p 1 mod4p é expresso como a soma de dois quadrados. Mas outros inteiros também possuem esta propriedade, por exemplo,

2 210 1 3

O próximo passo é caracterizar explicitamente quais inteiros positivos que podem ser representados como a soma de dois quadrados.

Teorema 2.2 – Seja o número positivo n que pode ser escrito como , onde é livre de quadrados. Então pode ser representado como a soma de dois quadrados se, e só se, não contém nenhum fator primo da forma

2n N m mn

m 4 3k .

Page 226: Numero 09 - Outubro de 2007

Demonstração:

Inicialmente suponhamos que não tem nenhum fator primo da forma 4 . Se m 3k 1m ,então e completamos. No caso em que , seja 2 20n N 1m 1 2 rm p p p a fatoração de como um produto de primos distintos. Cada um destes primos m ip , sendo iguais a 2 ou à forma 4 , podem ser escritos como a soma de dois quadrados. Agora, a identidade 1k

2 22 2 2 2a b c d ac bd ad bc

mostra que o produto de dois inteiros que são representados como uma soma de dois quadrados, é também representável como soma de quadrados. Assim, existem inteiros x e y

2 2tal que . Concluímos que m x y

2 22 2 2 2n N m N x y Nx Ny

uma soma de dois quadrados.

Agora, para a volta. Assumimos que n pode ser representado como uma soma de dois quadrados

2 2 2n a b N m

e seja p um primo ímpar divisor de m (sem perda de generalidade, podemos assumir que >1). Se , então a r , onde m ( , )d mdc a b ,d b sd 1,mdc r s . Adquirimos que

2 2 2 2d r s N m

e deste modo, sendo livre de quadrados, . Mas então m 2 |d N 2

22 2

2

Nr s m td

p

para algum inteiro t , que leva que

2 2 0 modr s p , pois é divisor de .p m

Agora a condição , implica que um dos , digamos é relativamente primo a

,mdc r s 1 r ou s rp . Digamos que, satisfaça a congruência r

Page 227: Numero 09 - Outubro de 2007

' 1 modrr p .

Quando a equação é multiplicada por , obtemos 2 2 0 modr s p 2'r

2' 1 0 modsr p

ou, para colocar diferentemente, . Como 1/ 1p 1 é um resíduo quadrático de , temos

que . Assim, pelo nosso desenvolvimento temos que não existe nenhum primo da forma que divide .

p

p 4 3k m1 mod4p

Corolário – Um inteiro positivo n é representado como uma soma de dois quadrados se, e só se, cada um de seus fatores primos da forma 4 3k apresentam-se com uma potência par.

Exemplo 2.2 – O inteiro 459 não pode ser escrito como a soma de dois quadrados, pois , com o primo 3 tendo expoente ímpar. De outro modo, 153 admite

representação 3 2459 3 17 3 17

2 2 2 2 2153 3 4 1 12 3 .

Um pouco mais complicado é o exemplo 25 7 13 17n . Neste caso, temos que

2 2 2 2 2 2 2 25 7 13 17 7 2 1 3 2 4 1n .

Duas aplicações da identidade aparecidas no teorema 2.2 dão que

2 22 2 2 2 23 2 4 1 12 2 3 8 14 52

e

2 22 2 2 2 22 1 14 5 28 5 10 14 33 42 .

Quando isto é combinado, concluímos que

2 2 2 27 33 4 231 28n 2 .

Page 228: Numero 09 - Outubro de 2007

3. Diferença de Quadrados

Existem certos inteiros positivos (obviamente, não primos da forma 4 ) que podem ser representados de mais de uma maneira com a soma de dois quadrados. O menor deles é

1

2

k

2 2 225 4 3 5 0 .

Se , então a relação mod2a b

2 2

2 2a b a bab

permite-nos encontrar uma variedade de tais exemplos. Tome n=153 como uma ilustração; daí

2 22 217 9 17 9153 17 9 13 4

2 2

e

2 22 251 3 51 3153 51 3 27 24

2 2

e, assim

2 2 213 4 27 242 .

Isto produz duas distintas representações

2 2 2 227 4 24 13 745.

Uma questão a ser feita é saber quais inteiros positivos admitem uma representação como uma diferença de dois quadrados. A resposta está disposta a seguir.

Teorema 3.1 – Um inteiro positivo pode ser representado como a diferença de dois quadrados se, e só se, não é da forma 4 2

nn k .

Demonstração:

Como 0 1 mod4a ou para todo inteiro a , segue que

2 2 0, 1 3 mod4a b ou .

Page 229: Numero 09 - Outubro de 2007

Assim, se 2 mod4n , nós não temos que 2n a b2 para todo e .a b

Voltando, suponha que o inteiro não é da forma n 4k 2 ; ou seja, . Se 0, 1 3 mod4n ou

1 3 mod4n ou , então e são ambos inteiros pares; daí, pode ser escrito como 1n 1n n

2 21 12 2

n nn

uma diferença de quadrados. Se 0 mod4n , então temos que

2 2

1 14 4n nn .

Corolário – Um primo ímpar é uma diferença de dois quadrados sucessivos.

Exemplos deste último corolário são dados por

2 2 2 2 211 6 5 17 9 8 29 15 142 .

Um outro ponto a ser mencionado é que a representação de um dado primo como a diferença de dois quadrados é única. Para ver isto, suponha que

p

2 2p a b a b a b

onde . Como 1 e são os únicos fatores de , necessariamente temos que 0a b p p

1a b e a b p

De onde podemos concluir que

1 12 2

p pa e b .

Assim, todo primo p pode ser escrito como a diferença de dois quadrados de dois inteiros em precisamente um modo, a saber

2 21 12 2

p pp .

Page 230: Numero 09 - Outubro de 2007

Uma situação diferente ocorre quando passamos de primos a inteiros arbitrários. Suponha que é um inteiro positivo que nem é primo e nem é da forma n 4 2k .

Começando com um divisor de , tomando d n ' ndd

(assumimos que ' ). Agora se

e são ambos pares, ou ambos ímpares, então

d d d

'd' '

2 2d d d d

e são inteiros.

Além disso, podemos escrever

2 2' ''2 2

d d d dn dd

Para uma ilustração, considere o inteiro 24. Daí,

2 22 212 2 12 224 12 2 7 5

2 2

e

2 22 26 4 6 424 6 4 5 1

2 2

dando-nos duas representações para 24 como uma diferença de quadrados.

4. Referências bibliográficas

[1] BURTON, D. M. Elementary Number Theory. 5ª ed. Mc-Graw-Hill Higher Education, 2002.

Page 231: Numero 09 - Outubro de 2007

Complexidade Algebrica em Demonstracoesde Geometria Euclidiana Plana: o Teorema

de Napoleao e Propriedades

Gabriela Aparecida dos Reis∗ Luciana Yoshie Tsuchiya†

Edson Agustini‡

Faculdade de Matematica - Famat

Universidade Federal de Uberlandia - Ufu - MG

Setembro de 2007

1 Introducao

Este trabalho de iniciacao cientıfica esta baseado na dissertacao de mestrado “Complexi-dade em Geometria Plana Euclidiana”, de S. M. R. Lopes, ref. [3] .

E muito comum no estudo da Geometria Euclidiana Plana encontrarmos varias de-monstracoes de uma mesma proposicao. Ferramentas matematicas simples como a Trigonome-tria, a Geometria Analıtica e os Numeros Complexos sao frequentemente utilizadas emdemonstracoes alternativas de resultados tradicionalmente provados por argumentos deGeometria Sintetica. Ferramentas matematicas mais sofisticadas envolvendo Transforma-coes Lineares e Grupos de Isometrias tambem podem ser utilizadas em varias demon-stracoes.

Em algumas das demonstracoes supracitadas e possıvel utilizar determinados polinomioscujas raızes representam casos particulares na qual a proposicao que se deseja provar setorna verdadeira e, a partir desses casos particulares, e possıvel demonstrar o caso geral.Considerando p como sendo o polinomio de menor grau que podemos deduzir em umadeterminada demonstracao de uma proposicao, podemos definir a complexidade algebricada proposicao como sendo o grau de p.

Nosso objetivo e estabelecer o conceito acima e exemplifica-lo por meio de uma demon-stracao do famoso:

Teorema de Napoleao: “Dado um triangulo ABC qualquer, sejam os triangulos equila-teros apoiados externamente (ou internamente) sobre cada um de seus lados. Entao, osbaricentros X, Y e Z destes triangulos equilateros formam um triangulo XYZ tambemequilatero, chamado Triangulo de Napoleao Externo (ou Interno).”

e de tres propriedades dos Triangulos de Napoleao:

[email protected] - Pet - Programa de Educacao Tutorial - Famat - Ufu.†[email protected] - Pet - Programa de Educacao Tutorial - Famat - Ufu.‡[email protected] Professor orientador.

Page 232: Numero 09 - Outubro de 2007

Propriedade 1: “Seja ABC um triangulo qualquer. Os Triangulos de Napoleao Externoe Interno de ABC tem o mesmo baricentro e este coincide com o baricentro do trianguloABC.”

que, assim como o proprio Teorema de Napoleao, possui complexidade algebrica 1.

Propriedade 2: “A diferenca entre as areas dos Triangulos de Napoleao Externo eInterno de ABC e igual a area do triangulo ABC.”

que possui complexidade algebrica 2.

Propriedade 3: “Sejam ABC um triangulo e ABP, BCQ e CAR os triangulos equilaterosapoiados externamente nos lados de ABC. Entao, os segmentos PC, QA e RB possuem omesmo comprimento e se encontram em um unico ponto. Alem disso, as retas que contemesses segmentos formam angulos congruentes entre si cujas medidas sao de 60◦.”

que possui complexidade algebrica 3.

2 O Teorema de Napoleao

Dado um triangulo ABC qualquer, sejam os triangulos equilateros apoiados externamente(ou internamente) sobre cada um de seus lados. Unindo-se os baricentros X, Y e Z dostriangulos equilateros obtemos o chamado Triangulo Externo (ou Interno) de Napoleao.

Teorema de Napoleao. O Triangulo Externo (ou Interno) de Napoleao XYZ de qualquertriangulo ABC e equilatero.

R

A B

C

Q

P

X

YZ

R'

B

C

A

Q'

P'

X'

Y'

Z'

Nas proximas subsecoes apresentamos cinco demonstracoes classicas do Teorema de Napoleao,sendo que a ultima ilustra o conceito de complexidade algebrica 1 desse teorema.

Page 233: Numero 09 - Outubro de 2007

2.1 Demonstracao do Teorema de Napoleao Usando Trigonome-tria

Considere a figura abaixo sendo t, u e v os comprimentos dos segmentos AX, AZ e CY,

respectivamente e a, b, c como sendo os comprimentos dos segmentos BC, AC e AB,

respectivamente.

R

A B

C

Q

P

X

YZ

u

u

t

v

y

x

z

Temos XAB = ZCA = 30o.

De fato, no triangulo AXB temos que AX ≡ XB, pois X e o baricentro do trianguloAPB. Logo, o triangulo AXB e isosceles e, consequentemente, os angulos de sua basesao congruentes. Alem disso, AX e a bissetriz de PAB e como PAB = 60o (triangulo

equilatero), segue que XAB = 30o. O mesmo se faz para CAZ = 30o.

Daı, podemos aplicar a lei dos Cossenos no triangulo AXZ:

y2 = u2 + t2 − 2ut cos(A + 60o). (1)

De forma analoga, temos:

x2 = u2 + v2 − 2uv cos(C + 60o) (2)

e

z2 = v2 + t2 − 2vt cos(B + 60o) (3)

A distancia do baricentro a um dos vertices de um triangulo vale2

3do comprimento

da respectiva mediana e, como em um triangulo equilatero o baricentro coincide com oortocentro, segue que:

t =2

3

√3

2c =

c√3,

u =2

3

√3

2b =

b√3

e

v =2

3

√3

2a =

a√3.

Page 234: Numero 09 - Outubro de 2007

Substituindo os valores de t, u e v em (1) , (2) e (3), temos:

y2 =

(b√3

)2

+

(c√3

)2

− 2

(b√3

)(c√3

)cos(A + 60o)

y2 =b2

3+

c2

3−

2bc

3cos(A + 60o)

3y2 = b2 + c2 − 2bc cos(A + 60o) (4)

3x2 = a2 + b2 − 2ab cos(C + 60o) (5)

3z2 = a2 + c2 − 2ac cos(B + 60o) (6)

Aplicando a Lei dos Cossenos aos triangulos ABR e BCR, podemos expressar o quadradodo comprimento do lado BR de duas maneiras diferentes:{ (

BR)2

= b2 + c2 − 2bc cos(A + 60o)(BR

)2= a2 + b2 − 2ab cos(C + 60o)

=⇒b2 + c2 − 2bc cos(A + 60o) = a2 + b2 − 2ab cos(C + 60o).

Por (4) e (5) , temos que o lado esquerdo e direto da expressao fica da seguinte forma:

3y2 = 3x2 =⇒ y2 = x2.

Como estamos nos referindo a medidas de segmentos, segue que y = x, ou seja, XZ = YZ.

Analogamente, considerando os triangulos ACQ e ABQ, concluimos que XY = ZY.

Como XY = YZ = XZ, concluımos que o triangulo XYZ e equilatero. �

2.2 Demonstracao do Teorema de Napoleao Usando GeometriaSintetica

Considere a figura abaixo onde temos os triangulos ABP, BQC e ACR inscritos cada umem um cırculo e seja o ponto O o ponto de inteseccao dos cırculos ABP e BCQ.

X

A B

C

ZY

R

Q

P

O

Page 235: Numero 09 - Outubro de 2007

O quadrilatero AOPB esta inscrito no cırculo ABP. Logo, a soma dos graus de seus angulosopostos e 180◦. Entao, temos:

AOB + P = 180◦ ⇒ AOB = 180◦ − P.

Da mesma forma no quadrilateroCOBQ temos:

BOC + Q = 180◦ ⇒ BOC = 180◦ − Q.

Temos tambem:

AOB + BOC + AOC = 360◦ ⇒ AOC = 360◦ − BOC − AOB.

Logo:

AOC = 360◦ −(180◦ − P

)−

(180◦ − Q

)= P + Q.

Mas P + Q + R = 180◦, ou seja, P + Q = 180◦ − R.

Assim, obtemos:

AOC = 180◦ − R ⇒ AOC + R = 180◦.

Donde conclui-se queAOC e R sao suplemetares.

Assim o quadrilatero AOCR esta inscrito em um cırculo que e concorrente com os cırculosABP e BCQ no ponto O.

Alem disso, de resultados da Geometria Euclidiana Plana sabemos que a reta que passapelos centros de dois cırculos que se intersectam e perpendicular a corda comum dosdois cırculos. Logo, XZ e perpendicular a OA no ponto que chamaremos de M e XY eperpendicular a OB no ponto que chamaremos de N.

Entao, temos o quadrilatero XMON, onde:

X + XMN + AOB + ONX = 360◦ ⇒X + 90◦ + AOB + 90◦ = 360◦ ⇒

AOB + X = 180◦.

Mas ja haviamos visto que AOB + P = 180◦. Logo:

X = P.

Analogamente, mostramos que Q = Y e R = Z e, como P = Q = R = 60◦, entao otriangulo XYZ e equilatero. �

2.3 Demonstracao do Teorema de Napoleao usando Arcos Ca-pazes

Na figura abaixo, sejam�

AB,�

BC e�

CA os arcos relativos aos cırculos de centros X, Y e Z,

que circunscrevem, respectivamente, os triangulos equilateros ABP, BCQ e CAR.

Page 236: Numero 09 - Outubro de 2007

C

A B

YZ

X

R

Q

P

Seja D ∈�

AB e E ∈�

AC de tal forma que o segmento DE passe pelo ponto A. Entao, DB

e EC fazem 60o com DE, em D e em E, respectivamente, pois os arcos�

AB e�

AC sao arcoscapazes dos segmentos DB e EC de um angulo de 60o.

Pelo Axioma das Paralelas de Euclides, as retas que contem os segmentos DB e EC seencontram num ponto F, formando um angulo de 60o, e alem do mais, F deve pertencer

obrigatoriamente ao arco�

BC, pois este e o arco capaz do segmento BC sobre um angulode 60o.

E

C

D

A

F

B

YZ

XM

N

T

Logo, existe um triangulo DEF que circunscreve o triangulo inicial ABC, e que contemum vertice em cada arco.Construindo as perpendiculares a DF pelos centros X e Y, obtemos os ponto M e N quesao, respectivamente, o ponto medio das cordas DB e BF, pois toda reta que passa pelocentro e corta perpendicularmente uma corda, o faz em seu ponto medio.Chamando de T o quarto vertice do retangulo XMNT, observamos que DF = 2MN = 2XT.

Observemos que XT e um cateto do triangulo retangulo XYT. Logo, nao e maior que ahipotenusa XY.Como XT e paralelo a DF, observamos que se D esta muito proximo de A, o trianguloretangulo XYT e externo ao Triangulo de Napoleao XYZ, mas se D esta muito proximode B, o triangulo retangulo XYT e interno ao triangulo de Napoleao XYZ.

Daı, concluımos que existe D ∈ AB tal que o lado XT do triangulo XYT coincide com ahipotenusa XY, atigindo seu comprimento maximo: XT = XY.

E ainda, como DF = 2XT, o comprimento maximo que DF pode assumir e 2XY. Analoga-mente, o comprimento maximo que FE e ED podem assumir e 2YZ e 2XZ, respectivamente.

Page 237: Numero 09 - Outubro de 2007

Como, o triangulo DEF e sempre equilatero, seus lados assumem seu comprimento maximoao mesmo tempo, ou seja, 2XT = 2YZ = 2XZ o que implica que XT = YZ = XZ.

Portanto, o triangulo XYZ e equilatero. �

2.4 Demonstracao do Teorema de Napoleao por Transformacoesno Plano

Podemos construir externamente triangulos equilateros sobre os lados do triangulo ABC

por meio de rotacoes. Obtemos os vertices P, Q e R rotacionando os vertices A,B e C deum angulo de 60o no sentido anti-horario em torno dos vertices B,C e A respectivamente.

R

A B

C

Q

P

X

YZ

60°

60°

60°

Considerando essa configuracao construida por rotacoes, onde X, Y e Z sao os baricentrosdos triangulos equilateros obtidos, indicaremos por RP (M) a rotacao de 120◦ no sentidohorario de um ponto M em torno de P, ou seja:

RP : R2 −→ R

2

M �−→ RP (M) = P + R (M − P)

onde R : R2 → R

2 e uma rotacao de 120◦ no sentido horario em torno da origem.

Geometricamente temos a figura abaixo.

P M

O

M-P

O+(M-P)

R(M-P)

P+R(M-P)=R (M)P

x

y

P=OP

M=OM

M-P=OM-OP=PM

120°

Page 238: Numero 09 - Outubro de 2007

Note que RP nao e um operador linear, pois nao fixa a origem. Sendo R um operadorlinear, a aplicacao acima permite-nos trabalhar com as propriedades de transformacoeslineares.Considerando I a representacao da aplicacao identidade em R

2, temos:

RX (P) = X + R (P − X)

= I (X) + R (P + (−I) X)

= I (X) + R (P) + R ((−I)X)

= I (X) + R (P) − R (X)

= I (X) − R (X) + R (P)

= (I − R) (X) + R (P)

Analogamente, obtemos:

RY (P) = (I − R) (Y) + R (P)

e

Rz (P) = (I − R) (Z) + R (P) .

Observe que a primeira rotacao leva A em B, a segunda leva B em C e a terceira leva C

em A, ou seja:

RX (A) = B

RY (B) = C

RZ (C) = A

120°

120°

120°

X

YZ

A R (C)= ZB R (A)=

X

C R (B)= y

Consequentemente, o ponto A e um ponto fixo na composta dessas tres rotacoes:

RZ (RY (RX (A))) = A.

Page 239: Numero 09 - Outubro de 2007

Temos, tambem:

RZ (Ry (RX (P))) = (I − R) (Z) + R (RY (RX (P)))

= (I − R) (Z) + R ((I − R) (Y) + R (RX (P)))

= (I − R) (Z) + R ((I − R) (Y) + R ((I − R) (X) + R (P)))

= (I − R) (Z) + R ((I − R) (Y)) + R (R ((I − R) (X) + R (P)))

= (I − R) (Z) + R ((I − R) (Y)) + R (R (I − R) (X) + R (R (P)))

= (I − R) (Z) + R ((I − R) (Y)) + R (R (I − R) (X)) + R (R (R (P)))

= (I − R) (Z) + R ((I − R) (Y)) + R2 (I − R) (X) + R3 (P)

Onde denotamos por R3 (P) a composta de tres rotacoes de 120◦ no sentido horario doponto P em torno da origem. Mas, como tres rotacoes de 120◦ corresponde a uma rotacaode 360◦, temos que R3 (P) e a translacao pelo vetor nulo e, portanto, e a transformacaoidentidade.Assim, seja:

M = (I − R) (Z) + R ((I − R) (Y)) + R2 (I − R) (X) .

Temos:RZ (RY (RX (P))) = M + P.

Observemos que M nao depende de P. Logo, RX ◦RY ◦RZ (P) e uma translacao pelo vetor−−→OM.

Fazendo M = A, ja vimos que a composta das tres rotacoes deixa o ponto A fixo, logo,

RX ◦ RY ◦ RZ (A) = Id e, consequentemente,−−→OM =

−→0 e o vetor nulo.

A composicao de RX com RY e uma rotacao de 240◦ no sentido horario. Chamaremos deO′ o centro da rotacao RX ◦ RY = TO′. Assim, TO′ : R

2 → R2 e a rotacao de um angulo de

240◦ e centro O′. Logo:TO′ (P) =

(I − R2

)(O′) + R2 (P) ,

sendo R2 : R2 → R

2 rotacao de 240o no sentido horario em torno da origem.Entao:

RY (RX (P)) = TO′ (P) .

Logo:

(I − R) (Y) + R (RX (P)) =(I − R2

)(O′) + R2 (P)

(I − R) (Y) + R ((I − R) (X) + R (P)) =(I − R2

)(O′) + R2 (P)

(I − R) (Y) + R ((I − R) (X)) + R2 (P) =(I − R2

)(O′) + R2 (P)

(I − R) (Y) + R ((I − R) (X)) =(I − R2

)(O′)

(I − R) (Y) + R ((I − R) (X)) = (I − R) ((I + R) (O′))

I (Y) − R (Y) + R (I (X) − R (X)) = (I − R) ((I + R) (O′))

I (Y) − R (Y) + R (X) − R2 (X) = (I − R) ((I + R) (O′))

I (Y) + R (X) − R (Y − R (X)) = (I − R) ((I + R) (O′))

I (Y + R (X)) − R (Y − R (X)) = (I − R) ((I + R) (O′))

(I − R) (Y + R (X)) = (I − R) ((I + R) (O′))

Page 240: Numero 09 - Outubro de 2007

Como (I − R) e um operador linear, e tambem uma aplicacao bijetora e, portanto, injetora.Logo, de:

(I − R) (Y + R (X)) = (I − R) ((I + R) (O′)) ,

concluımos que:

Y + R (X) = (I + R) (O′) .

Multiplicando ambos os lados da equacao pela inversa de (I + R) temos:

(I + R)−1

(Y + R (X)) = (I + R)−1

(I + R) (O′)

O′ = (I + R)−1

(Y + R (X)) .

Assim obtemos o ponto O′.

Mas afirmamos que O′ tambem e a imagem da rotacao de 60◦ no sentido horario de X emtorno de Y.

De fato:

Denotamos S : R2 → R

2 a rotacao de 60◦ no sentido horario em torno da origem. Daı,temos R : R

2 → R2 e a mesma que S2 : R

2 → R2. Entao:

O′ = (I + R)−1

(Y + R (X)) =(I + S2

)−1 (Y + S2 (X)

).

Mas a aplicacao que gira X em torno de Y no sentido horario de um angulo de 60◦ e:

SY (X) = Y + S (X − Y) .

Logo, devemos mostrar que O′ = SY (X) , ou seja:(I + S2

)−1 (Y + S2 (X)

)= Y + S (X − Y) .

Multiplicando ambos os lados por(I + S2

), temos:

(I + S2

) (I + S2

)−1 (Y + S2 (X)

)=

(I + S2

)(Y + S (X − Y))

Y + S2 (X) =(I + S2

)(Y + S (X − Y))

Y + S2 (X) =(I + S2

)(Y + S (X) − S (Y))

Y + S2 (X) = Y + S (X) − S (Y) + S2 (Y) + S3 (X) − S3 (Y)

S2 (X) − S2 (Y) = S (X) − S (Y) + S3 (X) − S3 (Y)

S2 (X − Y) = S (X − Y) + S3 (X − Y)(S3 − S2 + S

)(X − Y) = 0

A aplicacao(S3 − S2 + S

)e a aplicacao nula.

De fato: seja P ∈ R2 um ponto qualquer. Temos S3 (P) , S2 (P) e S (P) vetores de mesma

amplitude formando angulos de 120◦ entre si.

Page 241: Numero 09 - Outubro de 2007

P

S(P)

-S (P)2

S (P)2

S (P)3

-S (P)2

S (P)3

O x

y

Assim, (S3 − S2 + S

)(P) = S3 (P) − S2 (P) + S (P) = 0

e, entao, a equacao (S3 − S2 + S

)(X − Y) = 0

e verdadeira. Logo:O′ = SY (X) .

Donde concluımos que o triangulo XYO′ e equilatero.

Agora, basta mostrar que O′ = Z.

Temos:

RZ (RY (RX (P))) = RZ (TO′ (P))

= (I − R) (Z) + R (TO′ (P))

= (I − R) (Z) + R((

I − R2)(O′) + R2 (P)

)= (I − R) (Z) + R

((I − R2

)(O′)

)+ R3 (P)

= (I − R) (Z) + R(O′ − R2 (O′)

)+ I (P)

= (I − R) (Z) + R (O′) − R3 (O′) + P

= (I − R) (Z) + R (O′) − I (O′) + P

= (I − R) (Z) − (I − R) (O′) + P

= (I − R) (Z − O′) + P

Como a composta RX ◦ RY ◦ RZ e a identidade, segue que:

P = (I − R) (Z − O′) + P.

Daı:(I − R) (Z − O′) = 0.

Page 242: Numero 09 - Outubro de 2007

Mas ja vimos que (I − R) e uma aplicacao linear injetora, logo:

Z − O′ = 0 ⇒ Z = O′.

Portanto, o triangulo XYZ e equilatero. �

2.5 Demonstracao do Teorema de Napoleao utilizando o Con-ceito de Complexidade Algebrica

Demonstracao do Teorema para o triangulo externo:

Como homotetias e isometrias nao alteram a natureza dos Triangulos de Napoleao, con-sideremos o triangulo ABC no plano complexo de forma que o vertice A esteja na origeme o vertice B em 1 e o vertice C estara associado a um numero complexo z,como na figuraabaixo.

x

y

A=0 B=1

C=z

Para se fazer no plano complexo uma rotacao de um numero em torno da origem bastamultiplicar esse numero por uma constante complexa de modulo 1. Dessa forma, pode-seobter P a partir da rotacao de π

3no sentido anti-horario do vertice A em torno de B.

Daı:P = (A − B)eiπ

3 + B = (0 − 1)eiπ3 + 1 = 1 − eiπ

3 .

Verificando geometricamente:

R

A

(A-B)ei��� P=(A-B)ei���

+B

(A-B)B

C

Q

X

YZ

A=0B=0B

Page 243: Numero 09 - Outubro de 2007

E da mesma forma podemos obter Q pela rotacao de B em torno de C e R pela rotacaode C em torno de A. Entao, temos:

Q = (B − C)eiπ3 + C = (1 − z)eiπ

3 + z;

R = (C − A)eiπ3 + A = (z − 0)eiπ

3 + 0 = zeiπ3 .

As coordenadas dos baricentros X, Y e Z dos triangulos APB,BQC e CRA, respectiva-mente, podem ser obtidas atraves das medias aritmeticas das coordenadas dos verticesdesses triangulos e, como os vertices dos mesmos podem ser associados a expressoes afinsem z, temos:

X =A + P + B

3=

2 − eiπ3

3;

Y =B + Q + C

3=

(2 − eiπ

3

)z + 1 + eiπ

3

3;

Z =C + R + A

3=

(1 + eiπ

3

)z

3.

Para mostrar que o triangulo XYZ e equilatero, devemos verificar que Y pode ser obtido

atraves da rotacao no sentido anti-horario deπ

3do vertice X em torno de Z, ou seja,

Y = (X − Z)eiπ3 + Z ⇒ Y − (X − Z)eiπ

3 − Z = 0.

Substituindo X, Y e Z por suas expressoes correspondentes, obtemos:

2z − zeiπ3 + 1 + eiπ

3

3−

(2 − eiπ

3

3−

z + zeiπ3

3

)eiπ

3 −z + zeiπ

3

3= 0 ⇒

p (z) = (1 − eiπ3 + ei2π

3 )z + (1 − eiπ3 + ei2π

3 ) = 0

Notemos que p e um polinomio de grau 1 na variavel z, ou seja, p (z) = 0 e uma equacaopolinomial de grau 1 na variavel z. Logo, encontrando duas raızes para ela, chegamos aconclusao de que p (z) = 0 e, na verdade, uma identidade.Por exemplo, para z = eiπ

3 , temos:

(1 − eiπ3 + ei2π

3 )eiπ3 + (1 − eiπ

3 + ei2π3 ) = eiπ

3 − ei2π3 + eiπ + 1 − eiπ

3 + ei2π3 = 1 + eiπ = 0.

Para z = ei−π3 , temos:

(1−eiπ3 +ei2π

3 )ei−π3 +(1−eiπ

3 +ei2π3 ) = ei−π

3 −ei0+eiπ3 +1−eiπ

3 +ei2π3 = ei−π

3 +ei2π3 = 0.

As figuras abaixo representam as configuracoes geometricas correspondentes a essas solucoes:

Page 244: Numero 09 - Outubro de 2007

Logo, p (z) = 0 e, de fato, uma identidade. Portanto, qualquer valor atribuıdo a z satisfaza equacao, ou seja, o vertice C = z pode estar em qualquer lugar do plano complexo, oque significa que o teorema e valido para qualquer triangulo ABC. �

A construcao do triangulo ABC com vertices A = 0 e B = 1 no plano complexo so estadefinida para valores de C = z fora do eixo real. No entanto, a demonstracao acimapermite que consideremos triangulos degenerados, ou seja, que C esteja no eixo real.

A demonstracao para o Triangulo Interno de Napoleao associado ao triangulo ABC seguede maneira analoga a demonstracao acima.

3 Propriedades dos Triangulos de Napoleao

3.1 Complexidade Algebrica 1: A Coincidencia dos Baricentros

Propriedade 1: “Seja ABC um triangulo qualquer. Os Triangulos de Napoleao Externoe Interno de ABC tem o mesmo baricentro e este coincide com o baricentro do trianguloABC.”

Demonstracao

Considere a configuracao, na qual o triangulo inicial esta plano complexo de tal formaque o vertice A esta na origem, o vertice B no 1 e o vertice C associamos a um numerocomplexo z qualquer.

Ja vimos que P,Q e R pode ser obtido a partir da rotacao deπ

3no sentido anti-horario do

vertice A,B e C em torno do vertice B,C e A respectivamente. Logo, podemos escrever:

P = (A − B)eπ3

i + B = 1 − eπ3

i.

Q = (B − C)eπ3

i + C = (1 − z)eπ3

i + z

R = (C − A)eπ3

i + A = zeπ3

i.

Temos que as coordenadas dos pontos X, Y e Z sao dadas por:

X =A + B + P

3=

2 − eπ3

i

3

Y =B + C + Q

3=

(2 − eπ3

i)z + 1 + eπ3

i

3

Z =C + Z + R

3=

(1 + eπ3

i)z

3.

Geometricamente, considere os vertices A, B e P como sendo os respectivos vetores−−→OA,−→

OB e−→OP.

Page 245: Numero 09 - Outubro de 2007

R

A

PT

B

C

Q

X

Y

Z

Somando-se os tres vetores, obtemos o ponto T, ou seja, o vetor−→OT. Dividindo-o em tres

partes congruentes, verifica-se que o ponto X esta a um terco de T. Analogamente, tem-seo mesmo para Y e Z.

Portanto, o baricentro de um triangulo e a media geometrica da soma das coordenadasde seus vertices. Notemos que.as coordenadas X, Y e Z sao expressoes afins em z.

Temos que P′, Q′ e R′ tambem sao expressoes afins em z:

P′ = (B − A)eπ3

i + A = eπ3

i

Q′ = (C − B)eπ3

i + B = (z − 1)eπ3

i + 1

R′ = (A − C)eπ3

i + C = (1 − eπ3

i)z.

E, da mesma forma, os pontos X′, Y′ e Z′:

X′ =A + B + P′

3=

1 + eπ3

i

3.

Y′ =B + C + Q′

3= 2 + (1 + e

π3

i)z − eπ3

i.

Z′ =C + A + R′

3=

(2 − eπ3

i)z

3.

Chamando de G1 =X + Y + Z

3o baricentro do triangulo externo e G2 =

X′ + Y′ + Z′

3o

baricentro do triangulo interno, segue que ambos coincidem quando:

G1 − G2 = 0 ⇒2−e

π3

i

3+ (2−e

π3

i)z+1+eπ3

i

3+ (1+e

π3

i)z3

3−

1+eπ3

i

3+ 2+(1+e

π3

i)z−eπ3

i

3+ (2−e

π3

i)z3

3= 0 ⇒

p (z) =2−e

π3

i

3+ (2−e

π3

i)z+1+eπ3

i

3+ (1+e

π3

i)z3

3−

1+eπ3

i

3+ 2+(1+e

π3

i)z−eπ3

i

3+ (2−e

π3

i)z3

3= 0.

Page 246: Numero 09 - Outubro de 2007

Sendo p polinomio de grau 1, temos que a Propriedade 1 possui complexidade algebrica 1.

No entanto, nao procederemos como na demonstracao do Teorema de Napoleao atribuindodois valores particulares para z e concluindo que p e uma identidade. Neste caso, ebastante simples simplificar p.

Observemos que:

G1 =X + Y + Z

3

=2−e

π3

i

3+ (2−e

π3

i)z+1+eπ3

i

3+ (1+e

π3

i)z3

3

=2 − e

π3

i + 2z − eπ3

iz + 1 + eπ3

i + z + eπ3

iz

9

=z + 1

3.

e:

G2 =X′ + Y′ + Z′

3

=1+e

π3

i

3+ 2+(1+e

π3

i)z−eπ3

i

3+ (2−e

π3

i)z3

3

=1 + e

π3

i + 2 + z + eπ3

iz − eπ3

i + 2z − eπ3

iz

9

=z + 1

3.

e conclui-se que G1 = G2.

A demonstracao de que G (baricentro do triangulo incial) coincide com G1 e G2 e imediata,pois:

G =A + B + C

3=

0 + 1 + z

3=

1 + z

3.

3.2 Complexidade Algebrica 2: A Diferenca das Areas

Propriedade 2: “A diferenca entre as areas dos Triangulos de Napoleao Externo eInterno de ABC e igual a area do triangulo ABC.”

Demonstracao

Consideremos novamente a configuracao da triangulo ABC no plano complexo, onde A =

0, B = 1 e C = z. Ja provamos que os Triangulos Externo e Interno de Napoleao saoequilateros e que seus vertices sao expressoes afins em z. Portanto, podemos escrever seuslados como sendo l e l′, respectivamentea e:

l = ||αz + β||

el′ = ||δz + γ||,

onde α,β, δ, γ ∈ C.

Page 247: Numero 09 - Outubro de 2007

A area do triangulo incial ABC e dada da seguinte forma:

Area(ABC) =base × altura

2=

1 × h

2. (i)

Mas, observemos que:

x

y

h

h

c=z=x+hi

A=0 B=1

Se z = x + hi entao z = x − hi. Daı:

z − z = (x + hi) − (x − hi) = 2hi =⇒ h =z − z

2i.

Logo, podemos reescrever (i) da seguinte forma:

Area(ABC) =1

2

z − z

2i,

e

Area(XYZ) =

√3

4l2 =

√3

4||αz + β||2,

Area(X′Y′Z′) =

√3

4(l′)

2=

√3

4||δz + γ||2.

Queremos mostrar que:

Area(ABC) = Area(XYZ) − Area(X′Y′Z′),

isto e, que:1

2

z − z

2i=

√3

4||αz + β||2 −

√3

4||δz + γ||2. (ii)

Observemos que:√

3

4||αz + β||2 =

√3

4(αz + β)(αz + β)

=

√3

4(αz + β)(αz + β)

=

√3

4(αz + β)(αz + β)

=

√3

4(ααzz + αzβ + αzβ + ββ)

Page 248: Numero 09 - Outubro de 2007

e

√3

4||αz + β||2 = −

√3

4(δz + γ)(δz + γ)

= −

√3

4(δδzz + δzγ + δzγ + γγ).

Desenvolvendo a expressao (ii) temos:

1

2

z − z

2i=

√3

4(ααzz + αzβ + αzβ + ββ) −

√3

4(zz + αzγ + δzγ + γγ) ⇒

0 =(√

3αα −√

3δδ)

zz +

(√3αβ −

√3δγ −

1

i

)z

+

(√3αβ −

√3δγ +

1

i

)z +

√3ββ −

√3γγ ⇒

0 = −√

3

[(δδ − αα)zz +

(δγ − αβ +

1√3i

)z +

(δγ − αβ −

1√3i

)z + γγ − ββ

] ⇒0 = (δδ − αα)zz +

(δγ − αβ −

√3i

3

)z +

(δγ − αβ +

√3i

3

)z + γγ − ββ.

de forma que a Propriedade 2 e verdadeira se, e somente se, a expressao

P(z) = (δδ − αα)zz +

(δγ − αβ −

√3i

3

)z +

(δγ − αβ +

√3i

3

)z + γγ − ββ

e igual a zero para todo z ∈ C.

Temos que P e um polinomio de grau 2 nas variaveis x = Re(z) e y = Im(z). Logo:

P(x, y) = (δδ − αα)x2 + (δδ − αα)y2 +

(δγ − αβ −

√3i

3+ δγ − αβ +

√3i

3

)x+

+

(δγi − αβi −

√3

3− δγi + αβi +

√3

3

)y + γγ − ββ

ou seja:

(δδ−αα)x2+(δδ−αα)y2+(δγ − αβ + δγ − αβ

)x+

(δγi − αβi − δγi + αβi

)y+γγ−ββ = 0,

que e da forma(x − a)

2+ (y − b)

2= r2,

sendo:

a =δγ − αβ + δγ − αβ

2(δδ − αα)

b =δγi − αβi − δγi + αβi

2(δδ − αα)

r2 =

(δγ − αβ + δγ − αβ

2(δδ − αα)

)2

+

(δγi − αβi − δγi + αβi

2(δδ − αα)

)2

−(γγ − ββ

)

Page 249: Numero 09 - Outubro de 2007

Portanto, P e a equacao de uma circunferencia.

Sabemos que tres pontos nao colineares determinam uma unica circuferencia, portanto,se encontrarmos quatro raızes de P que nao estao em uma mesma circuferencia teremosque P(z) = 0 para todo z ∈ C.

Tomemos:

(i) z1 = 0.

R=RA=CZ=Z

R=P’

P=R’

X=Z’

Z=X’

B

Area(XYZ) = Area(X′Y′Z′) e Area(ABC) = 0.

(ii) z2 = 1.

Q=QB=CY=Y

R=P’

A

P=R’

X=Z’

Z=X’

Area(ABC) = Area(X′Y′Z′) e Area(XYZ) = 0.

(iii) z3 = eπ3

i.

Page 250: Numero 09 - Outubro de 2007

B=R’

C=P’

A=Q’

P

X

X =Y =Z’ ’ ’

R Q

Area(ABC) = Area(XYZ) e Area(X′Y′Z′) = 0.

(iv) z4 = e−π3

i.

B=R

P’

A=Q

C=P

X’

X=Y=Z

R’ Q’

Area(ABC) = Area(X′Y′Z′) e Area(XYZ) = 0.

Em todos os casos acima, Area(ABC) = Area(XYZ) − Area(X′Y′Z′), isto e, z1, z2, z3 ez4 sao quatro raızes de P que nao estao numa mesma circuferencia. Observemos a figuraabaixo.

Page 251: Numero 09 - Outubro de 2007

x

y

1

e�/3i

e-i�/3

0

60º

60º

60º

60º

60º

60º

Portanto, a Propriedade 2 e verdadeira para qualquer valor de z, e vale para qualquertriangulo ABC. �

3.3 Complexidade Algebrica 3: Tres Retas Concorrentes em umUnico Ponto

Propriedade 3: “Sejam ABC um triangulo e ABP, BCQ e CAR os triangulos equilaterosapoiados externamente nos lados de ABC. Entao, os segmentos PC, QA e RB possuem omesmo comprimento e se encontram em um unico ponto. Alem disso, as retas que contemesses segmentos formam angulos congruentes entre si cujas medidas sao de 60◦.”

R

A

I

B

C

Q

P

Demonstracao.

Vamos demonstrar primeiro que os segmentos AQ, BR e CP tem o mesmo comprimentoe que fazem entre si um angulo de 60◦.

Consideremos novamente a configuracao do triangulo ABC no plano complexo onde A = 0,

B = 1 e C = z (primeira figura da Propriedade 1).

Page 252: Numero 09 - Outubro de 2007

Temos que Q pode ser obtido a partir da rotacao deπ

3no sentido anti-horario do vertice

B em torno de C, ou seja:

Q = (B − C)eπ3

i + C.

Substituindo os valores de B e C na expressao de Q temos:

Q = (1 − z) eπ3

i + z.

Da mesma forma, o ponto R pode ser obtido a partir da rotacao deπ

3no sentido anti-

horario do vertice C em torno de A,ou seja:

R = (C − A)eπ3

i + A.

Substituindo pelos valores de C e A temos:

R = zeπ3

i.

Mas R tambem pode ser obtido rotacionando o ponto Q de2π

3no sentido anti-horario

em torno do vertice A e transladado pelo vetor−→AB obtendo a expressao:

R = (Q − A) e2π3

i + B,

ou ainda:

R − B = (Q − A) e2π3

i,

sendo A =−→0 , R =

−→AR, B =

−→AB e Q =

−−→AQ.

Daı:

−→AR −

−→AB =

(−−→AQ −

−→0)

e2π3

i ⇒−→BR =

−−→AQe

2π3

i,

ou seja, BR e o segmento AQ rotacionado de2π

3.

Geometricamente:

Page 253: Numero 09 - Outubro de 2007

A B

CQ

P

R=(C-A)e +A=(Q-A)e +Bi /3 i2 /3� �

(Q-A)ei2 /3�

Substituindo na expressao de R os valores de Q, A e B temos:

R =[(1 − z) e

π3

i + z]e

2π3

i + 1.

Igualando as duas expressoes de R obtemos a equacao:

[(1 − z) e

π3

i + z]e

2π3

i + 1 = zeπ3

i.

Notemos que essa e uma expressao afim em z, entao, se encontrarmos duas raızes parao qual a expressao e verdadeira temos uma identidade, isto significa que ela vale paraqualquer valor de z no plano complexo.

Vamos tomar, por exemplo, C = z1 = eπ3

i. Daı:

−eπi + e4π3

i + e2π3

i = 0[(1 − e

π3

i)e

π3

i + eπ3

i]e

2π3

i + 1 = eπ3

ieπ3

i(e

π3

i − e2π3

i + eπ3

i)

e2π3

i + 1 = e2π3

i

eπi − e4π3

i + eπi + 1 = e2π3

i

−eπi + e4π3

i + e2π3

i = 0.

Observacao: eπi = cos (π) + i sen (π) = −1.

Geometricamente:

Page 254: Numero 09 - Outubro de 2007

-ei�

e

i2�3

e

i4�3

-ei�

e

i4�3

+

e

i2�3

-ei�

e

i4�3

+ + =0

1

Outra raiz que satisfaz a equacao e C = z1 = e−π3

i, pois:[(1 − e

−π3

i)

eπ3

i + e−π3

i]e

2π3

i + 1 = e−π3

ieπ3

i(e

π3

i − e0 + e−π3

i)

e2π3

i + 1 = e0

eπi − e2π3

i + eπ3

i + 1 = 1

eπi − e2π3

i + eπ3

i = 0.

De fato, encontramos duas raızes para a expressao afim em z, portanto, para qualquertriangulo ABC, os segmentos AQ e BR sao congruentes e fazem entre si um angulo de60◦. A demonstracao de que CP tambem tem o mesmo comprimento e faz um angulo de60◦.com AQ e BR segue de maneira analoga.

Essa primeira parte da demonstracao tem grau de complexidade algebrica um, ja a se-gunda parte e mais complexa e tem grau tres. Vejamos.

Queremos mostrar agora que os segmentos AQ, BR e CP se encontram em um unicoponto I.

Primeiramente vamos encontrar as coordenadas dos pontos Q e R no plano complexo emfuncao de x e y, lembrando que z = x + yi.e eθi = cos (θ) + i sen (θ) .

Daı:

Q = [1 − (x + yi)]

(1

2+

√3i

2

)+ (x + yi)

=1

2−

1

2(x + yi) +

√3i

2−

√3i

2(x + yi) + (x + yi)

=1

2−

1

2x −

1

2yi +

√3i

2−

√3i

2x +

√3

2y + x + yi

=

(1

2x +

√3

2y +

1

2

)+

(−

√3

2x +

1

2y +

√3

2

)i.

Page 255: Numero 09 - Outubro de 2007

Podemos indicar um numero complexo x + yi como um par ordenado (x, y) , entao:

Q =

(1

2x +

√3

2y +

1

2,−

√3

2x +

1

2y +

√3

2

).

Analogamente, para R temos:

R = (x + yi)

(1

2+

√3i

2

)=

1

2x +

√3i

2x +

1

2yi −

√3

2y

=

(1

2x −

√3

2y

)+

(√3

2x +

1

2y

)i,

ou seja:

R =

(1

2x −

√3

2y,

√3

2x +

1

2y

).

Temos tambem que P e a rotacao do ponto A em torno de B, entao P = (A − B)eπ3

i + B.

Logo,

P = (0 − 1)

(1

2+

√3i

2

)+ 1 = −

1

2−

√3i

2,

ou seja:

P =

(−

1

2,−

√3

2

).

Sejam r, s, t as retas←→AQ,

←→BR,

←→CP, respectivamente, e α,β, γ ∈ R, tais que αAQ = AI,

βBR = BI e γCP = CI.

Parametrizando as retas temos:

r �−→ A + α(Q − A) = (0, 0) + α

(1

2x +

√3

2y +

1

2,−

√3

2x +

1

2y +

√3

2

)

s �−→ B + β (R − B) = (1, 0) + β

(1

2x −

√3

2y − 1,

√3

2x +

1

2y

)

t �−→ P + γ (C − P) =

(−

1

2, −

√3

2

)+ γ

(x +

1

2, y +

√3

2

)

As retas r, s e t se intersectam em um mesmo ponto, se e somente se, existem valores deα,β e γ tais que:

(0, 0) + α

(1

2x +

√3

2y +

1

2,−

√3

2x +

1

2y +

√3

2

)= (1, 0) + β

(1

2x −

√3

2y − 1,

√3

2x +

1

2y

)

(0, 0) + α

(1

2x +

√3

2y +

1

2,−

√3

2x +

1

2y +

√3

2

)=

(−

1

2,−

√3

2

)+ γ

(x +

1

2, y +

√3

2

)

Page 256: Numero 09 - Outubro de 2007

Daı: [0 + α

(1

2x +

√3

2y +

1

2

), 0 + α

(−

√3

2x +

1

2y +

√3

2

)]

=

[1 + β

(1

2x −

√3

2y − 1

), 0 + β

(√3

2x +

1

2y

)]e: [

0 + α

(1

2x +

√3

2y +

1

2

), 0 + α

(−

√3

2x +

1

2y +

√3

2

)]

=

[−

1

2+ γ

(x +

1

2

),−

√3

2+ γ

(y +

√3

2

)]Donde tiramos que:

1 + α

(−

1

2x −

√3

2y −

1

2

)+ β

(1

2x −

√3

2y − 1

)+ 0γ = 0

0 + α

(√3

2x −

1

2y −

√3

2

)+ β

(√3

2x +

1

2y

)+ 0γ = 0

1

2+ α

(1

2x +

√3

2y +

1

2

)+ 0β + γ

(−x −

1

2

)= 0

√3

2+ α

(−

√3

2x +

1

2y +

√3

2

)+ 0β + γ

(−y −

√3

2

)= 0

Colocando o sistema na forma matricial temos:⎡⎢⎢⎢⎢⎢⎢⎢⎢⎢⎢⎢⎢⎢⎢⎢⎣

1 −1

2x −

√3

2y −

1

2

1

2x −

√3

2y − 1 0

0

√3

2x −

1

2y −

√3

2

√3

2x +

1

2y 0

1

2

1

2x +

√3

2y +

1

20 −x −

1

2

√3

2−

√3

2x +

1

2y +

√3

20 −y −

√3

2

⎤⎥⎥⎥⎥⎥⎥⎥⎥⎥⎥⎥⎥⎥⎥⎥⎦

⎡⎢⎢⎣1

α

β

γ

⎤⎥⎥⎦ =

⎡⎢⎢⎣0

0

0

0

⎤⎥⎥⎦ .

Assim, temos que encontrar α,β, γ que satisfazem a equacao matricial.Sabemos que o sistema pode possuir uma unica solucao e, nesse caso, seu determinante ediferente de zero, ou possuir infinitas solucoes e seu determinante ser igual a zero.Esse sistema homogeneo nao possui a solucao trivial (nula), pois uma solucao possıveltem que ser da forma (1, α, β, γ) . Portanto, o determinante de M e igual a zero.

Page 257: Numero 09 - Outubro de 2007

Observemos que M e da forma:

M =

⎡⎢⎢⎢⎢⎢⎢⎢⎢⎢⎢⎢⎣

1 afim afim 0

0 afim afim 0

1

2afim 0 afim

√3

2afim 0 afim

⎤⎥⎥⎥⎥⎥⎥⎥⎥⎥⎥⎥⎦Calculando o determinante pelo Metodo de Laplace utizando a primeira coluna temos:

M = 1 (−1)2 det

⎡⎣afim afim 0

afim 0 afim

afim 0 afim

⎤⎦ + 0 (−1)3 det

⎡⎣afim afim 0

afim 0 afim

afim 0 afim

⎤⎦+

1

2(−1)

4 det

⎡⎣afim afim 0

afim afim 0

afim 0 afim

⎤⎦ +

√3

2(−1)

5 det

⎡⎣afim afim 0

afim afim 0

afim 0 afim

⎤⎦= (afim) (afim) (afim) − (afim) (afim) (afim)

+1

2[(afim) (afim) (afim) − (afim) (afim) (afim)]

√3

2[(afim) (afim) (afim) − (afim) (afim) (afim)] .

Assim, obtemos um polinomio nas variaveis x e y do tipo:

p (x, y) = α (x) y3 + β(x)y2 + γ (x) y + δ (x) ,

sendo α,β, γ, δ polinomios na variavel x, de forma que a Propriedade 3 e verdadeira semostrarmos que para todo (x, y) ∈ R

2 temos p(x, y) = 0.

Observemos que p(x, y) tem grau tres, entao se encontrarmos quatro pares (x, y) que sat-isfazem a equacao polinomial teremos uma identidade.e a propriedade vale para qualquerz = x + yi no plano complexo.Uma configuracao geometrica para o qual a Propriedade 3 e valida e aquela onde otriangulo ABC e isosceles.

P

A B

R

CQ

I

Page 258: Numero 09 - Outubro de 2007

Fixando x = x0 ∈ R, temos:

p (x0, y) = α (x0) y3 + β (x0) y2 + γ (x0) y + δ (x0)

p (x0, y) = q (y) = Ay3 + By2 + Cy + D,

sendo A,B, C, D constantes.

Fixemos agora x = x0 no intervalo aberto

]0,

1

2

[, como mostra a figura abaixo. Se

escolhermos C como sendo o ponto (x0, y1) ou (x0,y4) , obtidos a partir da interseccao dareta x = x0 com o cırculos de centro (0, 0) e raio 1, o triangulo ABC sera isosceles, poisdois de seus lados serao raio desse cırculo. O mesmo acontece se escolhermos C comosendo o ponto (x0, y2) ou (x0, y4) , obtidos a partir da interseccao da reta x = x0 com oscırculos de centro (1, 0) e raio 1.Entao, encontramos quatro raızes, y1, y2, y3 e y4 que satisfazem q (y) = 0. Logo, q (y) = 0

e uma identidade e vale para qualquer valor de y ∈ R.

x

y

y1

y2

y3

y4

0 x0 1½

Mas q (y) = p (x0, y) , entao p (x0, y) = 0 para todo y ∈ R.

Isto siginifica que α (x0) = β (x0) = γ (x0) = δ (x0) = 0 e como x0 e arbitrario segue que:

α (x) = β (x) = γ (x) = δ (x) = 0, para todo x ∈ R.

Consequentemente, p (x, y) = 0 para todo (x, y) ∈ R2. Logo, C = z = x + yi pode estar

em qualquer lugar do plano complexo.Conclusao: a Propriedade 3 e valida para qualquer triangulo ABC.

4 Referencias Bibliograficas

[1] Avila, G. Variaveis Complexas e Aplicacoes. Rio de Janeiro: LTC - Livros Tecnicose Cientıficos Editora. 1990.

[2] Callioli, C. A., Domingues, H. H. & Costa, R. C. F. Algebra Linear eAplicacoes. Sao Paulo: Atual Editora. 1983.

[3] Lopes, S. M. R. Complexidade em Geometria Plana Euclidiana. (Dissertacao deMestrado). Rio de Janeiro: PUC - Pontifıcia Universidade Catolica. 2002.

Page 259: Numero 09 - Outubro de 2007

Introducao a Teoria das Curvas AlgebricasAfins

Patrıcia Borges dos Santos 1 Cıcero Fernandes de Carvalho2

Faculdade de Matematica - FAMAT

Universidade Federal de Uberlandia - UFU - MG

Setembro de 2007

Resumo

Neste trabalho apresentamos algumas propriedades basicas das curvas algebricasafins como por exemplo, a irredutibilidade, a decomposicao em componentes irre-dutıveis, o grau da curva, entre outras. Tambem mostramos que e possıvel estimaro numero de pontos de intersecao de duas curvas algebricas sem componentes emcomum. Em essencia o trabalho mostra aplicacoes da teoria de domınios euclidianose domınios de fatoracao unica.

1 Introducao

Seja f(x, y) ∈ R[x, y] e considere o seguinte subconjunto de R2:

V (f) = {(x, y) ∈ R2 | f(x, y) = 0},

agora observe nos seguintes exemplos o que V (f) representa.

Exemplo 1.1 1. f(x, y) = y − ax2 − bx − c, com a �= 0 =⇒ V (f) e uma parabola.

2. f(x, y) = ax + by + c, com a, b ∈ R, (a, b) �= (0, 0) =⇒ V (f) e uma reta.

3. f(x, y) = x2 + y2 − 1 =⇒ V (f) e um cırculo.

4. f(x, y) = x2 + y2 =⇒ V (f) e um ponto.

5. f(x, y) = x2 + y2 + 1 =⇒ V (f) e um conjunto vazio.

Note que somente nos exemplos 1 , 2 e 3 e que faz sentido falar em curva, os outrosdois casos deixam de parecer tao estranhos quando passamos do plano real R

2 para oplano complexo C

2, que aqui chamaremos de plano afim. Nesse caso e natural permitirque os coeficientes de f(x, y) sejam numeros complexos arbitrarios.

Assim, considere agora f(x, y) ∈ C[x, y] e o seguinte subconjunto de C2:

V (f) = {(x, y) ∈ C2 | f(x, y) = 0}.

Observe que, se em 4 permitirmos que f ∈ C[x, y], entao teremos a seguinte fatoracao

f(x, y) = x2 + y2 = (x + iy)(x − iy).

1patricia [email protected] Programa de Educacao Tutorial (PETMAT)[email protected] Professor orientador de janeiro de 2007 a dezembro de 2007.

Page 260: Numero 09 - Outubro de 2007

Assim, V (f) = V (x+ iy)∪V (x− iy), ou seja, o conjunto de zeros consiste, no plano afim,de duas retas que intersectam na origem.

Da mesma forma, no exemplo 5 , a mudanca de variaveis x = ix1, y = iy1 torna oconjunto vazio no R

2 em um cırculo, agora em C2, com equacao:

x21 + y2

1 − 1 = 0,

visto que: x2+y2+1 = 0 =⇒ (ix1)2+(iy1)

2+1 = 0 =⇒ −x21−y2

1+1 = 0 =⇒ x21+y2

1−1 = 0.Ao passar de R

2 para C2 perdemos um pouco da intuicao, ja que as curvas algebricas

agora se parecem mais com superfıcies no espaco quadri-dimensional C2 ∼= R

4. Por outrolado, veremos que esta perda sera compensada por muitas vantagens tecnicas.

2 Curvas Algebricas Afins e suas Equacoes

Definicao 2.1 Um subconjunto C ⊂ C2 e chamado uma curva algebrica afim se existir

um polinomio f ∈ C[x, y] tal que grau f ≥ 1 e

C = V (f) = {(x, y) ∈ C2 | f(x, y) = 0}.

Veja que V (f) = V (λf) = V (fk) para λ ∈ C∗ e k ∈ N − {0}, ou seja, uma curva

algebrica afim nao e unicamente determinada por f . Quando f(x, y) e irredutıvel, estasera a unica ambiguidade, isto e, os polinomios da forma g(x, y) = λf(x, y)k, para λ ∈ C

e k ∈ N − {0}, sao os unicos para os quais V (f) = V (g). Isto sera uma consequencia doLema de Study abaixo.

Antes de apresentarmos o Lema de Study devemos ter em mente alguns resultados queserao importantes na demonstracao deste:

Teorema 2.2 Seja (D,ϕ) um domınio euclidiano. Entao:

1. D e um domınio fatorial.

2. Sejam a, b ∈ D\ {0} e seja d = mdc (a, b), entao:

• Existem e, f ∈ D tais que d = ea+fb.(Em particular, se a e b sao relativamenteprimos, i.e. se d = 1, entao existem e, f ∈ D tais que 1 = ea + fb).

• Tais e e f podem ser efetivamente calculados quando a divisao e efetiva.

3. D e um domınio principal.

Lema 2.3 (Lema de Gauss) Seja A um domınio de fatoracao unica e seja K ⊇ A seucorpo de fracoes. Seja f ∈ A[x] um polinomio primitivo nao constante.

1. Se f e redutıvel em K[x], entao tambem o e em A[x].

2. Se g ∈ A[x] e f | g em K[x], entao f | g em A[x].

As demonstracoes de 2.2 e 2.3 podem ser encontradas em [3] e [4] respectivamente.

Observacao 2.4 Veja que (C (x) [y] , ϕ), sendo ϕ a funcao grau, e um domınio euclidi-ano. E que todo polinomio irredutıvel e primitivo.

Page 261: Numero 09 - Outubro de 2007

Agora estamos prontos para enunciar e demonstrar o

Lema 2.5 (Lema de Study) Sejam f(x, y), g(x, y) ∈ C[x, y] polinomios nao constantese suponha que f(x, y) e irredutıvel. Entao V (f) ⊂ V (g) se e somente se f divide g emC[x, y].

Demonstracao. Se f e um divisor de g, isto e, g = f.h, entao V (f) ⊂ V (g).Reciprocamente, suponha por absurdo que V (f) ⊂ V (g) mas f nao divide g em C[x, y].

Sem perda de generalidade, podemos supor que f(x, y) nao e um polinomio apenas navariavel x. Assim podemos considerar f(x, y) como um polinomio nao constante de C[x][y](i.e. um polinomio nao constante na variavel y e com coeficientes no anel de polinomiosC[x]):

f(x, y) = f0(x) + f1(x)y + . . . + fn(x)yn,

com n > 0 e fn(x) �= 0.Seja C(x) o corpo quociente de C[x] e considere f(x, y) como um polinomio nao cons-

tante de C(x)[y]. Pelo Lema de Gauss, f(x, y) e irredutıvel em C(x)[y] e f nao divide g emC(x)[y]. Portanto mdc (f, g) = 1. Por 2.2 sabemos que C(x)[y] e um domınio principal, etambem que existem elementos α(x, y), β(x, y) ∈ C(x)[y] tais que

α(x, y)f(x, y) + β(x, y)g(x, y) = 1.

Lembramos que α(x, y) e β(x, y) sao polinomios na variavel y e com coeficientes nocorpo de fracoes C(x). Seja p(x) o mmc dos denominadores dos coeficientes de α(x, y) eβ(x, y). Observe que p(x) e nao nulo. Existem polinomios a(x, y), b(x, y) ∈ C[x, y] tais

que α(x, y) = a(x,y)p(x)

e β(x, y) = b(x,y)p(x)

. Portanto:

a(x, y)f(x, y) + b(x, y)g(x, y) = p(x).

Como p(x) e fn(x) sao nao nulos, existe x0 ∈ C tal que p(x0) �= 0 e fn(x0) �= 0.Observe que o polinomio f(x0, y) ∈ C[y] e nao constante. Como C e algebricamentefechado, existe y0 ∈ C tal que f(x0, y0) = 0. Como V (f) ⊂ V (g), temos tambem queg(x0, y0) = 0. Pela formula acima segue que p(x0) = 0, contradizendo a escolha de x0.

Observacao 2.6 Um enunciado analogo nos numeros reais e obviamente falso, por exem-plo para f = x2 + y2 e g = x, temos V (f) ⊂ V (g), mas f nao divide g.

Este lema tecnico e um precursor do Teorema dos Zeros de Hilbert (Hilbert Nullstel-lensetz ), que em casos especiais de curvas, diz:

Corolario 2.7 Se f ∈ C[x, y] nao e constante, entao V (f) �= ∅.

Demonstracao. Suponha V (f) = ∅. Se h e um fator irredutıvel de f , entao V (h) ⊂V (f), ou seja, V (h) = ∅. Pelo Lema de Study, h divide qualquer g, pois V (h) = ∅ ⊂ V (g),qualquer que seja g. Mas isto e impossıvel.

Observacao 2.8 Note que se V (f) �= ∅, entao V (f) contem infinitos pontos. De fato,seja (x0, y0) ∈ V (f) entao para todo λ ∈ R, tem-se f(λx0, λy0) = 0, isto e, (λx0, λy0) ∈V (f).

Page 262: Numero 09 - Outubro de 2007

3 Componentes irredutıveis

Das numerosas consequencias do Lema de Study, a primeira que nos discutiremos e adecomposicao de uma curva algebrica em suas componentes. Ja que aneis de polinomiossobre corpos sao domınios de fatoracao unica, cada f ∈ C[x, y] admite fatoracao

f = fk11 . · · · .fkr

r ,

onde os f� sao irredutıveis e nao sao dois a dois associados. Esta fatoracao e unica amenos de multiplicacao por constante e da ordem em que os fi ocorrem.

Portanto V (f) = V (f1) ∪ · · · ∪ V (fr), em outras palavras, a curva definida por fpode ser decomposta nas componentes V (f�) . As proximas definicoes e resultados daraosignificado mais preciso as componentes.

Definicao 3.1 Uma curva algebrica C ⊂ C2 e chamada redutıvel se existem curvas

algebricas planas C1, C2 tais que C1 �= C2 e C = C1∪C2. Nesse caso dizemos que C1 e C2

sao componentes proprias de C. Se C nao admitir componentes proprias, dizemos que Ce irredutıvel, isto e, para toda decomposicao C = C1 ∪ C2 segue que C1 = C2.

Lema 3.2 Uma curva algebrica C = V (f) ⊂ C2 e irredutıvel se e somente se existir

k ∈ N − {0} e um polinomio irredutıvel g ∈ C[x, y] tais que f = gk.

Demonstracao. Seja C irredutıvel e seja f = f1.f2, onde f1 e f2 sao relativamenteprimos e nao constantes. Se h e um fator irredutıvel de f1, entao temos que V (h) ⊂ V (f1),mas como C e irredutıvel temos V (f1) = V (f2) e pelo Lema de Study segue que h | f2.Mas isto nao e possıvel pois f1 e f2 sao relativamente primos.

Reciprocamente, suponha que C seja redutıvel, isto e, V (f) = V (f1) ∪ V (f2) eV (f1) �= V (f2). Entao existem fatores irredutıveis hi de fi que nao sejam associadosentre si. A inclusao V (hi) ⊂ V (f) e o Lema de Study implicam que f tem pelo menosdois fatores primos distintos, o que contraria a hipotese de f ser irredutıvel.

Teorema 3.3 Toda curva algebrica C ⊂ C2 admite representacao C = C1 ∪ . . . ∪ Cr,

onde C1, . . . , Cr sao curvas algebricas irredutıveis. A representacao e unica a menos daordem em que os Ci ocorrem.

Demonstracao. Seja C = V (f) e seja f = fk11 . · · · .fkr

r a fatoracao de f em primos.Ja vimos que C = V (f) = V (f1)∪ . . .∪V (fr) e pelo lema anterior segue que cada V (fi)e irredutıvel.

Para mostrar a unicidade basta mostrar que toda curva irredutıvel C ′ ⊂ C e algumadas Ci. Mas se C ′ = V (f ′), com f ′ irredutıvel, entao C ′ e irredutıvel. Pelo Lema deStudy f ′ e um fator primo de f , mas pela unicidade da fatoracao de polinomios segue quef ′ tem que ser algum dos fi.

Definicao 3.4 Os Ci, como em 3.3, sao chamados componentes irredutıveis da curva C.

Como vimos as componentes irredutıveis de uma curva algebrica sao unicamente de-terminadas. Pelo Lema de Study, podemos tambem determinar os possıveis fatores irre-dutıveis de um polinomio dado.

Page 263: Numero 09 - Outubro de 2007

Corolario 3.5 Seja C = V (f) ⊂ C2 uma curva algebrica e seja f = fk1

1 . · · · .fkrr a

fatoracao de f em fatores irredutıveis. Se C = V (g) para algum outro polinomio g,entao:

g = λf l11 . · · · .f lr

r

onde λ ∈ C∗ e l� ∈ N − {0}.

Demonstracao. Temos que C = V (f) = V (f1) ∪ · · · ∪ V (fr) e por hipotese queC = V (g). Para cada i ∈ {1, . . . , r} temos que V (fi) ⊂ V (f) = V (g), onde cada fi efator irredutıvel de f . Desse modo como V (fi) ⊂ V (g), temos pelo Lema de Study que

fi | g, para todo i ∈ {1, . . . , r}. Assimn∏

i=1

f lii | g e portanto g =

n∏i=1

λf lii como querıamos.

Isto nos da uma completa visao das possıveis equacoes para C.

Definicao 3.6 Por analogia com polinomios de uma variavel, nos definimos f = f1. · · · .fr

como sendo o polinomio minimal da curva. Este e unico, a menos da unidade

Observacao 3.7 Temos a seguinte propriedade algebrica:

I (C) ={h ∈ C[x, y] | h|C = 0

}e um ideal no anel de polinomios. I (C) e chamado ideal de C, e um ideal principal e egerado pelo polinomio minimal (segue do corolario acima).

Agora vamos usar o polinomio minimal para definir o grau de uma curva algebrica:

Definicao 3.8 Se C = V (f) ⊂ C2 e uma curva algebrica e f o polinomio minimal,

entao grau (C) = grau (f) e o grau da curva C. Se f nao e necessariamente o polinomiominimal, dizemos que e o grau do divisor.

Em sequencia daremos o significado geometrico do grau, considerando intersecoes decurvas com retas.

4 Intersecao de uma Reta com uma Curva Algebrica

Afim

Seja a reta L ⊂ C2, dada pela parametrizacao:

ϕ : C −→ L ⊂ C2

t �−→ (ϕ1 (t) , ϕ2 (t)),

onde ϕi ∈ C [T ] sao polinomios lineares.Dada uma curva algebrica plana C = V (f) ⊂ C

2, o nosso objetivo e encontrar umacota superior para o numero de pontos de L ∩ C, isto e, uma cota para # (L ∩ C) . Paraisso vamos supor que f(x, y) ∈ C[x, y] e um polinomio de grau n e, L �⊂ C.

Para encontrar os pontos de L∩C, basta obter as raızes do polinomio em uma variavelcomplexa:

g(t) := f (ϕ1 (t) , ϕ2 (t)) ,

Page 264: Numero 09 - Outubro de 2007

isto e, os zeros de g correspondem aos pontos de intersecao de C com L.Como supomos que L �⊂ C, entao g(t) nao e identicamente nulo, e portanto # (L ∩ C)

e um conjunto finito. Observe tambem que grau g ≤ grau f , uma vez que alguns termosde f podem ser cancelados quando substituımos ϕ(t). Assim, podemos estimar quantossao os pontos de L ∩ C.

Como # (L ∩ C) e determinado pelo numero de raızes de g(t), e ja que este possui nomaximo n raızes tem-se:

# (L ∩ C) ≤ n.

Portanto, se C ⊂ C2 e uma curva algebrica de grau n e L ⊂ C

2 e uma reta tal queL �⊂ C, entao # (L ∩ C) ≤ n.

Observacao 4.1 Um resultado analogo a este nos numeros reais nos da uma maneirade mostrar que certos subconjuntos de R

2 ou C2 nao podem ser curvas algebricas, como

mostra o exemplo abaixo.

Exemplo 4.2 A senoide, a cicloide e a hipocicloide com razao de raios irracional, saoexemplos de curvas que nao sao algebricas. Em cada caso existem retas que nao estaocompletamente contidas nas curvas, mas que intersectam-nas em infinitos pontos.

O limitante dado para # (L ∩ C) raramente e alcancado nos numeros reais, mas nosnumeros complexos ele e “quase sempre” obtido. Existem dois motivos possıveis paraL ∩ C possuir menos de n pontos:

1. O grau de g(t) pode ser menor do que n.

2. O grau de g(t) pode ser n, mas g(t) pode ter raızes multiplas.

O segundo problema pode ser resolvido contando os pontos de intersecao com as suasrespectivas multiplicidades (que serao definidas abaixo), e o primeiro considerando-se ospontos de intersecao no infinito. Um exemplo simples disso e quando C e uma reta paralelaa L, neste caso havera apenas o ponto de intersecao no infinito.

Definicao 4.3 Seja L ⊂ C2 uma reta com parametrizacao linear ϕ(t) = (ϕ1 (t) , ϕ2 (t)),

onde ϕi ∈ C [t] sao polinomios lineares. Seja f(x, y) ∈ C[x, y] e suponha que L �⊂ C =V (f). Seja g(t) := f(ϕ(t)). Dado um ponto P = (x(t0), y(t0)) ∈ L ∩ V (f) (ou seja, t0 euma raiz de g(t)), definimos a multiplicidade de intersecao de L e C = V (f) em P comosendo a multiplicidade de t0 como raiz de g(t).

Exemplo 4.4 Seja L ⊂ C2 uma reta passando pela origem cuja parametrizacao e dada

porϕ : C −→ L ⊂ C

2

t �−→ (αt, βt),

com α, β ∈ C, e (α, β) �= (0, 0). Seja f ∈ C[x, y] dado por f(x, y) = y − x2. Veja que

g(t) = f(ϕ(t)) = βt − α2t2

e entao, para α �= 0, temos

g(t) = 0 ⇐⇒ βt − α2t2 = 0 ⇐⇒ t(β − α2t) = 0 ⇐⇒ t = 0 ou t =β

α2.

Page 265: Numero 09 - Outubro de 2007

Assim o numero de pontos de intersecao de L e V (f) para α �= 0 e 2. No entanto paraα = 0, temos g(t) = βt, e entao

g(t) = 0 ⇐⇒ βt = 0 ⇐⇒ t = 0,

ou seja, existe apenas 1 ponto de intersecao entre L e V (f). Note que nesse caso graug(t) < 2.

E natural questionar se tambem e possıvel obter uma estimativa para o numero depontos de intersecao de duas curvas algebricas quaisquer. A resposta a essa perguntasera dada pelo Teorema de Bezout, que sera apresentado na proxima secao como umaaplicacao da teoria de resultante de polinomios.

5 A Resultante e o Teorema de Bezout

O Teorema de Bezout fornece uma cota superior para o numero de pontos de intersecao deduas curvas algebricas, que e o produto dos graus destas curvas. A versao mais geral nosdiz que duas curvas V (f) e V (g) sem componentes irredutıveis em comum e com grausm e n, respectivamente, se intersectam em exatamente mn pontos, levados em conta os“pontos no infinito” e as “multiplicidades de intersecao”.

Apresentaremos aqui um caso particular desse teorema e para sua demonstracao seranecessario desenvolver a teoria de resultante de dois polinomios.

Definicao 5.1 Seja D um domınio. Sejam

f(x) = a0xn + a1x

n−1 + · · · + an, a0 �= 0

g(x) = b0xm + b1x

m−1 + · · · + bm, b0 �= 0

dois polinomios em D[x] de grau ≥ 1. A resultante de f(x) e g(x), denotada por Rf,g, eo elemento do domınio D dado pelo seguinte determinante:

Rf,g =

∣∣∣∣∣∣∣∣∣∣∣∣∣∣∣∣∣∣∣∣∣∣∣∣∣

a0 a1 . . . an−1 an

a0 . . . an−1 an...

......

......

......

... a0

b0 b1 . . .... . . . bm−1 bm... . . . . . . bm−1 bm...... . . . . . .b0 . . . . . . . . . . . . . . . bm

∣∣∣∣∣∣∣∣∣∣∣∣∣∣∣∣∣∣∣∣∣∣∣∣∣sendo que existem m linhas de ai

′s e n linhas de bi′s e as linhas sao completadas com

zeros. A resultante entre um polinomio f(x) e sua derivada f ′(x) (quando f ′(x) nao econstante), e chamada discriminante de f(x).

Page 266: Numero 09 - Outubro de 2007

Veremos no proximo teorema que dados dois polinomios sempre e possıvel calcular aresultante deles e consequentemente, quando D for fatorial, e possıvel determinar se elestem ou nao um fator comum de grau ≥ 1.

Teorema 5.2 Seja D um domınio. Sejam

f(x) = a0xn + a1x

n−1 + · · · + an, a0 �= 0

g(x) = b0xm + b1x

m−1 + · · · + bm, b0 �= 0

dois polinomios em D[x] de grau ≥ 1. Entao as seguintes condicoes sao equivalentes:

1. Rf,g = 0.

2. Existem polinomios

0 �= f1(x) ∈ D[x] de grau ≤ n − 10 �= g1(x) ∈ D[x] de grau ≤ m − 1

tais que f1(x)g(x) = g1(x)f(x).

3. Se D e um domınio fatorial, f(x) e g(x) possuem um fator comum em D[x] de grau≥ 1.

Demonstracao. (1 ⇐⇒ 2) Encontrar 0 �= f1(x) = α1xn−1 + α2x

n−2 + . . . + αn

e 0 �= g1(x) = β1xm−1 + β2x

m−2 + . . . + βm em D[x] tais que f1(x)g(x) = g1(x)f(x) eequivalente a encontrar uma solucao nao trivial em D do seguinte sistema de (n + m)equacoes nas incognitas β1, β2, . . . , βm, α1, α2, . . . , αn:

(termo em xn+m−1)(termo em xn+m−2)

. . .

. . .(termo constante )

⎧⎪⎪⎪⎪⎨⎪⎪⎪⎪⎩a0β1 − b0α1 = 0

a1β1 + a0β2 − b1α1 − b0α2 = 0. . .. . .

anβm − bmαn = 0

Agora, e facil ver que existe uma solucao nao-trivial deste sistema em D se e somentese existe uma tal solucao nao-trivial no corpo de fracoes K de D, ou seja, pela regra deCramer, se e somente se o determinante da matriz dos coeficientes do sistema e nulo. Istoconclui a prova pois a resultante e o determinante de uma matrz que e a transposta damatriz do sistema a menos de multiplicacao por −1 das linahs envolvendo bj

′s.

(3 ⇒ 2) Como f(x) e g(x) possuem um fator em comum p(x) em D[x] de grau ≥ 1,entao temos:

f(x) = p(x)f1(x) com f1(x) ∈ D[X], grau f1(x) < ng(x) = p(x)g1(x) com g1(x) ∈ D[X], grau g1(x) < m

e, claramente, f1(x)g(x) = g1(x)f(x) (Note que neste sentido nao se utilizou que D efatorial).

(2 ⇒ 3) Sejam f1(x), g1(x) ∈ D[x] tais que f1(x)g(x) = g1(x)f(x). Sendo D[x] fato-rial, todos os fatores irredutıveis de de grau geq1 aparecem no produto f1(x)g(x); nemtodos eles podem aparecer em f1(x), pois, por hipotese temos grau f1(x) < grau f(x);assim pelo menos um dos fatores irredutıveis de grau ≥ 1 de f(x) aparecem em g(x).

Page 267: Numero 09 - Outubro de 2007

Corolario 5.3 Sejam D e D′ dois domınios, D fatorial. Sejam f(x), g(x) ∈ D[x] de grau≥ 1. Entao, f(x) e g(x) Tem um fator em comum de grau ≥ 1 em D[x] se e somente seeles tem um fator comum de grau ≥ 1 em D′[x].

Demonstracao. Se f(x) e g(x) tem um fator comum de grau ≥ 1 em D′[x], entao aresultante Rf,g = 0 (segue da implicacao (3 ⇒ 1) de 5.2, para a qual nao se precisa suporD′ fatorial, como observamos acima); logo f(x) e g(x) tem um fator comum de grau ≥ 1em D[x] (implicacao (1 ⇒ 3) de 5.2). A recıproca e clara.

Proposicao 5.4 Nas hipoteses de 5.2 tem-se que a resultante Rf,ge uma soma de termosdo tipo ±ai1 . . . aimbj1 . . . bjn com i1 + . . . + im + j1 + jn = nm.

A demonstracao de 5.4 pode ser encontrada em [3], assim como outros resultados sobreresultante. Vamos agora aplicar os resultados acima a polinomios em C[x, y] = C[y][x],lembrando que C[y] e domınio euclidiano, e logo domınio fatorial.

Teorema 5.5 (Teorema de Bezout) Sejam f(x, y), g(x, y) ∈ C[x, y] e sejam V (f) eV (g) as curvas algebricas associadas de graus n, m ≥ 1. Se f(x, y) e g(x, y) nao possuemfator irredutıvel em comum, entao

#(V (f) ∩ V (g)) ≤ nm.

Demonstracao. Para a prova deste teorema vamos precisar da resultante (denotadapor Rf,g(y)) de f(x, y) e g(x, y) considerados como polinomios em C[y][x], i.e. Rf,g(y) e odeterminante da matriz dada em 5.1, onde ai = ai(y) e bi = bi(y). Como f(x, y) e g(x, y)nao tem fator comum em C[y][x], segue de 5.3 que f(x, y) e g(x, y) nao tem fator comumem C(y)[x] (em 5.3 tome D = C[y] e D′ = C(y), onde C(y) denota o corpo das fracoes deC[y]). Portanto, pelo teorema 2.2, existem a, b ∈ C(y)[x] tais que

1 = af + bg.

Multiplicando por um denominador comum d(y) ∈ C[y] para a e b, obtemos

d(y) = a1(x, y)f(x, y) + b1(x, y)g(x, y)

com a1(x, y), b1(x, y) ∈ C[x, y].Se (x, y) ∈ C

2 e tal que f(x, y) = g(x, y) = 0, entao d(y) = 0. Assim existe somenteum numero finito de ordenadas possıveis para um ponto em C

2 da intersecao das curvasdeterminadas por f e por g, a saber as raızes em C do polinomio d(y). Agora, para umaordenada fixa y0 ∈ C, existem no maximo n pontos em C

2 da curva determinada porf(x, y) com esta ordenada y, a saber os pontos (x, y) ∈ C

2 tais que x0 seja uma raiz def(x, y0). Fica assim provado que

#(V (f) ∩ V (g)) < ∞.

Como C e algebricamente fechado, entao C e infinito. Como o numero de pontos deintersecao e finito o numero de retas passando por dois destes pontos tambem e finito.Tomando como reta y = 0 uma que nao seja paralela a nenhuma destas retas, obtemos umsistema de coordenadas no qual pontos distintos da intersecao tem ordenadas distintas.

Page 268: Numero 09 - Outubro de 2007

Logo: #(V (f)∩V (g)) = #{y ∈ C | f(x, y) e g(x, y) tem uma raiz em comum em C} ≤#{y ∈ C | f(x, y) e g(x, y) tem um fator em comum em C[x]}.

Observe que por 5.2, se f(x, y) e g(x, y) tem um fator em comum em C[x], entaoRf,g(y) = 0. Assim, #{y ∈ C | f(x, y) e g(x, y) tem um fator em comum em C[x]} =#{y ∈ C | Rf,g(y) = 0}.

Agora como Rf,g(y) e um polinomio de C[y], segue que #{y ∈ C | Rf,g(y) = 0} =grau(Rf,g(y)), e finalmente, por 5.4, tem-se que grau(Rf,g(y)) ≤ nm, pois grau ai(y) ≤ ie grau bj(y) ≤ j.

Portanto #(V (f) ∩ V (g)) ≤ nm.

Observacao 5.6 Na forma mais geral do Teorema de Bezout, temos a hipotese de que ascurvas V (f) e V (g) nao podem ter componentes irredutıveis em comum. Ja nessa versaomais fraca, a hipotese era que f(x, y) e g(x, y) nao possuem fator irredutıvel em comum.Essas condicoes sao equivalentes, e e o que mostraremos a seguir.

Proposicao 5.7 A condicao de que f(x, y) e g(x, y) nao tem fatores irredutıveis emcomum e equivalente a condicao de que V (f) e V (g) nao tem componentes irredutıveisem comum.

Demonstracao. De fato, suponha que V (f) e V (g) tenha a componente V (h) em co-mum. Assim, h e irredutıvel, pois caso contrario, h = k.l e entao V (h) = V (k) ∪ V (l),i.e. V (h) seria redutıvel. Agora como V (h) ⊂ V (f) e V (h) ⊂ V (g), temos pelo Lema deStudy que h | f e h | g, ou seja, h e fator irredutıvel comum. A demonstracao e analogaquando V (f) e V (g) possuem mais de uma componente em comum.

Reciprocamente, suponha que f e g possuam fator irredutıvel em comum, a saber h.Assim, f = k.h e g = l.h, consequentemente, V (f) = V (k) ∪ V (h) e V (g) = V (l) ∪ V (h),ou seja, V (f) e V (g) possui componente irredutıvel em comum. Analogo para quando fe g possuem mais de um fator irredutıvel em comum.

A versao mais geral deste resultado e dada considerando-se ao inves do plano afimC

2 o plano projetivo P2(C). Nesse “novo plano” da-se sentido aos pontos no infinito, e e

possıvel atribuir multiplicidades de intersecao de maneira que o numero total de pontoscomuns as duas curvas, contados com multiplicidade, seja igual ao produto dos grausdessas curvas, o que em essencia e o enunciado do Teorema de Bezout.

Referencias

[1] Araujo, C., Introducao as curvas algebricas planas, Notas da Jornada de IniciacaoCientıfica, IMPA (2006)

[2] Fischer, G., Plane algebraic curves, AMS (2001).

[3] Garcia, A. & Lequain, Y., Algebra um curso de introducao, IMPA (1988)

[4] Vainsencher, I., Introducao as curvas algebricas planas, Colecao Matematica Univer-sitaria, IMPA (2005).

Page 269: Numero 09 - Outubro de 2007

FAMAT em Revista

Revista Científica Eletrônica daFaculdade de Matemática - FAMAT

Universidade Federal de Uberlândia - UFU - MG

Número 09 - Outrubro de 2007www.famat.ufu.br

Problemas e Soluções

���

Page 270: Numero 09 - Outubro de 2007

Comitê Editorial da Seção Problemas e Soluções

do Número 09 da FAMAT EM REVISTA:

Luiz Alberto Duran Salomão (coordenador da seção) Márcio José Horta Dantas

Marcos Antônio da Câmara

Page 271: Numero 09 - Outubro de 2007

Problemas Propostos

33. Demonstre que a soma dos cubos de três números inteiros consecutivos é divisível por 9.

34. Em um tetraedro regular tomam-se seções paralelas a duas de suas arestas que não se intersectam. Determine a seção de área máxima.

35. A função xxf cos)( , definida para 0x , é periódica? Justifique sua resposta.

36. De quantas maneiras 2n, sendo n um natural, pode ser expresso como a soma de quatro quadrados de números naturais? Justifique sua resposta.

Page 272: Numero 09 - Outubro de 2007

Resoluções dos problemas da revista número 8

29. Em um momento inicial, duas velas tinham a mesma altura h, encontrando-se, uma da outra, a uma distância a. A distância entre cada uma das velas e a parede mais próxima é também igual a a. Com que velocidades movem-se as sombras das velas nas paredes, se uma vela queima durante o tempo t1 e a outra durante o tempo t2?

Resolução:

Se t1 = t2, obviamente as duas sombras movem-se à mesma velocidade (que é a velocidade

com que as velas diminuem, ou seja, 21 th

th ). Sem perda de generalidade, admitamos que t1

< t2. Na figura acima, x e y representam os comprimentos das velas 1 e 2, respectivamente, em um tempo t, enquanto r e s são os comprimentos das suas respectivas sombras, no mesmo

instante. Assim, 1th

dtdx e

2th

dtdy .

Da figura acima, temos que a

xya

rx , o que acarreta yxr 2 . Portanto,

21

2th

th

dtdr é a velocidade com que se move a sombra da vela 1.

Ainda da figura acima, temos que axs

arx

2, o que acarreta rxs 23 . Portanto,

211

223th

th

th

dtds =

21

2th

th é a velocidade com que se move a sombra 2.

Page 273: Numero 09 - Outubro de 2007

30. Dado um pentágono convexo, mostre que é possível escolher três de suas diagonais de modo que com elas se possa construir um triângulo.

Resolução: Se ABCDE é um pentágono convexo, como na figura abaixo, seja BE a sua diagonal de comprimento máximo.

Pela desigualdade triangular, temos que EP + PB > EB e PD + PC > CD. Daí, somando membro a membro as duas desigualdades acima, temos que (BP + PD) + (EP + PC) > BE + CD o que acarreta BD + CE > BE + CD > BE. Daí, temos que é possível construir-se um triângulo com os segmentos BDBE, e CE , diagonais do pentágono convexo ABCDE.

31. Seja W um conjunto de pontos do plano. Supondo que todo ponto de W é ponto médio de um segmento que tem suas extremidades em W, demonstre que W é infinito.

Resolução: Suponha que W seja finito. Assim, existem dois pontos A e B em W de modo que a distância AB = m seja máxima dentre todas as distâncias entre pares de pontos de W. Por hipótese, B é ponto médio de algum segmento CD , sendo C e D pontos de W.

Na figura acima, A’ representa o simétrico de A com relação ao ponto B. Como os triângulos CBA e DBA’ são congruentes (caso LAL), temos que AC = A’D. Agora, pela desigualdade triangular (no triângulo AA’D), temos que AC + AD = A’D + AD > AA’ = m + m. Daí, pelo menos uma das parcelas AC ou AD tem que superar m. Isto contradiz a maximalidade de m.

Page 274: Numero 09 - Outubro de 2007

32. Seja q um número natural maior do que 1. Se m e n são números inteiros positivos, demonstre que qm-1 é divisor de qn-1 se, e somente se, m é divisor de n.

Resolução: Inicialmente, suponha que m seja divisor de n. Assim, existe um inteiro positivo ktal que n = km. Portanto,

11111 21 mkmkmmkmmkn qqqqqqq , ou seja, qm-1 é divisor de qn-1.Reciprocamente, admita agora que qm-1 seja divisor de qn-1. Pelo algoritmo da divisão, existem e são únicos dois inteiros k e r de modo que rkmn , onde mr0 . Além disso, pela hipótese, k1 . Suponha, por absurdo, que r0 . Como

111 122121 qqqqqqqqqqq rkmnmnmnmmnn

segue que 11

1111

11

21

21

21

21

qqqqqq

qqqqqqqq

qq

mm

nn

mm

nn

m

n

11

21

12

qqqqqqqq mm

rmknmnmn não é um inteiro, uma vez que r < m e,

por conseguinte, 0 <1

121

1

qqqqq

mm

r

< 1. Temos, então, uma contradição com a

hipótese.

Page 275: Numero 09 - Outubro de 2007

A seguir, apresentamos uma segunda resolução para o problema de número 32. Essa resolução é uma adaptação da que nos foi enviada pelo leitor Rafael Alves Figueiredo, discente do 6o período do Curso de Matemática da Universidade Federal de Uberlândia. Os editores da Revista agradecem ao Rafael por sua contribuição.

32. Seja q um número natural maior do que 1. Se m e n são números inteiros positivos, demonstre que é divisor de se, e somente se, m é divisor de n.1mq 1nq

Resolução:Pelo Algoritmo da Divisão, existem e são únicos dois inteiros q e r de modo que

n=qm + r, sendo . Agora, por hipótese, é divisor de e, daí, mr0 1mq 1nq1mod1 mn qq . Como, obviamente, 1mod1 mm qq , segue que 1mod1 mmq qq . Assim, temos

.1mod1 mnmqmqnmqnr qqqqqqConseqüentemente, é divisor de Porém, isso só pode ocorrer se r =0,isto é, se m for divisor de n.

1mq .1rq

Por hipótese, m é um divisor de n. Assim, existe um inteiro positivo k tal que n = km. Portanto,

11111 21 mkmkmmkmmkn qqqqqqq , ou seja, qm-1 é divisor de qn-1.

nmqq nm |1|1 .

Page 276: Numero 09 - Outubro de 2007
Page 277: Numero 09 - Outubro de 2007

FAMAT em Revista

Revista Científica Eletrônica daFaculdade de Matemática - FAMAT

Universidade Federal de Uberlândia - UFU - MG

Número 09 - Outubro de 2007www.famat.ufu.br

Eventos

� !

Page 278: Numero 09 - Outubro de 2007

Comitê Editorial da Seção Eventos

do Número 09 da FAMAT EM REVISTA:

Maria Luiza Maes (coordenadora da seção) Marcos Antônio da Câmara Márcio José Horta Dantas

Page 279: Numero 09 - Outubro de 2007

EVENTOS

VII Semana da Matemática da Universidade Federal de Uberlândia - VII SEMAT/UFU

Período: 27 a 30 de novembro de 2007

Local de realização: Universidade Federal de Uberlândia - UFU

Campus Santa Mônica, Anfiteatro do Bloco 3Q - Uberlândia - MG

Organização: Faculdade de Matemática da UFU - FAMAT

Diretório Acadêmico da FAMAT/UFU - DAMAT

Programa de Educação Tutorial da FAMAT/UFU - PETMAT

Programação

Primeiro Dia 27/11/07 - terça-feira

07h30min - 08h30min Entrega de Material aos participantes

08h30min - 09h00min Abertura da VII SEMAT

09h00min - 09h30min Coffee-Break

09h30min - 10h30min Palestra: “Euler, sua obra e seu tempo” - Prof. Dr. Geraldo Severo de

Souza Ávila – Membro da Academia Brasileira de Ciências

10h40min - 12h00min Mini-curso: “Aplicações da decomposição singular de matrizes no

melhoramento genético de plantas” - Prof. Dr. João Batista Duarte - EA/UFG - Goiânia - GO

12h00min - 14h00min Almoço

14h00min - 15h00min Mesa Redonda: “FAMAT em ações extra-curriculares” - Coordenação:

Profa. Dra. Sezimária de Fátima Pereira Saramago - FAMAT/UFU - Uberlândia - MG

15h00min - 16h20min Mini-curso: “Introdução à teoria dos jogos” - Prof. Dr. Marcos

Antônio da Câmara e grupo PETMAT - FAMAT/UFU - Uberlândia - MG

16h20min – 16h50min Coffee-Break

16h50min - 18h10min Mini-curso: “Polinômios sobre corpos p-ádicos: uma breve

introdução” - Prof. Dr. Hemar Teixeira Godinho - IE/UnB - Brasília - DF

Page 280: Numero 09 - Outubro de 2007

Segundo Dia 28/11/07 - quarta-feira

08h00min - 09h20min Mini-curso: “Episódios recentes da geometria euclidiana” - Prof. Ms.

Sérgio Alves - IME/USP - São Paulo - SP

09h20min - 09h50min Coffee-Break

09h50min - 10h40min Sessão de Comunicações

10h40min - 12h00min Mini-curso: “Aplicações da decomposição singular de matrizes no

melhoramento genético de plantas” - Prof. Dr. João Batista Duarte - EA/UFG - Goiânia - GO

12h00min - 14h00min Almoço

14h00min - 15h20min Mini-curso: “Triangulações regulares: aspectos teóricos e

computacionais” - Prof. Dr. Luis Gustavo Nonato - ICMC/USP - São Carlos - SP

15h20min - 16h20min Palestra: “Introdução à modelagem de risco em finanças” - Prof. Dr.

Francisco Louzada Neto - CCET/UFScar - São Carlos - SP

16h20min - 16h50min Coffee-Break

16h50min - 18h10min Mini-curso: “Polinômios sobre corpos p-ádicos: uma breve

introdução” - Prof. Dr. Hemar Teixeira Godinho - IE/UnB - Brasília - DF

Terceiro Dia 29/11/07 - quinta-feira

08h00min - 09h20min Mini-curso: “Episódios recentes da geometria euclidiana” - Prof. Ms.

Sérgio Alves - IME/USP - São Paulo - SP

09h20min - 09h50min Coffee-Break

09h50min - 10h40min Sessão de Comunicações

10h40min - 12h00min Mini-curso: “Aplicações da decomposição singular de matrizes no

melhoramento genético de plantas” - Prof. Dr. João Batista Duarte - EA/UFG - Goiânia - GO

12h00min - 14h00min Almoço

14h00min - 15h20min Mini-curso: “Triangulações regulares: aspectos teóricos e

computacionais” - Prof. Dr. Luis Gustavo Nonato - ICMC/USP - São Carlos - SP

15h20min - 16h20min Palestra: “A gloriosa história da geometria” - Prof. Dr. Cláudio

Gorodski - IME/USP - São Paulo - SP

16h20min - 16h50min Coffee-Break

16h50min - 18h10min Mini-curso: “Polinômios sobre corpos p-ádicos: uma breve

introdução” - Prof. Dr. Hemar Teixeira Godinho - IE/UnB - Brasília – DF

Page 281: Numero 09 - Outubro de 2007

Quarto Dia 30/11/07 - sexta-feira

08h00min - 09h20min Mini-curso: “Episódios recentes da geometria euclidiana” - Prof. Ms.

Sérgio Alves - IME/USP - São Paulo - SP

09h20min - 09h50min Coffee-Break

09h50min - 11h20min Sessão de Comunicações

11h20min - 12h00min Mini-curso: “Introdução à teoria dos jogos” - Prof. Dr. Marcos

Antônio da Câmara e grupo PETMAT - FAMAT/UFU - Uberlândia - MG

12h00min - 14h00min Almoço

14h00min - 15h20min Mini-curso: “Triangulações regulares: aspectos teóricos e

computacionais” - Prof. Dr. Luis Gustavo Nonato - ICMC/USP - São Carlos - SP

15h20min - 16h20min Palestra: “Possibilidades do software CABRI 3D” - Prof. Dr. Vincenzo

Bongiovanni - CCE/PUC - São Paulo - SP

16h20min - 16h50min Coffee-Break

16h50min - 18h00min Encerramento e Momento Musical

Comissão Organizadora

Profa. Dra. Dulce Mary de Almeida - FAMAT - UFU (coordenadora)

Prof. Dr. Cícero Fernandes de Carvalho - FAMAT - UFU

Prof. Dr. Luís Antˆonio Benedetti - FAMAT - UFU

Prof. Dr. Marcos Antônio da Câmara - FAMAT - UFU

Profa. Dra. Maria Teresa Menezes Freitas - FAMAT - UFU

Prof. Dr. Rogério de Melo Costa Pinto - FAMAT - UFU

Prof. Dr. Walter dos Santos Motta Junior - FAMAT - UFU

Mariana Fernandes dos Santos Villela - discente do Grupo PET/SESu da FAMAT - UFU

Virgíınia Helena Ribeiro Miranda - discente do Diretório Acadêmico da FAMAT - UFU

Informações: http://www.famat.ufu.br/semat (site da VII SEMAT em construção)

V Encontro Sul Fluminense de Educação Matemática – ESFEM

Período: 5 e 6 de outubro de 2007

Informações: http://www.uss.br/web/page/Venc_matematica.asp

Page 282: Numero 09 - Outubro de 2007

IV Congresso Internacional de Ensino da Matemática

Período: 25, 26 e 27 de outubro de 2007

Informações: http://www.ulbra.br/ciem07/

Semana da Matemática e Física 2007

Período: 22 a 26 de outubro de 2007

Informações: http://www.unitau.br/eventos/semana-da-matematica-e-fisica-2007

VI Semana de Matemática da UESC

Período: 1 a 5 de outubro de 2007

Informações: http://www.uesc.br/eventos/visemana/

Congresso da Sociedade Latino Americana de Biologia Matemática

Congreso de la Sociedad Latinoamericana de Biología Matemática

XIV CLAB - IX ELAEM

Período: 13 a 16 de novembro de 2007

Informações: http://www.ime.unicamp.br/~biomat/clab2007.htm

4ª Semana Acadêmica – UFU na Contemporaneidade

Informações: http://www.semanaacademica.ufu.br/

15º Simpósio Internacional de Iniciação Científica da Universidade de São Paulo

Período: Humanas 21, 22 e 23/novembro - USP Leste

Biológicas 22 e 23/novembro - Ribeirão Preto

Engenharias e Exatas 26, 27 e 28/novembro - São Carlos

Agropecuárias 28 e 29/novembro – Pirassununga

Informações: http://www.usp.br/siicusp/

XI ENCONTRO NACIONAL DE EDITORES CIENTÍFICOS – XI ENEC

Período: 02 a 06 de outubro de 2007

Informações: http://www.lncc.br/abec/XIEnec.php

Page 283: Numero 09 - Outubro de 2007

V Conferência Nacional sobre Modelagem na Educação Matemática

Período: 08, 09 e 10 de novembro de 2007

Informações: http://www.iceb.ufop.br/niepem/eventos/indexcnmem.html

CBEm3 – Terceiro Congresso Brasileiro de Etnomatemática

Período: 26 a 29 de março de 2008

Informações: http://www.uff.br/cbem3/

XIV Encontro Nacional de Didática e Prática de Ensino

Período: 27 a 30 de abril de 2008

Informações: http://www.pucrs.br/eventos/endipe/

Symposium on the Occasion of the 100th Anniversary of ICMI

Período: 05 a 08 de março de 2008

Informações: http://www.unige.ch/math/EnsMath/Rome2008/

10º Simpósio de Educación Matemática

Período: 12 a 15 de maio de 2008

Informações: http://www.edumat.org.ar/

ICMI Study and IASE Round Table Conference

Período: 30 de junho a 04 de julho de 2008

Informações: http://www.ugr.es/~icmi/iase_study/

ICME 11 International Congress on Mathematical Education

Período: 06 a 13 de julho de 2008

Informações: http://icme11.org/

Page 284: Numero 09 - Outubro de 2007
Page 285: Numero 09 - Outubro de 2007

FAMAT em Revista

Revista Científica Eletrônica daFaculdade de Matemática - FAMAT

Universidade Federal de Uberlândia - UFU - MG

Número 09 - Outubro de 2007www.famat.ufu.br

Reflexões Sobre oCurso de Matemática

"#$

%

Page 286: Numero 09 - Outubro de 2007

Comitê Editorial da Seção Reflexões sobre o Curso de Matemática do Número 09 da FAMAT EM REVISTA:

Márcio José Horta Dantas (coordenador da seção) Marcos Antônio da Câmara

Valdair Bonfim

Page 287: Numero 09 - Outubro de 2007

A BELEZA DA MATEMÁTICA I

Luís Antonio Benedetti

O presente texto não é propriamente uma reflexão sobre o curso de matemática, mas

um ensaio sobre a matemática e seu conteúdo, pequena contribuição que será enriquecida no

próximo número da revista da FAMAT.

A matemática pode ser definida como uma ciência que tem por finalidade o estudo das

propriedades dos seres abstratos, isto é, dos números, das figuras geométricas e das funções.

Uma breve análise histórica sobre a origem e evolução da matemática mostra, de forma cabal,

que desde a antiguidade o homem utiliza os seus princípios para facilitar a vida dos

indivíduos e para organizar a sociedade. Assim, egípcios utilizaram a matemática para a

construção das pirâmides, diques de irrigação e para o estudo da astronomia. Os gregos

também utilizaram a matemática para fundamentar o pensamento filosófico e para criar novas

realidades matemáticas. Pitágoras, um grande filósofo de sua época e que tem seu nome

ligado a um dos mais significativos teoremas pode ser citado como exemplo.

Qualquer que seja o período histórico analisado é possível constatar que, de uma

maneira ou de outra, a matemática é parte integrante e essencial da cultura. Por óbvio, não foi

construída de um momento para o outro, a sua edificação se deu lentamente, através da

criatividade, da imaginação e da busca contínua pelo rigor. O seu método axiomático –

conjunto de axiomas, definições, teoremas, proposições, organizados de forma perfeitamente

racional e lógica, pode ser considerado a maior conquista do conhecimento e é base para todo

desenvolvimento científico da humanidade, ele se constitui de uma infinidade de estruturas

esteticamente belas, apreciadas apenas pelas mentes que são treinadas a percebê-las.

Na esteira de sua evolução, a matemática tornou-se uma ciência estruturada e

organizada, presente em áreas como a física, medicina, música, engenharia, arquitetura,

pintura, etc. Para qualquer lado que se olhe do conhecimento científico e mesmo artístico é

possível vislumbrar beleza matemática. Mas, a despeito de sua relevância, ela não está pronta

ou acabada, uma vez que a todo momento surgem novas fronteiras para a consolidação desta

ciência, sempre em contínua evolução. Eis que conceber uma sociedade sem a utilização da

matemática e o mesmo que conceber uma sociedade sem a música e as artes em geral, por

exemplo.

Page 288: Numero 09 - Outubro de 2007

A possibilidade do surgimento de novas realidades matemáticas confere a esta ciência

um caráter peculiar e nem sempre visível a todos, sobretudo para os não matemáticos: a

beleza da matemática. Em verdade, a sua beleza encontra-se oculta e implícita, visível apenas

para aqueles que conseguem chegar ao âmago de suas teorias e métodos.

Existem vários exemplos que confirmam o quanto essa ciência vem revestida de

beleza. Este é o caso, por exemplo, da Análise, Álgebra e Geometria, ramos da matemática

que se complementam e se combinam de forma complexa e harmoniosa.

Os exemplos não se esgotam ai. Um teorema possui uma beleza de tal maneira

perceptível para um matemático que, em muitos casos, podem colocar em segundo plano a

sua aplicação prática. Para o não matemático, a beleza de um teorema se encontra oculta,

embora a sua importância e aplicabilidade sejam levadas em consideração.

Há, na atualidade, um amplo arcabouço teórico sobre a matemática e as suas

aplicações. Entretanto, estudos e pesquisas que avaliam e comprovam a beleza da matemática

ainda são escassos. Considerando o fato que a matemática foi protagonista na história da

cultura, torna-se necessário aprofundar mais neste assunto. Para tanto, recorrer aos escritos de

Eudoxo, Descartes, Gauss, Hilbert, Poincaré, dentre outros pensadores que fizeram história

em relação à matemática, pode ser o ponto de partida para mostrar como essa ciência é bela,

sobretudo no seu modo de criação.

Aspecto interessante em relação à discussão sobre a beleza da matemática é que

alguns filósofos gregos da antiguidade já tinham essa percepção. Nesta linha de raciocínio,

Platão abriu caminho para que a beleza da matemática pudesse ser reconhecida no

pensamento filosófico, ao unir a verdade, o bem e a beleza. A partir deste momento a

matemática passou a ser analisada sob um enfoque diferente, ou seja, a ciência e a arte

(estética) passaram a trilhar um caminho semelhante. Evidentemente, no período em análise

nem todos tinham essa visão da matemática. Aristipo, por diversas vezes, negou a relação

existente entre a matemática e o belo e Aristóteles, ferrenho defensor do belo tratou de refutar

as observações deste filósofo com frases sobre a beleza que se eternizaram no tempo.

Mas o que vem a ser precisamente a beleza na matemática? Para responder tal

questionamento é preciso mencionar que a matemática sempre foi compreendida como algo

preciso, rigoroso, exato, coerente e útil. Essa concepção não está errada, caso contrário ela

não seria o alicerce de muitas ciências. Mas se a matemática deve ser rigorosa, também deve

se primar pela elegância, dado que nada impede que uma demonstração seja, ao mesmo

Page 289: Numero 09 - Outubro de 2007

tempo, exata e bela. Em suma, a matemática vai além da exatidão, pois ela também combina

leveza, elegância, naturalidade e a inspiração de novos valores.

Até o advento da ciência contemporânea, a beleza da matemática vinha associada à

racionalidade. Hoje, a sobriedade, a simplicidade na resolução de uma equação, ainda que

essa seja complexa, são elementos que caracterizam a beleza desta ciência. Há quem defenda

o ponto de vista que a beleza da matemática reside na simplicidade e não em sua

complexidade propriamente dita.

Por todos esses fatores, os matemáticos devem procurar passar para as gerações

futuras não apenas os conceitos e princípios desta ciência, que teve um papel fundamental no

desenvolvimento da tecnologia e em diversos setores do saber científico e artístico. Também

devem chamar a atenção para a beleza da matemática, seus adornos e todos os demais

elementos, que conferem a ela uma particularidade única em termos de beleza. Somente assim

a beleza da matemática tornar-se-á explicita até mesmos para os não matemáticos.

Page 290: Numero 09 - Outubro de 2007
Page 291: Numero 09 - Outubro de 2007

FAMAT em Revista

Revista Científica Eletrônica daFaculdade de Matemática - FAMAT

Universidade Federal de Uberlândia - UFU - MG

Número 09 - Outubro de 2007www.famat.ufu.br

Em Sala de Aula

&'�

Page 292: Numero 09 - Outubro de 2007

Comitê Editorial da Seção Em Sala de Aula

do Número 09 da FAMAT EM REVISTA:

Márcio José Horta Dantas (coordenador da seção) Marcos Antônio da Câmara

Page 293: Numero 09 - Outubro de 2007

Índice de Trabalhos

Ornamentos: uma aplicação da modelagem matemática para o ensino 295 Edinei Leandro dos Reis, Érika Cristina de Freitas e Rosana S. da M. Jafelice

Modelagem no Ensino Médio: Cubagem de Madeira 311Lóren Grace Kellen Maia Amorim, Mariana Martins Pereira e Rosana S. da M. Jafelice

Diagnóstico Médico Fuzzy de Doenças Infantis 329 Mariana Fernandes dos Santos Villela, Patrícia Borges dos Santos e Rosana S. da M. Jafelice

Fluxo Sanguíneo: Uma Aplicação da Integral de Riemann 347 Mariana Fernandes dos Santos Villela, Patrícia Borges dos Santos e Rosana S. da M. Jafelice

O Uso de Modelagem Matemática no Cálculo do Volume de uma Maçã 365 Alessandra Ribeiro da Silva, Carlos Henrique Tognon, Milena Almeida Leite Brandão e Rosana S. da M. Jafelice

O uso de técnicas de otimização para determinar uma dieta alimentar saudável e econômica 385 Alessandra Ribeiro da Silva, Carlos Henrique Tognon, Milena Almeida Leite Brandão e Rosana S. da M. Jafelice

Implicações da atividade de ensino na formação inicial de professores 401 Lóren Grace Kellen Maia Amorim, Mariana Martins Pereira, Fabiana Fiorezi de Marco Matos

Matemática E Xadrez: possibilidades no ensino fundamental 415 Rafael de Souza Duarte e Maria Teresa Menezes Freitas

Tecnologia de informação e comunicação no ensino de cálculo 431 Patrícia Oliveira Costa e Arlindo José de Souza Júnior

Aplicação simultânea de dois métodos de classificação étnico-racial 441 Aurélia Aparecida de Araújo Rodrigues, Corina Ilda Silva Ferreira, Willian Araújo Moura e Gabriela Vieira Lopes

O Estudo Matemático do Comportamento das Abelhas 447 Eduardo Henrique Siqueira Molinero, Lucas Dias Marques e Rosana S. da M. Jafelice

Page 294: Numero 09 - Outubro de 2007
Page 295: Numero 09 - Outubro de 2007

Ornamentos: uma aplicação da modelagem matemática para o ensino

Universidade Federal de UberlândiaFaculdade de Matemática

Edinei Leandro dos [email protected]

Érika Cristina de [email protected]

Rosana S. da M. [email protected]

Introdução

Desde a antiguidade vários povos com culturas das mais diversas, utilizavam figuras geométricas como elementos decorativos, nas construções arquitetônicas, nas manifestações artísticas e até mesmo nos seus utensílios domésticos [1].

Com o desenvolvimento das culturas verifica-se que a disposição destas figuras geométricas torna-se mais trabalhada e complexa. Podemos observar mais tarde que algumas civilizações desenvolveram um tipo diferente de ornamentos1, utilizando para isso repetições em um plano de uma mesma figura geométrica, de forma que estas repetições preenchessem todo o plano. Se tentarmos cobrir totalmente um plano com figuras que não se sobrepõem, o resultado é um mosaico.

Existem somente três formas de se obter mosaico com polígonos regulares de um mesmo tipo (triângulos eqüiláteros, quadriláteros e hexágonos regulares), mas se admitirmos outras condições (a combinação de polígonos, por exemplo), surge novas possibilidades. Ao longo dos tempos, diferentes culturas têm estudado os mosaicos por motivos do tipo intelectual (na Grécia), decorativos (em Roma) e religioso-filosóficos (no Islão) [2].

Mosaico artístico (Figura 1) é um embutido de pequenas pedras ou de outras peças (pequenos pedaços de vidro, mármore ou cerâmica) formando determinado desenho [3]. Neste trabalho vamos dar ênfase no estudo dos mosaicos com padrões geométricos.

Figura 1 – Calçadão de Copacabana [4]. Figura 2 – Pão de Açúcar (Rio Mosaico 2006, 2 ed.) [5].

Uma referência mundial de utilização dos mosaicos é o palácio de Alhambra. A Alhambra (Castelo Vermelho) (em Árabe, é um antigo palácio e complexo de

1 O ornamento é um elemento acessório, não fundamental, em uma composição artística, em especial na composição arquitetônica e no design.

Page 296: Numero 09 - Outubro de 2007

fortificações dos monarcas islâmicos de Granada, no sul de Espanha, ocupando o alto de uma colina arborizada, a sudeste da cidade. O nome Alhambra deriva provavelmente da cor dos tijolos do muro, secos ao sol e feitos de argila e gravilha de que são feitas as muralhas exteriores. Segundo outros autores, o adjetivo relembra o clarão avermelhado das tochas que iluminaram os trabalhos de construção que se prolongavam ininterruptamente, noite adentro, durante anos; outros associam o nome ao fundador, Mahomed Ibn-al-Ahmar; outros, ainda derivam-no da palavra árabe Dar al Amra, Casa do Senhor. O palácio foi construído principalmente entre 1248 e 1354, nos reinados de Ibn-al-Ahmar e seus sucessores; os nomes dos principais artistas e arquitetos são desconhecidos ou de conhecimento duvidoso [6].

Figura 3 – Vista externa do Palácio de Alhambra [7].

Metodologia

Para realizar esta pesquisa sobre “Ornamentos” buscamos informações sobre mosaicos, rosetas e faixas em diversos sites, artigos e em vídeos educacionais.

Utilizamos também um vídeo educacional do MEC para elaborar uma atividade pedagógica para utilização em sala de aula.

Objetivos

Mostrar os grupos de simetria no plano (faixa, roseta e mosaico) e algumas aplicações, analisar estruturas geométricas dos grupos de simetria e desenvolver uma atividade pedagógica para utilização na sala de aula.

Referencial Teórico

Para analisarmos os ornamentos, necessitamos de algumas definições de movimentos que podem ser realizados no plano:

Translações: movimento de certa distância, em uma direção e sentido determinados. A direção é determinada por um vetor [8]. Ver Figura 4.

Page 297: Numero 09 - Outubro de 2007

Figura 4 – Translação de um objeto [8].

Rotações: giros em volta de um determinado ponto e de certa amplitude angular. A rotação de 180º é conhecido também como simetria central [8]. Ver Figura 5.

Figura 5 – Rotação de um objeto [8].

Reflexões: caracteriza-se por ter um eixo que atua como se fosse um espelho, onde a parte considerada é refletida, mantendo-se a mesma distância em relação ao eixo [8]. Ver Figura 6.

Figura 6 – Reflexão de um objeto [8].

Translações refletidas ou glissoreflexões: resulta da composição de uma reflexão euma translação na direção da reflexão [8]. Ver Figura 7.

Figura 7 – Translação refletida de um objeto [8].

Page 298: Numero 09 - Outubro de 2007

Quando o mosaico é gerado por rotações e translações refletidas, podemos dizer que ele foi gerado por uma isometria inversa.

Para obtermos um motivo ou ornamento, aplicamos uma ou mais propriedades de isometria em uma figura ou elemento gerador (menor parte de uma forma). Na Matemática consideramos três tipos de ornamentos: faixa, roseta e mosaico [9].

A faixa é um ornamento ilimitado, composto entre duas retas paralelas. A simetria fundamental para sua composição é a translação. A combinação com as demais simetrias permite criar sete tipos de faixas [9]. Segue a listagem dos tipos de faixas:

G1: translações;

G2: rotações de 180°;

G3: reflexões horizontais;

G4: reflexões verticais;

G5: rotações de 180° e reflexõeshorizontais;

G6: rotações de 180° e translações refletidas horizontais; e

G7: translações refletidas horizontais.

Figura 8 – Os sete tipos de faixas [10].

No Anexo I apresentamos o “Fluxograma de Washburn e Crowe para a classificação das faixas monocromáticos” [10].

A roseta é um ornamento limitado, composto em um círculo. A simetria fundamental para sua composição é a rotação. Entretanto, é possível fazer um outro tipo de roseta combinando a rotação e a reflexão [9]. A seguir temos um exemplo de roseta:

Figura 9 – Exemplo de roseta [11].

Em relação aos grupos de simetria para gerar mosaico, temos 17 possibilidades. Estes 17 grupos de simetria no plano podem ser classificados a partir do número de rotações que são realizadas para gerar o mosaico (Ordem 1, 2, 3, 4 ou 6). A seguir separamos os grupos de simetria em relação à sua ordem e mostramos como são gerados os 17 tipos de mosaicos no plano.

Page 299: Numero 09 - Outubro de 2007

Ordem 1: não são gerados por rotações (p1, cm, pm, pg)Ordem 2: rotações de 180º (p2, cmm, pmm, pgg, pmg).Ordem 3: rotações de 120º (p3m1, p31m, p3).Ordem 4: rotações de 90º (p4, p4m, p4g)Ordem 6: rotações de 60º (p6, p6m) [12].

A Notação Cristalográfica para os grupos de simetria utilizam símbolos para fazer a distinção entre grupos de mosaicos. As letras p ou c significam a célula primitiva ou central. O número depois de p é a maior ordem de rotação, por exemplo, se for 6, então é uma rotação que representa 1/6 de uma volta. A letra m é a reflexão perpendicular (espelho) do eixo x. A letra g é uma glissoreflexão. O eixo x é na verdade a borda vertical esquerda de uma célula. O número 1 não representa simetria perpendicular em relação ao eixo x, mas em relação adeterminado ângulo [13]. No Anexo II apresentamos o “Fluxograma para classificação dos padrões planos monocromáticos” [14].

P1: é o grupo mais simples. Ele é gerado apenas a partir de translações, não tendo isometrias inversas. A base geradora desse mosaico é um paralelogramo. Ver Figuras 10 e 11.

Figura 10 – Base geradora do mosaico tipo p1 [8]. Figura 11 – Exemplo de mosaico do tipo p1 [15].

Cm: é gerado a partir de isometrias inversas. É um dos dois grupos com base geradora sendoum losango. Ver Figuras 12 e 13.

Figura 12 - Base geradora do mosaico tipo cm [8]. Figura 13 – Exemplo de mosaico do tipo cm [15].

Pm: é gerado por translações e reflexões. Sua base geradora é o retângulo. Ver Figuras 14 e 15.

Figura 14 – Base geradora do mosaico tipo pm [8]. Figura 15 – Exemplo de mosaico do tipo pm [15].

Page 300: Numero 09 - Outubro de 2007

Pg: é gerado a partir de translações e translações refletidas. Não possui reflexão e sua base geradora é um retângulo. Ver Figuras 16 e 17.

Figura 16 – Base geradora do mosaico tipo pg [8]. Figura 17 – Exemplo de mosaico do tipo pg [15].

P2: é gerado a partir de translações e rotações de 180º. A base geradora é um paralelogramo. Ver Figuras 18 e 19.

Figura 18 – Base geradora do mosaico tipo p2 [8]. Figura 19 – Exemplo de mosaico do tipo p2 [15].

Cmm: é gerado a partir de isometrias inversas e rotações de 180º. É o outro grupo com base geradora sendo um losango. Ver Figuras 20 e 21.

Figura 20 – Base geradora do mosaico tipo cmm [8]. Figura 21 – Exemplo de mosaico do tipo cmm [15].

Pmm: é gerado a partir de reflexões e rotações de 180º. Sua base geradora é um retângulo Ver Figuras 22 e 23.

Figura 22 – Base geradora do mosaico tipo pmm [8]. Figura 23 – Exemplo de mosaico do tipo pmm [15].

Pmg: é gerado a partir de isometrias inversas e rotações de 180º. Sua base geradora é um retângulo. Ver Figuras 24 e 25.

Figura 24 – Base geradora do mosaico tipo pmg [8]. Figura 25 – Exemplo de mosaico do tipo pmg [15].

Page 301: Numero 09 - Outubro de 2007

Pgg: é gerado a partir de translações refletidas e rotações de 180º. Sua base é um retângulo. Ver Figuras 26 e 27.

Figura 26 – Base geradora do mosaico tipo pgg [8]. Figura 27 – Exemplo de mosaico do tipo pgg [15].

P3: é o grupo gerado com rotações de 120º. A base geradora é um paralelogramo. Ver Figuras 28 e 29.

Figura 28 – Base geradora do mosaico tipo p3 [8]. Figura 29 – Exemplo de mosaico do tipo p3 [15].

P3m1: é gerado por isometrias inversas e rotações de 120º. Possui simetrias em relação aos eixos que formam 60º passando pelos centros de rotação. Sua base geradora é um paralelogramo. Ver Figuras 30 e 31.

Figura 30 – Base geradora do mosaico tipo p3m1 [8]. Figura 31 – Exemplo de mosaico do tipo p3m1 [15].

P31m: é gerado por isometrias inversas e rotações de 120º. Possui simetrias em relação aos eixos que formam 60º, uns passam pelos centros de rotação e outros não. Sua base geradora é um paralelogramo. Ver Figuras 32 e 33.

Page 302: Numero 09 - Outubro de 2007

Figura 32 – Base geradora do mosaico tipo p31m [8]. Figura 33 – Exemplo de mosaico do tipo p31m [15].

P4: é gerado por translações e rotações de 90º. A base geradora é um quadrado. Ver Figuras 34 e 35.

Figura 34 – Base geradora do mosaico tipo p4 [8]. Figura 35 – Exemplo de mosaico do tipo p4 [15].

P4m: é gerado por isometrias inversas e rotações de 90º. Os eixos de simetria formam ângulos de 45º entre si e cortam o centro da rotação de 90º. A base geradora é um quadrado. Ver Figuras 36 e 37.

Figura 36 – Base geradora do mosaico tipo p4m [8]. Figura 37 – Exemplo de mosaico do tipo p4m [15].

P4g: é gerado por isometrias inversas rotações de 90º. Seus eixos de simetria são perpendiculares e não passam pelos centros de rotação. A base geradora é um quadrado. Ver Figuras 38 e 39.

Figura 38 – Base geradora do mosaico tipo p4g [8]. Figura 39 – Exemplo de mosaico do tipo p4g [15].

Page 303: Numero 09 - Outubro de 2007

P6: é gerado por translações e rotações de 60º. Sua base geradora é um paralelogramo. Ver Figuras 40 e 41.

Figura 40 – Base geradora do mosaico tipo p6 [8]. Figura 41 – Exemplo de mosaico do tipo p6 [15].

P6m: é gerado por isometrias inversas e rotações de 60º. Os centros das rotações de 60º são cortados por 6 eixos de simetria, formando ângulos de 30º. Ver Figuras 42 e 43.

Figura 42 – Base geradora do mosaico tipo p6m [8]. Figura 43 – Exemplo de mosaico do tipo p6m [15].

Curiosidades

Escher, arquiteto de outros mundos

Mauritus Cornelis Escher, nasceu em Leeuwarden na Holanda em 1898, faleceu em 1970 e dedicou toda a sua vida às artes gráficas. Na sua juventude não foi um aluno brilhante, nem sequer manifestava grande interesse pelos estudos, mas os seus pais conseguiram convencê-lo a ingressar na Escola de Belas Artes de Haarlem para estudar arquitetura. Foi lá que conheceu o seu mestre, um professor de Artes Gráficas judeu de origem portuguesa, chamado Jesserum de Mesquita.

Com o professor Mesquita, Escher aprendeu muito, conheceu as técnicas de desenho e deixou-se fascinar pela arte da gravura. Este fascínio foi tão forte que levou Mauritus a abandonar a Arquitetura e a seguir as Artes Gráficas. Quando terminou os seus estudos, Escher decide viajar, conhecer o mundo! Passou por Espanha, Itália e fixou-se em Roma, onde se dedicou ao trabalho Gráfico. Mais tarde, por razões políticas muda-se para a Suíça, posteriormente para a Bélgica e em 1941 regressa ao seu país natal.

Estas passagens por diferentes países, por diferentes culturas, inspiraram a mente de Escher, nomeadamente a passagem por Alhambra, em Granada, onde conheceu os azulejos mouros. Este contato com a arte árabe está na base do interesse e da paixão de Escher peladivisão regular do plano em figuras geométricas que se transfiguram, se repetem e refletem, pelas pavimentações. Porém, no preenchimento de superfícies, Escher substituía as figuras abstrato-geométricas, usadas pelos árabes, por figuras concretas, perceptíveis e existentes na

Page 304: Numero 09 - Outubro de 2007

natureza, como pássaros, peixes, pessoas, répteis, etc. Podemos observar isso nas Figuras 44 e 45.

Figura 44 – Ornamentos de Escher [16]. Figura 45 – Ornamentos de Escher [16].

Escher, sem conhecimento matemático prévio, mas através do estudo sistemático e da experimentação, descobre todos os diferentes grupos de combinações isométricas que deixam um determinado ornamento invariante. A reflexão é brilhantemente utilizada na xilografia "Day and Night", uma das gravuras mais emblemáticas da carreira de Escher(Figura 46).

Figura 46 - “Day and Night” - Xilogravura de 1938 [16].

Se nos fixarmos no losango branco central a baixo, automaticamente somos levados até ao céu, e o que de início era uma simples figura geométrica rapidamente se transforma num pássaro. Os pássaros brancos voam para a direita em direção à noite que recobre uma pequena aldeia holandesa à beira de um rio. Os pássaros negros, por sua vez, sobrevoam uma imagem iluminada pelo sol, que é exatamente a imagem refletida da paisagem noturna.

Aos poucos, Escher, vai sendo cada vez mais ousado e além da “dança” com a geometria, vai também ao encontro do infinito. A divisão regular da superfície aparece misturada a formas tridimensionais, geralmente num ciclo sem fim, onde uma fase se dilui na outra. A litografia "Reptiles" é um bom exemplo disso (Figura 47).

Page 305: Numero 09 - Outubro de 2007

Figura 47 - "Reptiles" - Litografia de 1943 [16].

Entre toda a espécie de objetos está o seu próprio caderno de esboços colocado sobre uma mesa, no qual se vê um desenho: um mosaico de figuras em forma de répteis num contraste de três cores. Subitamente um dos répteis ali desenhados, sai do papel e dá vida a um ciclo tridimensional retornando depois à bidimensionalidade do caderno de esboços.

Fascinado pelos paradoxos visuais, Escher chegou à criação de mundos impossíveis. Nesses trabalhos, o artista joga com as leis da perspectiva para produzir surpreendentes efeitos de ilusão de óptica. Nos seus desenhos somos levados a novos universos, a lugaresverdadeiramente misteriosos! Para Escher a realidade pouco interessa, antes pelo contrário, prefere criar mundos impossíveis que apenas pareçam reais. Pó isso se tornou uma espécie de mágico das artes gráficas.

Escher suscitou a atenção por parte de muitos matemáticos, cientistas e cristalógrafos. O mais curioso é que Escher não tinha uma formação específica nestas áreas, mas elas aparecem nas suas criações! Cada vez mais assediado pelos matemáticos, Escher acabou muitas vezes por se inspirar em suas novas descobertas. Por exemplo, "Waterfall" foi baseada na figura do tribar, uma construção geometricamente impossível, criada pelo matemático Penrose.

Figura 48 - "Belvedere" - Litografia de 1958 [16].

O rapaz que está sentado no banco tem em suas mãos um objeto com a forma de cubo que, visto de cima, representa uma realidade diferente da de quando visto por baixo. Ele observa pensativamente o objeto impossível e não parece aperceber-se de que o belvedere (Figura 48), atrás das suas costas, é construído desta forma. No piso inferior, no interior da casa, está encostada uma escada pela qual sobem duas pessoas. Mas chegadas a um piso acima, estão de novo ao ar livre e têm de voltar a entrar no edifício [16].

Page 306: Numero 09 - Outubro de 2007

São todos estes “condimentos” matemáticos aliados à mente artística de Escher que resultam num trabalho tão original e extraordinário. Escher foi reconhecido pelo mundo, pelos seus desenhos de ilusões espaciais, de construções impossíveis, onde a geometria se transforma em arte ou a arte em geometria [16].

"Apesar de não possuir qualquer conhecimento ou treino nas ciências exatas, sinto muitas vezes que tenho mais em comum com os matemáticos do que com os meus colegas artistas".

M. C. Escher [16].

Aplicações em sala de aula

Se observarmos os caminhos que nos levam aos mais variados lugares, perceberemos a presença de ornamentos em diversos objetos. Portões, muros, calçadas, casas entre outros. A partir dessa observação, podemos introduzir o tema “Ornamentos” nas aulas de Geometria para alunos de diferentes níveis de instrução.

Comecemos pelo nível Fundamental de Ensino. Para alunos do Ensino Fundamental, podemos utilizar os mosaicos para ensinar vários conceitos sobre Geometria. Podemos começar mostrando os conceitos mais simples, que podem formar malhas. As figurasgeométricas (triângulos, quadriláteros, hexágonos) e suas propriedades (arestas, vértices, pontos médios, diagonais, alturas, medianas, mediatrizes, ângulos, entre outras), as isometrias diretas e inversas (translação, reflexão, rotação e glissoreflexão) são conceitos geométricos importantes que podem ser trabalhados com alunos nesse nível de ensino.

Para auxiliar o professor, sugerimos o vídeo educacional produzido pelo Ministério da Educação e Cultura (MEC), chamado “Nas malhas da geometria” [17], da série “Mão na Forma”. Nesse vídeo, podemos ver um exemplo da utilização das malhas na sala de aula em uma turma do Ensino Fundamental. Além disso, o professor pode aprofundar mais seus conhecimentos buscando os outros vídeos da série no site http://www.dominiopublico.gov.br,obtendo vídeos, textos, sons e figuras que vão auxiliar em sua prática docente.

O professor de Matemática pode propor também um trabalho multidisciplinar na sua escola, com os professores de Artes e História, fazendo um projeto mais aprofundado sobre o estudo de Ornamentos. Cada área pode utilizar os mosaicos como ponto de partida para exploração de importantes conceitos desenvolvidos ao longo da história.

No Ensino Médio, sugerimos ao professor o estudo mais aprofundado dos conceitos geométricos dos ornamentos. Utilizando este trabalho e as referências aqui citadas, o professor pode elaborar uma aula onde seus alunos irão explorar as formações de faixas, rosetas e mosaicos. Uma tarefa interessante é buscar construir juntamente com os alunos os fluxogramas de notação dos padrões de faixas e mosaicos apresentados como anexos neste trabalho.

Para alunos a nível de Ensino Superior, especificamente alunos do curso deMatemática, é interessante utilizar os Ornamentos com padrões para introduzir conceitos das estruturas algébricas das isometrias realizadas para gerar cada “motivo”. O professor pode mostrar a relação da simetria com a teoria dos grupos (simetrias do retângulo, do quadrado, do triângulo eqüilátero, entre outras).

Conclusão

Através dos conceitos apresentados neste trabalho podemos perceber o quanto a Geometria está presente em nossas vidas. Precisamos resgatar o ensino de Geometria nas

Page 307: Numero 09 - Outubro de 2007

escolas, melhorando o nível das disciplinas de Geometria dos cursos de Licenciatura em Matemática. Através da utilização de mosaicos podemos trabalhar diversos conceitos de Geometria em todos os níveis de ensino.

Ressaltamos a importância do desenvolvimento de material pedagógico que auxilie o professor em suas aulas, como os vídeos e outros materiais que discorremos no texto deste trabalho. Além disso é necessário que existam cursos de aperfeiçoamento de professores para que estes se mantenham atualizados em relação ao que ocorre com a Educação e mais especificamente com a Matemática e a Geometria.

Esperamos que este trabalho sirva como base para que professores possam planejar atividades interessantes para suas aulas de Geometria, além de servir como material pedagógico para aprofundamento no tema “Ornamentos”.

Bibliografia

[1] SANZ, Antonio Pérez. Movimientos en el plano. Serie de Matemáticas: Más por Menos. Programa da Televisión Educativa de TVE-2 "La Aventura del Saber". Madrid: 1996. 1 vídeo (12’23” min.), color, espanhol.

[2] MUZÁS, José María Sorando. Matemáticas em tu mundo. Disponível em <http://es.geocities.com/mundo_matematicas/FOTOGRAFIAS/fotografia_mosaicos.htm>Acesso em: 13 jul. 2007.

[3] WIKIPÉDIA, A Enciclopédia livre. Mosaico. Disponível em<http://pt.wikipedia.org/wiki/Mosaico>. Acesso em 14 jul. 2007.

[4] YAHOO, Esportes. Pan 2007 (XV Jogos Pan-Americanos) – Praias. Disponível em <http://br.esportes.yahoo.com/pan2007/guiario/praias>. Acesso em 24 jul. 2007.

[5] RIO MOSAICO 2006. Pão de Açúcar. 2 ed. Disponível em<http://brasilartesao.com.br/emfoco/riomosaico/index.htm >. Acesso em 25 jul. 2007.

[6] WIKIPÉDIA, A enciclopédia livre. Alhambra. Disponível em<http://pt.wikipedia.org/wiki/Alhambra>. Acesso em 13 jul. 2007.

[7] OREGON, University of. Images Alhambra. Disponível em<http://www.uoregon.edu/~laskaya/images>. Acesso em 24 jul. 2007.

[8] NOLLA, R. e MASIP, R. Mosaics Periòdics a la Casa Castellarnau. Departament d’Educació de Tarragona. Disponível em<http://www.xtec.es/iesponsdicart/castellarnau/Matematiques/WEBCAMAT/Documents/LlibreCA_Ramon_A5w.pdf> Acesso em: 14 jul. 2007.

[9] BIEMBENGUT, M. S. e HEIN, N. Modelagem Matemática no Ensino. 3 ed. São Paulo: Contexto, 2003.

[10] FERREIRA, Susana. Transformações geométricas e simetrias. Coimbra: 2000. Disponível em <http://www.prof2000.pt/users/j.pinto/textos/Frisos_pavimentacoes.pdf>.Acesso em 24 jul. 2007.

Page 308: Numero 09 - Outubro de 2007

[11] MOSAIC MARBLE. Mosaic Geometrical Forms. USA/Canadá: 2003. Disponível em<http://www.mosaicmarble.com/index.php?cPath=23&osCsid=3ceaf66fdf2f9fb711148607cbd5a515>. Acesso em 24 jul. 2007.

[12] LISSANEDDINE, Fátima. Mosaicos Árabes. Espanha. Disponível em<http://www.pangea.org/crpbdln/jjcc/treballs02/pdf%27s/51.pdf>. Acesso em 24 jul. 2007.

[13] SOUTHERN POLYTECHNIC, State University. Identifying the 17 Plane Symmetry Groups. Georgia: 2004. Disponível em <http://www.spsu.edu/math/tile/symm/ident17.htm>. Acesso em 24 jul. 2007.

[14] VELOSO, Eduardo. Padrões e Frisos – Ficha de Leitura. Portugal: 2001. Disponível em <http://www.prof2000.pt/users/edveloso/paginas/propostas/P07.html>. Acesso em 24 jul. 2007.

[15] MASIP, Ramon. Resum classificació mosaics. Espanha. Disponível em <http://www.xtec.es/~rmasip1/pdfs/resum.pdf>. Acesso em 24 jul. 2007.

[16] RAPOSO, A. C. P.; DUARTE, A. L. B. e ROSÁRIO, M. I. F. Artistas Matemáticos, Matemáticos Artistas. Disponível em<http://www.educ.fc.ul.pt/icm/icm2000/icm33/Escher.htm>. Acesso em 25 jul. 2007.

[17] MINISTÉRIO DA EDUCAÇÃO - MEC. Nas malhas da geometria. Série Mão na Forma: 2000. 1 vídeo (12’45” min.), color, português. Disponível em<http://www.dominiopublico.gov.br/pesquisa/DetalheObraForm.do?select_action=&co_obra=20839>.Acesso em 26 jul. 2007.

Page 309: Numero 09 - Outubro de 2007

Anexo IFluxograma de Washburn e Crowe para a classificação das faixas monocromáticos

Page 310: Numero 09 - Outubro de 2007

Anexo II

Fluxograma para classificação dos padrões planos monocromáticos(Washburn e Crowe)

Page 311: Numero 09 - Outubro de 2007

MODELAGEM NO ENSINO MÉDIO: CUBAGEM DE

MADEIRA

Universidade Federal de Uberlândia

Faculdade de Matemática

Lóren Grace Kellen Maia Amorim Mariana Martins Pereira Rosana Sueli da Motta Jafelice [email protected] [email protected] [email protected]

INTRODUÇÃO

Este trabalho mostra a utilização da modelagem no ensino médio (Modelação no

ensino), procurando mostrar uma aplicação da matemática no cotidiano. O texto

descreve algumas etapas da modelagem e um método desenvolvido para mostrar a

validade do método de cubagem utilizado pelo madeireiro e apresenta também uma

atividade que tem por objetivo auxiliar o professor no processo de ensino aprendizagem

de como ajudar o aluno na construção do conhecimento em relação ao volume do cone.

A intenção, quando procuramos compreender o método de cubagem da madeira

utilizado pelo madeireiro exibido em (BIEMBENGUT, 2003) é proporcionar ao aluno

um ambiente diferente para que o mesmo desenvolva sua aprendizagem de forma

compreensiva e significativa. O desenvolvimento deste projeto que fora intitulado

“Modelagem no ensino médio: Cubagem de Madeira” propiciou um espaço de

aprendizagem em Geometria Espacial.

Nesse trabalho trataremos do relato da experiência e dificuldades de elaboração

do referido projeto, bem como, da reflexão sobre os saberes movimentados e os

desdobramentos decorrente destes.

Para a realização do projeto o desafio era o de elaborar uma proposta de uma

atividade para alunos do ensino médio, envolvendo o ensino de Matemática através da

modelagem. Muito tempo foi necessário para se chegar à decisão de que havia no grupo

o desejo e a necessidade de desenvolver algo que pudesse ser trabalhado com o aluno,

deste nível de ensino, de maneira fácil, prática e prazerosa. A utilização da informática

se despontou como propício para explorar os conceitos de “Geometria Plana e Espacial”

Page 312: Numero 09 - Outubro de 2007

e, além disso, despertar o interesse dos alunos. Acreditava-se que este conteúdo abriria

um leque enorme de possibilidades para a realização de um trabalho interessante e

estimulador. Mas que material seria esse? Após a dedicação de várias horas discutindo e

realizando leituras e pesquisas, em diferentes textos e sites, optou-se pela construção de

uma atividade de ensino no ambiente computacional na tentativa de tornar real à

proposta imaginada.

Pensávamos que compreender a modelagem do método de cubagem utilizado

pelo madeireiro e a construção da atividade de ensino no ambiente computacional seria

fácil, porém quando começamos a desenvolver o trabalho, tivemos algumas surpresas,

pois não foi tão trivial perceber a matemática utilizada na abordagem de

(BIEMBENGUT, 2003) e nem na construção da atividade. Durante a elaboração da

mesma descobrimos o quanto é importante o professor desenvolver uma atividade antes

de propô-la a seus alunos, pois assim poderá identificar e entender que conteúdo

Matemático é possível ser explorado, e quando os alunos indagá-lo o professor não será

pego de surpresa.

Outro ponto relevante na produção da apresentação se relaciona a descoberta,

durante a preparação, sobre os vários conteúdos de Matemática possíveis de serem

explorados além daqueles pensados inicialmente. A idéia inicial proposta evidenciava

apenas o volume do cone, do cilindro e do prisma. Entretanto, a experiência nos levou a

descobrir que outros conteúdos estavam relacionados e poderiam ser também

explorados, tais como: perímetro, área, semelhança de triângulo.

Modelos Matemáticos e Situações Problemas Envolvendo Modelagem Matemática

Para (BASSANEZI, 2004), a Modelagem Matemática de uma situação problema

real deve seguir uma seqüência de etapas visualizadas na Figura 1.

Page 313: Numero 09 - Outubro de 2007

Figura 1 (BASSANEZI, 2000, p.27)

Na Figura 1 as setas contínuas indicam a primeira aproximação. A busca de um

modelo matemático que melhor descreva o problema estudado torna o processo

dinâmico, indicado pelas setas pontilhadas.

1. Experimentação: É uma atividade essencialmente laboratorial onde se processa

a obtenção de dados;

2. Abstração: É o procedimento que deve levar à formulação dos Modelos

Matemáticos;

3. Resolução: O modelo matemático é obtido quando se substitui a linguagem

natural das hipóteses por uma linguagem matemática coerente – é como num

dicionário, a linguagem matemática admite “sinônimos” que traduzem os

diferentes graus de sofisticação da linguagem natural;

Page 314: Numero 09 - Outubro de 2007

4. Validação: É o processo de aceitação ou não do modelo proposto. Nesta etapa,

os modelos, juntamente com as hipóteses que lhes são atribuídas, devem ser

testados em confronto com os dados empíricos, comparando suas soluções e

previsões com os valores obtidos no sistema real. O grau de aproximação

desejado destas previsões será o fator preponderante para validação;

5. Modificação: Alguns fatores ligados ao problema original podem provocar a

rejeição ou aceitação dos modelos. Quando os modelos são obtidos

considerando simplificações e idealizações da realidade, suas soluções

geralmente não conduzem às previsões corretas e definitivas, pois o

aprofundamento da teoria implica na reformulação dos modelos. Nenhum

modelo deve ser considerado definitivo, podendo sempre ser melhorado, poder-

se-ia dizer que um bom modelo é aquele que propicia a formulação de novos

modelos, sendo esta reformulação dos modelos uma das partes fundamentais do

processo de modelagem.

Genericamente, (BIEMBENGUT; HEIN, 2005), apresentam o modelo de

Modelagem Matemática, Figura 2, no qual matemática e realidade são dois conjuntos

disjuntos e a modelagem é o meio de fazê-los interagir.

Figura 2 (BIEMBENGUT; HEIN, 2005, p. 13)

Essa interação, que permite representar um fenômeno através da linguagem

matemática (modelo matemático), envolve uma série de procedimentos, que podem ser

agrupados em três etapas, subdivididas em seis subetapas, a saber:

Page 315: Numero 09 - Outubro de 2007

a) Interação

Reconhecimento da situação-problema;

Familiarização com o assunto a ser modelado referencial teórico.

b) Matematização

Formulação do problema hipóteses;

Resolução do problema em termos do modelo;

c) Modelo matemático

Interpretação da solução;

Validação do modelo avaliação.

Se o modelo não atender às necessidades que o geraram, o processo deve ser

retomado na segunda etapa – Matematização – mudando-se ou ajustando hipóteses,

variáveis, etc. Veja a Figura 3:

Figura 3 (BIEMBENGUT; HEIN, 2005, p. 15)

É importante ao concluir o modelo, a elaboração de um relatório que registre todas

as fases do desenvolvimento, a fim de propiciar seu uso de forma adequada

(BIEMBENGUT,1999).

Page 316: Numero 09 - Outubro de 2007

COMPREENDENDO O PROCESSO DE CUBAGEM DE MADEIRA

O nosso intuito ao realizar este trabalho foi o de utilizar a modelagem como

meio de auxiliar no processo de ensino-aprendizagem. Também consideramos a

oportunidade de discutir por meio deste projeto a possibilidade real do professor deixar

um pouco de lado o quadro negro e as fórmulas, atuando como mediador para que o

aluno construa o seu conhecimento a partir das aplicações e manuseio do material.

Abaixo descrevemos a modelagem do método de cubagem da madeira de forma

a explanar toda matemática utilizada, os objetivos do objeto de aprendizagem proposto

e os procedimentos em cada etapa do trabalho.

Segue abaixo o método de cubagem utilizado pelo madeireiro segundo

(BIEMBENGUT, 2003).

Segundo o madeireiro, o procedimento para calcular a metragem cúbica de

madeira ou tábua que obterá do tronco de uma árvore após o corte é o seguinte:

a) primeiro, estima o ponto central do tronco da árvore;

b) com um cordel (barbante), a partir desse ponto, encontra o perímetro do

tronco (circunferência);

c) a seguir, dobra o cordel (relativo ao perímetro encontrado) em quatro partes

iguais 2 r = 4l.

2 r = 4l l = r/2

d) num ato contínuo, eleva ao quadrado a medida desse quarto da circunferência;

Page 317: Numero 09 - Outubro de 2007

e) e, finalmente, multiplica o valor desse quarto cordel ao quadrado, pela medida

da altura da árvore obtendo, então, o volume ou o número de m³ da madeira.

Qual a validade do método do madeireiro?

Nesse processo, o madeireiro "aproxima" primeiro o tronco (de cone) a um

cilindro. Essa aproximação se dará como perímetro, a média entre os perímetros das

bases menor e maior do tronco.

Posteriormente, efetua o cálculo do volume de um prisma de base quadrada.

Com isso, a diferença entre os volumes é significativa. Vejamos por quê:

ao dividir o cordel em quatro partes e elevá-lo ao quadrado, o madeireiro calcula

a área de um quadrado, ou seja, “transforma” o círculo em um quadrado.

Embora os perímetros sejam iguais, as áreas são diferentes.

ao multiplicar a área (Aq) pela altura (h), determina o volume de um prisma e

não de um cilindro. A razão é de4

.

Page 318: Numero 09 - Outubro de 2007

Nesse caso, o volume obtido pelo método do madeireiro é menor do que o

volume do tronco. Isto porque o volume do cilindro é igual a /4 do volume do prisma.

Outro fato interessante é que o corte para a obtenção de tábuas, nessa madeireira,

era feita de forma hexagonal. Isto é, cortava-se uma tábua e, em seguida, girava-se o

tronco em um ângulo (aproximadamente) de 60° , seguindo o processo até não ser mais

possível retirar tábuas.

Por esse processo, o volume de um prisma hexagonal é

Se compararmos os volumes, veremos que:

Volume do cilindro > volume do prisma hexagonal > volume do prisma

quadrangular.

Numa análise superficial, observamos que o madeireiro "paga" pelo tronco,

como se fosse um prisma de base quadrangular, corta-o como um prisma de base

hexagonal e "ganha" efetuando seus cálculos a partir do cilindro, pois o tronco é

transformado em madeira e lenha.

Nesse momento poderá ser abordado os seguintes volumes:

hL²2

33

Page 319: Numero 09 - Outubro de 2007

Volume do prisma: O volume de um prisma é dado por

V(prisma) = A(base).h

Volume do cilindro: Em um cilindro, o volume é dado pelo produto da área da

base pela altura.

V = A(base). h

Se a base é um círculo de raio r, então:

V = r² h

Volume do cone: O volume do cone é obtido por 1/3 do produto da área da base

pela altura, então:

V = (1/3) r³

Volume do tronco de um cone: O volume de um tronco de cone reto é igual à

diferença entre os volumes do cone (maior) e do cone (menor), isto é:

Matematizando com dados numéricos

Vamos tomar a medida de uma árvore de eucalipto e passar ao cálculo do

volume, supondo que o tronco de eucalipto seja "aproximadamente" um tronco de um

cone reto. Fazendo:

raio maior ( R ) = 0,30 m; raio menor ( r ) = 0,25 m; altura ( h ) = 4,8 m

1) O volume de um tronco de cone reto é igual à diferença entre os volumes

do cone (maior) de altura (4,8 m + x) e do cone (menor) de altura x, isto é:

Substituindo os valores dos raios, temos:

hrHRVVVV tcCt ²²3

hrHRVVVV tcCt ²²3

3432,00275,0

3²25,08,4²30,0 xxxVt

Page 320: Numero 09 - Outubro de 2007

Uma vez que os triângulos ABC e ADE são semelhantes podemos obter o valor de x,

por:

Portanto, o volume do tronco (V):

V 0,364 1,143m³

2) Tomando a tora como cilindro, o volume (V2)

3) Obtendo o volume de um prisma hexagonal, por ser este o processo de corte

do tronco.

Um hexágono regular de (lado L ) é composto por seis triângulos eqüiláteros.

Calculando a área de um triângulo eqüilátero.

248,425,03,0 x

xx

xxh

rR

³140,1³363,08,4²275,0

²

2

2

mmVV

hrV

Page 321: Numero 09 - Outubro de 2007

Como a altura do triângulo eqüilátero é h =

Seja At a área do triângulo eqüilátero e AH área do hexágono

Assim, o volume do prisma hexagonal (V3) será:

Pelo método do madeireiro, temos:

Considerando que o raio na metade do tronco seja a média entre os raios inferior

e superior, temos que:

Comparando os volumes, observamos:

V1 > V2 > V3 > V4

1,143 > 1,140 > 0,94 > 0,896 m³

Numa análise superficial, poderíamos dizer que:

a) o madeireiro compra o tronco de árvore por 0,896m³;

,2

3L

²2

334

3²6

43²

2

32

LLA

LLL

A

H

t

³94,08,4²275,02

33

²2

33

3

3

mV

hLV

hnciacircunferêV2

4 4

³896,0³²09075,08,4455,0

55,02

25,030,022

2

2

4 mmV

mrRnciacircunferê

Page 322: Numero 09 - Outubro de 2007

b) tem um aproveitamento em madeira de 0,943m³ e,

c) ao aproveitar a casca, obtém também mais

Vcasca = Vc - VH = 1,140 – 0,943 = 0,197m³

Comparando (b) e (a)

Comparando (c) e (a)

Ou seja, aparentemente há uma diferença não "contabilizada" de 5,25% de

madeira ou de 21,9% ao se considerar, também, a casca. Esse cálculo leva-nos a pensar,

num primeiro momento, que o método do madeireiro não vale.

Analisemos como é feito o corte das tábuas.

A cada tábua cortada, a lâmina da serra transforma cerca de 1 cm de espessura da

madeira em pó. Supondo que a espessura de cada tábua seja 2,5 cm. Em volume de pó,

corresponde aproximadamente a 48 prismas de 1 cm de espessura; 4,8 m de

comprimento e largura variando, mais ou menos, entre 24,6 cm e 4,3 cm.

Observe a Figura 4:

%25,50525,0896,0047,0

896,0896,0943,0

%23,272723,0896,0244,0

896,0896,0140,1

Page 323: Numero 09 - Outubro de 2007

Figura 4

Temos que L = 27,5 cm (média dos raios)

A tábua tem 2,5 cm de espessura, encontrando a altura do triângulo eqüilátero

teremos:

Como a espessura é 2,5cm teremos:

Assim, 2x = 24,6 cm

Logo a primeira tábua a ser cortada terá a largura de 24,6cm.

Observe o triângulo vermelho, Figura 5:

Figura 5

Encontrando k (espessura de cada tábua), temos que:

81,232

5,27)5,27(2

22h

3,1231,2181,23

25,27

xx

Page 324: Numero 09 - Outubro de 2007

Agora vamos calcular a quantidade de tábuas (n) que poderão ser retiradas na

base menor, cujo raio é de 25 cm.

Temos um triângulo eqüilátero de lados 25 cm, logo a sua altura será de:

L = 21,65 cm.

Assim teremos que n = 21,65/2,5 8

Logo, podemos retirar 8 tábuas de cada prisma.

Encontrando a menor largura do prisma

4,481,381,235,27 x

xPortanto a largura das tábuas irá variar entre 24,6cm e 4,4cm.

Ou seja, a largura depende do número de tábuas.

L= 27,5 – 2,89n

onde n é o número de tábuas tiradas.

O volume de pó entre duas tábuas em cm³:

Ou seja, a largura depende do número de tábuas.

L= 27,5 – 2,89n

onde n é o número de tábuas tiradas.

O volume de pó entre duas tábuas em cm³:

89,2)45,1(5,2 222 kk

³5566089,25,27²4808

1cmncmVi

i

Page 325: Numero 09 - Outubro de 2007

Considerando que o corte da madeira é feito girando o tronco, o volume de pó de

serra será aproximadamente:

V(pó) = 6 X (55660) = 333964,8 cm³ = 0,33 m³

Comparando:

1,140 m³ (madeira mais casca) - 0,33 m³ (pó) = 0,81 m³ (volume de madeira)

Em percentagem, representa aproximadamente

Segundo o madeireiro, a perda é em torno de 20%.

Tomando o valor determinado pelo cálculo de volume feito pelo método do

madeireiro e subtraindo do valor "real":

1,140 m³ - 0,896 m³ = 0,244 m³ de perda

O que representa, em percentual:

Ou seja, uma perda em torno de 21%.

Concluímos que é válido o método de cubagem de madeira do madeireiro, e a

experiência mostra que é um modelo matemático, pois "aproxima" o tronco de cone (no

caso da árvore) a um prisma de base quadrada para saber o volume ou o número de

metros cúbicos de tábuas que conseguirá obter de uma árvore.

ATIVIDADE

Diante da grande dificuldade dos alunos em compreender a Matemática e, além

disso, a concepção de muitos alunos de diferentes níveis como sendo esta área um

%9,28140,1

10033,0

%4,21140,1

100244,0

Page 326: Numero 09 - Outubro de 2007

‘bicho-de-sete-cabeças’, consideramos interessante que o aluno tenha a oportunidade de

aprender interagindo e refletindo, evitando assim, um aprender mecânico, repetitivo e

aquele fazer sem saber o que faz e por que faz. Nesse sentido, optamos por desenvolver

um trabalho sobre o uso da modelagem e da informática, por acreditarmos que com essa

ferramenta as aulas de Matemática poderão ser mais interativas, despertando a

curiosidade, a criatividade e estimulando os alunos a fazerem perguntas.

Atividade 1: Nessa atividade o aluno irá escolher um cone, no qual um está

cheio de areia e o outro cheio de água, em seguida irá movimentar o cone até o cilindro,

esse processo será feito três vezes, se o aluno colocar menos de três o cilindro ficará

vazio se passar de três o conteúdo escolhido transbordará.

O objetivo dessa atividade é que o aluno compreenda como encontrar o volume

de um cone sabendo o volume do cilindro.

Atividade 2: Nessa atividade o aluno irá escolher uma altura, na tela aparecerá

uma serra elétrica que irá cortar o cone em uma certa altura. Depois do corte teremos

um cone menor e um tronco de cone.

Em seguida o cone e o tronco de cone irão encher então os alunos terão que

colocar o conteúdo no cone maior.

O objetivo dessa atividade é que o aluno compreenda como encontrar o volume

do tronco de cone.

Page 327: Numero 09 - Outubro de 2007

Atividade 3: Nessa atividade teremos a simulação do corte de uma árvore

quando o aluno passar o mouse sobre a tora no chão aparecerá o ponto médio entre a

base maior e a base menor. Em seguida teremos um barbante que contorna a tora

exatamente nesse ponto.

O aluno nesse momento terá que arrastar o barbante e escolher em quantos

pedaços esse se divide.

Se a escolha for três ele perceberá com animação que o volume ocupado

pelo prisma de base triangular é bem menor que o volume do tronco.

Se a escolha for quatro ele perceberá com animação que o volume

ocupado pelo prisma quadrangular é menor que o volume do tronco,

porém maior que o volume do prisma de base triangular.

Se a escolha for seis ele perceberá com animação que o volume do

prisma de base hexagonal será maior que o volume do tronco.

Page 328: Numero 09 - Outubro de 2007

CONSIDERAÇÕES FINAIS

A experiência relatada neste texto nos mostrou evidências da possibilidade real

de oferecer aos alunos do ensino médio uma aula mais dinâmica, em que os mesmos

participam ativamente de todo o processo de construção do conhecimento. Além disso,

se sobressaíram nessa caminhada de aprendizagem e desenvolvimento profissional, a

possibilidade e a vantagem da utilização da modelagem para proporcionar aulas de

Matemáticas mais interativas, que despertam curiosidades e estimulam os alunos a

fazerem perguntas, descobrirem semelhanças / diferenças, criarem hipóteses e chegarem

às próprias soluções.

Pensamos que o projeto em si tem suas potencialidades, mas se não houver a

mediação do professor a modelagem e a atividade de ensino no ambiente

computacional, por si só, não contribuirá para o processo de ensino-aprendizagem. Para

finalizar, acreditamos que o professor, com a mediação adequada, poderá explorar

diversos conceitos de matemática no método de cubagem a madeira.

BIBLIOGRAFIA

BIEMBENGUT, M. S. Modelagem Matemática no ensino / Maria Sallet Biembengut,

Nelson Hein. – 3ª ed. – São Paulo: Contexto, 2003.

FREITAS, M.T.M .A escrita no processo de formação contínua do professor de

Matemática. 2006. 299f. Tese (Doutorado em Educação: Educação Matemática) – FE,

Unicamp, Campinas (SP).

Page 329: Numero 09 - Outubro de 2007

Diagnóstico Médico Fuzzy de Doenças Infantis

Universidade Federal de Uberlândia Faculdade de Matemática

Mariana Fernandes dos Santos Villela Patrícia Borges dos Santos

[email protected] [email protected]

Rosana Sueli da Motta Jafelice [email protected]

Introdução A Modelagem Matemática tem como objetivo interpretar e compreender os mais

diversos fenômenos do nosso cotidiano e poder descrevê-los, analisá-los e interpretá-los com o propósito de gerar discussões reflexivas sobre tais acontecimentos que cercam nosso cotidiano. Neste trabalho, a modelagem é realizada através da Teoria dos conjuntos Fuzzy, o qual tem por objetivo o diagnóstico médico fuzzy de doenças infantis tais como, catapora, caxumba, coqueluche e meningite. Para isto, foi necessário a colaboração de especialistas, neste caso pediatras, e a partir de sinais e sintomas apresentado pelos pacientes, simulamos a atuação do médico no diagnóstico de seus doentes, com o intuito de ajudar este em suas tomadas de decisões e optar por exames laboratoriais. Além disso, realizamos o estudo de dois modelos de propagação de doenças transmissíveis (epidemias) os quais são, SIR (Suscetível Infectado Recuperado) e SIRS (Suscetível Infectado Recuperado Suscetível) que servem para exemplificar o a propagação de doenças estudadas no Diagnóstico Médico Fuzzy. Conjuntos fuzzy

Histórico Em 1965, com uma publicação de Lotfi A. Zadeh ("Fuzzy Sets", Information and Control, Vol. 8, pp. 338-353) surgiu uma nova teoria de conjuntos. Professor da Universidade da Califórnia, Berkeley, considerado um grande colaborador do controle moderno, Zadeh criou uma teoria de conjuntos em que não há descontinuidades, ou seja, não há uma distinção abrupta entre elementos pertencentes e não pertencentes a um conjunto, os são os Conjuntos Nebulosos. Começava aí a se desenvolver a Teoria Fuzzy (Nebulosa), para tratar de variáveis "imprecisas", ou definidas de forma "vaga".

Zadeh percebeu que a modelagem de muitas atividades relacionadas a problemas industriais, biológicos ou químicos seria complexa demais se implementada da forma convencional. Os sistemas fuzzy foram utilizados, com sucesso, em algumas aplicações que se tornaram exemplos clássicos. Destaca-se a primeira aplicação que se tornou pública:

• Em 1974 o professor Mamdani, do Queen Mary College, da Universidade de Londres, implementou um controle de uma máquina a vapor, baseado em lógica fuzzy. Até então, não se tinha conseguido automatizar essas máquinas com outras técnicas de controle, nem mesmo com algoritmo PID.

Page 330: Numero 09 - Outubro de 2007

Com o tempo, outras aplicações foram surgindo. No oriente, onde a cultura fez com que os conceitos da lógica nebulosa fossem aceitos com maior facilidade do que no mundo oriental, investiu-se muito em soluções baseadas em modelagem e controle fuzzy, e, além disso, inúmeras aplicações surgiram principalmente no Japão.

Apesar de os estudos teóricos terem se desenvolvido na Europa e nos Estados Unidos, as aplicações nunca tiveram lá a mesma ênfase que tiveram no oriente, principalmente no Japão, que investiu muito no desenvolvimento de tecnologias baseadas na Teoria Fuzzy.

Hoje, empresas como Boeing, General Motors, Allen-Bradley, Chrysler, Eaton e Whirlpool têm procurado soluções diversas na Teoria Fuzzy. Controle de refrigeradores de baixa potência, transmissão automotiva, e motores elétricos de alta eficácia fazem parte de suas linhas de pesquisa.

Nos Estados Unidos, a Agência de Proteção Ambiental estuda o uso de controle Fuzzy em motores. A NASA tem estudado a aplicação da Teoria Fuzzy na ancoragem automática de suas naves no espaço. Simulações mostram que um Sistema Fuzzy pode reduzir significativamente o consumo em motores a combustão [2]. Definição Um subconjunto fuzzy A de U é definido em termos de uma função pertinência u que a cada elemento x de U associa um número ( )u x , entre zero e um, que é chamado o grau de pertinência de x em A. Assim o conjunto A é definido da seguinte maneira:

[ ]: 0,1Au U → .

Os valores ( ) 1Au x = e ( ) 0Au x = significam a pertinência e a não pertinência do elemento x a A. Operações entre conjuntos fuzzy Sejam A e B subconjuntos clássicos de U representados pelas funções características Au e Bu , respectivamente. Os conjuntos

{ ; ou }A B x U x A x B∪ = ∈ ∈ ∈ , { ; e }A B x U x A x B∩ = ∈ ∈ ∈ , ' { ; }A x U x A= ∈ ∉ .

Definição1: Sejam A e B conjuntos fuzzy. As funções de pertinência que representam os conjuntos fuzzy união (Figura 1), intersecção (Figura 2) e complementar (Figura 3) de conjuntos fuzzy são dados por, x U∀ ∈ ,

( ) ( )max{ , }A B A Bu u x u x∪ = ,

( ) ( )min{ , }A B A Bu u x u x∩ = ,

( ) ( )' 1 }.A Au x u x= −

Page 331: Numero 09 - Outubro de 2007

Figura 1: Representa a união dos conjuntos fuzzy.

Figura 2: Representa a intersecção dos conjuntos fuzzy.

Page 332: Numero 09 - Outubro de 2007

Figura 3: Representa o complementar dos conjuntos fuzzy.

Exemplo:

Seja U um conjunto universo composto por pacientes de uma clínica, identificados pelos números 1, 2, 3, 4 e 5. Sejam A e B os conjuntos fuzzy que representam os pacientes com febre e dor, respectivamente. A Tabela 1, abaixo, representa a união, intersecção e complemento.

Paciente Febre ( )Au Dor ( )Bu A Bu ∪ A Bu ∩ 'Au 'A Au ∩

1 0.7 0.6 0.7 0.6 0.3 0.3 2 1.0 1.0 1.0 1.0 0.0 0.0 3 0.4 0.2 0.4 0.2 0.6 0.4 4 0.5 0.5 0.5 0.5 0.5 0.5 5 1.0 0.2 1.0 0.2 0.0 0.0

Tabela 1: União, intersecção e complementar dos conjuntos A e B. Normas Triangulares Generalizando os operadores de união e intersecções têm as normas triangulares, que podem ser definidas da seguinte maneira [1]: Definição2: Uma co-norma triangular (s-norma) é uma operação binária

[ ] [ ] [ ]: 0,1 0,1 0,1s X → satisfazendo: • Comutatividade: xsy = ysx • Associatividade: xs(ysz) = (xsy)sz • Monotonicidade: Se x ≤ y e w ≤ z então xsw ≤ ysz • Condições de Fronteira: xs0 = x, xs1=1.

Temos como exemplo de uma s-norma o operador max.

Page 333: Numero 09 - Outubro de 2007

1-União padrão (Figura 4) [ ] [ ] [ ]: 0,1 0,1 0,1s X → com xsy = max(x; y).

00.2

0.40.6

0.81

0

0.5

10

0.2

0.4

0.6

0.8

1

União padrão

Figura 4: s-norma ‘União Padrão’.

2- Soma Algébrica (Figura 5)

[ ] [ ] [ ]: 0,1 0,1 0,1s X → com xsy = x+y-xy.

00.2

0.40.6

0.81

0

0.5

10

0.2

0.4

0.6

0.8

1

Soma algébrica

Figura 5: s-norma ‘Soma Algébrica’.

Page 334: Numero 09 - Outubro de 2007

3- Soma Limitada (Figura 6)

[ ] [ ] [ ]: 0,1 0,1 0,1s X → com xsy = min(1; x + y).

00.2

0.40.6

0.81

0

0.5

10

0.2

0.4

0.6

0.8

1

Soma limitada

Figura 6: s-norma ‘Soma Limitada’.

4- União Drástica (Figura 7)

[ ] [ ] [ ]: 0,1 0,1 0,1s X → com

se 0 se 0

1 caso contrario

x yxsy y x

=⎧⎪ =⎨⎪⎩

.

00.2

0.40.6

0.81

0

0.5

10

0.2

0.4

0.6

0.8

1

União drástica

Figura 4: s-norma ‘União Drástica’.

Page 335: Numero 09 - Outubro de 2007

Definição3: Uma norma triangular (t-norma) é uma operação binária [ ] [ ] [ ]: 0,1 0,1 0,1t X → satisfazendo:

• Comutatividade: xsy = ysx • Associatividade: xs(ysz) = (xsy)sz • Monotonicidade: Se x ≤ y e w ≤ z então xsw ≤ ysz • Condições de Fronteira: xs0 = 0, xs1=x.

Temos como exemplo de uma s-norma o operador min. 1- Intersecção Padrão (Figura 8)

[ ] [ ] [ ]: 0,1 0,1 0,1t X → com xty = min(x; y).

00.2

0.40.6

0.81

0

0.5

10

0.2

0.4

0.6

0.8

1

Intersecção padrão

Figura 8: t-norma ‘Intersecção Padrão’.

2- Produto Algébrico (Figura 9)

[ ] [ ] [ ]: 0,1 0,1 0,1t X → com xty = xy.

00.2

0.40.6

0.81

0

0.5

10

0.2

0.4

0.6

0.8

1

Produto algébrica

Figura 9: t-norma ‘Produto Algébrico’.

Page 336: Numero 09 - Outubro de 2007

3- Diferença Limitada (Figura 10) [ ] [ ] [ ]: 0,1 0,1 0,1t X → com xty = max(0; x + y ; 1).

00.2

0.40.6

0.81

0

0.5

10

0.2

0.4

0.6

0.8

1

Diferença limitada

Figura 10: s-norma ‘Diferença Limitada’.

4- Intersecção Drástica (Figura 11)

[ ] [ ] [ ]: 0,1 0,1 0,1t X → com

se 1 se 1

0 caso contrario

x yxsy y x

=⎧⎪ =⎨⎪⎩

.

rara

00.2

0.40.6

0.81

0

0.5

10

0.2

0.4

0.6

0.8

1

Intersecção drástica

Figura 11: s-norma ‘Intersecção Drástica’.

Page 337: Numero 09 - Outubro de 2007

Relações Fuzzy Estudos de associações, relações ou interações, entre os elementos de diversas classes é de grande interesse na análise e compreensão de muitos fenômenos do mundo real. Matematicamente, o conceito de relação é formalizado a partir da teoria de conjuntos. Desta forma, intuitivamente pode-se dizer que a relação será fuzzy quando optamos pela teoria dos conjuntos fuzzy e será clássica quando optamos pela teoria clássica de conjuntos para conceituar a relação em estudo. Qual dos modelos adotar, entre estes dois, depende muito do fenômeno estudado. Porém, a opção pela teoria de conjuntos fuzzy sempre tem maior robustez no sentido de que esta inclui a teoria clássica de conjuntos.

Uma relação clássica segue a função característica da lógica clássica. Sendo assim, uma relação de amizade entre duas pessoas, por exemplo (ver [2]), designadas como “amigos” considera que nas relações humanas ou alguém é seu amigo ou não o é, o que é uma simplificação da realidade. Uma relação de amizade fuzzy entre duas pessoas considera o grau de amizade entre elas, sendo assim dois ou mais indivíduos podem se relacionar com diferentes graus de amizade, desde 1,0 ( são certamente amigos) até 0,0 ( não são amigos). Formalmente, uma relação fuzzy R entre duas variáveis, e yx X Y∈ ∈ , é definida por uma função que mapeia o par ordenado ( ,x y ) no espaço X Y× para o seu grau na relação, ou seja,

[ ]: 0,1R X Y× → . Esta definição é facilmente generalizada para relações de dimensões superiores. Um exemplo importante de relações fuzzy em sistema de diagnósticos é aquela que relaciona sintomas a doenças, o qual é o foco do nosso trabalho. Definição 4: Uma relação fuzzy R, sobre 1 2 ... nU U U× × × , é qualquer subconjunto fuzzy do produto cartesiano 1 2 ... nU U U× × × . Se o produto cartesiano for formado por apenas dois conjuntos, 1 2U U× , a relação é chamada de fuzzy binária sobre 1 2U U× . Assim, uma relação fuzzy é definida por uma função de pertinência [ ]1 2: ... 0,1R nU U Uϕ × × × → .

A principal vantagem na opção pela relação fuzzy é que a relação clássica indica apenas se há ou não relação entre dois objetos, enquanto uma relação fuzzy além de indicar se existe ou não relação, indica também o grau desta relação. Uma noção que será muito importante para o nosso trabalho, é o produto cartesiano entre conjuntos fuzzy. Definição 5: O produto cartesiano fuzzy 1 2 ... nA A A× × × dos subconjuntos fuzzy 1 2, ,..., nA A A de 1 2, ,..., nU U U , é a relação fuzzy R cuja função de pertinência é

( ) ( ) ( ) ( )1 21 2 1 2, ,..., ^ ^ ...^

nR n A A A nu x x x u x u x u x= onde ^ é a t-norma min. Composição Relações de Fuzzy Considere R e S duas relações fuzzy binárias em 1 2 ... nU U U× × × , respectivamente. Definição 6: A composição RoS é uma relação fuzzy binária em 1 3U U× , com função de pertinência dada por

( ) ( ) ( )( )2 2

1 3 1 2 2 3, max min , , ,RoS R Sx Uu x x u x x u x x

∈⎡ ⎤= ⎣ ⎦ .

Agora , definiremos um caso especial da composição max-min, que utilizamos no trabalho para elaborar o diagnóstico médico fuzzy. Definição 7: Sejam 1U e 2U dois conjuntos , F( 1U ) e F( 2U ) as classes dos conjuntos fuzzy de

1U e 2U , respectivamente, e R uma relação binária sobre 1 2U U× . Então a relação R define

Page 338: Numero 09 - Outubro de 2007

um funcional de F( 1U ) em F( 2U ) que a cada elemento 1A ∈ F( 1U ), faz corresponder o elemento 2A ∈ F( 2U ) a função de pertinência é dada por:

( ) ( ) ( ) ( ) ( )( )2 111 1

2 2 1 1 2max min , ,A A RR A x Uu x u x u x u x x

∈⎡ ⎤= = ⎣ ⎦ .

Diagnóstico Médico O objetivo desta aplicação, e deste trabalho, é propor um sistema fuzzy que imite a atuação de um médico no diagnóstico de seus pacientes, a partir dos sintomas que estes apresentam. Com o intuito de ajudar o médico a tomar decisões e optar por exames laboratoriais mais detalhados. Para isto, foi preciso a interferência de um especialista na área, que neste caso consultamos dois pediatras Dr. Georges Ishac Abdallah e Dr. Márcia F. Lopes. O trabalho trata-se de estabelecer um diagnóstico de doenças infantis. A idéia básica é relacionar os sintomas ou sinais de pacientes com as possíveis doenças, as quais são cataporas, caxumbas, coqueluches e meningites. Esta aplicação pode ser resumida da seguinte maneira:

Considere os seguintes conjuntos universais:

• U = conjuntos dos pacientes do médico 1; • S = conjuntos dos pacientes do médico 2; • V = conjunto dos sintomas; • W = conjunto das doenças. Foram analisadas as informações de dois diferentes médicos, os quais obtivemos conhecimento de sete pacientes 1 2 3 4 5 6 7, , , , , e P P P P P P P , com sintomas s1, s2, s3 ,s4 ,s5, s6 ,s7 ,s8 ,s9 ,s10 ,s11 ,s12 ,s13 ,s14 ,s15 ,s16 ,s17 e s18 que apresentaram os diagnósticos

1 2 3 4, , e d d d d , onde: � s1 = pintas vermelhas no corpo � s10 = infecção das glândulas

salivares � s2 = coceira � s11 = tosse seca � s3 = febre � s12 = coriza � s4 = cansaço � s13 = dor muscular � s5 = cefaléia � s14 = fraqueza � s6 = perda de apetite � s15 = dor ao mastigar ou engolir � s7 = rigidez na nuca � s16 = mal estar � s8 = calafrios � s17 = vômito � s9 = confusão mental � s18 = dor de garganta � d1 = catapora � d3 = coqueluche � d2 = caxumba � d4 = meningite

ΕντEntrada (sintomas)

Base de conhecimento

Saída (Diagnóstico)

Page 339: Numero 09 - Outubro de 2007

A média das relações fuzzy sintomas X doenças de ambos os médicos é dada pela seguinte tabela 2: s d

s1 s2 s3 s4 s5 s6 s7 s8 s9 s10 s11 s12 s13 s14 s15 s16 s17 s18

d1 1 1 0.45 0.4 0.5 0.4 0 0.1 0 0 0.2 0.3 0.05 0.2 0 0.1 0 0

d2 0 0 0.3 0.15 0.7 0.5 0 0.25 0 0.8 0.1 0 0.4 0.4 0.9 0.3 0.05 0.75

d3 0 0 0.9 0.45 0.25 0.25 0 0.15 0 0 1 .55 0.1 0.1 0 0.6 0.05 0

d4 0.2 0 0.95 0.5 0.8 0.8 1 0.75 0.4 0 0 0 0.3 0.1 0 0.85 0.8 0

Tabela 2: Relação fuzzy sintomas x doenças.

Médico 1 ( Dr. Georges Ishac Abdallah) s P

s1 s2 s3 s4 s5 s6 s7 s8 s9 s10 s11 s12 s13 s14 s15 s16 s17 s18

P1 0 0 0.7 0.5 0.1 0.2 0 0.5 0 0 1 0.5 0.1 0.5 0 0 0 0

P2 0 0 0.5 0.7 0.9 0.5 0.9 0.3 0.9 0 0.5 0.1 0.6 0.5 0 0.8 0.7 0

P3 0 0 0.5 0.3 0.8 0.7 0 0.2 0 1 0.5 0.2 0.3 0.5 0.9 0.7 0.3 0.8

P4 1 0.8 0.9 0.3 0 0.7 0 0.3 0 0 0 0 0.2 0.3 0 0.1 0 0

P5 1 0.5 0.9 0.2 0 0.1 0 0.5 0 0 0 0.5 0.1 0.2 0 0 0 0

P6 0 0 0.3 0.2 0.1 0.1 0 0.1 0 0 1 0.3 0.1 0.1 0 0.1 0 0

P7 0 0 0.5 0.1 0.1 0.1 0 0.1 0 0 1 0.5 0.1 0.1 0 0.1 0.3 0

Tabela 3: Relação fuzzy pacientes x sintomas.

Por exemplo, o diagnóstico médico do paciente P1, via relação fuzzy R, é facilmente obtido através da definição 6. Assim, de acordo com os sinais e sintomas apresentados, o paciente P1 pode ter uma das doenças di, com i = 1, 2, 3 e 4 com os respectivos graus de possibilidades (pela Tabela 3):

( ) ( ) ( ) ( )( )11 1 11 18max min , , 0.45R i P iR P i

u d u d s u s≤ ≤

⎡ ⎤= =⎣ ⎦

( ) ( ) ( ) ( )( )11 2 21 18max min , , 0.3R i P iR P i

u d u d s u s≤ ≤

⎡ ⎤= =⎣ ⎦

( ) ( ) ( ) ( )( )11 3 31 18max min , , 1.0R i P iR P i

u d u d s u s≤ ≤

⎡ ⎤= =⎣ ⎦

( ) ( ) ( ) ( )( )11 4 41 18max min , , 0.7R i P iR P i

u d u d s u s≤ ≤

⎡ ⎤= =⎣ ⎦

Assim, de acordo com os sintomas apresentados, o paciente P2 pode ter também uma das doenças di, com i = 1, 2, 3 e 4 , com os respectivos graus de possibilidades:

Page 340: Numero 09 - Outubro de 2007

( ) ( ) ( ) ( )( )22 1 11 18max min , , 0.5R i P iR P i

u d u d s u s≤ ≤

⎡ ⎤= =⎣ ⎦

( ) ( ) ( ) ( )( )22 2 21 18max min , , 0.7R i P iR P i

u d u d s u s≤ ≤

⎡ ⎤= =⎣ ⎦

( ) ( ) ( ) ( )( )22 3 31 18max min , , 0.6R i P iR P i

u d u d s u s≤ ≤

⎡ ⎤= =⎣ ⎦

( ) ( ) ( ) ( )( )22 4 41 18max min , , 0.9R i P iR P i

u d u d s u s≤ ≤

⎡ ⎤= =⎣ ⎦

Desta forma, obtêm-se os diagnósticos para todos os pacientes:

� ( ) ( )1

0.45;0.3;1.0;0.7R Pu =

� ( ) ( )2

0.5;0.7;0.6;0.9R Pu =

� ( ) ( )3

0.6;0.9;0.6;0.8R Pu =

� ( ) ( )4

1.0;0.5;0.9;0.95R Pu =

� ( ) ( )5

1.0;0.3;0.9;0.9R Pu =

� ( ) ( )6

0.3;0.3;1.0;0.3R Pu =

� ( ) ( )7

0.45;0.3;1.0;0.5R Pu =

Portanto, nota-se que o paciente P1, pela teoria aplicada, tem maior possibilidade de estar com coqueluche, o paciente P2 pode estar com meningite, P3 pode estar com caxumba, P4 e P5 podem estar com catapora e , P6 e P7 podem estar com coqueluche. Segundo o especialista os pacientes realmente possuíam as respectivas doenças.

Médico 2 (Dr. Márcia F. Lopes)

s P 1s 2s 3s 4s 5s 6s 7s 8s 9s 10s 11s 12s 13s 14s 15s 16s 17s 18s

1P 1.0 0.5 0.9 0 0 0 0 0 0 0 0 0 0 0 0 0 0 0

2P 0 0 1.0 0 1.0 0 1.0 0 0 0 0 0 0 0 0 0 1.0 0

3P 1.0 0.7 1.0 0.9 0 0 0 0 0 0 0 0 0 0.9 0 0 0 0

4P 0 0 1.0 0 0 1.0 0 0 0 0 1.0 0 0 0 0 0 1.0 0

5P 0 0.7 0 0 1.0 1.0 1.0 0 1.0 0 0.9 0 0 0 0 0 1.0 0

6P 1.0 0.5 1.0 0 0 0 0 0 0 0 0 0 0 0.5 0 0 1.0 1.0

7P 1.0 0.9 0.9 0 0.5 0 0.5 0.5 0.3 0 0.4 0.5 0 0.4 0 0.5 0.7 0

Tabela 4: Relação fuzzy pacientes x sintomas. Por exemplo, o diagnóstico médico do paciente P1, via relação fuzzy R, é facilmente obtido através da definição 6. Assim, de acordo com os sinais e sintomas apresentados, o

Page 341: Numero 09 - Outubro de 2007

paciente P1 pode ter uma das doenças di, com i = 1, 2, 3 e 4 com os respectivos graus de possibilidades (Tabela 4):

( ) ( ) ( ) ( )( )11 1 11 18max min , , 1.0R i P iR P i

u d u d s u s≤ ≤

⎡ ⎤= =⎣ ⎦

( ) ( ) ( ) ( )( )11 2 21 18max min , , 0.3R i P iR P i

u d u d s u s≤ ≤

⎡ ⎤= =⎣ ⎦

( ) ( ) ( ) ( )( )11 3 31 18max min , , 0.9R i P iR P i

u d u d s u s≤ ≤

⎡ ⎤= =⎣ ⎦

( ) ( ) ( ) ( )( )11 4 41 18max min , , 0.9R i P iR P i

u d u d s u s≤ ≤

⎡ ⎤= =⎣ ⎦

Assim, de acordo com os sintomas apresentados, o paciente P2 pode ter também uma das doenças di, com i = 1, 2, 3 e 4 , com os respectivos graus de possibilidades:

( ) ( ) ( ) ( )( )22 1 11 18max min , , 0.5R i P iR P i

u d u d s u s≤ ≤

⎡ ⎤= =⎣ ⎦

( ) ( ) ( ) ( )( )22 2 21 18max min , , 0.7R i P iR P i

u d u d s u s≤ ≤

⎡ ⎤= =⎣ ⎦

( ) ( ) ( ) ( )( )22 3 31 18max min , , 0.9R i P iR P i

u d u d s u s≤ ≤

⎡ ⎤= =⎣ ⎦

( ) ( ) ( ) ( )( )22 4 41 18max min , , 1.0R i P iR P i

u d u d s u s≤ ≤

⎡ ⎤= =⎣ ⎦

Desta forma, obtêm-se os diagnósticos para todos os pacientes:

� ( ) ( )1

1.0;0.3;0.9;0.9R Pu =

� ( ) ( )2

0.5;0.7;0.9;1.0R Pu =

� ( ) ( )3

1.0;0.4;0.9;0.9R Pu =

� ( ) ( )4

0.45;0.5;1.0;0.95R Pu =

� ( ) ( )5

0.7;0.7;0.9;1.0R Pu =

� ( ) ( )6

1.0;0.75;0.9;0.95R Pu =

� ( ) ( )7

1.0;0.5;0.9;0.9R Pu =

Portanto, nota-se que o paciente P1, pela teoria aplicada, tem maior possibilidade de estar com catapora, os pacientes P2 e P5 podem estar com meningite, P3 pode estar com catapora, P4 pode estar com coqueluche e, P6 e P7 podem estar com catapora. Segundo a especialista os pacientes realmente possuíam as respectivas doenças. Note que a resposta da composição é também um conjunto fuzzy, ou seja, a composição nem sempre responde qual doença o paciente possui, porém fornece a distribuição de possibilidades do paciente no conjunto de doenças dado que ele apresenta certa distribuição de possibilidades no conjunto de sintomas. Outra propriedade importante da relação fuzzy é que após ter diagnósticos de novos pacientes, estes podem ser incluídos na base de conhecimentos e assim aumentar a capacidade de se obter mais diagnósticos por meio da relação fuzzy R, tal como faz o médico.

Page 342: Numero 09 - Outubro de 2007

Apresentamos, na próxima seção, alguns sistemas de equações diferenciais relacionados com as epidemias de doenças. Modelo SIR (Suscetível Infectado recuperado) de Epidemiologia O estudo da propagação de doenças transmissíveis (epidemias) teve um desenvolvimento bastante lento até o século XIX, sendo finalmente assumido como pesquisa científica a partir dos trabalhos desenvolvidos pó Pasteur e Kock. A partir de 1927, os modelos matemáticos, formulados por Kermack-McKendric, ( ver [4]), consideram que uma epidemias com microparasitas (vírus e bactérias) ocorre em uma comunidade fechada através do contato entre pessoas infecciosas e pessoas sadias. A população hospedeira é subdividida em classes distintas (compartimentos) de acordo com a sanidade ou infecciosidade de seus elementos: S = S(t): pessoas sadias, mas suscetíveis à doença, podendo ser infectadas quando em contato com pessoas doentes; I = I(t): pessoas portadoras da doença (infecciosas); R = R(t): indivíduos imunes que já contraíram a doença e se recuperam, ou estão isoladas ou morreram. Supor que a comunidade seja fechada implica que a população total se mantém constante, isto é,

( ) ( ) ( )N S t I t R t= + + não varia com t. Este fato é característico das doenças cujo período de inclusão do parasita é relativamente pequeno. Para cada tipo de doenças podemos modelar sua velocidade de propagação através das interações entre as variáveis S, I e R. O processo epidemiológico pode ser esquematizado pelo sistema compartimental que resume as taxas de transições entre as três classes:

onde Iβ é a taxa de transmissão da doença (β>0), com β como o coeficiente de infecciosidade; α é a taxa de remoção (α>0) . Se considerarmos que:

a- Cada compartimento é composto de indivíduos homogêneos; b- Cada indivíduo infeccioso tem a mesma probabilidade de se encontrar com um

suscetível; c- Não ocorre nascimento na comunidade e a morte somente é causada pela doença.

Então o modelo matemático que descreve a epidemias, também chamado SIR ou modelo sem dinâmica vital, é dado por:

(I)

(II) (1)

(III)

dS SIdtdI SI IdtdR Idt

β

β α

α

⎧ = −⎪⎪⎪ = −⎨⎪⎪ =⎪⎩

I

S

R β α

Page 343: Numero 09 - Outubro de 2007

(I) os suscetíveis decrescem a uma taxa proporcional ao número de encontros com os

infecciosos. (II) os infectados aumentam do mesmo modo como os sadios diminuem e perdem os

que são curados ou mortos. (III) a variação dos retirados é proporcional à quantidade dos infectados.

Das doenças estudadas no Diagnóstico médico fuzzy, as que apresentam comportamento parecido com o modelo SIR são a catapora, caxumba e coqueluche. Em qualquer situação é fundamental conhecer os valores iniciais So=S(0)=100, Io=I(0)=10 e Ro=R(0)=10, e para resolução desse sistema, obtivemos os parâmetros do programa Populus. Temos então:

0.80.5

αβ

==

Utilizando o Matlab, temos como solução do sistema de equação diferencial ordinária (1) o seguinte gráfico (Figura 12):

0 0.1 0.2 0.3 0.4 0.5 0.6 0.7 0.8 0.9 1-20

0

20

40

60

80

100

120

tempo

popu

laçã

o

SuscetívelInfectadoRecuperado

Figura12: Resolução do sistema de equações diferenciais (1).

O modelo SIR tem como característica o fato em que se um indivíduo foi infectado e está recuperado, e este não se torna novamente suscetível a esta doença. Para os parâmetros considerados a Figura 12 mostra que o número de indivíduos suscetíveis torna-se cada vez menor até não existir mais pessoas suscetíveis, enquanto que a quantidade de indivíduos recuperados aumenta isso acontece, pois o sistema é fechado. Além disso, como o número de indivíduos recuperados aumenta, temos que a quantidade de pessoas infectadas diminuirá à medida que esse número de recuperados cresce. A Figura 12 mostra esse processo em um curto período de tempo.

Page 344: Numero 09 - Outubro de 2007

Modelo SIRS (Suscetível Infectado Recuperado Suscetível) de epidemiologia Um outro modelo de propagação de epidemia foi desenvolvido por Chimara (2003) através de um autômato celular probabilista que corresponde a um modelo SIRS, representando a situação em que recuperados são substituídos por suscetíveis, ou porque morreram (e um suscetível recém-nascido ocupa seu lugar) ou porque perderam a imunidade àquela doença. Nesse modelo foi considerada uma população de tamanho fixo e, estudando-se a influência dos parâmetros que representam as probabilidades de infecção, de cura e de morte causada pela doença. Das doenças estudadas no Diagnóstico Médico fuzzy, um exemplo que tem comportamento parecido com o modelo SIRS é a meningite.

O sistema que descreve o modelo SIRS é dado por:

(2)

dS IS RdtdI IS IdtdR I Rdt

β γ

β α

α γ

⎧ = − +⎪⎪⎪ = −⎨⎪⎪ = −⎪⎩

Em qualquer situação é fundamental conhecer os valores iniciais So=S(0)=50, Io=I(0)=100 e Ro=R(0)=100, e para resolução desse sistema, obtivemos os parâmetros do programa Populus. Temos então:

0.70.10.6

αβγ

===

onde, β como o coeficiente de infecciosidade ( 0β > ); α é a taxa de remoção 0α > e γ a taxa de diminuição de imunidade. Utilizando o Matlab, temos como solução do sistema de equação diferencial (2), Figura 13:

I S

R

γ

β α

Page 345: Numero 09 - Outubro de 2007

0 0.1 0.2 0.3 0.4 0.5 0.6 0.7 0.8 0.9 10

20

40

60

80

100

120

140

tempo

popu

laçã

o

SuscetívelInfectadoRecuperado

Figura 13: Resolução do sistema de equações diferenciais (2).

O modelo SIRS tem como característica o fato de que se um indivíduo foi infectado e se recupera, e este tornar-se suscetível novamente à doença considerada. Para os parâmetros considerados a Figura 13 mostra que o número de indivíduos suscetíveis torna-se, até certo tempo, cada vez menor e depois a quantidade de indivíduos estabiliza. A quantidade de indivíduos recuperados aumenta isso acontece, pois o sistema é fechado. Além disso, a medida que o número de indivíduos recuperados aumenta, temos que a quantidade de pessoas infectadas diminuirá. A Figura 13 mostra esse processo em um curto período de tempo. Conclusão O Diagnóstico Médico fuzzy apresentado feito neste trabalho teve por finalidade imitar a atuação do médico em seus diagnósticos de doenças infantis. Inclusive, tivemos uma boa aproximação do diagnóstico fuzzy de cada paciente, com o diagnóstico dado pelo médico. Além disso, apresentamos os modelos SIR e SIRS que se relacionam com as epidemias das doenças consideradas e com isso obtivemos exemplos de casos de SIR, os quais as doenças que se encaixam neste modelo são catapora, caxumba e coqueluche. No segundo modelo SIRS, temos como exemplo a meningite. Em ambos os casos a solução do sistema de equação diferencial é dado por gráficos, os quais mostram o comportamento de cada epidemia, sendo a comunidade fechada, nos dois modelos.

Page 346: Numero 09 - Outubro de 2007

Bibliografia [1] Jafelice, R.M., L.C.Barros, R.C.Bassanezi, Teoria dos Conjuntos Fuzzy com Aplicações, Notas em matemática aplicada – SBMAC, editora Plêiade, São Carlos, SP, 2005. [2] Barros, L.C., R.C.Bassanezi, Tópicos de Lógica Fuzzy e Biomatemática, Campinas, SP, 2006. [3] http://www.lps.usp.br/neo/fuzzy/fuzzy_historico.htm [4] Massad, E., R. X. Menezes, P. S. P., Silveira, N. R. S. Ortega, Métodos quantitativos em Medicina, Barueri, SP, 2004.

Page 347: Numero 09 - Outubro de 2007

Fluxo Sanguíneo: Uma Aplicação da Integral de Riemann

Universidade Federal de UberlândiaFaculdade de Matemática

Mariana Fernandes dos Santos Villela Patrícia Borges dos [email protected] [email protected]

Rosana Sueli da Motta [email protected]

Introdução

Neste trabalho o fluxo sanguíneo será apresentado como uma aplicação da integral de

Riemann. Iniciamos com uma breve biografia de Riemann, e, em seguida com as noções

necessárias para a definição de integral através das somas de Riemann. A seguir, é dada uma

explicação do funcionamento do sistema circulatório, e a partir daí, apresentaremos a Lei de

Poiseuille. Esta lei foi descoberta por Jean Louis Poiseuille (1799-1869), fisiologista e físico

francês, e nos dá uma expressão da velocidade do sangue como função do afastamento em

relação ao eixo central da artéria. Com isso, utilizando a integral de Riemann encontraremos a

expressão do fluxo sanguíneo, o qual é o objetivo deste trabalho [2].

A História de Riemann

Georg Friedrich Bernhard Riemann, filho de um pastor luterano, foi educado em

condições modestas. Era uma pessoa tímida e fisicamente frágil. Com boa instrução em

Berlim e depois em Göttingen, obteve seu doutoramento com uma tese sobre teoria das

funções de variáveis complexas, onde aparecem as equações denominadas de Cauchy-

Riemann, embora lá fossem conhecidas por Euler e D'Alembert. Neste trabalho já estabelece

o conceito de superfície de Riemann que desempenharia papel fundamental em Análise.

Riemann foi nomeado professor na Universidade de Göttingen em 1854, apresentou

um trabalho perante o corpo docente e que resultou na mais célebre conferência da história da

Matemática. Nele estava uma ampla e profunda visão da Geometria e seus fundamentos que

até então permanecia marginalizada.

Ao contrário de Euclides e em sentido mais amplo do que Lobachevsky, observou que

seria necessário tratar-se de pontos, ou de retas, ou do espaço não no sentido comum, mas

Page 348: Numero 09 - Outubro de 2007

como uma coleção de n-uplas que são combinadas segundo certas regras, uma das quais, a de

achar distância entre dois pontos infinitamente próximos.

Para Riemann, o plano é uma superfície de uma esfera e reta, o círculo máximo sobre

a esfera. Com os estudos de espaços métricos em geral com curvatura, tornou-se possível a

teoria da relatividade, contribuindo assim para o desenvolvimento da Física.

Riemann conseguiu muitos teoremas em Teoria dos Números, relacionando-os com

Análise, onde encontramos também a equação de Cauchy-Riernann que é uma concepção

intuitiva e geométrica da Análise, em contraste com a aritmetização de Weierstrass.

Por volta de 1854, realizou um estudo bem mais aprofundado sobre a integral e em sua

homenagem a integral estudada por ele passou a receber o nome de Integral de Riemann. Tal

nome serve para distinguir essa integral de outras que foram introduzidas mais tarde, como

por exemplo, a Integral de Lebesgue. A forma usada para introduzir o conceito de Integral de

Riemann nos cursos de Cálculo é a versão devida a Cauchy. O que justifica isto é que, ela é

simples e bastante acessível aos alunos de um curso de inicial de Cálculo, além de atender aos

propósitos de um curso desta natureza.

Nos cursos de Análise Matemática apresenta-se uma versão mais refinada, a Integral

de Darboux-Riemann, usando os conceitos de soma inferior, soma superior, integral inferior e

integral superior, que correspondem ao método de exaustão usando, respectivamente,

polígonos inscritos e polígonos circunscritos.

Mas, para que ninguém alimente idéias equivocadas, observamos que as diversas

definições da Integral de Riemann mencionadas são equivalentes e a diferença entre elas se

situa na adequação das definições para a obtenção das propriedades da referida Integral.

Em 1859, Riemann foi nomeado sucessor de Dirichlet na cadeira de Göttingen já

ocupada por Euler. Com seu estado de saúde sempre precário, acabou por morrer em 1866 em

conseqüência de uma tuberculose.

Integral de Riemann

Seja f: [a,b] →R limitada não negativa, isto é, f(x) > 0 ou f(x) = 0 para todo x em [a,b]

e tomemos uma partição: x0 = a < x1 < ... < xn = b, do intervalo [a,b] que tenha todos os n

subintervalos com o mesmo comprimento( )b a

dxn

−= .

Tomaremos apenas os primeiros pontos da partição e faremos uma análise geométrica

da curva no subintervalo [xo,x1] (veja Figura 1). Para os outros subintervalos ocorre uma

Page 349: Numero 09 - Outubro de 2007

situação similar. A área sob a curva no intervalo [xo,x1] pode ser obtida através da área S1 do

retângulo cuja base mede dx = x1-xo e a altura é a linha tracejada cuja medida é dada por f(c1)

onde c1 é um ponto em [xo,x1].

Figura 1: Representação da soma das áreas dos retângulos sob a curva.

Existe uma compensação da área "branca" que fica acima da curva e dentro do

retângulo que fica abaixo da curva e fora do retângulo. Em cada subintervalo Ij=[xj,xj+1] desta

partição tomamos um ponto genérico qualquer cj e formamos n retângulos, todos com as

bases de medida dx e alturas dadas por:

f(c1), f(c2), ..., f(cn).

Se a partição tem n subintervalos, denotamos por Sn a soma das áreas dos n retângulos:

( )1 21

( ) ( ) ... ( )n

n n jj

S f c dx f c dx f c dx f c dx=

= + + + =∑

sendo a soma realizada sobre todos os j=1,...,n. Se essas somas forem calculadas para todos os

valores de n, formaremos uma seqüência:

{S1, S2, ..., Sn, ...}.

Se esta seqüência numérica {Sn} é convergente para um número real bem definido,

diz-se que f é integrável no intervalo [a,b], e o valor do limite desta seqüência é denotado por:

( ) ( )1

limnb

ja xj

f x dx f c dx→∞

=

= ∑∫ (1)

A expressão da esquerda é a integral de f entre os limitantes de integração a e b e a

expressão da direita é o limite da seqüência de somas parciais Sn.

A integral definida por (1) é denominada Integral de Riemann e as somas

( )1

n

n jj

S f c dx=

=∑são chamadas de somas de Riemann.

Page 350: Numero 09 - Outubro de 2007

Temos nesta definição uma partição muito particular do intervalo [a,b], subdividindo-o

em partes iguais, podemos refazer o processo com intervalos de comprimentos diferentes,

sendo cada intervalo da forma [xj,xj+1] e comprimentos dxj=xj+1-xj. Assim, as somas de

Riemann Sn tomam a forma

( )1 1 2 21

( ) ( ) ... ( )n

n n n j jj

S f c dx f c dx f c dx f c dx=

= + + + =∑

Ao proceder desta forma temos que tomar uma precaução adicional, ou seja, não basta

tomar o limite de Sn quando n → ∞ , mas temos que acrescentar a condição que o maior dos

comprimentos dx1, ..., dxn deve convergir para zero. Com isto, temos a notação:

( ) ( )0

1

limnb

j ja pj

f x dx f c dx→

=

= ∑∫onde |P|=max{dx1,...,dxn}, isto é, é a norma da partição P, [4].

O sistema circulatório

Os animais têm de realizar, interruptamente, trocas de substâncias com o ambiente,

pois todas as suas células precisam receber nutrientes e oxigênio, e eliminar gás carbônico e

outros resíduos tóxicos produzidos no metabolismo, e isso, no homem se dá pelo o sistema

circulatório fechado.

O sistema circulatório possui diversas funções, as quais são: o transporte de nutrientes,

o transporte de oxigênio, a remoção do gás carbônico, a remoção das excreções, o transporte

de hormônios e o transporte de células e de anticorpos do sistema imunológico.

Os componentes desse sistema são: sangue, vasos sangüíneos e o coração.

Sangue

O sangue humano é constituído por um líquido amarelado, o plasma, e por três tipos

de elementos celulares, as hemácias, os leucócitos e as plaquetas (veja Figura 2).

No plasma 92% de seu peso é água, sendo o restante devido à presença de proteínas,

sais e substâncias diversas, tais como nutrientes, gases, excreções e hormônios. As hemácias,

também conhecida como glóbulo vermelho, são células especializadas no transporte de

oxigênio, e já os leucócitos ou glóbulos brancos são células responsáveis pela defesa do

organismo. E, por fim, as plaquetas são pequenas células ovais, as quais participam

ativamente do processo de coagulação do sangue.

Page 351: Numero 09 - Outubro de 2007

Figura 2: Composição do sangue.

Artérias, veias e capilares sangüíneos

As artérias são vasos que levam sangue do coração para os órgãos e tecidos do corpo,

sua parede é espessa e contém três camadas de tecidos, o endotélio, tecido muscular liso e o

tecido conjuntivo.

Os capilares sangüíneos são vasos muito finos que ligam as arteríolas (artérias

finíssimas, que se encontra nos órgãos e tecidos) às vênulas (vasos muito finos que se

conectam, no lado oposto às arteríolas, aos capilares sanguíneos, que se unem para formar

veias progressivamente maiores).

As veias são vasos que levam o sangue dos órgãos e tecidos de volta ao coração. A

parede das veias é formada por três camadas, equivalentes às da artéria. Entretanto, as

camadas medianas e externas das veias são menos espessas. Nas veias encontram-se válvulas

que impedem o refluxo do sangue, o que garante a circulação em um único sentido (Figura 3).

Figura 3: Esquema ilustrando diferenças entre artérias, veias e capilares.

Page 352: Numero 09 - Outubro de 2007

Coração

O coração é um órgão musculoso, do tamanho aproximado de um punho fechado e

com peso aproximadamente de 400g. Ele apresenta quatro cavidades internas, denominadas

câmaras cardíacas, duas superiores que são os átrios e duas inferiores, os ventrículos (veja

Figura 4).

O átrio direito se comunica com o ventrículo direito por meio da válvula tricúspide e o

átrio esquerdo comunica com o ventrículo esquerdo pela válvula bicúspide, os quais têm

como funções garantir a circulação do sangue no coração em um único sentido, dos átrios

para os ventrículos.

As câmaras do coração contraem-se e dilatam-se alternadamente, em média, 70 vezes

por minuto. A contração de uma câmara cardíaca é denominada sístole e seu relaxamento

diástole. A freqüência cardíaca varia de acordo com o grau de atividades e situação emocional

em que se encontra uma pessoa, e este controle da freqüência é feito pelo nódulo sino-atrial.

Figura 4: Representação do coração.

Fisiologia da circulação sangüínea

A circulação sanguínea pode ser descrita de uma forma simples do seguinte modo: o

sangue, após ser oxigenado nos pulmões dirige-se para a aurícula esquerda do coração

passando pelas veias pulmonares. Em seguida, é transferido para o ventrículo esquerdo

através da válvula mitral e deste é bombeado para todo o corpo. À saída do ventrículo

esquerdo, passa pela válvula aórtica, que dá passagem para a artéria aorta e é conduzido

através de uma rede complexa de artérias cada vez menores, indo alimentar todas as células.

Após as trocas gasosas, de nutrientes e de detritos existentes ao nível celular, o sangue

Page 353: Numero 09 - Outubro de 2007

regressa ao coração através de veias cada vez de maior dimensão, até entrarem no coração

através da veia cava em direção à aurícula direita.

A passagem da aurícula direita para o ventrículo direito é feita através da válvula

tricúspide e, a partir do ventrículo direito, o sangue passa ainda na válvula pulmonar que dá

acesso à artéria pulmonar que o conduz no sentido dos pulmões onde será oxigenado [1].

A biofísica da circulação sanguínea

A hidrodinâmica é a área da mecânica dos fluidos que estuda o seu movimento.

Existem essencialmente dois tipos de fluidos, um que é considerado ideal, ou seja, que não

tem viscosidade e os fluidos viscosos, aqueles que apresentam viscosidade.

A viscosidade é a grandeza que mede a fricção existente entre camadas adjacentes de

um fluido ou, de um ponto de vista prático, é a dificuldade ou facilidade com que um fluido

escorre.

A maioria dos fluidos apresenta viscosidade, em particular, a grande parte dos fluidos

biológicos, cujo exemplo que nos interessa é o sangue. Estes são caracterizados por uma

viscosidade não desprezível.

A conseqüência mais visível de se considerar a viscosidade de um fluido num

escoamento é o seu perfil de velocidade. Também relacionado com a viscosidade do fluido

está o tipo de escoamento que este apresenta. Na verdade, em fluidos reais, com viscosidade

não nula, verifica-se que para valores de velocidade do fluido abaixo de certo valor, o

escoamento é considerado laminar, isto é, todas as partículas do líquido se movem

paralelamente ao tubo e a velocidade aumenta uniformemente a partir de zero na parede, em

direção ao centro. No entanto, quando esse valor é ultrapassado, o escoamento passa a ser

turbulento.

No corpo humano a pressão do sangue se deve a contribuição da pressão estática, da

pressão dinâmica e da pressão mecânica. Em virtude do próprio peso do sangue as artérias e

veias estão sob a pressão estática, que dependerá da altura da coluna de sangue em relação ao

pé. A contribuição da pressão dinâmica é em virtude das diversas velocidades do sangue no

corpo. O efeito da pressão mecânica é em virtude do coração, que ao bombear o sangue para o

corpo está lhe exercendo certa pressão. No percurso do sangue haverá variações de pressão

sangüínea pelo corpo, muito em virtude dos efeitos da viscosidade. Um outro fato interessante

é que a pressão do sangue arterial (sangue rico em oxigênio) é maior que a do sangue venoso

Page 354: Numero 09 - Outubro de 2007

(sangue rico em gás carbônico). Isto se deve ao fato do sangue arterial ter o auxílio do coração

para ser bombeado para o resto do corpo, o que não ocorre com o sangue venoso.

Para aplicar à circulação sanguínea alguns dos resultados da hidrodinâmica, é

necessário analisar as propriedades do sangue e assumir algumas aproximações. Antes de

tudo, deve ter-se presente que o sangue, embora seja considerado como um fluido

homogêneo, na verdade, é constituído por diversas partículas em suspensão, o que, do ponto

de vista de análise do seu escoamento, torna a sua descrição particularmente difícil,

nomeadamente, quando os vasos que o conduzem são muito estreitos.

Um segundo ponto, prende-se com a elasticidade dos vasos que conduzem o sangue.

Apesar de se aceitar, que o sangue circula através de tubos rígidos, esta aproximação não é

verdadeira, uma vez que, como se sabe, as paredes dos vasos são extremamente elásticas,

sendo, inclusivamente, um fator importante de regulação do fluxo sanguíneo [6].

O cientista francês Jean Louis Poiseuille (1799-1869) se interessou bastante por

questões relacionadas com a circulação sanguínea e determinou experimentalmente como

variava a velocidade do sangue, o que posteriormente pôde ser deduzido teoricamente.

Consideremos o fluxo de sangue em um vaso sanguíneo. Um segmento de uma artéria

ou de uma veia pode ser encarado como um tubo cilíndrico de diâmetro constante.

Admitamos que a seção transversal seja um círculo de raio R. O sangue possui viscosidade

que é representado por η (letra grega eta). A viscosidade é medida em poise∗, a qual é cm-1 g

s-1 no sistema CGS (cm = centímetro, g = grama, s = segundo).

Também há atrito nas paredes do tubo. A velocidade do sangue em contato com a

parede do vaso é zero e a velocidade é máxima ao longo do eixo do centro do tubo. O fluxo

sanguíneo pode ser laminar, quando os vasos sanguíneos estão em condições normais, ou

turbulento, por exemplo, em um vaso que é parcialmente obstruído.

Agora admitamos um fluxo laminar. Seja r a distância a qualquer ponto do líquido a

partir do eixo do tubo (veja Figura 5). Então a velocidade v é uma função de r. Podemos

escrever v = v(r). O domínio da função é o intervalo 0 ≤ r ≤ R. Então a velocidade v (cm s-1)

é

2 2Pv = (R - r )

4ηL

Δ

onde L representa o comprimento do tubo (cm), PΔ a diferença de pressão entre os dois

extremos do tubo (cm-1 g s-2), R e η foram definidos anteriormente.

∗ A palavra poise para a unidade de viscosidade é uma abreviação de Poiseuille, vide bibliografia [5].

Page 355: Numero 09 - Outubro de 2007

Figura 5: Representação da distância r a partir do eixo do tubo.

Claramente, v = 0 para r = R. Para r =0 a velocidade alcança o seu máximo. Então a

imagem da função é 0 ≤ v ≤ 2PR

4ηL

Δ.

Para conceituar a lei de Poiseuille, é importante definir primeiramente fluxo, o qual é a

quantidade de fluido que passa por um determinado ponto da circulação em um dado período

de tempo. Da mesma forma esta definição serve para o fluxo sangüíneo que é, geralmente,

expresso em mililitros, ou litros por minuto e no total da circulação de uma pessoa adulta em

repouso é de cerca de 5.000 mL por minuto. A isto denomina-se débito cardíaco, porque

constitui a quantidade de sangue bombeada por cada ventrículo do coração num período

unitário de tempo. Portanto, é claro que essa mesma quantidade de sangue deve passar através

de ambas as circulações sistêmica e pulmonar.

O fluxo sanguíneo varia bastante nos diferentes tecidos e em determinados tecidos

necessitam de um fluxo bem maior do que outros. Tecidos como músculos esqueléticos

apresentam grandes variações no fluxo sanguíneo através dos mesmos em diferentes

situações: Durante o repouso o fluxo é relativamente pequeno, mas aumenta

significativamente durante o trabalho, quando o consumo de oxigênio e demais nutrientes

aumenta e a produção de gás carbônico e outros elementos também aumenta.

Com isso, a seguinte lei definida por Poiseuille é que o fluxo ϕ de um tubo cilíndrico

transportando um líquido viscoso com o raio R, comprimento L, pressão ΔP e coeficiente de

viscosidade η:

4RP.

8ηL

πϕ = Δ

Esta lei tem extrema importância para o estudo do fluxo sanguíneo [3], e será deduzida

na próxima secção.

Page 356: Numero 09 - Outubro de 2007

Fluxo Sanguíneo: Uma Aplicação da Integral de Riemann

Vamos agora calcular o volume de sangue que flui através de uma seção da artéria, ou

seja, o fluxo sanguíneo. Para tanto, dividamos o intervalo 0 < r < R em n subintervalos iguais,

de comprimento Δr, tal que rj seja o início do j-ésimo subintervalo. Estes subintervalos

determinam n anéis concêntricos, conforme Figura 6:

Figura 6: Representação de uma artéria subdividida em anéis concêntricos.

Quando Δr é pequeno, a área do j-ésimo anel é aproximadamente igual à área de um

retângulo cujo comprimento é a circunferência do menor perímetro do anel e cuja largura é

Δr, isto é,

jÁrea do j-ésimo anel 2 r rπ≅ Δ .

A multiplicação da área do j-ésimo anel (cm2) pela velocidade do fluxo sanguíneo

através dele fornece a razão (cm3 s-1) com que o sangue escoa. Como a velocidade do sangue

através do j-ésimo anel é aproximadamente igual a v(rj) cm s-1, segue-se que:

Fluxo sanguíneo

através do j-ésimo anel

⎛ ⎞⎜ ⎟⎝ ⎠

área do velocidade do sangue

j-ésimo anel através do j-ésimo anel

⎛ ⎞⎛ ⎞≅ ⎜ ⎟⎜ ⎟⎝ ⎠⎝ ⎠

( ) ( )j j2 r r v rπ≅ Δ

( ) ( )2 2j j

P2 r r R - r

4ηLπ

⎡ ⎤Δ≅ Δ ⎢ ⎥

⎣ ⎦

( )2 3j j

P2 R r - r r

4ηLπ Δ

≅ Δ

Page 357: Numero 09 - Outubro de 2007

O fluxo através da seção inteira é a soma das razões associadas a cada um dos n anéis

concêntricos, ou seja,

( )2 3j j

1

PFluxo 2 R r - r r

4ηL

n

j

π=

Δ≅ Δ∑

Por conseguinte, quando n cresce ao infinito, o somatório tende para o valor verdadeiro do

fluxo,

( )2 3j j

1

PFluxo = = lim 2 R r - r r

4ηL

n

nj

ϕ π→∞

=

ΔΔ∑

( )R 2 3

0

P= 2 R r - r r

4ηLdπ Δ

2 42

0

P R r= 2 r -

4ηL 2 4

R

π⎛ ⎞Δ⎜ ⎟⎝ ⎠

43R

= P /8ηL

cm sπ

Δ

Esta é a expressão matemática da Lei de Poiseuille. A dependência com o inverso da

viscosidade e do comprimento do tubo é natural: quanto mais comprido for o tubo, para uma

mesma diferença de pressão, menor deverá ser o fluxo. O mesmo se aplica à dependência com

a viscosidade: quanto mais viscoso for o fluido, menor deverá ser o fluxo. Curiosa é a

dependência do fluxo sanguíneo com o raio da seção reta ser com a quarta potência de R, [2] e

[3]!

Exemplo

Para termos uma visão mais ampla da Lei de Poiseuille, e da expressão da velocidade,

estudaremos um exemplo numérico, o qual foi escolhido por ser o mais realista possível.

Consideremos o sangue arterial com sua maior concentração de O2 ligado à

hemoglobina. Para o sangue humano sua viscosidade é um pouco inferior, à do sangue

venoso, em média η = 0,027 poise. O sangue flui através de uma arteríola (capilar arterial

largo) de comprimento L = 2 cm e raio R = 8 × 10-3 cm. Em uma extremidade, a pressão é

maior do que a outra e essa diferença é PΔ = 4 × 103 cm-1 g s-2. Então a velocidade é dada

por:

Page 358: Numero 09 - Outubro de 2007

( ) ( ) ( )3

2 2 -6 2 -1 4 2 -1P 4 × 10v = (R - r ) = (64 × 10 - r ) cm s v = 1,185 - 1,85× 10 r cm s

4ηL 4 × 0,027 × 2

Δ⇒

e a seguir ilustraremos a dependência da velocidade com a distância a qualquer ponto do

líquido a partir do eixo do tubo, ou seja, r, com o gráfico da Figura 7:

0 1 2 3 4 5 6 7 8

x 10-3

0

0.2

0.4

0.6

0.8

1

1.2

1.4

Distancia (cm)

Vel

ocid

ade

(cm

/s)

Figura 7: Gráfico da velocidade x distância em relação ao eixo central da artéria.

E o fluxo é dado por:

( )4 -3 4 -12

3 3 -4 3 1R × (8 × 10 ) × 4096 × 10= P = (4 × 10 ) = (4 × 10 ) = 1,1914 × 10

8ηL 8 × 0,027 × 2 0,432cm s

π π πϕ ϕ −Δ ⇒

Veja no gráfico da Figura 8 como o fluxo varia de acordo com a variação do raio da

arteríola.

Page 359: Numero 09 - Outubro de 2007

0 1 2 3 4 5 6 7 8

x 10-3

0

0.2

0.4

0.6

0.8

1

1.2x 10

-4

Raio (cm)

Flu

xo(c

m3 /s

)

Figura 8: Gráfico do fluxo x raio da artéria.

Artéria aorta e o fluxo sanguíneo

Vamos analisar o que se passa ao nível da artéria aorta, lembrando que artéria aorta é a

mais importante artéria do sistema circulatório do corpo humano. Dela se derivam todas as

outras artérias do organismo. A aorta se inicia no coração, na base do ventrículo esquerdo, e

termina à altura da quarta vértebra lombar, onde se divide nas artérias ilíacas comuns.

O poise (P), como havíamos dito, é a unidade de viscosidade dinâmica no sistema

CGS de unidades. A unidade análoga no Sistema Internacional de Unidades é o Pascal

segundo (Pa·s):1 Pa·s = 1 kg·m−1·s−1 = 10 P (poise) [5]. Assim, tendo em vista que o diâmetro

da artéria aorta é cerca de 2 cm, admitindo que o seu comprimento é aproximadamente 40 cm,

e que a diferença de pressão é 32.6 Pa, sabendo que a viscosidade do sangue é de

aproximadamente de η = 4 × 10-3 Pa s, facilmente se calcula a velocidade do sangue que nela

circula no eixo central e o fluxo sanguíneo:

( ) ( )22 2 -1-3

P 32.6v = (R - r ) = 0.01 v = 0,51 m s

4ηL 4 × 4 × 10 × 0.4

Δ⇒ e

( )4 -2 4

-5 3 1-3

R × (10 )= P = (32,6) = 8 × 10

8ηL 8 × 4 × 10 × 0,4m s

π πϕ ϕ ϕ −Δ ⇒ ⇒

Page 360: Numero 09 - Outubro de 2007

Infarto

O miocárdio (músculo do coração) recebe alimentos e oxigênio através das artérias

coronárias, os primeiros ramos da aorta, ou seja, o coração é o primeiro a usufruir de seu

próprio trabalho. Uma das formas de ocorrer um infarto é pelo acúmulo de colesterol

(lipoproteína de alta densidade) que pode se acumular nas paredes da aorta, dificultando a

nutrição do miocárdio, e, isto ocasiona uma redução na área transversal da aorta e produz uma

pressão dinâmica maior, ocasionando uma redução na pressão mecânica (veja Figura 9). Com

uma redução da pressão mecânica, ocorre um refluxo na coronária e conseqüentemente uma

isquemia (suspensão localizada de irrigação sanguínea devida à má perfusão circulatória

arterial). Sem receber nutriente e oxigênio o músculo cardíaco morre, isto é, ocorre o infarto

agudo do miocárdio. Existem outras formas de ocorrer infarto, como por exemplo, pela

aterosclerose (entupimento) das coronárias em virtude do acúmulo de LP(a) e LDL, dois tipos

de colesteróis, chamados de maus colesteróis.

Figura 9: Representação do coração após ocorrer infarto, e em destaque a artéria obstruída.

Com esta definição, vamos verificar o que acontece com o fluxo sanguíneo caso haja

um entupimento parcial de uma artéria.

Suponha que ocorra uma obstrução nessa artéria de 25%, assim vamos modelar duas

situações de obstrução cujas aproximações serão descritas nas Figuras 10 e 11.

Page 361: Numero 09 - Outubro de 2007

Assim, como feito anteriormente, vamos agora calcular o volume de sangue que flui

através de uma seção da artéria obstruída, ilustrada na Figura 10. Dividindo o intervalo

0 < r < R em n subintervalos iguais, de comprimento Δr, tal que rj seja o início do j-ésimo

subintervalo. Assim, a área do j-ésimo anel é aproximadamente igual à área de um retângulo

cujo comprimento é a circunferência do menor perímetro do anel e cuja largura é Δr. A

circunferência do menor perímetro do anel que estamos considerando agora é dado

por: j j j

32 r r r

2 2

π ππ − ≅ , e então a j

3área do j-ésimo anel é r r

2

πΔ .

Figura 10: Representação uma artéria com 25% de obstrução na área da seção transversal.

Logo, podemos obter o fluxo sanguíneo:

Fluxo sanguíneo

através do j-ésimo anel

⎛ ⎞⎜ ⎟⎝ ⎠

área do velocidade do sangue

j-ésimo anel através do j-ésimo anel

⎛ ⎞⎛ ⎞≅ ⎜ ⎟⎜ ⎟⎝ ⎠⎝ ⎠

( )j j

3r r v r

2

π⎛ ⎞≅ Δ⎜ ⎟⎝ ⎠

( )2 2j

3r r R -

2 jk rπ⎛ ⎞ ⎡ ⎤≅ Δ⎜ ⎟ ⎣ ⎦⎝ ⎠

( )2 3j j

3R r - r r

2k

π≅ Δ

onde,P

4ηLk

Δ= .

Assim como feito anteriormente, integrando essa expressão obteremos:

433 R

Fluxo = = /8

kcm s

πϕ .

Page 362: Numero 09 - Outubro de 2007

Da mesma maneira, vamos agora calcular o volume de sangue que flui através de uma

seção da artéria obstruída, ilustrada na Figura 11.

Figura 11: Representação uma artéria com 25% de obstrução na área da seção transversal.

Como a área da seção transversal da artéria será reduzida de 25% teremos:

2

2

R 100% 3

475%t R

t

ππ

−⇒ =

−.

Desse modo, a velocidade será dada por:

2 2 2 21 1

3v = (t - r ) v = R - r

4k k

⎛ ⎞⇒ ⎜ ⎟⎝ ⎠

,

onde, 1

P

4ηLk

Δ= .

Então o fluxo será dado por:

2 31 j j

1

3Fluxo = = lim 2 R - r

4

n

nj

k r rϕ π→∞

=

⎛ ⎞Δ⎜ ⎟⎝ ⎠

3R 4 2 310

3= 2 R - r

4k r r dπ ⎛ ⎞⎜ ⎟⎝ ⎠∫

3 42 4

21

0

3= 2 R -

4 2 4

R

r rkπ⎛ ⎞⎜ ⎟⎝ ⎠

41135 R

=512

kπ.

Page 363: Numero 09 - Outubro de 2007

Anemia

A anemia é uma anomalia caracterizada pela diminuição da concentração da

hemoglobina dentro das hemácias e pela redução na quantidade de hemácias no sangue. Isso

resulta em uma redução da capacidade do sangue em transportar o oxigênio aos tecidos, pois a

hemoglobina, uma proteína presente nas hemácias, é responsável pelo transporte de oxigênio

dos pulmões para os demais órgãos e tecidos e de dióxido de carbono destes para ser

eliminado pelo pulmão.

Os sintomas da anemia são variáveis, sendo os mais comuns fadiga, fraqueza, palidez

(principalmente ao nível das conjuntivas), déficit de concentração ou vertigens. Nos quadros

mais severos podem aparecer taquicardia, palpitações. Afeta também a gengiva (causando, em

casos mais graves, o seu sangramento).

Um dos sintomas acima, a taquicardia, se deve ao fato do sangue de uma pessoa

anêmica apresentar menor viscosidade e, consequentemente, um maior fluxo através de seus

vasos.

Desse modo, para verificar esse fato, usamos a equação de fluxo, assim:

43R

Fluxo = P /8ηL

cm sπ

Δ

com R o raio da artéria, L o comprimento da artéria, η a viscosidade do sangue e PΔ a

variação da pressão.

Como uma pessoa anêmica tem uma menor viscosidade, pela equação percebemos

que:

43R

Fluxo = P /8ηL

cm sπ

↑ Δ↓

diminui o valor do denominador, e então haverá um aumento do fluxo. Isto justifica o

aumento dos batimentos cardíacos.

Conclusão

Neste trabalho estudamos uma aplicação da Integral de Riemann em um fenômeno

biológico, demonstrando a Lei de Poiseuille e a fórmula do Fluxo Sanguíneo.

A partir destes conhecimentos modelamos duas situações de obstrução de uma artéria

e calculamos a “nova” fórmula do fluxo sanguíneo desta artéria obstruída.

Page 364: Numero 09 - Outubro de 2007

Referências Bibliográficas

[1] Amabis & Martho, Biologia do organismo 2. Editora Moderna, volume único.

[2] Batschelet, E., Introdução à matemática para biocientistas. São Paulo: EDUSP,1978.

[3] Hoffman, L. D.; Bradley, G. L., Cálculo: um curso moderno e suas aplicações. Rio deJaneiro: LTC,2002.

[4] http://wikipedia.org/wiki/Integral_de_Riemann

[5] http://pt.wikipedia.org/wiki/Poise

[6]http://w3.ualg.pt/~cmsilva/documentos/AulaTP_1_F%C3%ADsica_M%C3%A9dica.pdf

Page 365: Numero 09 - Outubro de 2007

O Uso de Modelagem Matemática no Cálculo do

Volume de uma Maçã Universidade Federal de Uberlândia

Faculdade de Matemática

Alessandra Ribeiro da Silva

[email protected]

Carlos Henrique Tognon

[email protected]

Milena Almeida Leite Brandão

[email protected]

Rosana Sueli da Mota Jafelice

[email protected]

IntroduçãoPresume-se que o cultivo da macieira (Figura 1), tenha-se iniciado há 25 milhões de

anos, tendo como centro de origem a região entre o Cáucaso e o leste da China. No império Romano, a cultura da macieira já estava bastante difundida. No entanto, é muito provável que o desenvolvimento das espécies atuais tenha-se iniciado após o final da última era glacial, portanto, há 20.000 anos. As migrações dos povos euroasiáticos colaboraram para a disseminação das formas primitivas das macieiras atuais.

Figura 1 e 2 - Macieira florida e plantação de maçãs, respectivamente [4].

O início das plantações de maçã no Brasil (Figura 2) ocorreu, provavelmente no município de Valinhos, estado de São Paulo, pelo fruticultor Batista Bigneti que, em 1926,tinha plantas da Cultivar Ohio Beauty.

Com a criação em 1928 da Estação Experimental de São Roque, em São Paulo, pelo Instituto Agronômico de Campinas, foi dado o passo inicial na pesquisa sobre macieira no Brasil.

Page 366: Numero 09 - Outubro de 2007

ObjetivosEste trabalho teve com objetivo calcular o volume de uma maçã utilizando vários

métodos e modelar o processo de resfriamento da maçã através da formulação de uma equação que expresse seu comportamento.

Considerações

Desde o plantio até a armazenagem da maçã, há vários fatores que podem ser analizados, por exemplo a escolha de terreno, o solo, a aração, herbicídas, colheita e armazenagem. Mas consideraremos apenas este último.

O objetivo do armazenamento é manter a qualidade interna e externa da fruta, assegurarando o perfeito funcionamento das câmaras de conservação, por meio da observação periódica dos equipamentos de refrigeração e controle de gases.

O armazenamento das frutas é feito nas câmaras frigoríficas. Antes de entrar na câmara fria, a maçã recebe um banho, atravessando um tanque de água gelada (-3°C), sobre uma esteira circulante, durante 25 minutos, saindo numa temperatura média de 6.5°C.

A temperatura média da câmara é de 1.5° C e tem capacidade para armazenar 600 bins(caixas). As maçãs podem permanecer na câmara de 5 a 8 meses até a sua comercialização. Se as maçãs forem comercializadas imediatamente após a colheita, então dispensa-se o trabalho do banho e do armazenamento em câmaras.

Inicia-se então a secagem e classificação. As frutas são retiradas da câmara fria e levadas para o classificador onde são separadas as estragadas. Recebem um jato de água passando dali para a desumidificação e polimento. Em seguida, vão para o secador com temperatura de 45°C e, finalmente, é feita a classificação.

A classificação é feita pelo peso e também pelo tamanho das maçãs que são acondicio-nadas em caixas com capacidade de 20kg. Cada caixa comporta de 88 a 250 unidades.

Curiosidades

1) Há mais de 7.500 espécies e variedades de maçãs, veja Figura 3. As diferentes espécies encontram-se em climas temperados e subtropicais.

Figura 3 - Variedades de maçãs [4].

2) As macieiras não florescem em áreas tropicais, por exemplo, as variedades da família Gala necessitam de um inverno com cerca de 700 horas de frio com temperaturas de 7,2°C;

3) A maçã fermentada é utilizada para elaborar bebidas alcoólicas (Figura 4), como a sidra asturiana, o Calvados francês e a sagardua basca;

Page 367: Numero 09 - Outubro de 2007

Figura 4 - Elaborado de Normandia.

4) A maçã possui as seguintes vitaminas: B1, B2 e Niacina, e também contém sais minerais como Fósforo e Ferro.

Nota Histórica e Definições

Para uma melhor compreensão do conteúdo deste trabalho, faz-se necessário neste momento uma introdução histórica no que diz respeito ao assunto Cálculo Diferencial e Integral, alguns resultados sobre centróides, o Teorema de Pappus e um dos princípios fundamentais da hidrostática. É o que se segue imediatamente.

A derivada e a integral são duas noções básicas do Cálculo Diferencial e Integral. Do ponto de vista geométrico, a derivada está ligada ao problema de traçar a tangente a uma curva enquanto que a integral está relacionada com o problema de determinar a área de certas figuras planas, mas também possui muitas outras interpretações possíveis.

O Cálculo Diferencial e Integral foi criado por Isaac Newton (1642-1727) e Wilhelm Leibniz (1646-1716). O trabalho destes cientistas foi uma sistematização de idéias e métodos surgidos principalmente ao longo dos séculos XVI e XVII, os primórdios da chamada “Era da Ciência Moderna”, que teve início com a Teoria heliocêntrica de Copérnico (1473-1543). Na realidade, a grande descoberta de Newton e de Leibniz foi que a Matemática, além de lidar com grandezas, é capaz de lidar com a variação das mesmas.

A idéia básica do conceito de integral já estava embutida no método da exaustão atribuído a Eudoxo (406-355 a.C.), desenvolvido e aperfeiçoado por Arquimedes (287-212 a.C.), grande matemático da escola de Alexandria. O método da exaustão consiste em "exaurir" a figura dada por meio de outras de áreas e volumes conhecidos. O inconveniente do método de exaustão de Arquimedes é que para cada novo problema havia a necessidade de um tipo particular de aproximação.

O que permitiu a passagem do método de exaustão para o conceito de integral foi a percepção que em certos casos, a área da região pode ser calculada sempre com o mesmo tipo de aproximação por retângulos (Figura 5).

Page 368: Numero 09 - Outubro de 2007

Figura 5 - Calculando área por aproximação de retângulos.

Esta foi uma descoberta conceitual importante, mas em termos práticos, a descoberta fundamental foi a possibilidade de exprimir a integral de uma função em termos de uma primitiva da função dada e este fato é conhecido pelo nome de Teorema Fundamental do Cálculo. A idéia ou o conceito de integral foi formulado por Newton e Leibniz no século XVII, mas a primeira tentativa de uma conceituação precisa foi feita por volta de 1820, pelo matemático francês Augustin Louis Cauby (1789-1857). Os estudos de Cauchy foram incompletos, mas muito importantes por terem dado início à investigação sobre os fundamentos do Cálculo Integral, levando ao desenvolvimento da Análise Matemática e da teoria das funções.

Por volta de 1854, o matemático alemão Bernhard Riemann (1826-1866) realizou um estudo bem mais aprofundado sobre a integral e em sua homenagem a integral estudada por ele passou a receber o nome de Integral de Riemann. Tal nome serve para distinguir essa integral de outras que foram introduzidas mais tarde, como por exemplo, a Integral de Lebesgue. A forma usada para introduzir o conceito de Integral de Riemann nos cursos de Cálculo é a versão devida a Cauchy. O que justifica isto é que, ela é simples e bastante acessível aos alunos de um curso inicial de Cálculo, além de atender aos propósitos de um curso desta natureza.

Agora veremos como a integração pode ser utilizada no cálculo de centróides. Considere a distribuição contínua de massa numa região R (chapa fina de material homogêneo) do plano xy com densidade superficial massa por unidade de área) constante, conforme a Figura 6.

Figura 6 - Uso de integração para o cálculo de centróides [3].

O momento dessa região em relação ao eixo y e em relação ao eixo x é dada pelas expressões:

b

ay dxxfxM )(

d

cx dyygyM )(

Page 369: Numero 09 - Outubro de 2007

respectivamente, onde f(x)dx é a área do retângulo vertical e sua massa é f(x)dx, g(y)dy é a área do retângulo horizontal e sua massa é g(y)dy.

A massa total da chapa pode evidentemente ser expressa de duas maneiras,

( ) ( ) .b d

a c

m f x dx g y dy

O centro de massa ,x y da chapa é agora definido por

( )

( )

b

yab

a

x f x dxM

xm

f x dx e

( )

( )

d

c xd

c

y g y dyMym

g y dy.

Como a densidade é constante podemos eliminá-la por cancelamento e as fórmulas tornam-se:

( )

( )

b

ab

a

xf x dxx

f x dx e

( )

( )

d

cd

c

yg y dyy

g y dy.

Exemplos

1) Cálculo do centróide de um retângulo. Considere o retângulo de altura h e base b e portanto de área hb, conforme Figura 7.

Figura 7 - Centróide de um retângulo [3].

2 20

0

1 1 1 1 1Temos: e de modo análogo, encontramos2 2 2

1 1 1 , logo o centróide é o ponto , que é obviamente o centro do retângulo.2 2 2

b

bx hdxx hx hb b

hb hb hb

y h b h

Page 370: Numero 09 - Outubro de 2007

2) Determinar o centróide da região do primeiro quadrante limitada pelos eixos e pela curva y = 4 - x2, conforme Figura 8.

Figura 8 - Centróide da região do primeiro quadrante limitada pelos eixos e pela curva y = 4 - x2 [3].

222 3

00

222 2 4

00

1 16 Usando o retângulo vertical, vemos que a área da região é (4 ) 4 . 3 3

3 3 1 3 Logo, (4 ) 2 . 16 16 4 4

Analogamente,

A x dx x x

xdAx x x dx x x

A4

0

3 usando um retângulo horizontal, temos 4 . 16

Para calcular essa integral, fazemos a substituição u = 4 - y. Assim, y = 4 - u e dy = -du e os novos limites de

xdAy y ydy

A

44 4 41 2 1 2 3 2 3 2 5 2

00 0 0

integração serão 4 e 0:

3 3 3 3 8 2 3 64 64 84 (4 )( ) (4 ) .16 16 16 16 3 3 16 3 5 5

3 8 Portanto, o centróide é o ponto , .4 5

y y ydy u u du u u du u u

Dois belos teoremas geométricos relacionando centróides com sólidos e superfícies

de revolução foram descobertos no século quatro antes de Cristo, por Pappus de Alexandria, o último dos grandes matemáticos gregos. Neste trabalho utilizaremos apenas um deles que passamos a descrever.

Primeiro Teorema de Pappus: Considere uma região plana que está inteiramente de um lado de uma reta do plano. Se essa região é girada ao redor da reta que desempenha a função de eixo, então o volume do sólido gerado dessa maneira é igual ao produto da área da região pela distância percorrida pelo centróide ao redor do eixo[3].

Voltemos nossa atenção agora para outro matemático grego, Arquimedes (287 a.C. - 212 a.C.), este, além de matemático era inventor. Nasceu na cidade-estado grega de Siracusa, na ilha da Sicília e foi o mais importante matemático da Antiguidade. Em Física, no seu Tratado dos Corpos Flutuantes, estabeleceu as leis fundamentais da

Page 371: Numero 09 - Outubro de 2007

estática e da hidrostática. Um dos princípios fundamentais da hidrostática é assim enunciado: "todo corpo mergulhado total ou parcialmente em um fluido sofre uma impulsão vertical, dirigido de baixo para cima, igual ao peso do volume do fluido deslocado, e aplicado no centro de impulsão." O centro de impulsão é o centro de gravidade do volume que corresponde à porção submersa do corpo. Isto quer dizer que, para o objeto flutuar, o peso da água deslocada pelo objeto tem de ser maior que o próprio peso do objeto. Conta-se que certa vez, Hierão, rei de Siracusa, no século III a.C. havia encomendado uma coroa de ouro, para homenagear uma divindade que supostamente o protegera em suas conquistas, mas foi levantada a acusação de que o ourives o enganara, misturando o ouro maciço com prata em sua confecção. Para descobrir, sem danificar o objeto, se o seu interior continha uma parte feita de prata, Hierão pediu a ajuda de Arquimedes. Este pôs-se a procurar a solução para o problema, a qual lhe ocorreu durante um banho. A lenda afirma que Arquimedes (Figura 9) teria notado que uma quantidade de água correspondente ao seu próprio volume transbordava da banheira quando ele entrava nela e que, utilizando um método semelhante, poderia comparar o volume da coroa com os volumes de iguais pesos de prata e ouro: bastava colocá-los em um recipiente cheio de água, e medir a quantidade de líquido derramado. Feliz com essa fantástica descoberta, Arquimedes teria saído à rua nu, gritando Eureka! Eureka! (Encontrei! Encontrei!).

Figura 9 - Arquimedes.

Outro matemático importante foi Pappus de Alexandria (Figura 10) e foi conhecido por seu trabalho Synagoga ou Coleção. Ele foi um egípcio helenizado nascido em Alexandria, Egito. Entretanto, muito pouco se conhece sobre sua vida e os escritos gravados sugerem que ele era professor.

Page 372: Numero 09 - Outubro de 2007

Figura 10 - Pappus de Alexandria.

Vejamos agora algumas definições que serão necessárias para o cálculo do volume de

um sólido de revolução.

1-iiii

n10

i1-i1-n10

1n0

xxxonde1},xmax{:pordadaéb],[a,de},x,...,x,{xpartiçãoumadenormaA)2

.1,x xex...xxase},,...,{ntopontilhamecomb],[a,fechadointervalodopartiçãoumaé}x,...,{x1)

ni

nib i

n

Figura 11 - Sólido obtido por rotação de uma curva.

ba dx. Veja Figura 11.

n

i ixif 2f(x)][1

2))((Vpordado

volumepossuib, xea xretasaseeixoof(x),ycurvapelalimitada

regiãodaeixo,dotornoemrotaçãopelaobtidorevoluçãodesólidoO

b].[a,x0,f(x)que talecontínuaRb][a,:fSeja3)

0lim

Page 373: Numero 09 - Outubro de 2007

Exemplos

644

)14

0

44

0

34

0

2

23

23 xdxxdxxVxy

0 0.5 1 1.5 2 2.5 3 3.5 40

1

2

3

4

5

6

7

8

Figura 12 - Gráfico da função y = x1.5.

34

322)2

3

0

3222

0

22222 axxadxxaVxayayxaa

Figura 13 - Uso da integração para o cálculo do volume de uma esfera [3].

Metodologia A aproximação do volume de uma maçã será feita utilizando-se conceitos de cálculo

diferencial e integral, conhecimentos de geometria espacial e um teorema, conhecido como teorema de Pappus.

É importante também ressaltar que a maioria dos problemas levantados neste processo de modelagem diz respeito à geometria do objeto em estudo, no caso a maçã. Este destaque para a parte visual é importante, visto que assim se consegue uma melhor compreensão do que está acontecendo além de aguçar a imaginação geométrica.

Para modelar o processo de resfriamento da maçã serão utilizadas equações de diferenças [1].

Os modelos matemáticos utilizados para o cálculo do volume de uma maçã estão colocados em uma seqüência que obedece a um nível gradativo de dificuldade e complexidade conceitual.

No entanto, isto não significa necessariamente que o resultado obtido para a

Page 374: Numero 09 - Outubro de 2007

aproximação do volume da maçã seja tão mais preciso quanto maior for a complexidade do modelo.

Desenvolvimento Existem vários métodos matemáticos para calcular o volume de uma maçã. Logo,

escolhemos os seguintes métodos para este cálculo: teorema de Pappus, fórmula do volume da esfera, fatiando uma maçã e usando integração. Este estudo foi realizado baseado em um modelo apresentado em [1].

1. Problema: Como calcular o volume de uma maçã?

Figura 14 - Etapas de uma modelagem [1].

1º Método: Utilizando a fórmula do volume da esfera

Envolvendo a maçã com um barbante (Figura 15) obtemos uma circunferência cujo comprimento é de 26.2cm. Sabendo que o comprimento de uma circunfência é dado pela fórmula 2 R temos que R = 4.1698cm.

34 Volume da esfera: 3

V r .

Volumeda esfera!

Fatiando amaçã!

Teorema dePappus!

Integração!

Page 375: Numero 09 - Outubro de 2007

Figura 15 - Medindo a circunferência da maçã com um barbante [1].

Aplicando a fórmula do volume de uma esfera obtemos um valor "aproximado" superior ao volume da maçã:

3 3max 4 3.1416 (4.1698) 3 303.6934 .V cm

Cortando-se a maçã ao meio (no sentido longitudinal), mede-se o raio r do círculo inscrito na face plana da maçã: r = 2.95cm, e obtém-se um valor mínimo para o volume da maçã:

33min 5364.1073/)95.2(1416.34 cmV

Calculando a média, entre o volume máximo e este mínimo, segue que:

3(303.6934 107.5364) 2 205.6149maçaV cm .

2º Método: Utilizando o teorema de Pappus

Pelo teorema de Pappus temos que o volume do sólido de revolução é igual ao produto da área da região pela distância d percorrida pelo centróide ao redor do eixo. Como d = 2 h e sendo A a área da região temos que V = 2 hA.

A Figura 16 mostra uma meia fatia de maçã e h é determinado experimentalmente medindo a distância entre o eixo da maçã (a partir do centróide) até a borda e considerando a metade deste comprimento.

Determinamos geometricamente a área A através de um papel milimetrado:

2 322.875 2.1 2 301.8292 .A cm e h cm V hA cm

Figura 16 - Volume da maçã pelo Teorema de Pappus [1].

Page 376: Numero 09 - Outubro de 2007

3º Método: Fatiando a maçã

(i) Retângulos internos (Figura 17).

321

1

2 5.235)( cmrVi

i

4.2Usamos 0.2 e 21 fatias cilíndricas.0.2

cm

Figura 17 - Fatiando a maçã [1].

(ii) Retângulos externos (Figura 18).

321

1

2 06.247)( cmrVi

i

Volume total (235.5 + 247.06)/2 = 241.28 3cm .

Figura 18 - Fatiando a maçã [1].

4º Método: Usando integração

(i) Aproximando a configuração do corte central da maçã por uma circunferência (Figura 19).

O volume de cada fatia é dado por

2iV = y x .

Page 377: Numero 09 - Outubro de 2007

Volume total: 4.14.1 3

2 3

0 0

2 2 16.81 288.69633xV y dx x V cm

Figura 19 - Usando integração para calcular o volume da maçã [1].

(ii) Aproximando por uma parábola y = ax2 + bx + c (Figura 20).

1 2 3Os pontos dados da curva são: 4.1, 0 , 0, 2.7 1,3.2 .P P e PDesta maneira, como P2 = (0, 2.7) temos que y = ax2 + bx + 2.7 e P1 e P3 nos fornecem

o sistema: 16.81 4.1 2.7

0.5a b

a b Resolvendo o sistema temos que a = -0.3737 e b = 0.8737 e, portanto,

20.3737 0.8737 2.7.y x x

Figura 20 - Aproximando o formato da maçã por uma parábola [1].

Usando integral, pode-se determinar o volume do sólido de revolução da parábola (“aproximadamente” metade do volume da maçã). Assim,

4.12 2 3

maça0

V =2 ( 0.3737 0.8737 2.7) 169.2408 .x x dx cm

Page 378: Numero 09 - Outubro de 2007

Conclusão Parcial

Cabe ressaltar que neste caso específico, de calcular volume de uma maçã, um processo mecânico seria o mais indicado para a avaliação, tanto em termos de simplicidade como de precisão.

Este processo, devido a Arquimedes, é o seguinte: Mergulha-se a maçã num recipiente cheio de água e o volume do líquido deslocado é igual ao volume da maçã. Com a utilização deste experimento, o volume encontrado para a maçã foi de 3310 .cm

2. Um Exemplo de Modelo Variacional

Para se fazer a formalização de um modelo variacional o conteúdo matemático que é utilizado baseia-se nas equações diferenciais ordinárias e equações de diferenças.

Processo de resfriamento da Maçã

Para que a maçã possa ser estocada ela deve primeiramente ser submetida a um processo de resfriamento, o qual é feito com a utilização de um tanque de resfriamento. A Figura 21 mostra os elementos que compõem o sistema de resfriamento com água.

Figura 21- Tanque de refrigeração [5].

O processo de resfriamento é uma das mais importantes etapas pós colheita que consiste na remoção rápida de calor do campo dos frutos antes do armazenamento ou comercialização. A maioria das câmaras de armazenagem não possui suficiente capacidade de refrigeração e nem o movimento de ar com velocidade suficiente para efetuar um resfriamento rápido dos produtos recém armazenados. Desta forma, este pré-resfriamento, geralmente, é uma operação separada e que necessita de equipamentos de maior capacidade de refrigeração. A Tabela 1 relaciona as condições para o armazenamento refrigerado de alguns tipos de maçãs.

Page 379: Numero 09 - Outubro de 2007

Cultivares Temperatura (°C)

Umidade Relativa (%)

Período de armazenamento

Gala e mutações 0 94-96 4-5 mesesFuji -1 a 0 92-96 6-7 meses

Golden Delicious 0 94-96 5-6 mesesBelgolden 0 94-96 5-6 mesesBraeburn 0 92-96 6-7 meses

Tabela 1

O Brasil, apesar de ser um país tropical, dispõe de poucos resfriadores comerciais. Além disso, pela falta de conhecimento dos produtores, o armazenamento ainda é feito de forma bastante precária e o pré-resfriamento dos frutos geralmente não é efetuado. Este fato, juntamente com a entrada de novas cargas ainda não resfriadas na unidade de armazenamento, faz com que o processo de resfriamento na câmara seja muito demorado e irregular, principalmente em função da oscilação de temperatura.

Antes da maçã entrar na câmara fria, que está à uma temperatura média de 1.5°C, o fruto recebe um banho num tanque à uma temperatura de -3°C. A passagem pelo tanque é feita sobre uma esteira circulante e dura cerca de 25 minutos.

O objetivo deste banho é fazer com que a temperatura da maçã alcance cerca de 6°C. Na saída do tanque, a temperatura da maçã é avaliada (por amostragem) e, caso não tenha atingido o valor ideal para estocagem, o lote de maçã deve passar novamente pelo tanque. Este processo de retorno ao tanque, além de atrasar a estocagem, ocupa uma maior mão-de-obra e por conseguinte acarreta prejuízos ao agricultor. Este transtorno ocorre porque a temperatura do meio ambiente é variável e a velocidade da esteira é constante (a máquina é construída para atender à termperatura ambiente de, no máximo, 26°C).

Em um primeiro momento, temos o seguinte problema: “Se a maçã entra no tanque a uma temperatura T0 (temperatura inicial), quantos

minutos deve permanecer neste banho para sair com uma temperatura de 7°C?” Para se tratar desta questão, usa-se a lei de resfriamento de Newton. Esta supõe que a

variação da temperatura é proporcional à diferença de temperatura do objeto e do ambiente (em condições ideais).

O Modelo Matemático que traduz a lei de resfriamento de Newton pode ser dado por uma equação de dierença, da seguinte maneira [1]:

1 ( ) (1) t t t aT T K T Tonde:

tT : temperatura da maçã no instante t; 0T : temperatura inicial (quando entra no tanque);aT : temperatura ambiente (do tanque) igual a -3°C;

K = coeficiente de resfriamento da maçã.

Solução: A equação (1) pode ser reescrita por

1 ( 1) (2)t t aT K T K Tque é uma fórmula de recorrência para qualquer valor Tt, uma vez que Ta = -3 e T0 é dado. A solução de (2) pode ser obtida usando-se o processo de recorrência:

Page 380: Numero 09 - Outubro de 2007

)1(

)1(

210

20

323

02

12

01

aaabTaT

babbaTabaTTbabTabaTT

TKbeKatomandobaTT

nnnn

a

1 1

O termo entre parêntesis de (3) é a soma de uma progressão geométrica de razão 1, então, como a soma dos termos de uma P.G. de razao 1 é dada por

1 ( 1), onde é o primeiro tenn

a a

S s a a s

0

0

rmo da P.G., segue imediatamente que:

( 10) ( 1) , ou (4)

( 1 ( 1) )

n nn

nn

T a T b a a

T a T b a b a (5) Se considerarmos que a temperatura média inicial da maçã é 25°C e que, depois de passar pela esteira durante 25 minutos, sua temperatura é T25 = 6.5°C, podemos calcular o valor de K= a + 1.

De (5), podemos escrever

0( 1) ( ) (6)nn a aT K T T T

Logo,

-0.0423k28

5.91k28

5.9ln)1(kln

285.9ln)1(kln25

285.9)1(k328)1(k5.6

251

251

2525

Considerando a solução (6), pode-se escrevê-la como: 0(0.95768) ( ) (7)t

t a aT T T Tcom T0 e Ta dados.

(3)

Page 381: Numero 09 - Outubro de 2007

Figura 22 - Temperatura da Maçã no Tanque x Tempo.

Observando o gráfico da Figura 22 que relaciona a temperatura da maçã no tanque com o tempo em que esta permanece imersa, verifica-se que quanto maior o tempo (em minutos) que a maçã fica no banho menor é a temperatura (em °C), como desejado.

Para se encontrar o tempo que a maçã deve permanecer no tanque de resfriamento em função da temperatura final Ttf (depois de passar pelo tanque), usa-se a equação (7) e obtém-se:

0 0

(0.95768) 23.1259 ln (8)f ft a t at

a a

T T T Tt

T T T T

Se Ta = -3 e considerando-se fixa a temperatura Ttf = 6.5 no fim do banho, pode-se colocar t em função de T0 (temperatura inicial da maçã).

A Tabela1, fornece os valores de t para Ttf = 6.5°C e Ttf = 7°C. O valor de t* é o tempo ideal, superestimado para a maçã permanecer no tanque.

0

0

Da Tabela 1, observa-se que, se 26°C , então 25 minutos no tanque é tempo suficiente para se ter 7°C.

Se 26°C < < 32°C, o banho deveria durar até 30 minutos; e se o diatf

TT

T

0

estiver bem quente onde 32°C 38°C, então o tempo necessário para a maçã atingir a temperatura de 7°C chega a ser 33 minutos.

T

Page 382: Numero 09 - Outubro de 2007

Tf = 6.5ºC Tf = 7ºC T0 ln(9.5/(T0+3)) tc t ln(10/(T0+3)) tc t t* 19 -0,83975 19,42 19'25'' -0,78845 18,23 18'14'' 19'20 -0,8842 20,45 20'27'' -0,83291 19,26 29'15'' 20'21 -0,92676 21,43 21'26'' -0,87547 20,25 20'15'' 21'22 -0,99675 22,37 22'32'' -0,91629 21,2 21'12'' 22'23 -1,006804 23,28 23'17'' -0,9555 22,1 22'6'' 23'24 -1,44545 24,15 24'15'' -0,99325 22,97 22'58'' 23'25 -1,08091 25 25' -1,03 23,8 23'48'' 24'26 -1,09199 25,8 25'48'' -1,0647 24,6 24'36 25'27 -1,149905 26,59 26'36'' -1,09812 25,4 25'24'' 26'28 -1,1826954 27,35 27'31'' -1,1314 26,17 26'10'' 27'29 -1,2144441 28,08 28'5'' -1,1632 27 27' 28'30 -1,2452157 28,8 28'48'' -1,19392 27,6 27'3'' 28'31 -1,2750687 29,49 29'29'' -1,22378 28,3 28'18'' 29'32 -1,3040562 30,16 30'10'' -1,25276 29 29' 30'33 -1,3322271 30,81 30'49'' -1,28093 29,62 29'37'' 31'34 -1,3596261 31,44 31'26'' -1,30833 30,25 30'15'' 31'35 -1,3862943 32,06 32'4'' -1,335 30,87 30'52'' 32'36 -1,412227 32,62 32'37'' -1,36098 31,5 31'3'' 32'37 -1,437588 33,25 33'15'' -1,3863 32 32' 33'38 -1,4622803 33,8 33'48'' -1,41098 32,63 32'37'' 33'

Tabela 2 - Temperatura inicial x Tempo necessário para atingir Ttf.

Figura 22 - Temperatura inicial x Tempo necessário para atingir Ttf.

Analisando o gráfico da Figura 22 se verifica que quanto maior for a temperatura inicial da maçã maior é o tempo necessário para que ela alcance tanto a temperatura final 6.5°C quanto 7°C. E ainda quanto menor a temperatura final maior deve ser o tempo de duração no banho.

Temperatura inicial

Tem

po n

eces

sário

par

a at

ingi

r Ttf

Page 383: Numero 09 - Outubro de 2007

Conclusão

Durante o processo de desenvolvimento do trabalho verificamos a importância de entender conceitos matemáticos para aplicá-los de uma maneira adequada e correta nas situações problemas que foram encontradas durante o percurso de modelagem de tais situações.

Além disso, é conveniente mencionar que foi necessário fazer um embasamento histórico para as questões abordadas aqui, com o objetivo de proporcionar ao leitor uma melhor compreensão dos fatos e da metodologia utilizada.

Finalmente, cabe ressaltar que todo processo de modelagem teve como suporte um conteúdo matemático, para que assim os modelos pudessem ser executados. Este processo também contou com o auxílio de conceitos específicos sobre o assunto tratado. Comparando os seguintes métodos: Teorema de Pappus, fatiando uma maçã, volume da esfera e integração com o princípio de Arquimedes observa-se que o 1º método teve uma aproximação melhor enquanto que a aproximação por uma parábola foi o menos preciso. Em termos operacionais o 2° método apresentou dificuldades de execução em relação aos demais.

Durante o processo de estocagem da maçã é necessário o seu armazenamento a uma temperatura de 6.5°C. Para tanto, utilizamos equações de diferenças para expressar matematicamente a temperatura desta no tanque e com isto descobrir o tempo necessário no banho. Assim, com os resultados obtidos o agricultor poderá reduzir seus gastos tanto com mão de obra quanto em relação a atrasos na estocagem.

Bibliografia

[1] R.C.Bassanezi. Ensino-Aprendizagem com Modelagem Matemática. Editora Contexto, 2004.

[2] Shenk, Al.. Cálculo e geometria analítica: volume 1. Editora Campus, 1991. [3] Simmons, George F..Cálculo com geometria analítica : volume 1. Editora

McGraw-Hill, Ltda,1987.[4] site http://pt.wikipedia.org/wiki/Ma%C3%A7%C3%A3[5] site http://www.scielo.br/img/revistas/cta/v23n2/2a12f02.gif

Page 384: Numero 09 - Outubro de 2007
Page 385: Numero 09 - Outubro de 2007

O USO DE TÉCNICAS DE OTIMIZAÇÃO PARA DETERMINAR UMA DIETA ALIMENTAR SAUDÁVEL E ECONÔMICA.

Universidade Federal de Uberlândia Faculdade de Matemática

Alessandra Ribeiro da Silva

[email protected]

Carlos Henrique Tognon

[email protected]

Milena A. Leite Brandão

[email protected]

Rosana Sueli M. Jafelice

[email protected]

Introdução

Uma alimentação equilibrada ou balanceada é aquela que oferece numa mesma refeição pelo menos um alimento de cada grupo (Energéticos, Construtores e Reguladores), pois assim conseguimos todos os nutrientes que nosso corpo precisa para viver em harmonia. Isso significa que o consumo de uma variedade de alimentos (Figura 1) é essencial para a obtenção do equilíbrio de nutrientes indispensáveis para satisfazer as necessidades fisiológicas e psicológicas de um indivíduo. Assim, é necessário uma dieta composta de proteínas, carboidratos, gorduras, fibras, cálcio e outros minerais, como também rica em vitaminas. Para isto necessitamos de uma dieta variada, que tenha todos os tipos de alimentos, sem abusos e também sem exclusões. Esta dieta pode ser constituída por três grupos básicos de alimentos, os alimentos energéticos,

construtores e reguladores.

Figura 1: Diversidade de alimentos.

Os Alimentos Energéticos fornecem energia. Alguns exemplos dos alimentos deste grupo são óleo, manteiga, margarina, bacon, açúcar, mel, pão, cereal matinal, biscoito, bolo, doces, sorvete, arroz, macarrão, milho, batata, mandioca, farinhas e outros.

Os chamados Alimentos Construtores auxiliam no crescimento e restabelecimento dos tecidos. Os alimentos que fornecem os nutrientes necessários à construção destes tecidos estão neste grupo. Alguns exemplos clássicos são carnes (boi, frango, porco, peixe, outros), leite e derivados (iogurte, queijo, requeijão, outros), ovos, feijão, ervilha, soja e outros.

Page 386: Numero 09 - Outubro de 2007

Também se tem os denominados Alimentos Reguladores. São aqueles que regulam o funcionamento do corpo. O organismo precisa de nutrientes para regular seu funcionamento, para prevenir certas doenças como gripes e resfriados e para ajudar na digestão dos alimentos. Os nutrientes reguladores são as vitaminas (por exemplo, A, B, C, D, E, K) e os minerais (ferro, cálcio, sódio, potássio, zinco e outros). Este grupo é composto por alimentos tais como as frutas (banana, limão, laranja, maçã e outras), legumes e verduras (cenoura, chuchu, abobrinha, alface, couve, agrião e outros).

Metodologia É frequentemente desejável descrever o comportamento de algum sistema ou

fenômeno da vida real em termos matemáticos, quer sejam eles físicos, sociológicos ou mesmo econômicos. A descrição matemática de um sistema ou fenômeno, chamada de modelo matemático, é construída levando-se em consideração determinadas metas. Por exemplo, talvez queiramos encontrar as quantidades necessárias para se ter uma boa alimentação gastando o mínimo possível por meio da análise de nutrientes nos alimentos presentes em diversos cardápios. Desta forma, escolhemos um cardápio de um site [1] e outro fornecido pelo recordatório de 24 horas realizado por um paciente da pós-graduanda em nutrição clínica Juliana Dias Borges e através de técnicas de otimização, usando o software Matlab, faremos a análise destes cardápios.

A pesquisa de preços foi feita no supermercado Extra e no Sacolão Center.

ObjetivosO objetivo deste trabalho é verificar se os cardápios escolhidos conciliam uma boa

alimentação com um custo mínimo avaliando as quantidades necessárias de cada alimento e se satisfazem todos os nutrientes necessários. E também resolver alguns problemas matemáticos relacionados à alimentação.

Pirâmide Alimentar Flexível e cheia de opções, a Pirâmide Alimentar (Figura 2) pode ser o seu guia para

uma dieta equilibrada e alimentação saudável [4]. Ela foi desenvolvida pelo departamento de agricultura americano e oferece orientação simples e fácil para você escolher seu cardápio respeitando as sete diretrizes:

1) Coma uma diversidade de alimentos. 2) Mantenha um peso saudável. 3) Escolha uma dieta com pouca gordura, colesterol e gordura saturada.4) Escolha uma dieta rica em vegetais, frutas e grãos. 5) Use açúcar com moderação. 6) Use sal com moderação. 7) Se consome bebidas alcoólicas, beba com moderação.

Page 387: Numero 09 - Outubro de 2007

Figura 2: Pirâmide alimentar.

Na próxima seção, definimos a teoria de Programação Linear que será utilizada para a resolução do problema que vamos estudar.

Preliminares

Sistemas lineares

Um dos problemas que aparece com elevada freqüência nas aplicações da Matemática é a resolução de sistemas de equações lineares. Um sistema de equações lineares é uma coleção finita de n variáveis e n equações lineares (todas nas mesmas variáveis), consideradas em conjunto e normalmente apresentadas na forma:

nnnnnnn

nn

nn

nn

bxaxaxaxa

bxaxaxaxabxaxaxaxabxaxaxaxa

332211

33333232131

22323222121

11313212111

Grupo dos cereais, raízes e tubérculos!

Grupo das frutas!

Grupo das verduras e legumes!

Grupo do leite, queijo e iogurtes!

Grupo das carnes e ovos!

Grupo das leguminosas!

Grupo dos açúcares!

Grupo dos óleos e gorduras!

Page 388: Numero 09 - Outubro de 2007

O sistema acima também pode ser representado na forma matricial:

nnnnnnn

n

n

n

b

bbb

x

xxx

aaaa

aaaaaaaaaaaa

3

2

1

3

2

1

321

3333231

2232221

1131211

Ou seja, o sistema linear pode ser escrito como: bxA

Sendo,

A = matriz dos coeficientes, nnA ;

x = vetor das variáveis (ou incógnitas), nx ;b = vetor dos termos independentes nb .

Uma solução de um sistema de equações lineares nas variáveis nxxx ,,, 21 é uma seqüência ordenada n,,, 21 de números tais que as substituições nix ii ,,1,transformam todas as equações do sistema em identidades verdadeiras. Resolver um sistema de equações lineares é determinar todas as suas soluções ou provar que não existe nenhuma. Um sistema de equações lineares que tenha pelo menos uma solução diz-se possível (determinado se só tiver uma, indeterminado se tiver mais do que uma). Um sistema de equações lineares que não tenha nenhuma solução diz-se impossível.

Definimos operação elementar sobre um sistema linear como sendo:

i) a permuta de duas de suas equações;

ii) a substituição de uma de suas equações por si mesma previamente multiplicada por uma constante não nula;

iii) a substituição de uma de suas equações pela soma de si mesma com uma outra previamente multiplicada por uma constante.

Dizemos que um sistema encontra-se na forma escalonada se o número de coeficientes iniciais nulos em cada equação, a partir da segunda, for maior do que na precedente.

Se um sistema linear de m equações e n incógnitas foi escalonado e, retiradas as equações do tipo 0=0, restaram p equações e n incógnitas, então:

i) se uma das equações restantes for da forma:

0x1+0x2 + ... + 0xn = i , com 0i , o sistema é impossível ;

ii) se não houver nenhuma equação da forma acima, o sistema é possível, sendo

determinado se p = n;

Page 389: Numero 09 - Outubro de 2007

indeterminado se p<n.

Programação Linear

A Programação Linear (PL) estuda métodos eficientes para a análise e solução de problemas de otimização (maximização ou minimização) cujo modelo matemático é de um tipo particularmente simples (linear), mas muito eficiente na prática. Num problema típico de PL temos: i) um número finito de variáveis xi.ii) um número finito de restrições ou vínculos, do tipo desigualdades (lineares) a que as variáveis devem satisfazer. iii) procura-se maximizar ou minimizar uma certa função real do tipo linear nas variáveis xi,que é chamada função objetivo do problema. Um passo fundamental para a resolução de um Problema de Programação Linear (PPL) é a modelagem do problema seguido do método de resolução do modelo.

Modelo

Sejam x1, x2, x3, x4, x5, o número de unidades dos alimentos s1, s2, s3, s4, s5respectivamente de uma dieta diária. O custo por dia desta dieta, em reais, será expresso pela função linear Q(x) = c1x1 +c2x2 + c3x3 + c4x4 + c5x5 chamada função objetivo. As restrições para as vitaminas são dadas

a11x1+ a12x2 +a13x3 +a14x4 a15x5 b1a21x1+ a22x2 +a23x3 +a24x4 a25x5 b2a31x1+ a32x2 +a33x3 +a34x4 a35x5 b3

Como não podemos consumir uma quantidade negativa de alimentos, temos que x1 0,x2 0, x3 0, x4 0, x5 0. Nosso problema será dado por:

Min Q(x) = c1x1 + c2x2 + c3x3 + c4x4 + c5x5a11x1 + a12x2 + a13x3 + a14x4 + a15x5 b1a21x1 + a22x2 + a23x3 + a24x4 + a25x5 b2a31x1 + a32x2 + a33x3 + a34x4 + a35x5 b3

x i 0, i = 1, 2, ... ,5 Uma forma mais compacta de expressar um PPL é a forma vetorial (matricial), ou seja,

Min Q(X) = CX AX BX 0

Abaixo seguem algumas definições: i) O conjunto S = {X : AX B (ou AX B; X 0)} é chamado de região de viabilidade (conjunto das soluções possíveis) ii) Um ponto que pertença a S é chamado solução viável.

Page 390: Numero 09 - Outubro de 2007

iii) Seja Q(X) = CX. Um ponto X* S tal que Q(X*) = max ou min{CX / X S }é chamado solução ótima e o valor Q(X*) é chamado de valor ótimo.

Obs.: a) Q(X) = CX é sempre linear em X. b) Podemos ter uma solução ótima, infinitas soluções ótimas ou nenhuma. c) Um problema de maximização (ou minimização) é dito ilimitado se sup{ CX / X S}=(inf{ CX / X S }= - .

Resolução Gráfica de um PPL

Problemas de PL que envolvem 2 variáveis (ou até 3 variáveis) podem ser resolvidos graficamente, porém este método é impraticável em espaços de dimensão maior. Basicamente o problema a ser resolvido é: - variáveis: x, y - desigualdades (lineares) envolvendo x e y. - função objetivo f(x, y) = ax + by + c. Estamos procurando um ponto (x, y) da região de viabilidade que nos forneça o maior (ou menor) valor k para f(x, y). Temos que f(x, y) = k representa retas paralelas, chamadas curvas de nível da função objetivo f. Sabemos que o vetor gradiente de f é perpendicular às curvas de nível de f. Procedimento resumido para resolução gráfica de um PPL( 2 )

a) Encontra-se a região de viabilidade de S. b) Traça-se o vetor gradiente de f c) Traçam-se as retas perpendiculares ao vetor gradiente (curvas de nível de f) d) O ponto mais “distante” na direção oposta ao vetor gradiente f e que intercepta pelo

menos um ponto de S é a solução ótima, no caso de minimização.

Exemplo 1:

)ominimizaçãde(caso)0,1(

o)maximizaçãde(caso)3/4,3/1()2,1(

2y2x-1y-x1yx

12y x y)f(x,

*2

*1

XX

f

Região Viável! f

y + x = 1

2x + y = 2

x - y = - 1

Page 391: Numero 09 - Outubro de 2007

Figura 3: Resolução gráfica do exemplo 1.

Exemplo 2:

Após uma pesquisa montamos a Tabela 1 a seguir, que mostra os valores nutricionais de alguns alimentos: arroz, feijão, peito de frango empanado congelado, suco de laranja pasteurizado e adoçado, pão tipo francês e margarina sem sal.

Arroz (125g)

Feijão(86g)

Frango(85g)

Iogurte(340g)

Mamão(140g)

Bolacha(34g)

VDR

Energia (Kcal) 192 283 183 238 56 151 2000Carboidratos(g) 14 53 0.1 33 15 26 300 Proteínas(g) 13 17 24 9 1 3 75 Lipídeo(g) 9 1 8 8 0.1 4 55

Tabela 1: Valores nutricionais de alguns alimentos.

Para montar uma dieta é necessário determinar as quantidades x1, ..., x6 (em porções) de cada alimento, necessárias para compor o VDR (valores diários de referência). Isso corresponde a resolver o sistema linear (1).

554x0.1xx88x x9x753x x9xx2417x13x30026x15x33x0.1x53x14x2000151x56xx382183x283x192x

654321

654321

654321

654321

(1)

Observe que o sistema possui quatro equações, correspondentes ao número de nutrientes, e seis incógnitas, correspondentes ao número de alimentos. A melhor maneira de resolver o sistema é por escalonamento, transformando o sistema ns forma escalonada reduzida.

2.770.27x7.41x x3.03-0.42x4.35x x

2.590.2x2.33x x6.050.56x x10.19x

654

653

652

651

(2)

O sistema (2) é possível indeterminado, isto é, possui infinitas soluções. Os valores para x1, ..., x4 dependem de valores escolhidos para x5 e x6, ditas variáveis livres. Assim, podemos expressar x1, ..., x4 em termos de x5 e x6. Temos então:

654

653

652

651

0.27x7.41x2.77x0.42x4.35x3.03-x0.2x-2.33x-2.59x0.56x x10.19-6.05x

(3)

Observamos, no entanto, que nem toda solução matemática é utilizável na situação prática, já que numa dieta é necessário escolher x5 0 e x6 0 de modo que também tenhamos x1 0, ..., x4 0. Assim, a partir do sistema (3) obtemos as condições:

Page 392: Numero 09 - Outubro de 2007

26.107.44x2x7.2110.36xx

12.9511.65x-x10.18 x18.19-x

56

56

56

56

(4)

Cada uma das inequações (4) corresponde a um semiplano no sistema de eixos x5x6.Os valores de x5 e x6 que satisfazem simultaneamente todas as inequações pertencem à região de interseção dos semiplanos. Essa região está hachurada na Figura 4.

Figura 4: Resolução gráfica do exemplo 2.

De acordo com a Figura 4, uma possível dieta pode ser obtida escolhendo x5 = 0.1 e x6=7. Substituindo esses valores em (4), obtemos:

x1 = 1.11; x2=0.957; x3 = 0.345 e x4=1.621Isto corresponde a, aproximadamente, 139g de arroz, 82g de feijão, 29g de frango,

551g de iogurte, 14g de mamão e 238g de bolacha.

Análise dos Cardápios

Após a definição dos cardápios montamos a Tabela 2, que mostra os valores nutricionais de cada alimento [2] com as quantidades indicadas pelo site [1] e pela nutricionista. Desta forma, com base nessas informações modelamos o problema e após a montagem dos sistemas lineares utilizaremos o software Matlab para encontrar a solução que mais se aproxima das quantidades ótimas preservando um cardápio saudável e um custo mínimo. Ressaltamos que os resultados que se seguem são apenas dados experimentais e que a exemplo de outros cardápios deve-se prevalecer ainda é o bom senso. Portanto, a atitude mais correta e segura é procurar um especialista que indicará a dieta mais adequada ao seu organismo (não consideramos na análise dos dados pessoas com patologias, por exemplo, diabetes).

x6=10.26+27.44x5

x6=7.21-10.36x5

x6=12.95-11.65x5

x6=10.8-18.19x5

x6

x5

Page 393: Numero 09 - Outubro de 2007

Cardápio do site Kcal Carb. Proteínas Lipídeos Cálcio Fibra Café da manhã iogurte - 170g 0,700 0,0970 0,0270 0,023 0,00101 0,002bolacha água e sal - 13,64g 4,320 0,6870 0,1010 0,144 0,0002 0,025mamão - 140g 0,400 0,1040 0,0050 0,001 0,00022 0,01Lanche iogurte - 170g 0,700 0,0970 0,0270 0,023 0,00101 0,002Almoço coxa de frango assada com pele-170g 2,150 0,0010 0,2850 0,104 0,00008 0arroz carreteiro - 187,5g 1,280 0,2810 0,0250 0,002 0,00004 0,016feijão carioca - 86g 0,760 0,1360 0,0480 0,005 0,00027 0,085alface - 40g 0,090 0,0170 0,0060 0,001 0,00014 0,01tomate cru com semente - 80g 0,150 0,0310 0,0110 0,002 0,00007 0,012Café da tarde maçã - 120g 0,560 0,1520 0,0030 0 0,00002 0,013Jantacarne moída cozida - 90g 2,120 0,0000 0,2670 0,109 0,00004 0feijão carioca - 86g 0,760 0,1360 0,0480 0,005 0,00027 0,085arroz carreteiro - 187,5g 1,280 0,2810 0,0250 0,002 0,00004 0,016cenoura cozida - 110g 0,300 0,0670 0,0080 0,002 0,00026 0,026salada de repolho - 55g 0,170 0,0390 0,0090 0,001 0,00035 0,019

Cardápio da Nutricionista Kcal Carb. Proteínas Lipídeos Cálcio Fibra Café da manhã leite com achocolatado - 278g 0,83 0,142 0,021 0,022 0,0007 0,006pão francês - 50g 3 0,586 0,08 0,031 0,00016 0,023queijo minas - 90g 2,64 0,032 0,174 0,202 0,00579 0Lanche barra de cereal - 25g 3,65 0,838 0,072 0,01 0,00143 0,041Almoço lagarto cozido - 65g 2,22 0 0,329 0,091 0,00004 0arroz carreteiro - 187,5g 1,28 0,281 0,025 0,002 0,0004 0,016feijão carioca - 86g 0,76 0,136 0,048 0,005 0,00027 0,085chuchu - 25g 0,19 0,048 0,004 0 0,00008 0,01Café da tarde pêra - 132g 0,53 0,14 0,006 0,001 0,00008 0,03Jantaarroz carreteiro - 187,5g 1,28 0,281 0,025 0,002 0,00004 0,016lasanha - 200g 1,64 0,0325 0,058 0,012 0,0001 0,016

Page 394: Numero 09 - Outubro de 2007

Tabela 2: Cardápios.

Considere as seguintes informações nutricionais:

Tabela 3: Valores diários recomendados.

Para montar uma boa alimentação é necessário determinar as quantidades (em gramas) de cada alimento, necessárias para compor o VDR (Valores Diários de Referência). Isso corresponde a resolver os sistemas lineares (5) e (6).

)5(

300,019x0,026x0,016x0,085x0,013x x0,0120,01x0,085x0,016x x0,002 x0,010,025x0,002x

10,00035x x0,00026 x0,00004 x0,00027 x0,00004 x0,000020,00007x0,00014x

0,00027x0,00004x0,00008x0,00101x0,00022x x0,00020,00101x550,001x0,002x x0,002 x0,0050,109x0,002x

x0,001 x0,005 x0,002 x0,1040,023x0,001x x0,1440,023x630,009x0,008x x0,025 x0,0480,267x x0,003 x0,011

x0,006 x0,048 x0,025 x0,285 x0,027 x0,0050,101x0,027x3750,039x x0,067 x0,281 x0,136 x0,152 x0,031

x0,017 x0,136 x0,281 x0,0010,097x x0,104 x0,6870,097x25000,17x0,3x1,28x0,76x2,12x

0,56x0,15x0,09x0,76x1,28x2,15x x0,70,4x4,32x0,7x

15141312

1098764321

1514

1312111098

7654321

15141312119

87654321

1514131211109

87654321

15141312109

87654321

1514131211

10987654321

o site:Cardápio d

onde x1 é a quantidade(em gramas) de iogurte; x2 é a quantidade(em gramas) de bolacha água e sal;

x3 é a quantidade(em gramas) de mamão; x4 é a quantidade(em gramas) de iogurte; x5 é a quantidade(em gramas) de coxa de frango assado com pele; x6 é a quantidade(em gramas) de arroz carreteiro; x7 é a quantidade(em gramas) de feijão carioca; x8 é a quantidade(em gramas) de alface; x9 é a quantidade(em gramas) de tomate sem semente; x10 é a quantidade(em gramas) de maçã; x11 é a quantidade(em gramas) de carne moída; x12 é a quantidade(em gramas) de feijão carioca; x13 é a quantidade(em gramas) de arroz carreteiro; x14 é a quantidade(em gramas) de cenoura cozida; x15 é a quantidade(em gramas) de repolho;

Kcal Carb. Proteínas Lipídeos Cálcio Fibra Valores Diários de

Referência 2000-2500 300-375 63-126 55 1 25-30

Page 395: Numero 09 - Outubro de 2007

A função objetivo é dada por: F(X) = 0,00275 x1+0,01575 x2+ 0,00199 x3+ 0,00275 x4+0,00348 x5+ 0,001296 x6+ 0,00358 x7+ 0,002178 x8+ 0,00239 x9+ 0,00248 x10+0,00749 x11+ 0,00358 x12+ 0,001296 x13+ 0,00129 x14+ 0,00039 x15.

Utilizando-se o software Matlab encontramos a seguinte solução para o sistema (5):

X = [321,42 83,86 198,42 321,42 160 180,81 80,41 30,3 70,06 217,42 70 80,41 200,82 113,41 102,37], o que representa aproximadamente:

Café da manhã: duas garrafinhas de iogurte, 18 bolachas de água e sal e 200g de mamão (mais da metade);

Lanche da manhã: duas garrafinhas de iogurte; Almoço: duas coxas de frango (assada com pele), 3 colheres de arroz, uma concha de

feijão (50% de caldo), 4 folhas de alface e 3 fatias e meia de tomate; Lanche da tarde: uma maçã e meia. Janta: 4 colheres de carne moída, 1 concha de feijão, 3 colheres de arroz, 1 cenoura

cozida e102g de repolho;

Note que na Tabela 4 temos que a solução do sistema se aproxima da faixa estabelecida para cada nutriente considerado. No caso da proteína, o VDR é tomado como 10% das calorias [5], mas segundo a nutricionista, é permitido chegar até 20% o que corresponderia a 126g e, portanto estaria dentro do intervalo desejado. Em relação ao custo este valor não é o real, pois diariamente quando compramos certos produtos pagamos pela mercadoria total e não apenas por parte dela.

Tabela 4: Faixa para os nutrientes comparada com os valores obtidos na solução.

Kcal Carb. Proteínas Lipídeos Cálcio Fibra Custo diário

VDR 2000-2500 300-375 50-63 55 1 25-30 -

Solução 2181,176 317,2008 111,8026 53,39727 0,862794 34,00553 6,593818

Page 396: Numero 09 - Outubro de 2007

300.016x0.016x0.03x0.01x0.085x0.016x0.041x0.023x0.006x10.0001x0.00004x0.00008x

0.00008x0.00027x0.0004x0.00004x0.00143x0.00579x0.00016x0.0007x

550.012x(6)0.002x0.001x0.005x0.002x0.09x0.01x0.202x0.031x0.02x

630,058x0,025x0,006x0,004x0,048x0,025x0,329x0,072x0,174x0,08x0,021x

3750,325x0,281x0,14x0,048x0,136x0,281x0,838x0,032x0,586x0,142x

25001,64x1,28x x0,530,19x0,76x1,28x2,22x3,65x2,64x3x0,83x

11109876421

11109

8

7654321

11

1097654321

1110

987654321

11

1098764321

11

10987654321

tautricionisCardápio/n

onde x1 é a quantidade(em gramas) de leite com achocolatado; x2 é a quantidade(em gramas) de pão francês;

x3 é a quantidade(em gramas) de queijo minas; x4 é a quantidade(em gramas) de barra de cereal; x5 é a quantidade(em gramas) de lagarto cozido; x6 é a quantidade(em gramas) de arroz carreteiro; x7 é a quantidade(em gramas) de feijão carioca; x8 é a quantidade(em gramas) de chuchu; x9 é a quantidade(em gramas) de pêra; x10 é a quantidade(em gramas) de arroz carreteiro; x11 é a quantidade(em gramas) de lasanha;

A função objetivo é dada por F(X1)= 0,00495 x1+0,00466 x2+ 0,00864 x3+ 0,048 x4+0,008 x5+ 0,001296 x6+ 0,00358 x7+ 0,00069 x8+ 0,00298 x9+ 0,001296 x10+ 0,01245x11.

Já neste caso a solução para o sistema (6) é dada por:

X1 = [316,97 74,63 80 47,64 55 243,62 94,69 63,67 170,22 243,62 236,52], o que representa aproximadamente:

Café da manhã: 1 copo e meio de leite com achocolatado , 1pão (francês) e meio e duas fatias de queijo minas;

Lanche da manhã: 2 barras de cereais (25g cada); Almoço: 55g de lagarto cozido, 4 colheres de arroz, um pouco mais de uma concha de

feijão (50% de caldo), 64g de chuchu; Lanche da tarde: uma pêra e meia; Janta: 4 colheres de arroz, 237g de lasanha.

Note que na Tabela 5 temos que a solução do sistema se aproxima melhor da faixa estabelecida para cada nutriente considerado. No caso dos lipídeos, segundo a nutricionista,

Page 397: Numero 09 - Outubro de 2007

pode-se ingerir outros alimentos que não alterem muito as quantidades dos demais nutrientes, por exemplo, colocar um pouco de azeite na salada visto que a quantidade de lipídeos desejada não foi alcançada. Observe ainda que o custo diário é maior que o anterior.

Tabela 5: Faixa para os nutrientes comparada com os valores obtidos na solução.

Caso não os indivíduos não sigam as dietas saudáveis, podem ter que enfrentar problemas de obesidade. O exemplo 1 a seguir ilustra este tipo de problema.

Exemplo 1: Um ator de cinema que pesa 120Kg precisa fazer um severo regime para emagrecer em virtude do seu papel num novo filme a ser rodado. O diretor exige que ele perca a terça parte do seu peso no máximo em três meses, seguindo uma dieta racional que emagreça proporcionalmente ao peso de cada dia. Nestas condições, sabendo-se que iniciada a dieta, o artista emagrecerá 20Kg em 40 dias, quantos dias serão necessários para que ele comece a atuar no filme. Seja P = P(t) = peso que se perde em um instante arbitrário t. Assim, P(t + h) = peso que se perde em um instante arbitrário t + h e P(t + h) - P(t) = variação do peso que se perde no instante h.

20)40(0)0(

)120(

Assim,

a.instantânevariaçãodtdP(t))()(

0lim

:quenote Eunitário.instantenoperdesequepesodo variação)()(Portanto,

PP

PkdtdP

htPhtP

h

htPhtP

Kcal Carb. Proteínas Lipídeos Cálcio Fibra Custo diário

VDR 2000-2500 300-375 50-63 55 1 25-30 -

Solução 2180,044 315,5989 79,7935 35,38516 0,942485 30,94475 9,801251

Page 398: Numero 09 - Outubro de 2007

.956.882.1ln

5.1ln403/28012040120120quesegue40P(t)seAssim,

120120)(forma,Desta

.2.1ln40/165120120)40(Assim,

20.P(40)queTemos.120120)(Logo,.120120)0(

quesegue0P(0)Como.120)(

120)()(120ln)(120lnP(t)-120

dtdP

,

)2.1ln40/1()2.1ln40/1()2.1ln40/1(

)2.1ln40/1(

4040

10

1

1

1

tt

eeeetP

keeP

etPcecPectP

ectPckttPktPdtdkDaí

ttt

t

kk

ktk

kt

kt

Enfim, serão necessários 88 dias para que o ator comece a atuar no filme.

Podemos resolver, matematicamente, problemas de concentração de glicose. É o que segue no próximo exemplo.

Exemplo 2: Por infusão, a concentração de glicose no sangue é aumentada a uma taxa constante, R (em mg/minuto). Ao mesmo tempo, a glicose é convertida e excretada a uma taxa que é proporcional a presente concentração de glicose. Conseqüentemente, se tCCrepresenta esta concentração, obtemos a equação diferencial

KCRdtdC

(5)com uma constante 0K [3].

Temos que: R : taxa constante de aumento da concentração de glicose no sangue. C : concentração de glicose no sangue no instante t .

A equação diferencial é solucionada da seguinte forma:

.

)0(

KRCK

dtdC

KRKCdtdC

(6)

A fim de simplificar, consideremos pKR . Então, separamos as variáveis:

., KdtpC

dCdtpCKdC

Por integração, temos:

AKtpCKdtpC

dC ln,

Assim,

Page 399: Numero 09 - Outubro de 2007

KReSC tK

(7)

Na expressão (7), S é uma constante.

Também, (7) é a solução geral de RKCdtdC .

Podemos representar graficamente a solução geral desta equação. Para tanto, utilizamos os seguintes valores: 4,10 KS e 5R . Vamos considerar somente valores positivos para S ; conseqüentemente teremos somente valores positivos para tCC . A representação gráfica de uma solução particular desta equação é dada a seguir.

0 1 2 3 4 5 6 7 8 9 101

1.5

2

2.5

3

3.5

4

4.5

5

5.5

6

Tempo percorrido em minutos

Con

cent

raça

o de

glic

ose

no s

angu

e

ç g ç p p

Figura 5: Gráfico da equação (7).

Conclusão

Analisando os resultados encontrados percebemos que o cardápio do site apresentou menor custo, porém o cardápio da nutricionista satisfez melhor as faixas dos nutrientes. Assim, pode-se adotar o cardápio do site complementando com alimentos que aumentassem as quantidades necessárias de cálcio e lipídeos sem alterar muito as quantidades dos demais nutrientes ou adotar o cardápio da nutricionista gastando um pouco mais, no entanto atendendo melhor os valores diários de referência.

Entretanto, deve ficar claro que as necessidades calóricas e de nutrientes de uma pessoa variam de acordo com sexo, atividade física, idade e, conseqüentemente, podem ser inferiores ou superiores aos valores estipulados.

Page 400: Numero 09 - Outubro de 2007

Bibliografia

[1] http://www.tiojoao.com.br/cardapios.asp?link=3&sublink=3#

[2] http://www.unicamp.br/nepa/taco/contar/taco_versao2.pdf

[3]BATSCHELET, Edward. Introdução à Matemática para Biocientistas. Tradução: Vera M. A. P. da Silva e Junia Maria P. Quitete. São Paulo: Editora da Universidade de São Paulo, 1978.

[4] http://www.copacabanarunners.net/piramide.html

[5] http://www.agroinfo-br.com/conteudo.asp?cod=37

Page 401: Numero 09 - Outubro de 2007

IMPLICAÇÕES DA ATIVIDADE DE ENSINO NA

FORMAÇÃO INICIAL DE PROFESSORES

Universidade Federal de Uberlândia Faculdade de Matemática

Fabiana Fiorezi de Marco Matos Lóren Grace Kellen Maia Amorim Mariana Martins Pereira [email protected] [email protected] [email protected]

RESUMO

Este artigo resultou de uma pesquisa realizada na Universidade Federal de Uberlândia, no

curso de Licenciatura/Bacharelado em Matemática. Enfatiza a importância do uso de

ambientes computacionais e relata a produção de uma atividade de ensino no Macromedia

Flash explorando o conceito de área. Destaca que tais ambientes, quando cuidadosamente

planejadas as atividades, são recursos pedagógicos eficazes para a construção do

conhecimento matemático.

Palavras-chave: atividade de ensino, formação inicial de professores, ambiente

computacional.

INTRODUÇÃO

Ao refletirmos sobre o estado atual da Educação no Brasil, deparamos, na maioria das

vezes, com uma realidade sem aparentes atrativos para crianças, adolescentes e adultos: uma

educação considerada por eles desagradável e desinteressante. É possível, porém, propormos

experiências que procurem redimensionar crenças e valores, com o propósito de transpor tal

situação, considerando o aluno como ser integral, ou seja, o emocional e o cognitivo juntos

formando o aprendiz.

A preocupação dos estudantes, em geral, de buscar uma fórmula para solucionar

situações-problema parece-nos ser decorrente do ensino de mecanismos de repetição das

formas abstratas dos conceitos científicos matemáticos, o chamado “ensino

tradicional”. Percebemos, em nosso exercício de professoras, que várias escolas utilizam a

pedagogia do treinamento (mostrar o conceito, demonstrar seu funcionamento, treiná-lo,

avaliá-lo (avaliação somativa)) ao invés de valorizar o processo de saber pensar sobre

Page 402: Numero 09 - Outubro de 2007

conceitos matemáticos mediante a resolução de problemas (LIMA, 1998).

Segundo LANNER DE MOURA et all. (2003a)

“a simples existência objetiva dos conceitos matemáticos não determina a sua existência no nosso subjetivo, do mesmo modo que o simples fato de lidarmos com um eletrodoméstico qualquer - uma televisão, um aspirador de pó etc - não nos torna pensadores em eletrônica. Assim, podemos conjecturar sobre a existência de um espaço vazio de compreensão entre a manipulação mecânica e cotidiana de um conceito e a sua (re)criação subjetiva. (...) É de se pensar que quanto mais intensificamos a prática mecânica, mais o conceito que a embasa torna-se invisível ao pensamento.”

Com estas reflexões, neste artigo, apresentamos os resultados de uma pesquisa

realizada com alunos do curso de Licenciatura/Bacharelado em Matemática da Universidade

Federal de Uberlândia, na disciplina Informática e Ensino.

APRESENTAÇÃO DA DISCIPLINA

A disciplina Informática e Ensino é oferecida semestralmente e faz parte das

disciplinas do 2º período do curso de Licenciatura e Bacharelado em Matemática da

Universidade Federal de Uberlândia (UFU) e foi oferecida pela primeira vez no 1º semestre

de 2006.

No referido semestre, os objetivos da disciplina visavam:

Propiciar aos alunos subsídios teóricos que permitissem a percepção e a

conscientização sobre a utilização da informática na sociedade e na educação, especialmente

em relação ao papel do professor, do aluno e de ambientes computacionais de aprendizagem.

Investigar novas tecnologias aplicadas ao ensino de matemática;

Provocar a mudança de postura didática/metodológica do futuro professor face às

ferramentas tecnológicas de apoio ao ensino;

Discutir as potencialidades e limitações de softwares na produção de atividades

interativas de ensino com os alunos.

Compartilhar com os colegas de disciplina seu planejamento, suas experiências,

reflexões e saberes elaborados durante a produção de atividades interativas de ensino.

Os participantes desta disciplina eram alunos de diversos períodos, pois neste

semestre a Universidade passava por uma transição havendo a implementação de um novo

currículo e os alunos de períodos finais precisavam cumprir esta disciplina.

Page 403: Numero 09 - Outubro de 2007

METODOLOGIA DA DISCIPLINA

A disciplina Informática e Ensino utilizou o yahoo groups para servir como meio de

comunicação entre professora-pesquisadora/alunos e como apoio às aulas, sendo que toda

informação referente às aulas e ao conteúdo a ser trabalhado eram disponibilizados no

ambiente.

A dinâmica da disciplina consistiu na discussão e reflexão sobre a bibliografia

sugerida no curso e cada aluno foi orientado a elaborar mapas conceituais dos textos teóricos

relacionando-os com: a) inserção de novas tecnologias em ambiente escolar e seus reflexos no

currículo de matemática dos ensinos fundamental e médio e nos cursos de formação de

professores, b) softwares educacionais: critérios de usabilidade; avaliações técnicas, c)

calculadoras e multi-mídia em ambiente escolar e com as interlocuções realizadas em sala de

aula.

Além disso, os alunos foram orientados a elaborar reflexões em diários pessoais

sobre as aulas e sua aprendizagem mediante cada discussão. Estas reflexões eram lidas e

comentadas e questionadas, tanto teórica como metodologicamente, por escrito no próprio

diário, focando o desenvolvimento profissional do aluno, pela professora-pesquisadora e

sempre retornavam aos alunos para que pudessem verificar e repensar situações de

aprendizagem, constituindo-se uma atividade de avaliação formativa.

Durante o semestre os alunos puderam também explorar e vislumbrar estratégias de

trabalho e de atividades com os ambientes computacionais S-Logo, Cabri Géomètre II e

Winplot e vivenciar a criação e construção de Webquest’s1.

Além destas atividades, os alunos deveriam, como trabalho de final de disciplina,

produzir atividades de ensino de Matemática, em ambientes computacionais, em grupos de

quatro ou cinco pessoas.

ATIVIDADE DE ENSINO NA FORMAÇÃO INICIAL DO PROFESSOR DE

MATEMÁTICA EM AMBIENTES COMPUTACIONAIS

Optamos pela utilização de atividades de ensino (MOURA, 2000, 2002) pelo fato

destas apresentarem uma abordagem que considera o aluno em todo seu movimento de

aprendiz, considerando, além do aspecto cognitivo, outros de natureza distinta deste como as

formas sensitivas do pensamento: sensações e percepções, assim denominadas por Kopnin

(1978). Estas dizem respeito a movimentos subjetivos do pensamento do aluno, ainda não

objetivamente expressos por deduções e linguagem lógica; uma abordagem que contempla a

1 Maiores informações podem ser encontradas no site http://br.geocities.com/edineileandro/webquest.swf.

Page 404: Numero 09 - Outubro de 2007

existência de um momento anterior à solução propriamente dita do problema, em que são

considerados as emoções, além de sentimentos, frustrações, dúvidas, hesitações, alegrias e

desejo de querer resolver a situação encontrada. Durante todo este processo, o aluno tanto

aceita sugestões e propostas de colegas quanto as nega, tanto hesita e responde quanto afirma

e interroga, tanto fica perplexo com o desconhecido, quanto é contundente no que sabe.

Para entendermos o conceito de atividade de ensino, buscamos Leontiev (1988) que

aborda atividade como uma unidade de formação na qual as necessidades emocionais, afetivas

e materiais dirigem a ação do sujeito. Este autor define atividade como os processos

psicologicamente caracterizados por aquilo a que o processo, como um todo, se dirige (seu

objeto), coincidindo sempre com o objetivo que estimula o sujeito a executar esta atividade,

isto é, o motivo (p.68). Moura (2000) complementa dizendo que a

atividade é regida por uma necessidade que permite o estabelecimento de metas bem definidas. O estabelecimento de objetivos por sua vez permitirá a criação de estratégias para se chegar a cumprir as metas. É aí que aparece o conjunto de ações necessárias para levar a bom termo os objetivos a serem alcançados. Estas ações devem fazer parte de um plano no qual se inclui o uso de instrumentos, sejam eles simbólicos ou não, que servirão como auxiliares para a execução das ações. (p.24).

Nesta pesquisa, adotamos a definição de Moura (2000) para atividade de ensino.

Segundo esse autor, esta deve envolver o aluno em situações-problema e de reflexão que

gerem a necessidade do desenvolvimento de significados próprios do conceito em questão.

Conforme as palavras do próprio autor, a atividade de ensino contém alguns elementos:

Em primeiro lugar, ela precisa ser do sujeito. Isto é, deve provocar no sujeito uma necessidade de solucionar algum problema. Ou melhor ainda: ter sua nascente numa necessidade. Esta, por sua vez, só aparece diante de uma situação que precisa ser resolvida e para cuja solução exige uma estratégia de solução. Assim, ela exige um plano de ação. Nesse plano, o sujeito parte de conhecimentos que já possui e que lhe servem de instrumento para poder avaliar a situação vivenciada. É desse seu nível de conhecimento que parte para resolver o problema que lhe é colocado (p.34) (grifo nosso).

Em um outro trabalho, este mesmo autor enfatiza que atividade de ensino é

aquela que se estrutura de modo a permitir que sujeitos interajam, mediados por um conteúdo negociando significados, com o objetivo de solucionar coletivamente uma situação-problema. É atividade orientadora porque define elementos essenciais da ação educativa e respeita a dinâmica das interações que nem sempre chegam a resultados esperados pelo professor. Este estabelece os objetivos, define as ações e elege os instrumentos auxiliares de ensino, porém não detém todo o processo, justamente porque aceita que os sujeitos em interação partilhem significados que se modificam diante do objeto de conhecimento em discussão (MOURA, 2002, p.155).

Page 405: Numero 09 - Outubro de 2007

Diante desta caracterização de atividade de ensino a entendemos como uma situação

desencadeadora de um novo conhecimento para o sujeito, pois ela gera neste uma necessidade

que, a partir dos conhecimentos já elaborados e assimilados, proporcionará ao sujeito ter um

conhecimento diferente do inicial. Nesse processo, o sujeito precisa avaliar a situação

continuamente. Em síntese, Moura (2002) denomina de atividade de ensino aquela na qual o

sujeito estabelece objetivos, desencadeia ações, elege instrumentos e, por fim, avalia se

chegou a resultados adequados ao que era desejado (p.155). O autor destaca que a atividade

de ensino deve estar carregada de intencionalidade, um motivo que permita estabelecer metas

e objetivos bem definidos que possibilitem a criação de estratégias que irão compor o plano

de ação daquele que a propõe, pois sua finalidade maior é o ensino (de Matemática, no nosso

caso).

Diante do exposto, nosso objetivo principal consistiu em analisar o papel da atividade

de ensino como contribuinte para formação dos futuros professores de matemática por meio

da produção de uma atividade de ensino em ambiente computacional que possibilitassem o

desenvolvimento dos aspectos do saber pensar conceitos matemáticos e não somente enunciar

fórmulas matemáticas.

Acreditamos que a utilização de recursos computacionais permite ao professor uma

maneira diferenciada de ministrar suas aulas, deixando a mesmice das aulas tradicionais e

emergindo para uma aula mais dinâmica.

Os suportes oferecidos pelos ambientes ajudam a superar os obstáculos interligados ao

processo de construção do conhecimento matemático. E, quanto mais os ambientes tornam-se

ricos nos seus recursos, mais as idéias matemáticas serão significativas para os alunos.

Essa concepção é reforçada por Maltempi (2004), que vê que

o computador como algo que viabiliza a criação de situações mais propícias, ricas e específicas para a construção de conhecimentos. Estas situações geralmente estão relacionadas com o desenvolvimento de projetos, pois o aprendiz tem mais oportunidade de aprender quando está ativamente engajado na construção de um artefato sobre o qual possa refletir e mostrar a outras pessoas (p.265).

Concordamos também com Miskulin (1999) quando afirma que a Tecnologia não

consiste apenas em um recurso a mais para os professores motivarem as suas aulas, consiste

sim em um meio poderoso que pode propiciar aos alunos novas formas de gerarem e

disseminarem o conhecimento (p.159).

Os ambientes informatizados, na forma que se apresentam hoje, por si só, não

garantem a aquisição de conhecimentos. Para que haja avanço no conhecimento matemático,

Page 406: Numero 09 - Outubro de 2007

é importante que o professor tenha um planejamento das atividades a serem desenvolvidas.

Não basta colocar a disposição do aluno um programa em que este possa fazer vários cálculos

automaticamente, mas sim colocar a disposição do aluno uma tarefa em que o professor atue

sistematicamente no processo de aprendizagem.

Miskulin (1999), defende que

O ambiente, por mais rico e construtivo que seja, por si só, não é suficiente para promover contextos propícios para a construção do conhecimento. Nesse sentido, a mediação do professor desempenha um papel determinante, à medida que o professor cria situações desafiantes, recorta-as em vários problemas intermediários que possibilitam aos alunos deslocarem-se muitas vezes do problema principal, olhando-o e percebendo-o, sob uma outra perspectiva, possibilitando-lhes a busca de novos caminhos, a reavaliação constante de suas estratégias e objetivos, enfim, envolvendo-se cada vez mais, no processo de construção do conhecimento (p.184).

De acordo com as palavras da autora, percebemos que o professor exerce um papel

fundamental para que as idéias matemáticas se tornem significativas, atuando como mediador

de todo o processo.

Diante destas considerações, acreditamos que se faz necessário propiciar, em cursos de

formação de professores de matemática, vivências de produção de atividades de ensino, em

ambientes computacionais ou não, nas quais os futuros professores sejam estimulados,

desafiados e agentes de sua aprendizagem.

Para nossas análises, buscamos reflexões sobre as vivências sobre atividades de

ensino, em sala de aula, registradas em diários pessoais de alunos envolvidos na pesquisa e

posterior produção de uma atividade de ensino em ambiente computacional. O diário foi

sugerido com o objetivo de proporcionar aos alunos relembrar suas vivências matemáticas

escolares sobre diversos conceitos e, a partir delas, construir perspectivas para uma futura

prática pedagógica.

Mediante estas considerações, nossa questão de investigação se traduz em como

vivenciar e produzir atividades de ensino de Matemática, em ambientes computacionais, pode

se constituir numa atividade de formação de professores?

As atividades de ensino desenvolvidas foram adaptadas dos livros “A fração – A

repartição da Terra” (LIMA e MOISÉS, 1998), “Elementar é o essencial” (LIMA,

TAKAZAKI E MOISÉS, 1998); “A variável – ser e não ser” (LIMA E MOISÉS, 2000); O

cálculo algébrico: operações com “Número Algébrico” – A linguagem Algébrica 2 (LIMA e

MOISÉS, 1993) e conceitos como composição, decomposição, medida, volume (geometria

Page 407: Numero 09 - Outubro de 2007

espacial), área (geometria plana), números inteiros e função estavam envolvidos nas

atividades.

Todas as atividades foram vivenciadas e discutidas pelos alunos, seguindo a dinâmica

indivíduo-grupo-classe, onde neste terceiro momento a discussão era mediada pela professora.

Esta dinâmica tinha como objetivo integrar os alunos ao movimento conceitual devendo, num

primeiro momento, pensar individualmente sobre a situação encontrada, para em seguida,

discutir as sínteses que elaborou com seu grupo de trabalho. Este, por sua vez, elabora uma

síntese a partir das reflexões de todos os seus membros e, então, é feita uma discussão com o

grupo-classe para encontrar uma possível solução ou a solução mais adequada para cada

situação (LANNER DE MOURA et all, 2003a, 2003b).

UMA EXPERIÊNCIA

A proposta para o desenvolvimento do trabalho final da disciplina foi a produção de

uma atividade de ensino em ambiente computacional. Pelo fato de precisarmos criar uma

interação entre os alunos e atividade de ensino em ambiente computacional, decidimos utilizar

o software Flash2 para suprir nossas necessidades de interatividade. O que se pretendeu

destacar ao utilizarmos esse software foi demonstrar quão natural e intensa, se tornam nos

ambientes informatizados de ensino, as ações, reflexões e abstrações dos alunos.

Nossa preocupação inicial era compreender o significado de atividade de ensino

segundo Moura (2002), para assim partirmos para o desafio desenvolvê-la no ambiente

computacional escolhido.

Com o intuito de vivenciar as dificuldades enfrentadas pelos professores de

Matemática quando estes decidem propor uma aula dinâmica e interativa, elaboramos a

atividade de ensino intitulada “Construindo o conceito de área” (figura 1).

2 Macromedia Flash é um software desenvolvido e comercializado pelo Adobe. É utilizado para criação de animações interativas e pode ser utilizado junto com outras tecnologias como o XML, HTML entre outros. <http://www.adobe.com/br/products/flash>

Page 408: Numero 09 - Outubro de 2007

Fig. 1 – Tela inicial da atividade de ensino “Construindo o conceito de área”

Esta idéia surgiu após vivenciarmos uma atividade de ensino intitulada volume3,

mediada pela professora-pesquisadora da disciplina. Por meio desta atividade percebemos que

para se construir o conceito de volume é preciso proporcionar aos alunos situações nas quais

possam refletir sobre o que fazer para dar sentidos e significados próprios aos conteúdos

matemáticos e não apenas reproduzir mecanicamente os conceitos. Em outras palavras, o

professor deve cada vez mais respeitar o ritmo de aprendizagem de cada aluno, valorizando

também o saber-pensar e não só o saber-fazer (LIMA, 1998).

Na atividade produzida o aluno terá que preencher dois espaços retangulares,

escolhendo para isso o quadrado, o triângulo ou o círculo, procurando responder a seguinte

questão: Qual dos dois pátios possui a maior área?. Para tanto, ele poderá manipular

empiricamente as diferentes figuras de maneira a construir a relação necessária ao recobrir os

planos.

Dessa maneira, o aluno perceberá que apenas uma das três figuras - quadrado –

constitui-se a melhor para cobrir a superfície ocupando totalmente os planos (figura 2).

3 Atividade adaptada do livro “A fração – A repartição da Terra” (LIMA e MOISÉS, 1998).

Page 409: Numero 09 - Outubro de 2007

Fig. 2 – Tela de recobrimento de espaços

Esta atividade de ensino vem acompanhada de uma tela de questões que tem o

objetivo de levar o aluno a construir o conceito de área. Ao responder as questões, na ordem

proposta e tendo a mediação do professor, o aluno poderá sistematizar o conceito de área de

uma figura retangular (figura 3).

Fig. 3 – Tela de questões

Acreditamos que a atividade de ensino por nós produzida no ambiente computacional

pode favorecer a construção do conceito de área de figuras geométricas planas, pois leva o

Page 410: Numero 09 - Outubro de 2007

aluno a compreender que tal conceito consiste no preenchimento de uma determinada

superfície de acordo com a unidade de área adotada e que melhor irá se adequar à situação e,

só posteriormente, é sintetizado pela fórmula “base x altura”. Para nos orientar sobre o

conceito de medida recorremos a Caraça (2000), que afirma que para medir são necessárias

três fases e três aspectos distintos: escolha da unidade; comparação com a unidade;

expressão do resultado dessa comparação por um número. (p.30).

Esta postura difere-se da que é adotada pelos livros didáticos e até mesmo pelas

explicações de professores em sala de aula, que prioriza a apresentação e aplicações da

fórmula “base x altura”, restando ao aluno uma atitude passiva, isento de uma compreensão

efetiva e significativa do conceito abordado.

Durante o processo de elaboração da atividade de ensino enfrentamos muitos

obstáculos até que o caminho a ser trilhado pelo grupo fosse escolhido. O primeiro obstáculo

foi decidir qual conteúdo iríamos abordar no nosso projeto e essa escolha se deu após vários

rodízios de idéias onde a opinião de todos os integrantes do grupo foi relevante.

O segundo foi definir de que forma iríamos trabalhar com o tema escolhido. Mas aos

poucos as idéias foram se encaixando com aquilo que queríamos: iniciar a nossa atividade de

ensino com uma atividade interativa de maneira a desafiar o aluno a construir o conceito de

área. A dificuldade foi planejar a atividade, não a parte prática em si, mas as questões que

levarão o aluno a construir o conceito de área. Essa foi a etapa do trabalho que olhamos com

mais carinho porque a maneira de propor uma atividade, pode ou não permitir que o aluno

desenvolva seu conhecimento. E como a atividade de ensino tem uma ação formadora, é

preciso que o professor tenha cuidado para não “podar” o raciocínio do aluno durante a

atividade, mas sim mediar o processo de construção do conhecimento.

O terceiro obstáculo foi decidir qual programa utilizar para que o nosso objetivo fosse

alcançado. Para implementar essa atividade utilizamos o programa Flash, e esta escolha se

deu ao fato de termos alguns colegas que manipulam bem esse programa e estavam dispostos

a colaborar com o nosso grupo. A atividade produzida permite a visualização dos espaços a

serem preenchidos e suas respectivas áreas, além de proporcionar grande interatividade já que

permite ao aluno uma postura ativa, em que este é capaz de manipular figuras, observar e

comparar resultados e buscar a alternativa mais eficaz para descobrir a área procurada.

Posteriormente, o professor deve orientar o trabalho para a sistematização de tal conceito,

promovendo a aprendizagem e auxiliando a construção do conhecimento dos alunos de forma

prazerosa e dinâmica.

Page 411: Numero 09 - Outubro de 2007

Uma das experiências que nos marcou foi a realização do trabalho em grupo onde

pudemos compartilhar idéias e refletir sobre diferentes opiniões. Pensamos que o trabalho em

grupo, pelo menos potencialmente, pode produzir resultados melhores do que se cada um dos

membros atuasse individualmente.

O trabalho também nos proporcionou um momento para refletir sobre o tipo de

avaliação que seria adotada para essa atividade de ensino. Após algumas discussões com a

professora chegamos à conclusão que o tipo de avaliação para essa atividade não tem que ser

necessariamente uma prova, pois à medida que o professor acompanha o aluno no

desenvolvimento da atividade ele já o está avaliando.

Todo processo, desde a escolha da atividade de ensino, até a escolha de qual software

a ser utilizado, exigiu um grande empenho do grupo. Muitos questionamentos foram feitos

para que a atividade via computador não tivesse alguns “bugs” que pudessem ser percebidos

por nossos alunos e que comprometessem a mesma. Tudo isto muito contribuiu para a

formação dos integrantes do grupo enquanto futuros docentes, no momento em que

descobrimos novas possibilidades de ensino via computador que podem facilitar o ensino, a

aprendizagem e o interesse tanto por parte dos pelo professores.

CONSIDERAÇÕES FINAIS

Para elaborar uma atividade de ensino, exclusiva no computador, pudemos perceber

que o mais difícil é a criação de ambientes de aprendizagem, nos quais o aluno constrói o seu

conhecimento mediado pelo professor, ao invés de o professor transmitir informação ao

aluno.

O desenvolvimento do trabalho nos trouxe um crescimento muito grande, pois

começamos a “pensar sobre” (LIMA, 1998) o que estávamos fazendo e não aceitar tudo o que

nos foi ensinado durante toda vida escolar sem questionamentos. Percebemos também que

nem toda fonte de informação é confiável e que o professor precisa refletir, questionar e

escolher qual a melhor maneira de elaborar uma atividade.

Cada parte concluída do trabalho não se tornava uma porta fechada, pois a cada dia

que líamos, mudávamos alguma imagem, situação-problema, dicas, o que nos permite

concluir que o saber do professor está em constante movimento e aperfeiçoamento.

Page 412: Numero 09 - Outubro de 2007

REFERÊNCIAS BIBLIOGRÁFICAS

LANNER DE MOURA, A. R. et all. Movimento conceitual em sala de aula. In: Anais... XI

CIAEM - Conferência Interamericana de Educación Matemática, Blumenau, SC, 2003a.

LANNER DE MOURA, A. R. et all. Movimento conceitual: atividade de ensino e de

pesquisa In: Anais... VII EBRAPEM - Encontro Brasileiro de Estudantes de Pós-Graduação

em Educação Matemática, Rio Claro, 2003b.

LEONTIEV, A. N. Uma contribuição à teoria de desenvolvimento da psique infantil. In:

VYGOTSKY, L. S. et all. Linguagem, desenvolvimento e aprendizagem. Tradução Maria

da Penha Villa Lobos. São Paulo: Ícone, 1988.

LIMA, L. C. Da mecânica do pensamento ao pensamento emancipado da mecânica. In:

caderno do professor "Trabalho e Tecnologia", Programa Integrar – CUT, São Paulo, SP,

1998.

LIMA, L. C., TAKAZAKI, M. e MOISÉS, R. P. Elementar é o essencial. São Paulo:

CTEAC, 1998.

LIMA, L. & MOISÉS, R. P. A variável: ser e não ser. São Paulo: CETEAC, 2000.

______________________. A fração: repartindo o universo. São Paulo: CETEAC, 1998.

______________________. O cálculo algébrico: operações com “Número Algébrico” – A

linguagem Algébrica 2. São Paulo: CETEAC, 1993.

MALTEMPI, M. V. Construcionismo: pano de fundo para pesquisas em informática aplicada

à Educação Matemática. In: BICUDO, M. A. V.; BORBA, M. C. (orgs.). Educação

Matemática: pesquisa em movimento. São Paulo: Cortez, 2004.

MISKULIN, R. G. S. Concepções teórico-metodológicas sobre a introdução e a utilização

de computadores no processo ensino-aprendizagem da geometria. Tese de Doutorado.

Faculdade de Educação, UNICAMP, Campinas, SP, 1999.

MOURA, M. O. A atividade de ensino como ação formadora. In: CASTRO, A. D.;

CARVALHO, A. M. P. (orgs.). Ensinar a ensinar: didática para a escola fundamental e

média. São Paulo: Pioneira Thomson Learning, 2002.

Page 413: Numero 09 - Outubro de 2007

______________. O educador matemático na coletividade de formação: uma experiência

com a escola pública. Tese de Livre Docência. São Paulo, SP, Faculdade de Educação, USP,

2000.

Page 414: Numero 09 - Outubro de 2007
Page 415: Numero 09 - Outubro de 2007

MATEMÁTICA E XADREZ: possibilidades no ensino fundamental

Rafael de Souza Duarte – UFU - [email protected] Teresa Menezes Freitas – UFU - [email protected]

INTRODUÇÃO

O ensino de Matemática tem sido percebido por muitos alunos como algo monótono, em que o professor transfere conceitos fundamentais através de aulas tediosas e maçantes. Acreditamos que por algum tempo essa idéia tenha sido predominante, mas com as constantes evoluções e pesquisas não consideramos que seja justo que esse tipo de afirmação permaneça.Estudos envolvendo várias correntes teóricas, entre estas, o construtivismo e o interacionismo, com os seus representantes Piaget e Vygotsky, alertaram os educadores para a possibilidade de dar maior dinamicidade ao ensino da Matemática em sala de aula, fazendo com que o professor não tenha a função única de transferir o conhecimento para o aluno em um discurso “bancário” meramente transferidor do perfil do objeto ou do conteúdo (FREIRE, 1996, P. 26). Acreditamos que a utilização de materiais concretos, lúdicos e da tecnologia napedagogia moderna auxilia e contribui para a eficácia do aprendizado do aluno que, através do simples “brincar”, não apresenta limites, antes encontrados dentro da sala de aula em certas matérias, ou seja, permite ao aluno evoluir segundo seu próprio ritmo. Este texto discute as possibilidades da inserção do jogo, mais especificamente do xadrez em sala de aula, na perspectiva do ensino e aprendizagem da Matemática.

Cousinet, citado por Christofoletti (2005), afirma que o jogo e a brincadeira são atividades naturais da criança, portanto, recomenda-se que a atividade educativa se baseienessas atividades, não considerando todo o tempo o adulto que todo ser humano se tornará.

Piaget (apud GRANDO, 2005), afirma existir três tipos de jogos, assim denominados:jogos de exercícios, jogos simbólicos e jogos de regras. O último engloba os dois primeiros, tornando-se o mais importante dos jogos quando a criança alcança o período das operações concretas, pois a criança torna-se capaz de jogar respeitando as regras por consentimentomútuo, ressaltando a possibilidade social da proposta.

O xadrez, por ser um jogo de regras, impõe ao aprendiz normas de planejamento e estratégia, além de uma série de julgamentos que o jogador deve fazer, pois existe umlimitador que se relaciona a interdependência entre as jogadas (anteriores e do adversário).

Kamii (apud MARQUES, 2004), estabelece três características desejáveis em um jogo para desenvolvimento moral, cognitivo e emocional do aluno. Assim, afirma a autora:

1. Em relação aos adultos, gostaríamos que as crianças desenvolvessem suaautonomia através de relacionamentos seguros, nos quais o poder do adulto seja reduzido o máximo possível.2. Em relação aos companheiros, gostaríamos que as crianças desenvolvessem sua habilidade de descentrar e coordenar diferentes pontos de vista.3. Em relação ao aprendizado, gostaríamos que as crianças fossem alertas,curiosas, críticas e confiantes na sua capacidade de imaginar coisas e dizer o que realmente pensam. Gostaríamos, também, que elas tivessem iniciativa,elaborassem idéias, perguntas e problemas interessantes e relacionassem as coisas umas às outras. (p.15).

Desta forma, percebemos que o jogo de xadrez possui as três característicasressaltadas por Kamii, pois ao jogar uma partida a criança é totalmente responsável por suas decisões, não podendo o adulto interferir em questões de lances escolhidos pelo aluno. Em suas escolhas, a criança é levada a aprender que, na estratégia de jogo, não se pode pensar em

Page 416: Numero 09 - Outubro de 2007

apenas dar xeque-mate e sim, em problemas estratégicos que envolvam ganho de peças ou vantagem posicional, forçando-a a descentralizar seus pontos de vista em relação ao que está acontecendo no tabuleiro. Além dessas vantagens acreditamos que a mediação do professor em momentos oportunos contribui para o desenvolvimento no aluno da capacidade de análise da partida que, por meio de reflexão e comunicação com adversário e com o professor, detectaerros de estratégia, de raciocínio, entre outros.

Vale ressaltar a importância que o jogo de xadrez apresenta para a educação, poisestudos apontam a possibilidade deste jogo aprimorar habilidades de suma importância para o jovem estudante. Lasker (apud GIUSTI, 1999), relata algumas dessas habilidades: raciocíniológico, concentração, paciência, autocontrole (físico e mental), projeção de cenários futuros (formação de conjecturas para a matemática,) entre outros.

No decorrer de uma partida de xadrez, vários fatores influenciam nas decisões da criança, porém, vários destes trabalham em função da eficácia do raciocínio. Concentração, atenção e previdência são muito importantes nessa estrutura de base para a formação de uma estratégia vitoriosa. Essas habilidades contribuem para uma boa construção do raciocínio lógico da criança que, implicará em maior facilidade na resolução de questões matemáticas.

Já faz alguns anos que o primeiro autor deste artigo trabalha com o jogo de xadrez, ensinando seus alunos a história, teoria e prática deste jogo. O ingresso no Curso deMatemática na UFU propiciou um novo universo educativo, relacionando xadrez eMatemática de uma forma até então desconhecida pelo mesmo. Através de estudos da psicologia da educação, métodos de ensino da matemática e, em particular, a disciplina ‘Instrumentação para o ensino da Matemática’, percebeu-se a estreita relação existente entreEnsino de Matemática e Jogo de Xadrez.

Ao longo do primeiro semestre letivo de 2006, algumas leituras e vivências deexperiências de ensino foram realizadas na disciplina acima citada, sob a orientação da professora Maria Teresa Menezes Freitas – segunda autora deste artigo. Uma das atividades avaliativas desta disciplina se relacionava ao desenvolvimento de um projeto que envolvia a criação de propostas abrangendo materiais concretos, jogos, aspectos lúdicos ou umadinâmica diferenciada para o ensino da Matemática. Entre os itens a ser avaliado destacava-sea apresentação e discussão, ao fim do período letivo, do produto final dos estudos e pesquisasrealizados, podendo o grupo contar com a utilização dos recursos que se fizessem necessário.Assim, foi desenvolvido um projeto intitulado “Projeto Xadrez-Matemática” relacionado ao ensino da Matemática em que se associou o jogo a alguns conteúdos de Matemática do ensino básico.

Sob a orientação da professora, responsável pela disciplina acima citada, foi possívelaplicar em sala de aula as atividades abordadas no Projeto Xadrez-Matemática, com jovensmatriculados na 5ª e 7ª séries de um colégio em que o primeiro autor deste artigo faz parte do corpo docente.

A seguir, descrevemos os detalhes relacionados à experiência realizada.

MÉTODO UTILIZADO

Jogo: XadrezMaterial: Um jogo de xadrez a cada dois alunos.Objetivo: Dar xeque-mate ao rei inimigo. (vide anexo 1)

Page 417: Numero 09 - Outubro de 2007

O jogo de xadrez tem sido considerado um jogo complexo e, talvez por essa razão,exige um tempo maior de dedicação ao estudo de sua teoria para um aprendizado efetivo. No entanto, há a possibilidade de se ensinar Matemática durante esse processo sem que haja alguma dificuldade relacionada à teoria do jogo. Desse modo, o método utilizado teve um ano de duração com uma aula (50 minutos) por semana incorporada ao currículo escolar das crianças.

Para a experiência foram selecionadas vinte crianças de 5ª e 7ª séries de um colégio da rede privada em Uberlândia – MG que, durante o primeiro semestre letivo aprenderammovimentos e regras básicas do jogo de xadrez obtendo total noção das exigências do mesmo,adquirindo, assim, conhecimento para formar estratégias e táticas de jogo. Nesse momento, podemos notar certa semelhança com a Matemática, pois, através de problemas estratégicosencontrados pelo aluno foi possível auxiliá- lo a encontrar a maneira certa de resolvê- lo. Esse auxílio muito se assemelha ao método de resolução de problemas de George Polya, onde a criança é orientada a primeiro compreender, identificar o problema, segundo, a compor um plano, em seguida, executar este plano e por último analisar o resultado. Todas essas etapas foram verificadas em uma partida de xadrez tornando desta maneira, um meio eficiente de ensinar o aluno a entender problemas matemáticos e, conseqüentemente, melhorando aeficiência nestes. Veja quadro abaixo:

Processo de Polya Processo enxadrísticoCompreensão do problema Identificação de debilidades do

adversárioElaboração de um plano Elaboração de uma estratégiaExecução do plano Execução da combinação (seleção de

posições ganhadoras)Avaliação dos resultados Reflexão sobre o processo desencadeado,

análise da partida.

A partir do segundo semestre de 2006, iniciou-se o processo de ensino de conceitos matemáticos relacionando sempre que possível o xadrez.

DESENVOLVIMENTO

Com alunos da 7ª série, o plano cartesiano tornou-se bastante compreensível através de uma batalha enxadrística, semelhante à batalha naval. Sem uma sistematização formal do conteúdo em questão o aluno adquiriu satisfatoriamente as noções de coordenadas no plano.Acompanhe o exemplo:

Com o tabuleiro (mural) vazio, o professor inicia a brincadeira pedindo a um aluno que indique a coordenada da casa desejada falando a letra da coluna e o número da linha. Oprofessor poderá controlar as casas escolhidas pelos alunos anotando-as no quadro. Caso o aluno acerte uma peça, este poderá escolher mais uma casa e assim sucessivamente. Cada acerto corresponde a um ponto e no final da partida quem acertar a posição exposta no mural ganhará cinco pontos incentivando, desta maneira, a competição entre eles. Observe diagrama abaixo:

Page 418: Numero 09 - Outubro de 2007

Neste exemplo, a posição indicada no mural é de xeque-mate ao rei preto. Portanto, se o aluno acertar a posição da última peça, ele terá a chance de ganhar mais cinco pontos caso acerte o que a posição indica.

A partir desta brincadeira, foi possível inserir no contexto a identificação dos eixos coordenados, o eixo das abscissas e o das ordenadas, como traçar pontos no plano cartesiano e a noção de traçar gráficos de equações. Nesse momento a formalização do conteúdo tornou-senecessária para a continuação da brincadeira. Para fixar tais conceitos, a brincadeira foiprolongada, aumentando a competição e aprofundando um pouco mais no conteúdo. Por exemplo, o aluno que acertou a posição indicada no diagrama (último passo indicado acima)poderá ganhar mais pontos caso acerte informações corretas a respeito da colocação das peças como a torre na casa A8, informações estas especificadas pelo professor antes de recomeçar a brincadeira. Por exemplo : transformar a torre em um ponto T com coordenadas (A; 8). Acoordenada “A” corresponderia à abscissa do ponto T e a coordenada “8” seria a ordenada de T, entre outras a critério do professor. Cada informação certa corresponde a um ponto.

As letras pertencentes ao tabuleiro são facilmente substituídas por números para completa compreensão de plano cartesiano. Para inserir o conceito matemático em questão

Page 419: Numero 09 - Outubro de 2007

associando à prática do jogo, basta a introdução do estudo de notação algébrica de uma partida de xadrez.

Nesta atividade, a maior parte dos alunos obteve total compreensão do conceito matemático estudado, no entanto, dois alunos tiveram certa dificuldade em absorvertotalmente o conteúdo, encontrando dificuldade principalmente em localizar pontos comcoordenadas que tinham o número zero ou na abscissa ou na ordenada.

Para sanar essa dificuldade foi preciso debater com os alunos em qual fileira (coluna)do tabuleiro ficavam as peças com abscissa (ordenada) 1, 2 e assim em diante para perceberem que a coordenada do ponto com abscissa (ordenada) igual a zero ficaria sobre o eixo das abscissas (ordenadas).

Com alunos da 5ª série do ensino fundamental, iniciou-se primeiramente a noção de área, principalmente no aperfeiçoamento da utilização de unidades de área. Inicialmente, através de problemas simples como o cálculo da área do tabuleiro utilizando, por exemplo, uma casa A1, como unidade de área e, em seguida, elevando o nível de dificuldade dosproblemas construindo uma figura mais complexa, fornecendo ao aluno sólida compreensãorelacionada à unidade de área. A figura abaixo ilustra um exemplo desta proposta.

Calcular a área da seta em destaque utilizando as seguintes unidades de área:

Problemas semelhantes ofereceram oportunidade de estabelecer interações entrealunos e proporcionaram bom entendimento das áreas em relação às suas unidades. Emrelação ao item b, algumas crianças tiveram dificuldade em solucionar a questão efreqüentemente afirmavam: “mas professor, esse quadrado não cabe na figura toda”. No entanto, orientando, desafiando e estimulando o diálogo foi possível proporcionarencaminhamentos que fizessem com que os alunos concluíssem que, no quadrado (unidade de área) em questão, podia-se desmembrá- lo em oito triângulos da questão c, tornando-o umproblema mais simples.

Ainda com crianças da 5ª série foi possível trabalhar o conceito de frações no qual forautilizado o tabuleiro como um ‘geoplano’. Com aplicações semelhantes como, por exemplo, formação de ilhas se estabeleceu grande intimidade dos alunos com as frações e suasoperações fundamentais. Para o tabuleiro tornar-se um geoplano, basta transformar cada casa deste em um ponto do geoplano. Desta maneira, obtém-se um geoplano na forma quadrada8x8. (Vide Anexo 2).

Assim, foi possível mostrar que em uma adição ou subtração de frações de mesmo denominador, este não se altera e, também foi possível explorar o conceito de frações

Page 420: Numero 09 - Outubro de 2007

equivalentes e sua simplificação. Para a introdução do assunto basta substituir o elástico utilizado no ‘geoplano’ pelas peças do jogo de xadrez, proporcionando ao professor maior poder de interferência em uma partida de xadrez entre os alunos abordando-os com questões matemáticas relacionadas às frações. Veja diagrama abaixo.

? Indique quantas casas do tabuleiro as peças brancas ocupam. A seguir monte a fração que as peças brancas representam no tabuleiro.? E as peças pretas?? Qual fração, em casas, do tabuleiro representa todas as peças em jogo?? E as casas vazias?

Essas interferências possibilitaram a fixação e compreensão do conteúdo por meio de exercícios realizados de maneira descontraída e espontânea relacionados ao jogo, propiciandocom que a criança resolvesse as tarefas necessárias sem a pressão e o sentimento de obrigaçãopenosa de solucionar uma lista de exercícios ou a tarefa de casa.

Paralelamente às aulas e experiências com os estudantes da 5ª e 7ª séries, foi tambémrealizado um trabalho com as crianças da 4ª série visando, em um primeiro momento, a familiarização do jogo pelo jogo. Neste ambiente de brincadeira e descontração, foi possível fixar certos conceitos matemáticos relacionados ao jogo, como, relação lucro-prejuízo,maximização de lucro ou minimização de prejuízo conforme exemplo apresentado mais adiante no texto.

Por meio do jogo os alunos, sem mesmo perceber, utilizam alguns conceitos relacionadosà matemática para conseguir melhor vantagem em determinado momento da partida e,conseqüentemente, levam esse conhecimento para sua vida cotidiana.

Quanto à relação lucro-prejuízo em uma partida de xadrez, pode-se trabalhar desde o início da aprendizagem do jogo, pois é uma das maneiras de se obter vantagem suficiente para alcançar o objetivo principal do jogo, o xeque-mate. Observe que a cada peça presente no jogo associa-se um valor material (vide anexo 1). Desta maneira, em uma posição normal e freqüente de uma partida de xadrez, pode-se auxiliar o aprendiz de modo bem simples e claro.

Page 421: Numero 09 - Outubro de 2007

Veja diagrama abaixo:

Na posição indicada acima, o Peão da casa a3 ameaça capturar o Bispo situado em b4. Se o jogador (das peças pretas) capturar esse peão terá prejuízo, pois o Peão posicionado em b2 capturará o Bispo preto em seguida, ou seja, as brancas ganharão 3 pontos enquanto as pretas ganharão somente 1 ponto (3-1 = 2 pontos de prejuízo). Caso o Bispo preto capture o Cavalo branco que está em c3, o jogador das pretas não terá lucro nem prejuízo, pois o Cavalo também possui valor material igual a 3. Durante esse processo é possível introduzir tímida e informalmente o conceito de coordenadas no plano, conceito esse a ser formalizado no decorrer da 5ª série.

A partir de exemplos, como os citados acima, chegou-se a conceitos mais complexos envolvendo mais peças do jogo e com maior troca de peças, maior lucro ou prejuízo. Veja exemplo abaixo:

O Cavalo preto posicionado na casa f6 encontra-se ameaçado por duas peças brancas (Bispo em g5 e Cavalo em d5). Caso o Cavalo branco o capture, o jogador das pretas terá, para minimizar o prejuízo, duas opções a seguir: capturar o cavalo branco com a Dama (d8) ou capturá- lo com o peão de g7. Se o Cavalo branco for capturado pela Dama, então o jogador das peças pretas terá prejuízo de 7 pontos. Veja:

Page 422: Numero 09 - Outubro de 2007

Peças Capturadas

Pretas:

(3 pontos) + (3 pontos) = 6 pontos

Brancas:

(3 pontos) + (10 pontos) = 13 pontos

Porém, se o jogador (das peças pretas) capturar o Cavalo branco com o peão de g7, então ele minimizará o seu prejuízo a zero. Veja diagrama abaixo:

Agora, se o jogador (das peças brancas) capturar o Peão preto, então quem terá prejuízo será ele próprio, pois ele trocará o Bispo (3 pontos) pelo Peão (1 ponto) tendo um prejuízo de 2 pontos. Observe que, mesmo inconscientemente, a criança trabalha o tempo todo com lógica, mais um fator que torna esse jogo tão importante como ferramenta pedagógica.

CONSIDERAÇÕES FINAIS

O Projeto Xadrez-Matemática apresentado ao final do semestre letivo a todos os alunos da turma da disciplina optativa do curso – Instrumentação para o Ensino daMatemática - apresentou-se como uma experiência rica em que todos puderam discutir e salientar aspectos importantes relacionados à proposta que foram implantados na prática oportunamente.

Entre os aspectos evidenciados destacamos aquele que relaciona o desenvolvimento das habilidades nos alunos, principalmente, com o empenho e a orientação firme e segura porparte do professor que deve ter clareza da importância de sua mediação no processo de ensino aprendizagem.

Acreditamos que a amplitude pedagógica deste jogo milenar merece ser muito bem aproveitada pelo educador, seja na parte matemática, seja na socialização do indivíduo, pois estudos apontam que inclusive no âmbito esportivo é possível desenvolver no aluno valores como a consciência do saber ganhar e saber perder e do respeito ao adversário. Vale observar

Page 423: Numero 09 - Outubro de 2007

que o xadrez é um jogo que não tem discriminação em relação ao sexo, ou seja, uma garotapode jogar em igualdade com um garoto diferentemente ao que acontece em alguns esportesde competição.

O estudo e a prática do jogo de xadrez possibilitam, além de tudo, a substituição dealguns materiais concretos, minimizando o tempo despendido em uma aula, pois em um determinado momento, a criança já dominará as regras e objetivo do jogo, permitindo aoprofessor iniciar imediatamente a construção de novo conceito matemático sem se preocupar em ensinar as regras novamente. Dessa maneira, a criança desenvolve cada vez mais sua capacidade de concentração, raciocínio lógico, formação de conjecturas, abstração,autocontrole, paciência, autonomia, memória e, principalmente, sua criatividade eimaginação.

Portanto, além de contribuir para a formação de conceitos da matemática o jogo de xadrez é um eficiente meio para se formar um indivíduo social, com valores bem definidos e características importantes como pensamento crítico para conviver-se em sociedade.

Em anexo, apresentamos os aspectos relacionados ao jogo de xadrez na expectativa que outros professores usem sua criatividade para o desenvolvimento da Matemática em sala de aula de maneira original e, assim contribuindo para preservação de uma cultura milenar.

Page 424: Numero 09 - Outubro de 2007

ANEXO 1

Aspectos Iniciais do jogo - Movimentos e Regras

O xadrez é constituído de: 1 tabuleiro 8x8, 32 peças (16 brancas e 16 pretas) assim subdivididas:

Um Rei branco e um preto;

Uma Rainha branca e umapreta;

Duas Torres brancas e duas pretas;

Dois Bispos brancos e dois pretos;

Dois Cavalos brancos e dois pretos;

Oito Peões brancos e oito pretos;

Tabuleiro

O Movimento das peças

Bispo: movimenta-se em diagonal;Valor: 3 pontos;

Rei: movimenta-se de uma em uma casa para qualquer lado;Valor: a partida;

Page 425: Numero 09 - Outubro de 2007

Torre: movimenta-se em vertical ouhorizontal;Valor: 5 pontos;

Dama: movimenta-se em vertical,horizontal ou diagonal quantas casasquiser;Valor: 10 pontos;

Cavalo: anda uma casa como a Torre e em seguida uma como o Bispo (seguindo a mesma direção);Valor: 3 pontos;

Peão: anda somente uma casa na vertical por vez, captura uma casa na diagonal;Valor: 1 ponto.

Page 426: Numero 09 - Outubro de 2007

Abaixo, veja a forma de se ensinar o movimento e a existência de cada peça encontrada por uma professora.

“Dois reis – um de pele branca e outro de pele negra – descobriram terrasinexploradas. E começam a disputá-las porque querem aquele novo território. Eles usam uma capa muito pesada para se proteger na batalha, por isso, apesar de caminharem em qualquer direção, só andam uma casa por vez. As rainhas(damas), muito vaidosas, são corajosas e se movem para todos os lados do reino. Elas moram ao lado dos reis . Os melhores amigos das rainhas são os bispos, que vivem ao lado delas e têm um problema na perna, por isso só andam na diagonal. Para tentar vencer essa guerra, os reis construíram um castelo em cada extremo do reino e duas torres para protegê-los. Os cavalos dos dois reinos também são fortes e os únicos que pulam peças. Cada rei tem oito soldados (peões), que ficam na linha de frente e protegem o reino”.(GUIDI, 2006)

Disposição inicial das peças

As brancas sempre iniciam a partida. Cada lado tem direito a mover uma peça por lance. Os lances são alternados.

Objetivo do jogo

O objetivo do jogo consiste em dar xeque-mate no adversário. Quando uma peça ameaça o rei inimigo, ou seja, o rei está dentro do raio de ação da peça adversária, ele está em xeque. Se o rei não puder fugir para uma casa segura, ou colocar uma peça entre ele e a peça adversária para obstruir o xeque, ou capturar a peça que o está ameaçando, então ele está em xeque-mate. Exemplos:

Page 427: Numero 09 - Outubro de 2007

A dama branca ameaça o rei preto (xeque) e o rei preto pode fugir (casas marcadas em vermelho) para uma casa segura;

Posição de Xeque-mate

A torre branca ameaça o rei preto que não tem casas para se esconder, pois todas as casas que ele poderia ir estão ameaçadas, ou pela torre (linha verde) ou pela dama (linha azul).

ANEXO 2

Cada casa do tabuleiro (a1, por exemplo) de xadrez equivale a um ponto vermelho do geoplano quadrado.

As ilhas no geoplano são representadas da seguinte maneira:

Page 428: Numero 09 - Outubro de 2007

A ilha ilustrada acima representa quatro pontos vermelhos de um total de sessenta e quatro, ou seja, possui representação de fração na forma 4/64.

Enquanto que com o tabuleiro de xadrez a mesma ilha pode-se representar do seguinte modo:

No diagrama acima se usam as casas ocupadas por peças para representar os pontos envolvidos pelo elástico do geoplano, isto é, as peças dispostas no tabuleiro acima também representam 4/64 do tabuleiro.

Observe também que, quanto à multiplicação de frações, podemos tratá- la da seguinte maneira: as peças brancas ocupam duas casas, ou seja, metade da ilha (4/64). Então, para sabermos qual fração do tabuleiro as peças brancas representam, basta multiplicarmos ½ (metade da ilha) por 4/64 (fração que a ilha representa) e, assim, obteremos uma fração igual a 4/128. Simplificando essa fração, chegamos ao resultado de 2/64 que pode ser verificado facilmente pelo aluno no tabuleiro.

Com o exemplo citado acima, torna-se trivial a regra da multiplicação de frações que é expressa por “numerador multiplica numerador e denominador multiplica denominador”,pois o estudante a visualizará de forma geométrica (no tabuleiro), e não como uma simples regra-“sem-pé-nem-cabeça”.

Page 429: Numero 09 - Outubro de 2007

REFERÊNCIAS BIBLIOGRÁFICAS

CARNEIRO, C. F. e LOUREIRO, L. A importância do xadrez na educação das crianças.Editora Adonis, 2005;

CHRISTOFOLETTI, D. F. A. O jogo de xadrez na educação matemática inhttp://www.efdeportes.com/efd80/xadrez.htm Acesso em 20/12/2006

D’AGOSTINI, G. Xadrez Básico. Ediouro Publicações S.A., 2002;

FREIRE, P. Pedagogia da Autonomia: saberes necessários à prática educativa. São Paulo: Paz e Terra, 1996.

GIUSTI, P. Xadrez: da escola aos primeiros torneios. Barcarola Editora, 1999;

GRANDO, R. C. O jogo e a matemática no contexto da sala de aula. Editora Paulus, 2005;

GUIDI, S. 27/05/2006. O jogo do xeque-mate in jornal Folha de São Paulo;

MACHADO, R. M. Explorando o Geoplano. VIII Encontro Nacional de EducaçãoMatemática, 2004;

MARQUES, M. B. O jogo como alternativa para as aulas de matemática nas séries finais do ensino fundamental, VIII Encontro Nacional de Educação Matemática, 2004;

Page 430: Numero 09 - Outubro de 2007
Page 431: Numero 09 - Outubro de 2007

TECNOLOGIA DE INFORMAÇÃO E COMUNICAÇÃO NO ENSINO DE CÁLCULO.

Patrícia Oliveira Costa Professora da Universidade Federal de Uberlândia - Departamento de Matemática

38408-100, Campus Santa Mônica, Uberlândia, MG E-mail: [email protected]

Arlindo José de Souza Júnior Professor da Universidade Federal de Uberlândia - Departamento de Matemática

38408-100, Campus Santa Mônica, Uberlândia, MG E-mail: [email protected]

Palavras chaves: Calculo Diferencial e Integral, Modelagem e Ensino com Software.

Resumo:

Nesse artigo apresentamos a investigação que estamos realizando sobre a prática colaborativa de ensinar e aprender Cálculo Diferencial e Integral na Universidade.

O desenvolvimento desse projeto está ocorrendo através de reuniões períodicas entre dois professores da Faculdade de Matemática, de uma aluna do curso de Licenciatura em Matemática e de uma aula do curso de Engenharia Elétrica. Nestas reuniões são produzidos e socializados saberes docentes relacionados ao uso educacional da mídia informática (Web Quest, Software Modellus e Sistema de gerenciamento de Curso - Moodle).

Esta investigação está sendo aprimorada através de um processo de produção e análise de dados obtidos através de um acompanhamento sistemático. Nesse estudo estaremos apresentando a nossa reflexão de como o trabalho de projeto pode contribuir para a integração das Mídias no processo de ensinar e aprender Cálculo.

1. Introdução:

O projeto que está sendo desenvolvido é constituído de quatro objetivos principais:

O primeiro diz respeito ao fato da disciplina de Cálculo Diferencial e Integral ser historicamente considerada como problemática, devido, principalmente, ao seu alto índice de reprovação. Este fato vem provocando insatisfações tanto nos alunos como nos professores.

O segundo diz respeito à utilização do computador no processo de ensino-aprendizagem destas disciplinas, quer seja como elemento de motivação dos alunos, quer seja por se procurar realizar uma educação no mundo atual, e no mundo atual o computador esta presente.

O terceiro diz respeito ao desenvolvimento de práticas pedagógicas que incorporem as aplicações do Cálculo.

Page 432: Numero 09 - Outubro de 2007

O quarto diz respeito à dificuldade dos alunos em visualizações, das matérias de Cálculo que são em sua maioria ministradas apenas teoricamente, ou seja, a capacitação desse aluno na teoria e na aplicação.

Ultimamente, observa-se um número crescente de pesquisas envolvendo esta temática. A partir do final da década de noventa, observamos o surgimento de grupos no interior das Universidades Brasileiras que desenvolveram trabalhos educativos muito importantes relacionados a tecnologias da informação e comunicação (TIC) no processo de ensinar e aprender Cálculo Diferencial e Integral.

Há um grande número de professores e pesquisadores que utilizam softwares como o Modellus, Derive, Mathematica, Maple, dentre outros, na ação de ensinar e aprender Cálculo Diferencial e Integral, atendendo as necessidades tecnológicas de capacitação e de estilos de aprendizagem dos mais variados cursos. No decorrer dessa pesquisa procuramos conhecer e analisar uma diversidade de softwares.

2. Tecnologias de Informação e Comunicação – TICs.

Qual seria a finalidade do uso dos computadores e de outras tecnologias associadas?

Primeiramente devemos assegurar a motivação e a formação dos professores.Com o uso de computadores nas salas de aula, bibliotecas e salas de estudo, o aluno ocupará um papel mais central no processo de ensino/aprendizagem, sendo o seu ritmo mais respeitado e desenvolvendo-se uma aprendizagem mais eficaz e sólida. Ocasionando assim um ensino mais moderno e mais dinâmico, adaptado ao mundo globalizado de hoje.

Uma vantagem também óbvia é a diminuição das desigualdades entre os alunos do interior e os dos grandes centros, para não falar das diferenças entre países. Com a possibilidade de acesso à Internet, com todos os serviços que ela disponibiliza, e também com o uso de CD-ROMs, todos os alunos podem ter acesso a informação muito mais variada. Estas mudanças vão necessariamente ocorrer, mais cedo ou mais tarde, sob o risco, se tal não acontecer, da escola se afastar perigosamente da sociedade, ficando mesmo à margem dela.

Para que tudo isto se torne possível, a escola deve investir no professor, em cursos de capacitação, informática, softwares e modelagens e é claro que devemos contar com uma ajuda sistemática dos nossos governantes também, com projetos e decretos destinados a estes fins.

Com professores sensibilizados para o uso das novas tecnologias, as mudanças serão necessariamente mais rápidas. Os estudantes, ao comunicarem com pessoas em locais mais ou menos distantes, começam a compreender, a apreciar e respeitar as semelhanças e diferenças entre línguas, culturas e políticas. Interiorizam que não interessa a raça, o aspecto físico e o nível social, mas sim a troca de idéias e conhecimentos e o valor de algumas dessas idéias e conhecimentos. A visão do mundo e do lugar que nele ocupam vai-se forçosamente alterar. Os estudantes serão levados naturalmente a pensar em assuntos de interesse global.

3. O Trabalho:

Ao desenvolvermos um trabalho colaborativo utilizando um sistema de gerenciamento de cursos (Moodle) e o software (Modellus), procuramos realizar uma prática integrada em torno do processo de Ensinar e aprender Cálculo Diferencial e

Page 433: Numero 09 - Outubro de 2007

Integral. No planejamento dessa disciplina o trabalho de projetos foi organizado em torno do desenvolvimento de atividades. O nosso interesse nesta investigação tem sido o de compreender como as tecnologias da Informação e Comunicação, podem contribuir para o acompanhamento do processo de produção dos projetos dos alunos nos encontros presencias e virtuais.

Esta investigação1 está ocorrendo na disciplina de Cálculo Diferencial e Integral I, do curso de Química, da Universidade Federal de Uberlândia. O processo de produção de dados está se desenvolvendo através das observações das aulas, da aplicação de questionários e do acompanhamento do trabalho desenvolvido no sistema de gerenciamento de cursos.

Vários softwares podem ser trabalhados, dentre eles o Modellus, pois permite a visualização e cálculos relacionados nesta disciplina, em particular ferramentas de produção de gráficos de funções e de cálculo de integrais definidas por vários métodos numéricos com apoio visual. Ou seja, esse software é muito eficiente quando se trata de Cálculo levando em consideração a seguinte ementa: equações, funções, gráficos, limites, derivadas, integrais, áreas e volumes.

4. Software Modellus:

O Modellus, que pode ser encontrado no site http://phoenix.sce.fct.unl.pt/modellus é de origem portuguesa da Universidade Federal de Lisboa. Construído em linguagem C++, o software possui interface de janelas das quais possui sete tipos de distintas funções, integradas pelo mesmo modelo matemático:

Janela Modelo: Nessa janela digitamos as propriedades de todos os objetos a serem criados nas demais janelas. Podem-se escrever funções iterações e equações diferenciais ordinárias. Além de poder integrar numericamente. A variável independente é inicialmente designada por t, mas pode ser designada por qualquer outra letra.

Figura 1: Modelagem no MODELLUS

1 Esta pesquisa conta também com a colaboração de Maísa Gonçalves da Silva (Graduanda do curso de Matemática) e Maria Leonor Silva de Almeida (Graduanda do curso de Engenharia Elétrica) da Universidade Federal de Uberlândia.

Page 434: Numero 09 - Outubro de 2007

Janela Controle: Nesta janela encontram-se as opções relativas à variável independente.

Janela Condições Iniciais: Nessa janela podemos atribuir valores distintos aos parâmetros de uma família de funções que constituirão “casos”, que podem ser analisados separadamente ou sobrepostos nas demais janelas.

Janela Tabela: O programa permite por essa janela o acompanhamento dos valores das variáveis envolvidas no modelo nos diferentes casos.

Janela Notas: Permite ao autor do modelo inserir algum comentário sobre ele, funciona como um bloco de notas.

Janela Gráfico: Podem ver-se múltiplas representações de relações matemáticas. Por exemplo: equações, tabelas, gráficos e animações.

Janela Animação: Nesta janela estão integrados diversos recursos para permitir a criação de objetos que simulem o modelo matemático criado, permitindo visualizarmos e manipularmos os mesmos. Aqui, diversos recursos são oferecidos para que o usuário decida sobre a construção do modelo.

A versão dois do Modellus permite também a utilização de fotografias, vídeos, gráficos, etc., como registros de informação para construção de modelos. Uma vez colocadas essas imagens como “fundo” numa janela de “Animação”, podem utilizar-se “ferramentas de medida” de coordenadas, distâncias, áreas, declives e ângulos. As medidas assim efetuadas podem ser utilizadas para construir modelos, modelos esses que podem depois ser comparados com as imagens ou podem ser utilizados para construir animações.

Observamos que o software utilizado é de fácil manejo e de domínio público, tornando ainda mais fácil a familiarização dos ambientes computacionais com o ensino de Cálculo. Seguindo as críticas referentes ao software mencionado anteriormente, buscamos analisar outros recursos tecnológicos aplicados no ensino, deparamos com algumas experiências descritas por pesquisadores de instituições de ensino superior, que descreveram aplicações em sala de aula.

5. Plataforma Moodle:

A intenção geral era fazer com que a aula de Cálculo 1 ficasse ligada com a informática, para que o aluno tivesse acesso a todas as aulas, dia de provas, trabalhos e pudessem tirar dúvidas on – line com os professores e os demais alunos. Com uma coleta de dados expressiva e depois de muita pesquisa, resolvemos trabalhar com o MOODLE, sistema de gerenciamento de cursos, disponibilizado pela Universidade Federal de Uberlândia. Para os alunos terem acesso a esta plataforma de ensino, eles devem se cadastrar na mesma e em particular na Plataforma de Cálculo Diferencial e Integral (CDIMAT). Será disponibilizado a eles uma senha para cadastro na plataforma, e um login junto com uma senha que somente eles devem saber. A plataforma contém dezesseis atividades para os alunos, dentre elas:

Um questionário: é necessário saber a familiaridade dos alunos com o computador e a internet; Quatro fóruns: para que os alunos deixem suas opiniões em relação a aula, plataforma e software MODELLUS (afim de estreitar a relação entre aluno professor);Apresentação no Power Point da aula, contendo exemplos e explicações;

Page 435: Numero 09 - Outubro de 2007

Quatro listas de exercícios sobre o conteúdo da disciplina de Cálculo; Quatro Modelagens envolvendo funções, limites, derivada, otimização e Integral; Web Quest sobre a história do Cálculo.

A inclusão do MOODLE e do MODELLUS na disciplina de Cálculo iniciou – se na primeira semana de aula do primeiro semestre de 2007, (de acordo com o calendário da Universidade Federal de Uberlândia). A princípio os alunos tiveram aversão, principalmente aqueles que faziam a matéria pela segunda vez, mas agora utilizam comandos antes desconhecidos pra nós, modelando funções cada vez mais expressivas.

Figura 2: Início da Plataforma de Cálculo diferencial e Integral

Figura 3: Algumas atividades da Plataforma CDIMAT

Page 436: Numero 09 - Outubro de 2007

Figura 4: Introdução da Webquest: História do Cálculo

As aulas sobre a plataforma e o Modellus são ministradas em um laboratório de Ensino, disponibilizado pela direção do curso de Matemática – UFU, e as monitorias em um laboratório de informática cedido pela Coordenadora do Curso de Química – UFU. No contrato pedagógico, foi estabelecido que os trabalhos devem ser entregues na própria plataforma por um link em cada atividade, sendo cada aluno responsável por seu envio. A plataforma não aceita envio de trabalhos atrasados, cabendo a cada professor responsável determinar hora, data e tamanho do arquivo.

6. Resultados:

A princípio os alunos de tiveram aversão, principalmente àqueles que faziam a matéria pela segunda vez, estavam acostumados a aulas somente teóricas e sabiam que para obter resultados teriam que trabalhar muito. Tivemos longos debates, afim de discutimos qual seria a melhor maneira de orienta – los, para que não ficassem sobrecarregados. Para fazer as modelagens no Software, o aluno precisava conhecer bem o conteúdo a ser trabalhado, pois caso contrário, não saberia o porquê de uma parábola ficar mais aberta ou fechada, se o gráfico do logaritmo de 1x está cortando o eixo x no lugar correto, se as assíntotas do limite tocam a função ou mesmo se o Modellus calculou através de integral, a área da roda gigante corretamente, por exemplo. Com o passar do tempo, todos, inclusive eu, fomos aprimorando os conhecimentos teóricos e práticos, até que os alunos começaram a utilizar comandos antes desconhecidos pra nós, fazendo modelagens cada vez mais expressivas. O que mais me chamou atenção foi o fato de um aluno tirar zero na primeira prova de limites, o que me deixou muito preocupada. No começo do semestre parecia bem desmotivado, mas se mostrou muito interessado pela plataforma e pelo software Modellus, já que trabalhava em uma loja de materiais de informática junto com o pai. Com as aulas no laboratório e em sala de aula, a curiosidade e a vontade de modelar o

Page 437: Numero 09 - Outubro de 2007

conteúdo foram lhe despertando o interesse pelo estudo, até que suas modelagens eram as melhores apresentadas. Para fazer o trabalho da plataforma ele estava comparecendo em minha sala, depois da aula, em média três vezes por semana, para saber se em algum conteúdo da disciplina, ele poderia fazer ou não determinadas observações. Foi aí que ele começou a se entregar ao curso de Cálculo Diferencial e Integral, conseguiu notas boas nas duas últimas provas, seus trabalhos foram os mais elaborados e criativos, enfim ele conseguiu passar.

Figura 5: Trabalho do Modellus do aluno do primeiro período do curso de Química – UFU .

Figura 6: Trabalho do Modellus do aluno do primeiro período do curso de Química - UFU.

Page 438: Numero 09 - Outubro de 2007

Esta prática de ensino a distância faz com que os alunos se sintam mais a vontade, se abram mais, pois a aula virtual é de certa forma equiparada com as aulas presenciais. Nesse estudo observamos que existem diferentes propostas de se trabalhar com computadores no processo de ensinar e aprender Cálculo. Constatamos que essas práticas pedagógicas estão sendo trabalhadas em muitos cursos e que o trabalho com o computador é desenvolvido paralelamente as aulas convencionais de Cálculo. É importante compreender que um dos fatores que interfere neste propósito são os diferentes softwares utilizados.

7. Referências:

ARAÚLO. L. Jussara. Situações Reais e Computacionais: Os Convidados São Igualmente Bem-Vindos?. Universidade Estadual Paulista .MESQUITA. FILHO. Julio – UNESP _ RIO CLARO. Revista Bolema. Boletim de Educação Matemática. Ano 16. nº 19, 2003.

BARUFI, Maria C. Bonomi. A construção/negociação de significados no curso universitário inicial de Cálculo Diferencial e Integral. São Paulo, 1999. Tese (Doutorado em Educação), Faculdade de Educação, Universidade de São Paulo.

BEZERRA, J. Q. O computador e o cálculo diferencial. In: V ENCONTRO NACIONAL DE EDUCAÇÃO MATEMÁTICA (ENEM), 5., 1995, Aracaju. Anais...Aracaju, 1995, p. 85.

CARRILLO, Wenceslao, R. de Los Rios. EJECUCION Y EVALUACION DE UN PROYECTO DE ENSEÑANZA PROGRAMADA PARA EL MEJORAMIENTO DE LA ENSEÑANZA DEL CALCULO EN LA UNIVERSIDAD DE PANAMA.Campinas, 1980. Dissertação (Mestrado em Ensino de Ciências e Matemática), Convênio OEA- PREMEN - UNICAMP). Universidade Estadual de Campinas

CASSOL, Armindo. A educação matemática no ensino superior. In: ENCONTRO NACIONAL DE EDUCAÇÃO MATEMÁTICA (ENEM), 5., 1998, São Leopoldo. Anais... São Leopoldo, 1998. V. 1. p. 108-109.

PONTE, João P. Concepções do professor de matemática da formação. In: BROWN, M. , FERNANDES, D. , MATOS J. F. ,PPONTE, J. P. (Ed.) Educação Matemática. Temas de investigação. Lisboa: SEM-SPCE, 1992ª.

SOUZA JR. Arlindo. J. S.1993, Concepções do professor universitário sobre o ensino da matemática. Tese(Mestrado em Educação Matemática) - Rio Claro, Universidade Estadual Paulista,

SOUZA JR. Arlindo. J. S. Concepções do professor universitário sobre o ensino da matemática. Rio Claro, 1993. Dissertação( Mestrado em Educação Matemática) – Universidade Estadual Paulista.

VITALLE, Bruno. Computador na escola; um brinquedo a mais? Ciência Hoje, Rio de Janeiro, v. 13, n. 77, p. 19-25, out./nov. 1991.

Page 439: Numero 09 - Outubro de 2007

PAPERT, Seymour. A máquina das crianças: repensando a escola na era da informática. Porto Alegre: Artes Médicas, 1994.

RATTNER, Henrique. Informática na sociedade. São Paulo: Brasiliense, 1985.

REZENDE, W.M. O ensino de Cálculo: dificuldades de natureza epistemológica. In Machado, N.: Cunha, M.(org) Linguagem, Conhecimento, Ação- ensaios de epistemologia e didática. Escrituras, São Paulo, 2003.

RIPPER, Afira Viannna. O preparo do professor para as novas tecnologias. In: OLIVERRA, Vera Barros (org). Informática em Psicopedagogia. São Paulo: SENAC, 1996. p.54-83.

SILVA,Miriam Godoy P. O computador na perspectiva do desenvolvimento profissional do professor. Campinas, 1997.Tese (doutorado em Educação) – Faculdade de Educação, Universidade Estadual de Campinas.

VALENTE, J. A. Computadores e conhecimento; repensando a educação. Campinas: UNICAMP, 1993. 417p. cap.1: Diferentes usos do computador na educação, p.123. cap.2: Por Quê o computador na educação?, p.24-44. cap.7: Formação de profissionais na área de informática em educação, p.114-134.

_____________http://nautilus.fis.uc.pt/softc/Read_c/destaque/enstec.htm

Page 440: Numero 09 - Outubro de 2007
Page 441: Numero 09 - Outubro de 2007

Aplicação simultânea de dois métodos de classificação étnico-racial

Aurélia Aparecida de Araújo Rodrigues1

[email protected]

Corina Ilda Silva Ferreira2

[email protected]

Willian Araujo Moura3

[email protected]

Gabriela Vieira Lopes3

[email protected]

ResumoO objetivo deste trabalho é analisar os resultados de dois métodos de classificação étnico-racial: a autoclassificação e a classificação por terceiros. A pesquisa contou com a participação dos alunos curso de Psicologia da Universidade Federal de Uberlândia, ingressantes no primeiro semestre de 2006, os quais responderam simultaneamente a um amplo questionário. Os métodos de classificação apresentaram resultados divergentes para a classificação de alunos de cor preta.

Palavras-chave: classificação étnico-racial, autoclassificação, classificação por terceiros.

1. Introdução Atualmente, estão em discussão as possibilidades e os limites das políticas de ação

afirmativa e os resultados até agora atingidos por medidas universalistas adotados pelo Estado. Enquanto as políticas universalistas partem do pressuposto de que o ideal é dar o mesmo tratamento a todos, as de ação afirmativa tem como meta tratar de maneira diferente os diferentes, minimizando as formas de discriminação que impeçam o acesso a oportunidades e benefícios, de forma a se atingir a igualdade real (Tragtenberg, 2006). As políticas inclusivas (de ação afirmativa) trabalham com os conceitos de igualdade e de universalização tendo em vista a redução da desigualdade social (Curry, 2005).

No Brasil, há vários casos em que a implantação de políticas universalistas não promove necessariamente a igualdade real. Em se tratando da educação superior no país, Telles (2003) mostrou que os grandes beneficiários do boom do ensino superior ocorrido entre 1960 a 1999 foram os brancos, os quais apresentaram crescimento médio anual da proporção de diplomados cerca de quatro vezes maior do que os negros, usando dados do Instituto Brasileiro de Geografia e Estatística (IBGE).

No campo da saúde, pro exemplo, Batista et. al. (2004) relataram diferenças nas causas de óbitos segundo cor/raça no Estado de São Paulo e Leal et al. (2005) destacaram desigualdades raciais na assistência ao pré-natal e ao parto no município do Rio de Janeiro. Diferentes métodos de classificação étnico-racial têm sido avaliados, em estudo de saúde no Brasil e em outros países. Alguns autores consideram que a autoclassificação (comparada à categorização realizada por entrevistadores) é o método que melhor reflete a identificação do indivíduo com os diversos grupos raciais. Outros que apontam que a melhor

1 Orientadora. Professora Adjunto da Faculdade de Matemática da Universidade Federal de Uberlândia (UFU). 2 Aluno de graduação em Ciências Sociais da UFU. 3 Aluno de graduação em Psicologia da UFU.

Page 442: Numero 09 - Outubro de 2007

abordagem depende do objetivo do estudo. Assim, por exemplo, no caso de investigações sobre a discriminação racial, observadores externos expressariam melhor a forma como o indivíduo é visto pela sociedade (Williams, 2002 apud Kaplan e Bennett, 2003).

O padrão classificatório, no Brasil, é freqüentemente caracterizado pela fluidez (indeterminação, subjetividade e dependência contextual) em sua aplicação. Constata-se, por exemplo, que quanto maior a posição sócio-econômica do indivíduo, mais recorrente é a possibilidade dele se “embranquecer” (Silva, 1999).

Dentro deste contexto, o objetivo principal deste trabalho é analisar classificação étnico-racial em duas abordagens distintas: a autoclassificação e a classificação por terceiros. As análises foram feitas com base em dados obtidos do questionário que foi aplicado a um grupo de 30 estudantes simultaneamente. Será dada ênfase para a análise dos resultados referentes à classificação para a cor preta.

Espera-se, ainda, que este trabalho contribua com as discussões sobre a classificação étnico-racial e suas conseqüências nas diversas áreas sociais, políticas e econômicas.

Na literatura, há vários trabalhos discutindo questões relacionadas à classificação étnico-racial, mas poucos apresentam dados e análises estatísticas. Este trabalho trata-se de um estudo de caso, a análise feita aqui poderá ser estendida para outras situações.

2. Metodologia A pesquisa contou com a participação dos 30 alunos curso de Psicologia da

Universidade Federal de Uberlândia, ingressantes no primeiro semestre de 2006, os quais responderam simultaneamente a um amplo questionário, ver Apêndice. Os alunos autores deste trabalho aplicaram o questionário durante o segundo semestre de 2006, na ocasião eles estavam cursando a disciplina Introdução à Estatística no Curso de Psicologia.

O questionário continha dois itens distintos para a classificação étnico-racial, além de perguntas referentes ao racismo, classificação étnico-racial e características sociais (idade, estado civil, renda familiar e outros). Aqui serão analisados os resultados referentes à pergunta 4 e pergunta 11, que correspondem à autoclassificação e classificação por terceiros, respectivamente.

Em um dos blocos iniciais foi utilizada a pergunta proposta pelo IBGE, com suas opções de resposta para a classificação da cor. Cabe ressaltar, no entanto, que preto é a cor e negro é a raça. Não há cor negra, como muito se ouve. Há cor preta. De acordo com a convenção do IBGE, portanto, negro é quem se autodeclara preto ou pardo.

Pergunta 4 do questionário (autoclassificação): Qual a sua cor?( ) Amarela ( ) Branca ( ) Indígena ( ) Parda ( ) Preta Ao final do questionário, foi incluída uma pergunta onde deveria ser informado o

número de pretos nessa mesma turma . Pergunta 11 do questionário (classificação por terceiros): Em sua turma, quantos

estudantes você classificaria como pretos? ( )Nenhum ( )1 ( )2 ( )3 ( )4 ( )5 ( )Mais de 6

3. Resultados Nos itens que seguem, serão abordadas as respostas aos questionamentos levantados

na seção 2, a partir dos resultados encontrados nas tabelas 1 e 2 e nos gráficos 1 e 2 . Conforme observado no Gráfico 1 e Tabela 1, observa-se que 10 alunos se consideram

da raça negra, que é formada por indivíduos de cor preta ou parda.Na Tabela 1, dos trinta participantes que responderam ao questionário, apenas um

(3,3%) se autoclassificou de cor preta. No entanto, conforme Tabela 2, na classificação por terceiros, obteve-se classificações de zero a três estudantes de cor preta na turma, ou seja,

Page 443: Numero 09 - Outubro de 2007

6,7% dos entrevistados não identificaram alunos de cor preta na turma, 36,7% identificaram apenas um, 43,3% identificaram dois e 13,3% identificaram três.

Tabela 1: Autoclassificação étnico-racial dos alunos do curso de Psicologia-UFU ingressantes em 2006-1

Autoclassificação (cor) Freqüência Porcentagem

Indígena 0 0,0% Preta 1 3,0%

Amarela 4 13,3% Parda 9 30,0% Branca 16 43,3%

0

2

46

8

10

1214

16

18

Indígena Preta Amarela Parda Branca

Cor

Freq

üênc

ia

Gráfico 1: Autoclassificação étnico-racial dos alunos do curso Psicologia-UFU ingressantes em 2006-1

Tabela 2: Classificação por terceiros, referente a cor preta, de acordo com a classificação dos alunos do curso Psicologia-UFU ingressantes em 2006-1

Número de estudantes pretos

na turmaFreqüência Porcentagem

Nenhum 2 6,7% 1 11 36,7% 2 13 43,3% 3 4 13,3%

Page 444: Numero 09 - Outubro de 2007

0

2

4

6

8

10

12

14

Nenhum Um Dois Três

Nº de estudantes pretos

Freq

uênc

ia

Gráfico 2: Classificação por terceiros, referente a cor preta, de acordo com a classificação dos alunos do curso Psicologia-UFU ingressantes em 2006-1

4. Conclusão:De acordo com o resultados obtidos no estudo de caso deste trabalho, a classificação

por terceiros e a autoclassificação realizadas simultaneamente foram divergentes para a classificação de alunos de cor preta. No método de autoclassificação, apenas um estudante se considerou de cor preta. Na classificação por terceiros, o número de pretos no grupo variou de nenhum a três.

Critérios de classificação étnico-racial semelhantes têm sido usados em algumas universidades brasileiras para auxiliar na distribuição de vagas pela política de cotas raciais, que é um programa de ação afirmativa. Na etapa de inscrição para o vestibular, o candidato a uma vaga na universidade se autoclassifica, e numa outra etapa, ele é submetido à classificação por terceiros (com uso de fotos e comissões de classificação racial). As divergências de classificação étnico-racial que, eventualmente ocorrem durante esse processo, têm submetido os candidatos a constrangimentos e desencadeado críticas por parte da sociedade. No entanto, a classificação por terceiros é útil para identificar os possíveis oportunistas (brancos que se autoclassificam como negros para obter privilégios) no processo de seleção das universidades.

Então, se o objetivo é implantar programas de ação afirmativa para negros, pode-se aperfeiçoar o sistema de distribuição de vagas que utiliza política de cotas raciais, por exemplo, pode-se reduzir as divergências entre a autoclassificação e a classificação por terceiros praticada pelas comissões de classificação racial. Rosemberg (2004) sugere ações como explicitar em anúncio prévio o que é um programa de ação afirmativa, porque ele é necessário e a quem ele se destina.

A escolha da estratégia e do sistema de classificação deve levar em consideração vantagens e desvantagens de cada uma das alternativas, além dos objetivos da investigação. Por exemplo, ao utilizar as categorias do IBGE ganha-se em termos de comparabilidade com outros estudos, mas é preciso ponderar a possibilidade de tendência ao “branqueamento”, especialmente nos estratos de melhor posição sócio-econômica. Vale ressaltar, como apontam Kaplan e Bennett (2003), que qualquer conjunto de opções de resposta é apenas uma aproximação do que é socialmente considerado como categorias étnico/raciais. Assim, os pesquisadores devem ser cautelosos ao inferir que os indivíduos pertencem a estas categorias

Page 445: Numero 09 - Outubro de 2007

de maneira definitiva, ou que cada uma dessas categorias agrega um grupo absolutamente homogêneo (Maio et. al., 2005).

Apesar das limitações inerentes ao que se convencionou denominar de classificação étnico-racial, é de grande valia um estudo sobre a classificação racial no Brasil, pois através dela é possível, por exemplo, delimitar de que adoece (morbidade) e de que morre a população negra; e obter indicadores fundamentais para políticas de combate ao racismo em instituições de diversas áreas.

5. Referências BibliográficasBATISTA, L. E.; ESCUDER, M. M.; PEREIRA, J. C. A cor da morte: causas de óbito

segundo características de raça no Estado de São Paulo, 1999 A 2001. Revista de Saúde Pública, vol. 38 (5), pp. 30-36, 2004.

CURRY, C R J. Políticas inclusivas e compensatórias na educação básica. Cadernos de Pesquisa, vol. 35 (124), pp. 11-32, 2005.

KAPLAN, J. B.; BENNETT, T. Use of race and ethnicity in biomedical publication. JAMA,vol. 289, pp. 2709-2716, 2003.

LEAL, M. C.; DA GAMA. S. G.; DA CUNHA, C. B. Desigualdades raciais, sóciodemográficas e na assistência ao pré-natal e ao parto, 1999-2001. Revista de Saúde Pública, vol. 39 (1), pp. 100-107, 2005.

MAIO, M. C.; MONTEIRO, S.; CHOR, D.; LOPES, C. S. Resultados comparativos de dois métodos de autoclassificação no Rio de Janeiro, Brasil. Cad. Saúde Pública, vol 21(1), pp. 171-180, 2005.

MORETTIN, P. A.; BUSSAB, W. O. Estatística Básica. Saraiva: São Paulo, 2002. MUNANGA, K. A difícil tarefa de definir quem é negro no Brasil. Estudos avançados, vol.

18 (50), pp. 51-56, 2004. Entrevista. ROSEMBERG, F. O branco no IBGE continua branco na ação afirmativa? Estudos

avançados, vol. 18 (50), pp. 61-66, 2004.SILVA, N. V. Uma nota sobre “raça social” no Brasil. In Hasenbalg, C. A; Silva, N. V.;

organizadores. Cor e estratificação social. Editora Contracapa: Rio de Janeiro, pp. 107-125, 1999.

TELLES, E. Racial ambiguity among the Brazilian population. Ethn Racial Stud, vol. 25, pp. 415-441, 2002.

TELLES, E. Racismo à brasileira: uma nova perspectiva sociológica. Relume Dumará: Rio de Janeiro, 2003.

TRAGTENBERG, M.H. R.; BASTOS, J. L. D.; PERES, M. A. Políticas de ampliação do acesso e diversidade socioeconômica e étnico-racial na Universidade Federal de Santa Catarina. Acessado em set.2007. Disponível em http://www.acoes-afirmativas.ufsc.br/artigos/2006-09-30-artigo-pagina-comissao.pdf.

Page 446: Numero 09 - Outubro de 2007

Apêndice

Questionário N __________

1- Qual a sua idade? ____________

2- Qual o seu estado civil?( ) Casado ( ) Solteiro ( ) Divorciado ( ) Viúvo ( ) Outros

3- Qual a renda líquida mensal da sua família, em reais? ( ) Até 300 ( ) De 301 a 1000 ( ) De 1001 a 1500 ( ) De 1501 a 2000 ( ) De 2001 a 3000 ( ) De 3001 a 5000 ( ) Acima de 5000

4- Qual a sua cor?( ) Amarela ( ) Branca ( ) Indígena ( ) Parda ( ) Preta

5- Para você, as diferenças existentes no Brasil são raciais ou se referem à classe social? ( )Racial ( )Classe ( )Ambos

6- Você acredita que existe racismo em nosso país? ( )Sim ( )Não

7- Você se considera racista? ( )Sim ( )Não ( )Às vezes

8- Você convive com alguma pessoa racista? ( )Sim ( )Não

9- Você já sofreu algum preconceito racial? ( )Sim ( )Não

10- Você já presenciou alguma atitude racista? ( )Sim ( )Não

11- Em sua turma, quantos estudantes você classificaria como pretos? ( )Nenhum ( )1 ( )2 ( )3 ( )4 ( )5 ( )Mais de 6

Page 447: Numero 09 - Outubro de 2007

O Estudo Matemático do Comportamento das Abelhas

Universidade Federal de Uberlândia

Faculdade de Matemática

Eduardo Henrique Siqueira Lucas Dias Marques Rosana Sueli da Motta

Molinero Jafelice

[email protected] [email protected] [email protected]

IntroduçãoAbelha é a denominação comum de vários insetos pertencentes à ordem Hymenoptera,

da superfamília Apoidea, aparentados das vespas e formigas. O representante mais conhecido é a Apis mellifera, oriunda do Velho Mundo, criada em larga escala para a produção de mel. As espécies de abelhas nativas das Américas (Novo Mundo) não possuem ferrão.

Uma abelha visita dez flores por minuto em busca de pólen e do néctar. Ela faz, em média, quarenta vôos diários, tocando em 40 mil flores. Com a língua, as abelhas recolhem o néctar do fundo de cada flor e guardam-no numa bolsa localizada na garganta. Depois voltam à colmeia e o néctar vai passando de abelha em abelha.Desse modo a água que ele contém se evapora, ele engrossa e se transforma em mel, veja Figura 1.

Figura 1: Rainhas e operárias.

As abelhas são insetos sociais que vivem em colônias e são conhecidas há mais de 40.000 anos e as que mais se prestam para a polinização, ajudando enormemente a agricultura, produção de mel, geléia real, cera, própolis e pólen, são as abelhas pertencentes ao gênero Apis.

Inseto laborioso, disciplinado, a abelha convive num sistema de extraordinária organização: em cada colméia existem cerca de 80.000 abelhas e cada colônia é constituída por uma única rainha, centenas de zangões e milhares de operárias.

O objetivo do trabalho é estudar como as abelhas vivem, trabalham, são divididas, se reproduzem e modelar matematicamente a dinâmica da população de uma nova colméia.

Page 448: Numero 09 - Outubro de 2007

Um Estudo Geométrico da Colméia

Inicialmente, estudamos a geometria dos alvéolos.

- Geometria dos alvéolos

As abelhas constroem suas casas ou favos na forma de recipientes aglomerados de cera que se propagam um ao lado do outro.

Os recipientes, denominados alvéolos, tem a forma de um prisma hexagonal regular (faces laterais iguais e ângulos entre as faces iguais) aberto numa extremidade e formando um ápice triédrico na outra face.

Os alvéolos são usados tanto para o desenvolvimento populacional da colméia como para deposito de mel, produto obtido da transformação do néctar e pólen das flores.

O estudo sobre o mosaico de um favo foi realizado com base em [1].

- Mosaico de um favo

O corte transversal de um favo apresenta a configuração de um mosaico formado pela repetição de hexágonos regulares.

A pavimentação de um plano consiste em cobri-lo com uma mesma figura, sem deixar espaços vazios ou tendo figuras interseccionadas.

Se quisermos um mosaico, como na Figura 2, formado pela propagação de um só tipo de polígono regular (lados iguais e ângulos internos iguais), devemos escolher o polígono de modo que seu ângulo interno seja um divisor de 360º.

Vejamos alguns casos, Figuras 3, 4 e 5:

Figura 2: Mosaico de um favo. Figura 3: Quadrado tem ângulos

internos iguais a 90º.

Page 449: Numero 09 - Outubro de 2007

Figura 4: Triângulo eqüilátero tem Figura 5: Hexágono regular tem ângulos ângulos internos iguais a 60º. internos iguais a 120º.

Todo polígono regular pode ser inscrito em um circulo de modo que seus lados sejam cordas deste circulo. Assim, dado um polígono regular de n lados podemos sempre dividi-lo em n triângulos isósceles. Cada triângulo é formado considerando o lado do polígono como base e tendo vértice no centro do circulo que circunscreve o polígono, Figura 6.

Figura 6: Polígonos circunscritos.

Sabemos que um polígono regular pode se propagar, formando um mosaico, se 360/for um numero inteiro positivo, onde é o ângulo interno do polígono. Este número nos dá a quantidade de polígonos que tem vértice comum.

Sabemos que o ângulo interno deve ser menor que 180º. Por outro lado o polígono de menor número de lados é o triângulo eqüilátero, n = 3. Neste caso, temos o intervalo de 60º até 180º. Os divisores positivos de 360º são: 1, 2, 3, 4, 5, 6, 8, 9, 10, 12, 15, 18, 20, 24, 30, 36, 40, 45, 50, 60, 72, 90, 120, 180 e 360.

Concluímos então que temos as seguintes possibilidades: 60, 72, 90 e 120.

Analisemos a figura quando = 72º.

72º = (n – 2).180º/n 72n = 180n – 360 -108n = -360 n = 10/3 N

Com isso concluímos que: as abelhas constroem seus alvéolos na forma de prismas de base hexagonais. Das três possíveis escolhas das bases que restou, ou seja, dos valores possíveis de que poderiam ter sido escolhidos, optaram pelo polígono que tem o menor perímetro, com área A fixada, que é a base hexagonal onde = 60°.

Page 450: Numero 09 - Outubro de 2007

- Dança das abelhas

O estudo da dança das abelhas foi realizado com base em [3]. Uma abelha pode lembrar-se da rota de vôo a partir da posição do sol no céu, do odor e

da cor das flores. É capaz, também, de retornar à mesma fonte de alimento, no mesmo horário, do dia seguinte. O pesquisador Von Frisch foi quem descobriu a forma de comunicação das abelhas, ou seja, quando encontra uma boa fonte de néctar e pólen retorna para informar às demais a posição e o odor das flores. Ela toma como referência a posição do sol, isto é, o ângulo entre sua própria rota de vôo e uma linha horizontal da colméia, na direção do sol.

Sua forma de comunicação é denominada a “dançado requebrado”. Quando as flores estão a menos de 100 metros de distância da colméia, a dança é circular. Se o alimento está a mais de 100, a abelha corre para frente por uma pequena distância, retornando ao ponto inicial por um semicírculo, e volta descrevendo um outro semicírculo na direção oposta, dando uma idéia de oito, veja Figura 7. Se a dança é feita a 30° à direita da vertical significa que o alimento está a 30° à direita do sol [2].

Figura 7: A dança das abelhas.

Ao dançar na colméia, outras abelhas podem aprender a posição e o odor das flores, embora não aprenda sua cor e sua forma. O número de vezes por segundo que a abelha perfaz o circuito “dançando” indica a distância da florada em relação à colméia. Crane (1983) apresenta a duração de cada circuito da dança pela distância:

Distância (m) 200 500 1000 2000 3500 4500 Duração do circuito (s) 2,1 2,5 3,3 3,8 5,6 6,3

Como podemos localizar uma florada a partir da dança da abelha? Para calcular a distância da florada da colméia, procederemos utilizando coordenadas

polares.Exemplo: Se a fonte de alimento, por exemplo, estiver a 983,87m da colméia e

formando um ângulo de 60° no sentido horário em relação à direção do sol nascente (leste) podemos encontrar a distância em que a florada está da colméia em relação aos pontos cardeais, Figura 8.

Page 451: Numero 09 - Outubro de 2007

Figura 8: Localização da florada em relação à colméia em coordenadas polares.

As abelhas não usam coordenadas retangulares para comunicar a posição da fonte de alimentos. As coordenadas polares têm um papel importante no comportamento animal, principalmente na orientação de aves e peixes.

P: (x, y) coordenadas retangulares P: (r, ) coordenadas polares;

onde r = 22 yx (distância polar) e = arctgxy (ângulo polar).

Temos que a hipotenusa do triângulo retângulo (distância da colméia à florada) é 983,87m e o ângulo em relação ao eixo x (que aponta para o sul) é 30°.

Assim temos que:

sen = ry sen 30° =

87,983y 491,93m

cos = rx cos 30° =

87,983x 852,05m

Em coordenadas retangulares podemos dizer que a fonte de alimento está, aproximadamente, a 491,93m para leste e 852,05m para o sul, em relação à colméia.

A seguir um exemplo retirado de [2]. Problema: Uma abelha exploradora descobre uma fonte de mel ao entardecer. Esta

fonte está localizada a 850m a leste e 1200m ao sul da colméia. Que coordenadas polares serão sinalizadas pela abelha?

Resolução: Inicialmente utilizamos coordenadas polares, ou seja, extraímos de um sistema de coordenadas cartesianas (x, y), as coordenadas polares (r, ) de forma que:

x = r.cos

y = r.sen

Page 452: Numero 09 - Outubro de 2007

Sabemos que x = 1200 e y = 850 e que tg = xy =

1200850 0,71. Assim, temos

que: = arctg (0,71) 0,6174 35,37°. Como r = 22 yx , então r = 22 8501200 = 7225001440000 = 2162500 1470,54.

Podemos concluir então que, as coordenadas polares que serão sinalizadas pela abelha são (1470,54; 35,37°), como na Figura 9.

Figura 9: Localização da florada em relação à colméia em coordenadas retangulares e polares.

Comentário: O problema nos propôs encontrar a localização da florada em coordenadas polares, que são dadas no sentido anti-horário, mas, se o autor quisesse saber qual o ângulo entre a florada e o sol, a resposta seria 90° - 35,37° = 54,63°.

Dinâmica de uma Colméia

Quando se propõe analisar o crescimento populacional de uma comunidade qualquer, um dos objetivos é saber seu comportamento em cada instante e a previsão de seu tamanho no futuro. Cada população tem uma dinâmica de crescimento própria, isto é, uma “lei de formação” inerente à espécie.

Vamos apresentar modelos matemáticos distintos relativamente ao nível de conhecimento matemático baseados em [1], mas que expressam, essencialmente, o mesmo fenômeno; crescimento populacional de uma colméia.

- A Colméia

Entre apicultores, a expressão colméia significa abelhas alojadas racionalmente, com uma população equilibrada e distribuída em três castas: rainha, operárias e zangões.

A abelha rainha é responsável pela produção das operárias, dos zangões (que são os machos) ou novas rainhas, botando dois tipos de ovos. Os ovos fertilizados dão origem às operárias (fêmeas não reprodutoras) e os ovos não fertilizados originam os zangões. As rainhas são produzidas quando as larvas são alimentadas com nutrientes altamente protéicos (geléia real).

Page 453: Numero 09 - Outubro de 2007

A constituição de uma colméia em condições normais é a seguinte 1 rainha que pode viver até 5 anos; Até 400 zangões que são produzidos no final de verão, e sua quantidade depende da

abundância de alimento (vivem até 80 dias); 60000 a 80000 operárias. A longevidade de uma operária depende do clima e do seu

período de atividade. De um modo geral sua vida média varia de 38 a 42 dias. A capacidade de postura de uma rainha chega a 3000 ovos por dia, o que corresponde

a duas vezes seu próprio peso. Esta quantidade depende da área disponível para postura, da qualidade genética da rainha e das condições florais e climáticas existentes.

Quando uma rainha diminui a quantidade de ovos, as operárias responsáveis pela manutenção das larvas promovem o desenvolvimento de nova rainha. A nova rainha, depois do vôo nupcial em que é fecundada pelos zangões, retorna à colméia desalojando a rainha velha que sai para formar uma outra colméia. Acompanhando a velha rainha seguem um séqüito de aproximadamente 10000 operárias: é o enxame voador.

Para o estudo do crescimento da população em uma nova colméia consideraremos os seguintes dados e hipóteses:

Postura da rainha é constante: 2000 ovos por dia; Período entre a postura e o nascimento da abelha: 21 dias; Quantidade inicial de abelhas (operárias): 10000; Longevidade média de uma operária: 40 dias.

- Modelos

Um modelo matemático da dinâmica populacional de uma nova colméia deve ser apresentado, levando-se em consideração dois estágios distintos: o período de adaptação que é intermediário entre a postura inicial e o nascimento das primeiras operárias (21 dias), e o período de desenvolvimento quando nascem diariamente 2000 abelhas.

Em relação ao período inicial podemos estabelecer duas hipóteses distintas quanto ao índice de mortalidade das operárias:

H1) As abelhas têm idades equidistribuídas Neste caso estamos supondo que em cada grupo, distribuído por idade (dias de vida), existem exatamente a mesma quantidade de operárias. Desta forma, das 10000 abelhas iniciais, em cada dia morrerão, em média, 250 abelhas o que

corresponde a 401 de 10000.

Seja yn = y(n) a quantidade de operárias vivas no n-ésimo dia de existência de nova colméia, 0 n<21.

Podemos obter a expressão de y(n) recursivamente, isto é,

y0 = 10000

y1 = y0 – 250

y2 = y1 – 250 = y0 – 2 250

Generalizando, obtemos:

yn = y0 – n250

Page 454: Numero 09 - Outubro de 2007

Assim, obtemos um modelo matemático que nos dá a informação sobre a quantidade de abelhas “velhas” no n-ésimo dia de existência da colméia:

yn = 10000 – 250n , 0 n<21 (1)

Observação 1: O modelo (1) é discreto no sentido que a variável independente n (tempo) está tomando valores no conjunto dos números naturais N.

Observação 2: A equação (1) pode ser obtida, analisando a taxa de decaimento. Seja k > n, definimos y = yk - yn: quantidade de abelhas que morrem entre o k-ésimo

e o n-ésimo dia e n = k – n: um número de dias passados, então a razão incremental ny é

dada por:

250)250)(()1(...)]2()1[()]1([

)(...)()( 1211

nknk

nnkkkkyyyyyy

nkyy

ny nnkkkknk

ou seja, a razão entre a variação da quantidade de abelhas pela variação do tempo é constante. Isto significa que o resultado para um dia n qualquer poderia ser obtido por uma regra de três: “A quantidade de abelhas que morrem em n dias é proporcional a n”. Por exemplo, se em 1 dia morrem 250, em 21 dias morrerão 5250 abelhas:

1 25021 x

21x 5250250

A constante C = -250 é o coeficiente angular da reta:

y(t) = -250t + 10000 com 0 t 21, t R, (2)

que representa o modelo contínuo correspondente à equação (1). Chamamos a atenção para o fato de que a constante de “proporcionalidade” usada

numa regra de três é equivalente ao coeficiente angular de uma reta, ou seja, só podemos usar regra de três quando as variáveis estão relacionadas segundo a equação de uma reta.

H2) A mortalidade das abelhas é “proporcional” a quantidade que se tem de abelhas em cada instante.

Observe que com esta hipótese não podemos usar regra de três. A taxa de mortalidade

é401 = 0.025 e portanto, a taxa de sobrevivência é (0-0.025) = 0.975.

Podemos agora obter uma expressão de recorrência (modelo discreto) para yn com esta nova hipótese:

y0 = 10000

y1 = 0.975y0

y2 = 0.975y1 = (0.975)²y0

yn = (0.975)ny0, (3)

Page 455: Numero 09 - Outubro de 2007

Usando o fato de que ax = exlna, para todo x R, com a>0 a função potência (3) pode ser dada na forma exponencial:

yn = y0en.ln(0.975) = y0e-0.02532.n (4)

No caso continuo (tempo continuo) podemos escrever

y = y(t) = y0e-0.02532.t, 0 t 21. (5)

Tomando y0 = 10000 e t = 21 em (5), obtemos y(21) = 5876. Verificamos que, de acordo com as hipóteses consideradas, os valores de y21 são distintos – na prática tal diferença não é significativa, mesmo para o estudo do comportamento futuro da colméia.

O modelo matemático para o período de desenvolvimento da nova colméia leva em consideração que a partir do 21° dia nascem, 2000 abelhas.

Se A0 é a quantidade remanescente de operárias velhas depois de 21 dias, teremos para o 21° dia:

Y1 = y21 = A0 + 2000.

Considerando agora a taxa de sobrevivência igual a 0.975, podemos formar uma relação de recorrência a partir do valor A0:

Y2 = y22 = 0.975Y1 +2000 = 0.975A0 + 2000(0.975 +1)

Y3 = y23 = 0.975Y2 +2000 = (0.975)²A0 + 2000[(0.975)² + 0.975 + 1)].

E assim sucessivamente, chegamos a

Yn = (0.975)n-1A0 + 2000[(0.975)n-1 + (0.975)n-2 + ... + 0.975 + 1].

A expressão entre colchetes é a soma de uma progressão geométrica de razão igual a 0.975, o que nos permite simplificar, escrevendo:

Yn = (0.975)n-1A0 + 2000975.01

)975.0(1 n

= (A0 - 78000)(0.975)n-1 + 80000. (6)

Podemos pensar numa expressão contínua para Yn tomando:

y(t) = (A0 – 78000)e(t-21).ln(0.975) + 80000 (t 21)

ou seja,

y(t) = (A0 - 78000)e0.02532.(t – 21) + 80000 t 21. (7)

A expressão (7) nos dá a população da colméia num tempo t qualquer a partir do 21° dia.

Podemos notar que quando t cresce o valor de e-0.02532 tende a zero e portanto a população da colméia se estabiliza com 80000 operárias, o que mostra uma coerência com os dados experimentais.

Isto pode ser traduzido pela expressão matemática

Page 456: Numero 09 - Outubro de 2007

80000)(lim tyt

A reta y = 80000é uma assíntota horizontal da função y(t), denominada valor de estabilidade.

Juntando as duas partes do modelo contínuo de crescimento populacional das abelhas (equações (5) e (7)), podemos escrever;

y(t) = 10000 e-0.02532.t se 0 t<21(8)

y(t) = (A0 - 78000)e-0.02532.(t - 21) + 80000, se t 21

onde, A0 são as sobreviventes no 21° dia.

- Lei de formação de uma colméia

No Caso contínuo (tempo t como variável contínua) podemos usar a linguagem de derivadas e expressar a hipótese H2 da seguinte forma:

dtdy = -0.025y (9)

y(0) = 10000, 0 t 21

ondedtdy indica a variação instantânea da população de abelhas. O modelo (9) quer dizer que

até os primeiros 21 dias, a variação da população de abelhas (mortalidade) é proporcional à

quantidade presente em cada instante, com um índice de mortalidade igual a 401 = 0.025 e

uma população inicial de 10000 abelhas. A solução da equação (9) é obtida separando-se as variáveis e integrando

ydy = -0.025dt, logo

ydy = 025.0 dt,

ou

lny = -0.025t + k (k: constante de integração)

donde tiramos

y(t) = eke-0.025.t.

Usando a condição inicial y(0) = 10000, vem que ek = 10000. Assim,

y(t) = 10000e-0.025.t, 0 t<21 (10)

A solução (10) é aproximadamente igual à (5) obtida anteriormente.

Page 457: Numero 09 - Outubro de 2007

Para o período de crescimento da colméia, podemos fazer uma analogia com o “modelo exponencial assintótico” uma vez que, em ambas as situações as soluções são semelhantes.

Consideramos então a equação diferencial:

dtdy = k(L - y) (11)

onde, L = 80000 é a população limite, t 21, k = ln0.975 e y(21) = 7500 ( 5500remanescentes mais 2000 recém nascidas).

Separando variáveis e integrando a equação (11), obtemos

yLdy = kdt –ln(L – y) = kt + c.

Portanto, L – y = ece-kt.Considerando que a equação (11) está definida para t 21, podemos escrever

y(t) = L – ece-k.(t - 21), t 21.

Como y(21) = 7500, temos

-ec = 7500 – 80000 = -72500

Portanto,

y(t) = -72500e-0.02532.(t - 21) + 80000, para t 21. (12)

Conclusão

Desta forma, podemos dizer que a “lei de formação” de uma colméia nova é a seguinte:

“O crescimento populacional de uma colméia é proporcional à diferença entre a população máxima sustentável e a população dada em cada instante.”

Salientamos mais uma vez que nenhum modelo matemático é definitivo. Sempre podemos modificá-lo tornando-o mais realista. Por exemplo, no período de adaptação (início da colméia) a rainha não tem condições de colocar 2000 ovos por dia pois os alvéolos ainda nem estão construídos. Também, a hipótese simplificadora que considera uma postura constante da rainha, nesta colméia, pode ser modificada.

Page 458: Numero 09 - Outubro de 2007

Bibliografia[1] BASSANEZI, Rodney Carlos. Modelagem Matemática como Método de Ensino-

Aprendizagem. Editora Contexto, 2002.

[2] BATSCHELET, Edward. Introdução à Matemática para Biocientistas. Tradução: Vera M.

A. P. da Silva e Junia Maria P. Quitete. São Paulo: Editora da Universidade de São Paulo,

1978.

[3] BIEMBENGUT, Maria Salett & HEIN, Nelson. Modelagem Matemática no Ensino.

Editora Contexto, 2003.

Page 459: Numero 09 - Outubro de 2007

FAMAT em Revista

Revista Científica Eletrônica daFaculdade de Matemática - FAMAT

Universidade Federal de Uberlândia - UFU - MG

Número 09 - Outubro de 2007www.famat.ufu.br

Iniciação Científicaem Números

���

Page 460: Numero 09 - Outubro de 2007

Comitê Editorial da Seção Iniciação Científica em Números

do Número 09 da FAMAT EM REVISTA:

Maria Luiza Maes (coordenadora da seção) Márcio José Horta Dantas

Page 461: Numero 09 - Outubro de 2007

Projetos De Iniciação Científica Que Se Realizam Durante O Período

De Setembro De 2007 a Agosto De 2008

Orientador: Arlindo José de Souza Júnior

Orientado: Maísa Gonçalves da Silva

Título: O Ensino de Cálculo com o Auxílio da Informática

Início: novembro de 2006

Fim: outubro de 2007

Orientador: Aurélia Aparecida de Araújo Rodrigues

Orientado: Robson Silva Rossi

Título: Cálculo de medidas de desempenho de gráficos de controle para Variáveis utilizando

o Matlab

Início: novembro de 2006

Fim: outubro de 2007

Orientador: Aurélia Aparecida de Araújo Rodrigues

Orientado: Mateus Araújo Kappel

Título: Monitoramento de um processo de produção industrial utilizando Gráficos de controle

Início: abril de 2007

Fim: março de 2008

Orientador: César Guilherme de Almeida

Orientado: Ernani Magno de Freitas Júnior

Título: Técnica de Decomposição de Domínio e de Pré-Condicionamento de Matriz no

Cálculo da Velocidade de Darcy em Escoamentos em Meios Porosos

Início: março de 2007

Fim: fevereiro de 2008.

Page 462: Numero 09 - Outubro de 2007

Orientador: Cícero Fernandes de Carvalho

Orientado: Danilo Adrian Marques

Título: Álgebra Comutativa e Computação Algébrica

Início: agosto de 2006

Fim: novembro de 2007

Orientador: Cícero Fernandes de Carvalho

Orientado: Milena Almeida Leite Brandão

Título: Introdução aos códigos corretores de erros

Início: Dezembro de 2005

Fim: Dezembro de 2007

Orientador: Cícero Fernandes de Carvalho

Orientado: Patrícia Borges dos Santos

Título: Introdução à teoria das curvas algébricas planas

Início: janeiro de 2007

Fim: dezembro de 2007

Orientador: Edmilson Rodrigues Pinto

Orientado: Matheus Bartolo Guerero

Título: Estudo de Modelos de Regressão com apoio computacional

Início: março de 2007

Término: março de 2008

Orientador: Edmilson Rodrigues Pinto

Orientado: Weyder Orlando Brandão Junior

Título: Estudo de Modelos de Regressão com apoio computacional

Início: março de 2007

Término: março de 2008

Page 463: Numero 09 - Outubro de 2007

Orientador: Ednaldo Carvalho Guimarães

Orientado: Joaquim Ferreira Vieira Neto

Título: Ajuste de Distribuição de Probabilidade à Dados Climatológicos de

Uberlândia-MG

Início: fevereiro de 2007

Fim: janeiro de 2008

Orientador: Edson Augustini

Orientado: Adriele Giaretta Biase

Título: Introdução à Criptografia

Início: março de 2007

Fim: fevereiro de 2008

Orientador: Edson Augustini

Orientado: Luciana Yoshie Tsuchiya

Título: O conceito de complexidade em Geometria

Início: março de 2007

Fim: fevereiro de 2008

Orientador: Edson Augustini

Orientado: Gabriela Aparecida dos Reis

Título: O conceito de complexidade em Geometria

Início: março de 2007

Fim: fevereiro de 2008

Orientador: Edson Augustini

Orientado: Thiago Rodrigues da Silva

Título: Propriedades Geométricas da Aplicação Normal de Gauss em Superfícies Regulares

Início: março de 2007

Fim: fevereiro de 2008

Page 464: Numero 09 - Outubro de 2007

Orientador: Fabiana Fiorezi de Marco Matos

Orientado: Sheila Maria Fernandes Carrijo

Título: A utilização de Jogos no Ensino de Matemática : A Intervenção Pedagógica

Início: abril de 2007

Fim: março de 2008

Orientador: Luiz Alberto Duran Salomão

Orientado: Daniel Augusto Alves de Oliveira

Título: Cálculo de Probabilidades

Início: abril de 2007

Fim: março de 2008

Orientador: Luiz Alberto Duran Salomão

Orientado: Michelle Crescêncio de MIranda

Título: Como resolver problemas: elementos teóricos e heurísticas

Início: outubro de 2006

Fim: outubro de 2007

Orientador: Marcelo Tavares

Orientado: Eloar Correia de Lima

Título: Sazonalidade e Tendências em Séries Temporais de dados de IPC de Diferentes

Instituições

Início: fevereiro de 2007

Fim: janeiro de 2008

Orientador: Marcelo Tavares

Orientado: Maria Luiza Maes

Título: Avaliação do Comportamento de Aspectos Gerenciais das Micro e Pequenas

Empresas de Uberlândia por Meio de Técnicas Uni e Multivariadas

Início: agosto/2007

Fim: julho/2008

Page 465: Numero 09 - Outubro de 2007

Orientador: Marcos Antônio da Câmara

Orientado: Giselle Moraes Resende Pereira

Título: Aplicações da Teoria dos Grafos

Início: março de 2007

Fim: fevereiro de 2008

Orientador: Marcos Antônio da Câmara

Orientado: Maksuel Andrade Costa

Título: Programação Inteira

Início: março de 2007

Fim: fevereiro de 2008

Orientador: Rogério de Melo Costa Pinto

Orientado: Gustavo Silva Sages

Título: Avaliação do Desempenho dos Alunos da Engenharia Elétrica da UFU

Início: novembro de 2006

Fim: novembro de 2007

Orientador: Rosana Sueli da Motta Jafelice

Orientado: Mariana Fernandes Santos Villela

Título: O Estudo de Modelos Biológicos p-Fuzzy

Início: março de 2007

Fim: fevereiro de 2008

Orientador: Rosana Sueli da Motta Jafelice

Orientado: Karla Barbosa de Freitas

Título: O Estudo de Modelos Biológicos p-Fuzzy

Início: setembro de 2007

Fim: fevereiro de 2008

Page 466: Numero 09 - Outubro de 2007

Orientador: Sezimária F P Saramago

Orientando: Lúcio Aurélio Purcina

Tipo de Orientação: Doutorado

Projeto: Técnicas de Otimização Aplicadas à Solução de Grandes Sistemas Lineares

Início: agosto de 2005

Fim: agosto de 2009

Orientador: Sezimária F P Saramago

Orientando: Giovana Trindade S Oliveira

Tipo de Orientação: Doutorado

Projeto: Estudo da Topologia do Espaço de Trabalho de Robôs Manipuladores 3R

Início: março de 2007

Fim: março de 2011

Orientador: Sezimária F P Saramago

Orientando: Carlos Alberto da Silva Junior

Tipo de Orientação: Mestrado

Projeto: Uma contribuição ao Estudo da Programação Linear

Início: março de 2006

Fim: março de 2008

Orientador: Sezimária F P Saramago

Orientando: Bruno Nunes de Souza

Projeto: Métodos Iterativos Não-Estacionários: Aplicação de Técnicas de Otimização na

Solução de Sistemas Lineares

Início: março de 2007

Fim: fevereiro de 2008

Page 467: Numero 09 - Outubro de 2007

Orientador: Sezimária F P Saramago

Orientando: Aline Rocha de Assis

Projeto: Evolução Diferencial aplicada à Solução de Problemas de Otimização Multi-

Objetivo

Início: dezembro de 2005

Fim: dezembro de 2007

Orientador: Sezimária F P Saramago

Orientando: Alencar Soares Bravo

Projeto: Técnicas de Realidade Virtual aplicadas à Robótica.

Início: agosto de 2007

Fim: julho de 2008

Orientador: Sezimária F P Saramago

Orientando: Alessandra Ribeiro da Silva

Projeto: Métodos Iterativos Não-Estacionários: Aplicação de Técnicas de Otimização na

Solução de Sistemas Lineares

Início: fevereiro de 2007

Fim: dezembro de 2007

Orientador: Sezimária F P Saramago

Orientando: Antônio Dias Carrijo Neto

Projeto: Estudo da Topologia do Espaço de Trabalho de Robôs Manipuladores 3R

Início: outubro de 2005

Fim: dezembro de 2007

Orientador: Sezimária F P Saramago

Orientando: Sidney Araújo Mendonça

Projeto: Estudo da Topologia do Espaço de Trabalho de Robôs Manipuladores 3R

Início: outubro de 2005

Fim: dezembro de 2007

Page 468: Numero 09 - Outubro de 2007

Orientador: Weber Flávio Pereira

Orientado: Viviane Carvalho Mendes

Título: Construções Geométricas Antigas e Modernas

Início: setembro de 2007

Fim: agosto de 2008

Orientador: Weber Flávio Pereira

Orientado: Lívia Silva Rosa

Título: Uma Introdução aos Sistemas Dinâmicos Caóticos via Família

Quadrática

Início: setembro de 2007

Fim: agosto de 2008

Orientador: Márcio José Horta Dantas

Orientado: Rafael Alves Figueiredo

Título: Uma Introdução à Mecânica Analítica e à Dinâmica não Linear e o Problema do

Vibrador Centrífugo

Início: agosto de 2007

Fim: julho de 2008

Page 469: Numero 09 - Outubro de 2007

FAMAT em Revista

Revista Científica Eletrônica daFaculdade de Matemática - FAMAT

Universidade Federal de Uberlândia - UFU - MG

Número 09 - Outubro de 2007www.famat.ufu.br

E o Meu Futuro Profissional?

���

Page 470: Numero 09 - Outubro de 2007

Comitê Editorial da Seção E o Meu Futuro Profissional?

do Número 09 da FAMAT EM REVISTA:

Marcio José Horta Dantas (coordenador da seção) Maria Luiza Maes

Page 471: Numero 09 - Outubro de 2007

Entrevista com o professor João Carlos em relação ao curso de Matemática

no campus do Pontal

Neste número, Maria Luiza Maes, aluna do Curso de Matemática, entrevista o professor João

Carlos, professor da Faculdade de Matemática. O tema da entrevista é em relação ao curso da

Matemática implantado no campus do Pontal na Universidade Federal de Uberlândia.

Maria Luiza – Qual a infra-estrutura que vocês possuem? Tem biblioteca? Laboratórios?

João Carlos – Através de um Convênio firmado entre a FEIT-UEMG, FTM e UFU,

conseguimos para o funcionamento do primeiro ano dos cursos da FACIP sete salas de aula,

um laboratório de Biologia, um de Química e um de Informática. Temos uma biblioteca que

está funcionando no Bloco administrativo da FACIP e contamos também com a biblioteca da

FEIT e FTM, onde os alunos da UFU podem consultar os livros e periódicos. Para o segundo

semestre de 2007 estará em funcionamento mais sete laboratórios, sendo um de Ensino de

Matemática e outro de Desenho Geométrico, além de um auditório para 50 lugares. Para 2008

está previsto o funcionamento de mais cinco laboratórios e oito salas de aula. Um prédio com

1200 m2 está sendo edificado para o funcionamento das atividades dos cursos da FACIP na

FEIT, as obras devem terminar em dezembro de 2007. Nosso Plano Diretor será

encaminhado ao MEC ainda este ano e as obras do Campus deverá ter início em 2008. O

Campus do Pontal conta com uma área de 500.000 m2. Encaminhamos para a FINEP um

projeto de 1,5 milhões para construir um prédio com infra-estrutura para a pesquisa na

FACIP.

M. L. – Quantas vagas possuem o curso? E quais períodos?

J. C. – O Curso possui 80 vagas anuais, sendo 40 para o Diurno e 40 para o Noturno.

M. L. – Em sua opinião, você acha que estava na hora de ter o curso de Matemática no

Campus do Pontal?

Page 472: Numero 09 - Outubro de 2007

J. C. – A região do Pontal do Triângulo Mineiro carece de professores de Matemática com

uma formação de excelência como a que a UFU oferece. Sendo assim acredito que o curso de

matemática será fundamental para a transformação da qualidade do ensino de matemática

nesta região.

M. L. – Como é a estrutura do curso? (Licenciatura e Bacharelado)

J. C. – Os alunos cursam um ciclo comum de disciplinas até o quarto período do curso e a

partir daí eles fazem a opção de cursar a Licenciatura ou o Bacharelado.

M. L. – Sabemos que possui alunos que estudam à noite geralmente trabalham durante o dia,

como estes alunos estariam integralizando os estágios obrigatórios?

J. C. – O aluno que fizer a opção pela Licenciatura Noturna tem carga horária dos estágios

prevista no próprio turno que está cursando, ele não precisa cumprir os créditos do estágio em

outro turno ou no horário de trabalho.

M. L. – No curso noturno, caso os alunos reprovem em determinada matéria de que modo

eles podem suprir esta matéria sem que os mesmos não sejam prejudicados no próximo

semestre em relação das matérias coincidirem horários?

J. C. – Como existe na FACIP outros cursos como Física e Química algumas disciplinas

comuns a estes cursos serão oferecidas em períodos diferentes, assim os alunos poderão

cursá-las se assim achar necessário. Além disso, como temos o curso de matemática tanto no

Diurno quanto Noturno, o aluno poderá cursar as disciplinas de dependência em turnos

diferentes.

M. L. – Até o momento (agosto/2007), como está o índice de evasão do curso?

J. C. – No curso Diurno está em 12,12% e Noturno 19,4%.

M. L. – Qual foi a proporção candidato/vaga no vestibular?

J. C. – Matemática Diurno 0,43 e Matemática Noturno 1,07.

M. L. – Já foi encontrado algum problema na estrutura atual do curso? Se sim, quais foram?

J. C. – Até o momento não.

Page 473: Numero 09 - Outubro de 2007

M. L. – Qual a expectativa que você possui desse curso?

J. C. – Acredito que o curso vai causar um forte impacto na qualidade do ensino da

matemática, principalmente na região do Pontal do Triângulo Mineiro.

M. L. – O curso está atendendo as expectativas dos alunos?

J. C. – Temos uma Comissão Própria de Avaliação do Curso que aplicou um questionário no

primeiro semestre de 2007 e verificou que o curso tem atendido bem as expectativas dos

alunos.

Page 474: Numero 09 - Outubro de 2007
Page 475: Numero 09 - Outubro de 2007

FAMAT em Revista

Revista Científica Eletrônica daFaculdade de Matemática - FAMAT

Universidade Federal de Uberlândia - UFU - MG

Número 09 - Outubro de 2007www.famat.ufu.br

Merece Registro

Page 476: Numero 09 - Outubro de 2007

Comitê Editorial da Seção Merece Registro

do Número 09 da FAMAT EM REVISTA:

Marcos Antônio da Câmara (coordenador da seção)

Page 477: Numero 09 - Outubro de 2007

MERECE REGISTRO

A) MESTRADO EM MATEMÁTICA NA UFU

O curso de Mestrado em Matemática da UFU recebeu mais duas bolsas para os seus alunos, dessa vez da Capes. Assim, o curso, que tem seis alunos regulares, conta agora com cinco bolsas: 2 da Fapemig, 2 da Capes e uma bolsa fornecida pela UFU, via a Pró-Reitoria de Pesquisa. A única aluna do curso que não tem bolsa, tem vínculo empregatício e, portanto, não pode receber bolsa. Para o ano que vem, existe uma previsão de 4 bolsas novas, o que também será ótimo, se for concretizado.

B) CURSO DE APERFEIÇOAMENTO

De 23 a 27 de julho de 2007 realizou-se, na Universidade Federal de Uberlândia, um módulo do Curso de Atualização para Professores de Matemática do Ensino Médio. O curso é desenvolvido com a participação de professores do IMPA Instituto Nacional de Matemática Pura e Aplicada, com o suporte do Instituto do Milênio IM/AGIMB, da FINEP, e apoio da Rede Nacional de Pesquisas. Na Universidade Federal de Uberlândia, o módulo realizado em julho de 2007 inseriu-se no projeto de extensão cadastrado na Pró-Reitoria de Extensão sob o número SIEX-5183.

Nesse projeto, professores de matemática do ensino médio, que atuam na região, assistem na parte da manhã a aulas ministradas por professores no IMPA, e transmitidas via videoconferência, e na parte da tarde resolvem e discutem exercícios relativos aos conteúdos expostos pela manhã.

Nesse módulo tivemos 54 inscrições e 36 professores foram freqüentes. As aulas foram sobre temas retirados do livro “A Matemática do Ensino Médio volume 1”, editora da SBM, 2001, e foram ministradas pelos professores Eduardo Wagner, Elon Lages Lima e Paulo Cezar Pinto Carvalho.

C) VII ERMAC - VII Encontro Regional de Matemática Aplicada e Computacional

O VII ERMAC realizado de 20 a 22 de junho de 2007 na Universidade Federal de Uberlândia foi um momento histórico para as Faculdades de Matemática, Computação e o Instituto de Física, pois pela primeira vez estas unidades acadêmicas organizaram um evento conjuntamente. O VII ERMAC foi um sucesso e contou com a participação de aproximadamente 400 participantes incluindo a presença de pesquisadores, professores, alunos de pós-graduação e um grande número de alunos de graduação. A programação do evento foi diversificada contando com mini-simpósios em três áreas: Biomatemática, Otimização e Nanociência/Nanotecnologia). Além dos mini-simpósios, o público presente pode apreciar a apresentações de trabalhos científicos recentes em sessões técnicas. Professores de renome do meio acadêmico e científico ministraram mini-cursos e proferiram palestras com diferentes temáticas: Biologia Computacional, Equações Diferencias Parciais, Arte e Matemática, Nanotecnologia e Educação Matemática. A sessão de pôsteres se apresentou como um momento marcante do evento em que trabalhos de iniciação científica

Page 478: Numero 09 - Outubro de 2007

foram avaliados por professores da UFU das respectivas áreas envolvidas no evento. Ressaltamos ainda a realização da Mesa Redonda com o tema ‘Aplicações da Matemática em outras Ciências’ que contou com a participação de um professor de renome de cada faculdade das Ciências Exatas da UFU e teve uma expressiva participação de toda a comunidade acadêmica. Merece também registrar a participação dos alunos do Departamento de Música e Artes Cênicas da UFU, coordenados pelo Prof. Ms. Eduardo Tullio, que abrilhantaram o evento em momentos especiais: Abertura, Coquetel e Encerramento.

COMITÊ ORGANIZADOR

Rosana S. Motta Jafelice - FAMAT/UFU (Coordenadora) César Guilherme de Almeida - FAMAT/UFU Dulce Mary de Almeida - FAMAT/UFU Edmilson Rodrigues Pinto - FAMAT/UFU Fernando Antônio de Freitas - FAMAT/UFU Marcelo Rodrigues de Sousa - FACOM/UFU Maria Teresa Menezes Freitas - FAMAT/UFU Qu Fanyao - INFIS/UFU

COMITÊ CIENTÍFICO

César Guilherme da Almeida - FAMAT/UFU Eliana Xavier Linhares de Andrade - UNESP João Carlos Moreira - UFU/FACIP Márcia Aparecida Fernandes - FACOM/UFU Noélio Oliveira Dantas - INFIS/UFU Olímpio Hiroshi Miyagaki – UFV

COMISSÃO DE APOIO

Maísa Gonçalves da Silva - DAMAT/UFU Matheus Bartolo Guerrero - PETMAT/UFU Rene Felipe Keidel Spada - INFIS/UFU Gil Victor Teixeira Pinto - PET/COMPUTAÇÃO/UFU Helder R. Golveia Linhares - PET/COMPUTAÇÃO/UFU

Aproveitando este espaço de registro, a Equipe de Organizadores agradece aos palestrantes convidados, aos docentes e aos discentes pela participação no evento; a FAMAT, FACOM e INFIS pelo apoio e confiança na realização do VII ERMAC; aos Coordenadores das Graduações e dos Mestrados das Faculdades envolvidas na organização do evento; aos alunos do DAMAT, do PETMAT e ao tutor do PETMAT pelo apoio nas atividades de organização do VII ERMAC; aos funcionários da FAMAT pela dedicação na organização deste encontro e a todos os patrocinadores do evento.

Page 479: Numero 09 - Outubro de 2007

D) MARCA IMPRESSIONANTE

Trabalho é o mais acessado na área de Matemática há nove meses consecutivos.

Desde julho de 2005, o artigo Two new properties of ideals of polynomials and applications – que trata de ideais de polinômios entre espaços de Banach – do professor da Unidade Acadêmica de Matemática e Estatística da UFCG, Daniel Pellegrino, em parceria com o professor Geraldo Márcio de Azevedo Botelho, da Universidade Federal de Uberlândia, vem sendo a produção mais procurada para download (leitura ou gravação) na página eletrônica The ScienceDirect TOP25 Hottest Articles (top25.sciencedirect.com), que destaca os trabalhos científicos mais requisitados em várias revistas de diversas áreas do conhecimento. São nove meses consecutivos como o primeiro da lista de acesso para downloads na área de Matemática, da revista holandesa Indagationes Mathematicae, onde o trabalho foi publicado na edição de junho de 2005. Marca impressionante, segundo Pellegrino, por ser um trabalho desenvolvido numa área de pesquisa relativamente pequena.

Maiores detalhes:

http://www.jornaldaciencia.org.br/Detalhe.jsp?id=41946

ou no site (top25.sciencedirect.com) nos links SubjectArea: Mathematics e no Journal: Indagationes Mathematicae.

E) PIEEX

Quatro dos alunos do Curso de Matemática foram contemplados com bolsa no projeto PIEEX – 2007 nos seguintes projetos:

Mídias na Educação : O Estágio Colaborativo no Trabalho e Rede Rafael de Souza DuarteVirginia Helena Ribeiro MirandaOrientação: Prof. Arlindo José de Souza Júnior

Objetos de Aprendizagem no Cotidiano da Escola Pública Estadual Éliton Meireles de MouraMaisa Gonçalves da Silva Orientação: Profa. Maria Teresa Menezes Freitas

F) PIBEG

Projetos PIBEG da Faculdade de Matemática.

Titulo: Ações construtivas: uma experiência à luz da metodologia presente nas "Olimpíadas de Matemática".

Curso beneficiado: Licenciatura e Bacharelado em Matemática

Membros da equipe executora do Projeto:

Page 480: Numero 09 - Outubro de 2007

Prof. Dr. Walter dos Santos Motta Júnior - Coordenador/Orientador - FAMAT Tabata Saturnina Morais - Bolsista – FAMAT Viviane Carvalho Mendes - Bolsista - FAMAT

Título: Aperfeiçoamento das técnicas de ensino-aprendizagem da disciplina Cálculo Numérico.

Cursos beneficiados: Ciências da Computação, Engenharia Biomédica, Engenharia Civil, Engenharia Elétrica, Engenharia Mecânica, Engenharia Mecatrônica, Engenharia Química, Física dos Materiais e Matemática.

Membros da equipe executora do Projeto:

Profa. Dra. Sezimária F. Pereira Saramago – Coordenadora - FAMAT Prof. Dr. Alessandro Alves Santana – Colaborador - FAMAT Profa. Dra. Célia A.Zorzo Barcelos - Orientadora - FAMAT Prof. Dr. César Guilherme de Almeida – Orientador - FAMAT Paulo Balduino Flabes Neto – Bolsista - Eng. Mecânica Warlisson Inácio de Miranda – Bolsista - FAMAT

Título: Repensando o Ensino de Álgebra Linear na Universidade Federal de Uberlândia.

Cursos beneficiados: Todos os cursos da área de ciências exatas que possuem a disciplina Álgebra Linear em seus currículos: Matemática, Física, Física de Materiais, Computação, Engenharia Química, Engenharia Elétrica, Engenharia Biomédica, Engenharia Civil, Engenharia Mecânica, Engenharia Mecatrônica.

Membros da equipe executora do Projeto:

Prof. Dr. Valdair Bonfim – Coordenador e Orientador - FAMAT Profa. Dra. Lúcia Resende P. Bonfim – Orientadora - FAMAT Letícia Garcia Polac – Bolsista - FAMAT Otoniel Nogueira da Silva – Bolsista - FAMAT

G) PARTICIPAÇÃO EM BANCAS

O Prof. César Guilherme de Almeida participou no dia 17 de agosto, da banca de defesa da tese de Doutorado de Simone Sousa Ribeiro, IPRJ - UFRJ, Pós-graduação em Modelagem Computacional, intitulada "Novos Esquemas Centrias de Diferenças Finitas para a Simulaçao de Escoamentos Multifásicos em Reservatórios de Petróleo".

O Prof. César Guilherme de Almeida participou no dia 17 de agosto, da banca de qualificação de Doutorado de Paulo Sérgio A S Jesus, IPRJ - UFRJ, Pós-graduação em Modelagem Computacional, intitulada "Análise de Incertezas em Escoamentos de Fluidos em Meios Porosos".

Page 481: Numero 09 - Outubro de 2007

O Prof. César Guilherme de Almeida participou no dia 30 de agosto, da banca de defesa de dissertação de mestrado de Maria de Fátima Cursino Borges, UFU – Programa de Pós-Graduação em Educação, intitulada "Interdisciplinaridade e modelagem matemática: Saberes docentes em movimento na formação de professores".

O Prof. Cicero Fernandes de Carvalho participou em 20 de agosto de 2007 de banca de defesa de tese de doutorado na UFMG, onde a aluna Flaviana Andrea Ribeiro defendeu a tese intitulada "Feixes Livre de Torção sobre Curvas com Pontos Duplos Ordinários".

O Prof. Márcio J H Dantas participou no dia 30 de julho, da banca de defesa da tese de Doutorado de Danilo Carlos Pereira, UNICAMP- Eng. Mecânica, intitulada "Dinâmicanão-Linear e Controle de uma aeronave em Vôo Longitudinal".

A Profa. Rosana S Motta Jafelice participou, no dia 01/06/2007, da banca do exame de qualificação de doutorado no IMECC-UNICAMP de Maristela Missio, intitulada "O Uso de Controladores Fuzzy em EDP Aplicado a Febre Aftosa em Bovinos".

H) PRODUÇÃO CIENTÍFICA

PERIÓDICOS

Professor Dr. Cícero Fernandes de Carvalho CARVALHO, C., On V-Weierstrass sets and gaps. Journal of Algebra, v. 312, p.956-962,2007.

CARVALHO, C.; MUÑUERA, Carlos ; SILVA, Ercílio ; TORRES, Fernando, Near Orders and Codes. IEEE Transactions on Information Theory, v. 53, p. 1919-1924, 2007.

Professor Dr. Geraldo Márcio de Azevedo Botelho BOTELHO, G.;PELLEGRINO, D., Coincidence situations for absolutely summing non-linear mappings, Portugaliae Mathematica 64 (2007),175-191.

BARBOSA, J.; BOTELHO, G.; Diniz, D.; PELLEGRINO, D., Spaces of absolutely summing polynomials, Mathematica Scandinavica 101 (2007), 1-19.

Professor Dr. Edmilson Rodrigues Pinto PINTO, E. R., PONCE DE LEON, A. C. M., 2006, Modelagem conjunta da média e dispersão de Nelder e Lee como alternativa aos métodos de Taguchi, Pesquisa Operacional, v. 26, n. 2, pp. 203-224.

Professora Dra. Rosana Sueli da Motta Jafelice JAFELICE, R.M.; BASSANEZI, R.C., Curvas Padrões de Tratamento do HIV, Biomatemática 17 (2007), 55--64, Uma publicação do Grupo de Biomatemática da UNICAMP.

Professor Dr. Vitor Gonzalo L NeumannRational classes and divisors on curves of genus 2, V.G. Lopez Neumann (UFU) - Constantin Manoil (UNIGE - Genève - Suisse) manuscripta math. 120, 403 - 413 (2006)

Page 482: Numero 09 - Outubro de 2007

CAPÍTULO DE LIVRO

Professora Ms. Fabiana Fiorezzi de Marco Matos GRANDO, R. C.; MARCO, F. F. O movimento da resolução de problemas em situações com jogo na produção do conhecimento matemático. In: MENDES, J. R., GRANDO, R. C. (orgs.). Múltiplos olhares: matemática e produção de conhecimento. São Paulo: Musa Editora, 2007.

EVENTOS

INTERNACIONAIS

O Prof. Cicero Carvalho participou de 12 a 18 de Agosto de 2007 do IV Congresso Iberoamericano de Geometria Complexa, em Ouro Preto, onde apresentará o trabalho "Bounds for the set of V-Weierstrass gaps".

O Prof. César Guilherme de Almeida participou de 12 a 16 de agosto de 2007 do VII Latin American Workshop on Magnetism, Magnetic Materials and Their Applications, Rio de Janeiro, onde apresentou o trabalho "Electrical control of nanomagnetism in magnetic ions doped single quantum dots".

O Prof. Edmilson Rodrigues Pinto participou, de 02 a 10/06, do congresso Model Oriented Design and Analysis, na Espanha, onde apresentou o artigo: "Bayesian Ds-optimal designs for generalized linear models with varying dispersion parameter", Model-Oriented Design and Analysis-Physica Verlag, v. 8, pp. 183-190.

O Prof. Geraldo Márcio de Azevedo Botelho apresentou os seguintes trabalhos:

Factorization of holomorphic mappings through operator ideals: the non-surjective case, II Encontro de Análisis Funcional, Universidad de Murcia, Murcia, Espanha, 5 de maio de 2007.

Summability and estimates for polynomials and multilinear mappings, 22nd Summer Conference on Topology and its Applications, Universidad Jaume I, Castellón, Espanha, 24-27 de julho de 2007.

Ideals of homogeneous polynomials from a historical viewpoint (Invited Speaker), Annual Meeting of the Irish Mathematical Society, Dublin, Irlanda, 3-4 de setembro de 2007.

A Profa. Fabiana Fiorezzi de Marco Matos participou da XII Conferência Interamericana de Educação Matemática em Julho de 2007, onde apresentou o trabalho Atividade de ensino na formação inicial de professores.

NACIONAIS

O Prof. Alessandro A. Santana participou, no período de 03 a 06/09/2007, do CNMAC- Florianópolis, onde apresentou o trabalho: "Método de alta ordem - reconstrução de mínimos quadrados para a solução de equações de advecção - difusão em duas dimensões".

Page 483: Numero 09 - Outubro de 2007

O Prof. Arlindo J S Júnior participou de 18 a 21 de julho da mesa redonda no IX ENEM, em Belo Horizonte: "Mídias na Educação Matemática" e apresentou o trabalho científico : Objeto de Aprendizagem e Modelagem Matemática: Saberes Docentes no Cotidiano da Escola".

O Prof. Arlindo J. S. Júnior participou, no período de 03 a 06/09/2007, do CNMAC- Florianópolis, onde apresentou o trabalho: "Mídias na Educação: o Estágio Colaborativo no Trabalho em Rede".

A Prof. Aurélia A. Rodrigues participou de 28 a 31 de agosto de 2007 do XXXIX Simpósio Brasileiro de Pesquisa Operacional - SBPO, em Fortaleza - CE, onde apresentou o trabalho "Gráficos de Controle para Número de Defeituosos".

O Prof. Edmilson R. Pinto participou, de 22 a 27/07, do 52 RBRAS, em Santa Maria -RS, onde apresentou o artigo "Planejamento ótimo de experimento para modelos conjuntos da média e da dispersão".

O Prof. Edmilson R. Pinto participou de 28 a 31 de agosto de 2007 do XXXIX Simpósio Brasileiro de Pesquisa Operacional - SBPO, em Fortaleza - CE, onde apresentou o trabalho "Avanços Recentes em Planejamento Ótimo de Experimentos para Modelos Lineares Generalizados".

O Prof. Edson Agustini participou, no período de 30/07 a 04/08/2007, do 26 Colóquio Brasileiro de Matemática - Impa - Rio de Janeiro - RJ, onde apresentou o trabalho: "Teoria da Informação e Codificação Hiperbólica: mergulho isométrico de constelações de sinais de H2 em R6".

O Prof. Edson Agustini participou de 03 a 06 de setembro de 2007 do XXX Congresso Nacional de Matemática Aplicada e Computacional, Florianópolis, onde apresentou o trabalho "Constelações de Sinais em Planos Hiperbólicos: Mergulhos Isométricos em R6 e em S8".

O Prof. Ednaldo C. Guimarães participou, de 22 a 27/07, do 52ª RBRAS, em Santa Maria -RS, onde apresentou o artigo "Uso de análise de variância na determinação de sazonalidade e tendência índices inflacionários de diferentes instituiçãoes".

O Prof. Ednaldo C. Guimarães participou, de 30/07 a 02/08, do 35º CONBEA, em Bonito -MS, onde apresentou o artigo "Precipitação decendial no município de Uberlândia - MG: Análise descritiva da série temporal".

A Profa. Fabiana Fiorezi M. Matos participou, de 18 a 21/07, do IX Encontro Nacional de Educação Matemática., em Belo Horizonte, onde apresentou o artigo "Jogos Computacionais em aula de Matemática: uma perspectiva para a Resolução de Problemas".

A Profa. Izabel A. Almeida participou, no período de 23 a 25/05/2007, do ENEBI 2007 em Petrópolis -RJ , onde apresentou o artigo "Análise da Resistência à Fratura de Dentes Pré-Molares Tratados Endodonticamente utilizando o Método de Elementos Finitos".

Page 484: Numero 09 - Outubro de 2007

O Prof. Luciano Ferreira Silva participou, de 28/05 a 01/06, do SRV 2007 (Simposium on Virtual and Augmented Reality 2007), em Petrópolis, onde apresentou os artigos:

1) Ambientes distribuídos de Realidade Virtual como suporte a Aprendizagem Colaborativa e a Resolução de Problemas;

2) Realidade Virtual e o Algoritmo de Busca Tabu Aplicados ao Problema de Carregamento de Veículos.

O Prof. Marcelo Tavares participou, de 22 a 27/07, do 52 RBRAS, em Santa Maria -RS, onde apresentou o artigo "Modelo de Previsão para Preciptação Decendial do Município de Corumbaíba-Go".

O Prof. Márcio J. Horta Dantas participou, no período de 22/02 a 24/05/2007, do VI DINCON em S. José do Rio Preto (UNESP), onde apresentou o trabalho "Remarks on Energy Transfer for a Class of Coupled Oscillators".

A Prof. Patrícia O. Costa participou, no período de 03 a 06/09/2007, do CNMAC- Florianópolis, onde apresentou o trabalho: "Mídia e Informação no Ensino de Cálculo"

O Prof. Rogério M. C. Pinto participou, de 22 a 27/07, do 52ª RBRAS, em Santa Maria -RS, onde apresentou o artigo "Modelo de Previsão para Produção de Sucos de Cajú em uma Indústria na Região de Uberlândia".

O Prof. Rogério Sales Gonçalves participou, no período de 22/05 a 25/05/2007, do VI DINCON em S. José do Rio Preto (UNESP), onde apresentou o trabalho "Workspace Positioning and Orientation Optimization of a Parallel Robotic Structure".

I) PALESTRAS E MINI-CURSOS

O Prof. Luiz A. Duran Salomão ministrou, no dia 10/05/07, uma palestra na 3a. Semana da Matemática em Divinópolis (UEMG), intitulada "O Centro de Massa e a Reta de Euler".

O Prof. Luiz A. Duran Salomão ministrou, no dia 20/09 a palestra "Desigualdades, médias e valores extremos", no 5o. Encontro de Matemática da Unicerp - Patrocinio- MG.

A Profa. Rosana S. M. Jafelice proferiu no dia 10/09 no CEMEPE a palestra intitulada: "Modelagem Matemática no Ensino". Esta palestra faz parte do Programa de Formação Continuada para Docentes.

A Profa. Sezimária F. P. Saramago participou, no período de 22/02 a 24/05/2007, do VI DINCON em S. José do Rio Preto (UNESP), onde ministrou a palestra "A Study About the Optimum Path Planning of the Cassino Parallel Manipulator".

Page 485: Numero 09 - Outubro de 2007

O Prof. Walter dos Santos Motta Júnior ministrou o mini-curso "Seqüências Generalizadas" na VII Jornada de Matemática realizada de 11 a 13 de setembro de 2007 em São Carlos.

J) ORIENTAÇÕES - VIII CEMAT

Alunos do VIII Curso de Especialização em Matemática e seus respectivos orientadores de monografia de conclusão de curso, no período de abril de 2007 a março de 2008.

ALUNOS ORIENTADOR

Adriano Rodrigues Teixeira Valdair Bonfim

André Gustavo Cruz da Costa

Luís Antônio Benedetti

André Luiz Ribeiro Lúcia Resende Pereira Bonfim

Cleuber Divino Antônio Carlos Nogueira

Denise Nunes Melo Edmílson Rodrigues Pinto

Espedito Rodrigues César Guilherme de Almeida

Eva Paula P. Almeida Arlindo José de Souza Júnior

Fabiano Elias Reis Dulce Mary de Almeida

Hélen Cristina V. Freitas Luiz Alberto Duran Salomão

Juscélia Dias Mendonça Ednaldo Carvalho Guimarães

Keleey Silva Brito Mario Luiz de Mendonça Faria

Márcia Aparecida Alvarenga Fabiana Fiorezi de Marco Matos

Márcia Lemos Queiroz Maria Teresa Menezes de Freitas

Melissa da Silva Rodrigues Marcos Antônio da Câmara

Page 486: Numero 09 - Outubro de 2007

Entrevista com o professor Luiz Alberto Duran Salomão em relação às

Olimpíadas Brasileiras da Matemática das Escolas Públicas – OBMEP

Neste número, Maria Luiza Maes, aluna do Curso de Matemática, entrevista o

professor Luiz Alberto Duran Salomão, professor da Faculdade de Matemática. O tema da

entrevista é a Olimpíada Brasileira de Matemática das Escolas Públicas (OBMEP), uma

promoção do Ministério da Educação e do Ministério da Ciência e Tecnologia em parceria

com a Sociedade Brasileira de Matemática (SBM) e o Instituto de Matemática Pura e

Aplicada (IMPA), responsáveis por sua direção acadêmica. O entrevistado atua como

professor orientador dos bolsistas da OBMEP no pólo de Uberlândia pelo segundo ano

consecutivo e manifesta aqui sua opinião sobre esta importante realização que é muito mais

do que uma simples competição.

Maria Luiza – Qual é o objetivo da OBMEP?

Luiz Alberto – O objetivo principal da OBMEP é estimular o estudo da matemática através

de problemas motivadores que desafiem a criatividade dos alunos e atraiam o interesse dos

professores. Desse objetivo central, são esperados alguns desdobramentos não menos

importantes como, por exemplo, a revelação de talentos para a matemática, a formação de

futuros pesquisadores e professores dessa área do saber, o aperfeiçoamento de professores e a

melhoria da qualidade da Educação Básica, da qual a matemática é um dos principais

componentes.

M. L. – Os alunos bem classificados na OBMEP têm algum benefício decorrente de seus

desempenhos?

L. A. – Os 2001 alunos com melhor desempenho na OBMEP em todo o Brasil são

contemplados com bolsas de iniciação científica júnior, durante o período de um ano. Nesse

tempo, os bolsistas participam de um programa, sob orientação de professores designados

para este fim, no qual os estudantes têm oportunidade de desenvolver diversos estudos sobre

temas bastante variados da matemática. Como muitas vezes essas atividades são

desenvolvidas em universidades, temos aí um exemplo de forte integração entre as escolas

públicas e o ensino superior. O material que vem sendo utilizado nesse programa é produzido

pela própria OBMEP e está disponível no site www.obmep.org.br . Além disso, aos alunos

detentores de medalhas de ouro nas olimpíadas também têm sido oferecidos outros benefícios,

Page 487: Numero 09 - Outubro de 2007

como a participação em eventos especialmente organizados para eles, nos quais lhes são

oferecidos minicursos, palestras e visitas a importantes centros de ensino e pesquisa. Os

professores e as escolas dos alunos mais destacados também são premiados: os docentes

participam de encontros que têm sido realizados no IMPA, onde desenvolvem diversas

atividades voltadas para sua capacitação profissional; já as escolas, ganham bibliotecas,

laboratórios e outras benfeitorias. Todavia, não devemos deixar de mencionar um outro

benefício: trata-se da melhoria no ambiente escolar. Recentemente, fui convidado a participar

da homenagem que uma tradicional escola pública de Uberlândia prestava a um de seus

estudantes que, pela segunda vez, foi contemplado com bolsa de iniciação científica. Nessa

oportunidade, fiquei bastante impressionado com a beleza da homenagem: os estudantes

participaram expressivamente da cerimônia e, com a orientação das professoras, criaram um

sem-número de artefatos de natureza matemática que utilizaram para decorar o salão. Essa

reunião ocorreu alguns dias antes da realização da prova de 1a fase da OBMEP do corrente

ano e pude perceber que os estudantes a aguardavam com muita expectativa e vibração. Este é

um resultado que, embora seja difícil de se quantificar, é de valor inestimável.

M. L. – As provas estão de acordo com os Parâmetros Curriculares Nacionais – PCN’s?

L. A. – É claro que os objetivos dos PCN’s são muito mais amplos. Como se pode perceber

analisando-se brevemente as provas das OBMEP, essas não têm vínculos com conhecimentos

específicos, pautando-se principalmente pela avaliação de habilidades na resolução de

problemas, privilegiando sobretudo a criatividade dos alunos e seu potencial para estudos

posteriores em matemática. Contudo, os PCN’s enfatizam fortemente a questão da

interdisciplinaridade no desenvolvimento dos conteúdos de um modo geral e, nesse aspecto,

pode-se afirmar que as provas da OBMEP estão em plena conformidade com tais parâmetros,

trazendo questões interessantes e naturalmente contextualizadas.

M. L. – Qual a comparação que pode ser feita entre a OBMEP e a Olimpíada Brasileira de

Matemática (OBM)?

L. A. – A OBM é uma atividade que vem sendo desenvolvida pela SBM desde 1979 e é

através dela que são selecionados os participantes das olimpíadas internacionais, nas quais o

Brasil tem tido participações importantes. Todavia, com a criação da OBMEP, a SBM tem

conseguido atingir contingentes muito mais numerosos já que os estudantes das escolas

públicas não eram suficientemente incentivados a participar da OBM. No entanto, esse quadro

Page 488: Numero 09 - Outubro de 2007

já vem sendo revertido uma vez que, no corrente ano, os bolsistas da OBMEP poderão

participar da OBM já na sua 2a fase.

M. L. – O que a OBMEP espera dos alunos?

L. A. – A resposta a essa pergunta, de certa forma, está inserida nos próprios objetivos da

OBMEP. Evidentemente, espera-se que os alunos das escolas públicas sintam-se, através dos

desafios para a resolução de problemas, mais motivados para o estudo da matemática e que

isso repercuta em uma educação de melhor qualidade para os nossos jovens.

M. L. – As etapas do Polya ficam evidenciadas nas provas?

L. A. – Certamente você deve estar se referindo às quatro etapas da resolução de um

problema que G. Polya descreve em seu conhecido livro “A Arte de Resolver Problemas”,

não é mesmo? Primeiramente, eu gostaria de dizer que esse livro é uma preciosidade e deve

ser lido por todos que se interessam pelo ensino de matemática. No entanto, acredito que o

uso do termo “evidenciadas” não é o mais adequado pois poderia induzir o leitor dessa

entrevista a acreditar que, conhecendo as tais etapas, o aluno teria algo como um roteiro para

resolver o problema. Na verdade, isso não acontece. O que deve ficar claro é que a leitura do

livro, embora útil e agradável, jamais irá substituir a importância do treinamento, da

experiência e do exercício da criatividade na resolução de problemas.

M. L. – Qual é o tipo de exercícios que ocorrem nas provas?

L. A. – As provas de um modo geral requerem pouco conhecimento. O fundamental na

resolução dos exercícios é a criatividade. Contudo, o leitor dessa entrevista deve consultar as

provas e decidir por si mesmo se compartilha de minha opinião. O site da OBMEP

disponibiliza todas as provas anteriores.

M. L. – Você poderia nos explicar por que motivo um mesmo exercício pode aparecer nos

três níveis de prova?

L. A. – A explicação é que muitos dos problemas propostos nas provas têm uma vinculação

muito pequena com conteúdos matemáticos e, por isso, suas resoluções dependem quase que

somente da habilidade do candidato. Assim, tais questões podem ser adequadas a mais de um

nível.

Page 489: Numero 09 - Outubro de 2007

M. L. – Como você descreveria as provas da OBMEP?

L. A. – Acho que essa pergunta já foi respondida em parte. Além do que já foi dito, eu

poderia acrescentar que algumas questões, sobretudo de 1a fase, não são difíceis. Creio que

isso se deve à intenção de não afugentar os candidatos. Na 2a fase aparece questões um pouco

mais complicadas, sobretudo pelo fato da prova ser discursiva e requerer do candidato certo

domínio da linguagem. Conversando com alguns alunos que participaram de todas as

olimpíadas, tenho ouvido opiniões um pouco divergentes, mas parece-me que a opinião mais

freqüente é de que as provas, a cada ano, têm ficado um pouco mais difíceis.

M. L. – O aluno deve possuir qual tipo de conhecimento sobre resolução de problemas nas

provas?

L. A. – Na verdade, o aluno não precisa de conhecimentos sobre resolução de problemas. O

que ele precisa é gostar de resolver problemas, ter autoconfiança diante de um desafio e ter

adquirido alguma experiência na resolução de problemas. Além disso, alguns fatores como,

por exemplo, domínio da linguagem oral e escrita, capacidade de interpretação de textos e

uma boa formação inicial em aritmética e geometria são de grande valor para o desempenho

dos alunos.

M. L. – Para finalizar, diga-nos alguma coisa sobre sua experiência com a orientação dos

bolsistas da OBMEP.

L. A. – Para mim essa é a parte mais interessante das olimpíadas. A turma atual que eu e o

professor Jocelino Sato orientamos no pólo de Uberlândia tem dezenove alunos, com idades

variando de 12 a 18 anos. Os mais novos cursam a sexta série do Ensino Fundamental e, no

outro extremo, temos três bolsistas que já são universitários. Trabalhar com esses jovens é

para nós uma experiência muito rica. Temos, em média, dois encontros mensais com esses

bolsistas, com a duração de oito horas cada um. Nesses encontros, estes jovens, além de

estudar assuntos que lhes são inteiramente novos, são permanentemente desafiados a resolver

problemas não triviais. Freqüentemente ficamos admirados com a habilidade, o entusiasmo e

a criatividade desses jovens estudantes quando eles nos apresentam soluções absolutamente

originais para esses problemas. Acreditamos que esse programa, em um futuro muito

próximo, irá contribuir fortemente para a revelação de grandes talentos para a matemática e as

ciências de um modo geral.

Page 490: Numero 09 - Outubro de 2007
Page 491: Numero 09 - Outubro de 2007

Olimpıadas de Matematica

Prof. Antonio Carlos Nogueira

1 Introducao

Em cerca de 2500 a.C., os gregos realizavam festivais esportivos em honra a Zeus nosantuario de Olımpia - o que originou o termo olimpıada. O evento era tao importanteque interrompia ate as guerras. Os nomes dos vencedores das competicoes comecam a serregistrados a partir de 776 a.C. Atualmente, a cada quatro anos, atletas de centenas depaıses se reunem num paıs sede para disputarem um conjunto das mais variadas modali-dades esportivas. Sao os Jogos Olımpicos ou Olimpıadas.

Neste pequeno texto queremos destacar, na verdade, um outro tipo de Olimpıada: aOlimpıada de Matematica. Mas afinal, o que e uma Olimpıada de Matematica?

Podemos definir uma Olimpıada de Matematica como uma competicao equivalenteas esportivas (como natacao, futebol) ou como os concursos de literatura e festivais demusica. Como qualquer disputa, a Olimpıada de Matematica tambem exige preparacaoespecıfica. Os ”atletas”de matematica se preparam atraves da resolucao de problemas deMatematica, individualmente ou em grupos. Eles ”treinam”como objetivo de desenvolvera habilidade logica, a criatividade e a sociabilidade, bem como metodos adequados depensamento e de trabalho.

A importancia da resolucao de problemas e reconhecida mundialmente como um as-pecto central no processo de ensino-aprendizagem nao so de matematica mas de ciencias,em geral.

(...) a historia da matematica mostra que os avancos matematicos quase semprese originam de um esforco para resolver um problema especıfico. (Dieudonne,citado em [4])

A resolucao de problemas e o coracao desta disciplina. (Halmos, citado em [4])

A resolucao de problemas e a principal razao para estudar matematica, na linhade considera-la como um processo de conhecimentos previamente adquiridos parasituacoes novas e desconhecidas. Resolver problemas supoe apresentar questoes,analisar situacoes, traduzir resultados, desenhar diagramas e refutar provas e erros.(Blanco Nieto, citado em [4])

Page 492: Numero 09 - Outubro de 2007

Problemas que envolvem raciocınio e criatividade sao muito estimulantes e envolventese despertam o interesse de jovens estudantes. O objetivo da resolucao de problemas naoe a busca de uma solucao especıfica, mas sim o ato de facilitar o conhecimento das ha-bilidades basicas, os conceitos fundamentais e a relacao entre ambos. Isto propiciaraao ”resolvedor”de problemas a capacidade de lidar com situacoes semelhantes e tambemcom situacoes novas, a partir do momento que ele comeca a desenvolver uma atitudeinvestigativa e inquiridora. Este e o objetivo da Olimpıada de Matematica: mostrar aimportancia da matematica no desenvolvimento das ciencias atraves da resolucao de prob-lemas e tambem comecar a formar naqueles que os resolvem esta atitude investigativa.

Uma grande descoberta resolve um grande problema; mas na solucao de todoproblema ha uma certa descoberta. (Polya, citado em [4])

Um problema e algo que precisa da criatividade de quem aprende, exigindo-lhe a in-corporacao de elementos de aprendizagem precedentes para se chegar a solucao. Quandoum problema e resolvido, aprende-se algo novo.

2 Olimpıadas de Matematica: historico

Encontramos nos textos de Historia da Matematica que, em tempos mais antigos, matematicosdesafiavam uns aos outros propondo problemas e muitas vezes estas disputas eram real-izadas em pracas publicas.

Por volta de 1515, Scipioni del Ferro (1465-1526), professor de matematica da Uni-versidade de Bolonha, resolveu algebricamente a equacao cubica x3+mx = n.(...)Ele nao publicou o trabalho mas revelou o segredo a seu discıpulo Antonio Fior.Por volta de 1535, Nicolo Fontana de Brescia, mais conhecido como Tartaglia (otartamudo), devido a lesoes fısicas sofridas quando crianca que afetaram sua fala,anunciou ter descobert uma solucao algebrica para a equacao cubica x3+px2 = n.Achando que se tratava de blefe, Fior desafiou Tartaglia para uma disputa publicaenvolvendo a resolucao de equacoes cubicas. Com muito empenho, Tartaglia con-seguiu resolver tambem, faltando poucos dias para a disputa, a equacao cubicadesprovida do termo quadratico. Como no dia marcado sabia resolver dois tiposde cubicas, ao passo que Fior so sabia resolver um, Tartaglia triunfou plenamente.([3])

O relato acima e apenas um dos varios episodios de disputas envolvendo a Matematica.Assim, o que nasceu talvez por um capricho de certas pessoas tomou forma mais salutarcom a realizacao da 1a Olimpıada de Matematica, realizada na Hungria, no sec. XIX.

A partir de entao vem se organizando por todo o mundo competicoes de matematicaentre estudantes de diversos nıveis de escolaridade. Desde 1959, se realiza, anualmente,

Page 493: Numero 09 - Outubro de 2007

a Olimpıada Internacional de Matematica (IMO), da qual participam alunos do ensinomedio de cerca de 100 paıses (entre eles o Brasil). Outra competicao semelhante e aOlimpıada Ibero-Americana de Matmatica que conta com a participacao de mais de 20paıses da America Latina, alem de Espanha e Portugal. Em 1994, foi criada a OlimpıadaInternacional de Matematica para estudantes universitarios (a IMC).

No Brasil, em 1977, a Academia Paulista de Ciencias criou a Olimpıada Paulista deMatematica, que talvez tenha sido a semente para que, em 1979, fosse organizada, pelaSociedade Brasileira de Matematica (SBM), a primeira edicao da Olimpıada Brasileirade Matematica (OBM). A OBM esta hoje em sua 29a edicao. A partir daı, varias ini-ciativas regionais surgiram, de forma que foram implantadas varias Olimpıadas estaduais(Ceara, Sergipe, Rio de Janeiro, Minas Gerais, entre outros) e mesmo competicoes decarater regional (Olimpıada Regional de Rio Preto, Olimpıada Pessoense de Matematica,Olimpıada de Matematica de Sao Joao del Rey, etc).

Em 2005, foi criada a Olimpıada Brasileira de Matematica das Escolas Publicas (OB-MEP) com o aval dos Ministerios da Ciencia e Tecnologia e da Educacao. Como o proprionome diz uma Olimpıada de Matematica voltada exclusivamente para estudantes das es-colas publicas brasileiras.

3 O Brasil nas Olimpıadas

O Brasil tem tido participacao expressiva nas Olimpıadas Internacionais, como podemosver nas informacoes seguintes, fornecidas pela secretaria geral da OBM.

1. Olimpıada de Maio

• Paıses participantes: Todos os paıses ibero-americanos.

• Faixa etaria - escolar: Nıvel 1 12 a 13 anos; Nıvel 2 14 a 15 anos.

• Numero de estudantes por paıs por nıvel: 10

• Posicionamento: O Brasil tem mantido um amplo e consistente domınio. Sem-pre conseguimos obter a maior pontuacao dentre os participantes e a premiacaomaxima: 1 ouro, 2 pratas, 4 bronzes e 3 mencoes.

2. Olimpıada de Matematica do Cone Sul:

• Paıses participantes: Argentina, Bolıvia, Brasil, Chile, Paraguai, Peru e Uruguai.

• Faixa etaria - escolar: Estudantes ate 16 anos.

• Numero de estudantes por paıs: 4

• Posicionamento: O Brasil participa desta importante competicao desde 1988 etem dominado a competicao conquistando ate hoje um total de 65 medalhas,sendo 18 de ouro, 24 de prata e 23 de bronze.

3. Olimpıada Ibero-Americana de Matematica:

Page 494: Numero 09 - Outubro de 2007

• Paıses participantes: Todos os paıses ibero-americanos.

• Faixa etaria - escolar: Estudantes ate 18 anos.

• Numero de estudantes por paıs: 4

• Posicionamento: Em 2004 e 2005, o Brasil conseguiu o feito, ate entao inedito,de uma equipe ganhar 4 ouros. Tais fatos mostram a primazia brasileira nacompeticao. O Brasil participa desta olimpıada desde 1985 conquistando desdeentao um total de 77 medalhas, sendo 42 de ouro, 25 de prata e 10 de bronze.

4. Olimpıada Ibero-Americana de Matematica Universitaria:

• Paıses participantes: Argentina, Brasil, Colombia, Cuba, Equador, Espanha,Mexico, Peru, Portugal, Uruguai e Venezuela.

• Faixa etaria - escolar: Estudantes universitarios.

• Numero de estudantes por paıs: 10

• Posicionamento: O Brasil tem mantido um amplo e consistente domınio. Sem-pre conseguimos obter a maior pontuacao dentre os participantes e a premiacaomaxima: 1 ouro, 2 pratas, 4 bronzes e 3 mencoes.

5. Olimpıada Internacional de Matematica (IMO):

• Paıses participantes: Aproximadamente 100 dos cinco continentes. Todos ospaıses de maior destaque na ciencia mundial participam.

• Faixa etaria - escolar: Estudantes pre-universitarios de ate 19 anos.

• Numero de estudantes por paıs: 6

• Posicionamento: Inicialmente, vale a pena conhecer um pouco do panoramamundial. As grandes forcas mundiais sao China, Russia e Estados Unidos.Tambem tem grande forca, da antiga Cortina de Ferro: Bielo-russia, Bulgaria,Hungria, Polonia, Romenia e Ucrania; da Europa Ocidental: Alemanha, Francae Reino Unido; da Oceania: Australia; da America: Canada; da Asia: Caza-quistao, Coreia do Sul, India, Ira, Japao, Taiwan e Vietna; alem de Israel eda Turquia. Pode-se perceber entao que, para obter um posto entre os 20primeiros, devem ser vencidos paıses de enorme tradicao. Desde 2001, quandoos 6 brasileiros ganharam medalhas e ficamos em 16o lugar na classificacaogeral, o Brasil tem conseguido vencer a varias destas nacoes. Nestas ultimasedicoes conseguimos ficar a frente em uma ou mais ocasioes de Franca, Ale-manha, Reino Unido, Canada, Hungria, Australia, Israel, India, Ira, Polonia eCazaquistao. Temos lutado bastante para garantir uma posicao estavel entreas 20 maiores forcas mundiais. O Brasil participa desta importante competicaodesde 1979 conquistando desde entao um total de 75 medalhas, sendo 7 de ouro,13 de prata e 55 de bronze. Este ano o Brasil foi o melhor paıs classificado noambito ibero-americano.

6. Olimpıada Internacional de Matematica Universitaria (IMC):

Page 495: Numero 09 - Outubro de 2007

• Esta e uma olimpıada cuja participacao e por Universidade e nao por paıs.Cada equipe consta de um professor-lıder e 4 estudantes. 147 Instituicoes detodo o mundo tem participado da competicao, entre as quais podemos destacar:MIT, Princeton, Universidade de Moscou, Universidade Eotvos Lorand de Bu-dapeste, Ecole Polytechnique de Paris, Instituto Politecnico de Kiev, Uni-versidade de Bonn, Universidade Complutense de Madrid e Universidade deVarsovia. A edicao mais recente contou com a participacao de mais de 70 uni-versidades. Devido aos nossos bons resultados na IMO e, agora, no proprioIMC, temos uma autorizacao especial para enviar uma equipe brasileira comalunos de varias universidades e institutos de ensino superior.

• Posicionamento: O numero de medalhas conquistado pelos nossos estudantesem cinco anos de participacao e impressionante: 35 medalhas, sendo 1 de ouroespecial (Grand First Prize), 7 de ouro, 11 de prata, e 16 de bronze.

Mais informacoes sobre as Olimpıadas de Matematica no Brasil podem ser encon-tradas no site da OBM www.obm.org.br.

4 A OBMEP

No intuito de ampliar, a partir de experiencias bem sucedidas - por exemplo, o ProjetoNumeratizar, do Estado do Ceara - a participacao de estudantes de escolas publicas nasOlimpıadas de Matematica, foi criada, em 2005, a Olimpıada Brasileira de Matematicadas Escolas Publicas - OBMEP. Evento patrocinado pelo Governo Federal, atraves dosMinisterios da Educacao e da Ciencia e Tecnologia (MEC e MCT) a OBMEP tem sidoexecutada pelo Instituto Nacional de Matematica Pura e Aplicada (IMPA), vinculado aoMinisterio da Ciencia e Tecnologia, e pela Sociedade Brasileira de Matematica (SBM). Otrabalho que vem sido desenvolvido responde a um desafio extremamente enriquecedor,cujos resultados, espera-se, valorizarao estudantes e professores, bem como suas escolas,seus municıpios e seus estados.

4.1 Objetivos da OBMEP

A OBMEP e um projeto de inclusao social voltado para a Escola Publica, seus estudantese professores. Tem por objetivo oferecer oportunidades a uma parcela da sociedade que,em geral, tem pouco convıvio e acesso ao exercıcio e producao do conhecimento. Seuspressupostos basicos sao:

• a Matematica contribui, de modo determinante, para a formacao do indivıduo con-sciente e oferece multiplas oportunidades para o seu desempenho na sociedade;

• o talento para a Matematica e Ciencias esta aleatoriamente distribuıdo pelo Paıs,nao dependendo de cor, sexo ou classe social;

• premiar a competencia e o esforco de alunos e professores e a forma mais eficientede motiva-los e de resgatar a qualidade como valor na Educacao publica;

• aproveitar o potencial cientıfico de nossos jovens talentos e estrategico para o de-senvolvimento do Paıs.

Page 496: Numero 09 - Outubro de 2007

Os principais objetivos da OBMEP sao:

1. estimular e promover o estudo da Matematica entre alunos das escolas publicas;

2. identificar jovens talentos e fornecer oportunidades para seu ingresso nas areascientıficas e tecnologicas;

3. incentivar o aperfeicoamento dos professores das escolas publicas, contribuindo assimpara a sua valorizacao profissional;

4. contribuir para a melhoria do ensino da Matematica na rede publica;

5. contribuir para a integracao entre as escolas publicas, as universidades federais, osinstitutos de pesquisa e as sociedades cientıficas.

4.2 Descricao da OBMEP

A OBMEP consta da realizacao de uma competicao nacional de Matematica entre asescolas publicas seguida de treinamento para os alunos e os professores premiados. Ele edirigido as escolas publicas municipais, estaduais e federais.

A Olimpıada esta dividida em tres nıveis:

• Nıvel I - 5a e 6a series

• Nıvel II - 7a e 8a series

• Nıvel III - Ensino Medio.

A competicao e realizada em duas fases sendo que na primeira fase participam todosos alunos de todas as escolas inscritas. A prova, nesta etapa, consiste de 20 questoes demultipla escolha. Na 2a fase participam apenas 5% dos alunos, de cada escola, com melhordesempenho na 1a fase. A prova, nesta etapa, e discursiva e realizada sob a supervisao defiscais escolhidos pela SBM/IMPA em locais definidos pelos Coordenadores Regionais.

4.3 Alguns numeros

A OBMEP esta, hoje, em sua terceira edicao. No que segue relataremos alguns dadossobre a 1a e a 2a OBMEP e tambem alguns dados sobre a OBMEP - 2007.

4.3.1 OBMEP - 2005

Na OBMEP-2005 foram inscritos 10.520.830 alunos de 31.030 escolas distribuıdas em5.198 municıpios de todo o Brasil. No Estado de Minas Gerais foram inscritos 1.297.079alunos de 3498 escolas em 813 municıpios. A media de notas (nacional) da 1a fase foi10,9 pontos (num total de 20 pontos). Em Minas Gerais, a media foi 11,7 pontos. Paraa 2a fase foram classificados 457.718 alunos sendo 60.126 de Minas Gerais. Nesta etapaa media nacional de notas foi 23,5 pontos (considerando os tres nıveis) em 120 pontospossıveis. A media em Minas Gerais foi 29,6 pontos (terceira maior media do Brasil).

Page 497: Numero 09 - Outubro de 2007

4.3.2 OBMEP - 2006

Em 2006, na 2a Olimpıada Brasileira de Matematica das Escolas Publicas participaramda 1a fase 14.181.705 alunos de 32.655 escolas publicas em 5259 municıpios brasileiros.Na segunda fase, participaram 630.849 alunos de 29.661 escolas. Em Minas Gerais, par-ticiparam da 1a fase 1.653.094 alunos de 3.558 escolas de 808 municıpios. Na segundafase, participaram 77.292 alunos de 797 municıpios.

4.3.3 OBMEP - 2007

A OBMEP esta, hoje, na 3a versao (a OBMEP - 2007). Mais uma vez, os numerossurpreendem: foram inscritos na 1a fase da OBMEP deste ano cerca de 17,3 milhoes deestudantes de 38.450 escolas de todo o Brasil. Isto faz da OBMEP a maior competicao dematematica do mundo. A primeira fase foi realizada em 14 de agosto passado e a segundaesta programada para 20 de outubro.

4.3.4 Premiacao

A OBMEP premia alunos (medalhas - ouro, prata e bronze -, professores, escolas e mu-nicıpios). Os alunos melhores classificados recebem uma bolsa do tipo Iniciacao CientıficaJr. do CNPq (em 2005 e 2006 foram distribuıdas 2001 bolsas, na OBMEP 2007 estaoprevista 3.000 bolsas) e outros 30 mil alunos recebem mencao honrosa pela participacaono evento; os professores dos alunos mais bem posicionados participam de um curso deaperfeicoamento oferecido pelo IMPA; as escolas recebem laboratorios de informatica,livros e certificados de merito nacional.

Ressaltamos, no entanto, que a OBMEP nao termina com a aplicacao das provas da2a fase ou com a entrega dos premios aos vencedores. Os alunos ganhadores das bolsasda OBMEP participam de um programa de capacitacao, durante um ano, programa esteque e orientado, em geral, por professores das universidades brasileiras, e organizado esuprevisionado pelo IMPA.

5 Conclusao

As Olimpıadas de Matematica sao um instrumento extremamente eficiente para a de-scoberta de jovens com talento cientıfico. O aprimoramento de tais jovens leva-os a de-senvolver uma enorme capacidade de resolver problemas e de apresentar solucoes criativase inovadoras. Se bem orientados, estes jovens serao os cientistas do futuro e com certeza,profundamente envolvidos com o desenvolvimento nacional.

Referencias

[1] www.obmep.org.br

[2] www.obm.org.br

Page 498: Numero 09 - Outubro de 2007

[3] Eves, H.W. Introducao a historia da matematica , Campinas, SP : UNICAMP,1997.

[4] Huete, J.C.S. & Bravo, J.A.F. O ensino da matematica: fundamentosteoricos e bases psicopedagogicas, Ed. Artmed, 2006, Porto Alegre.